You are on page 1of 375

JEE MAIN 2021

KINEMATICS-1D
1. If the velocity-time graph has the shape AMB, what would be the shape of the corresponding
acceleration-time graph ? [JEE MAIN 2021 (FEB)]

(1) (2)

(3) (4)

2. A particle is projected with velocity v0 along x-axis. A damping force is acting on the particle
which is proportional to the square of the distance from the origin i.e., ma = –x2. The distance at
which the particle stops : [JEE MAIN 2021 (FEB)]
1 1 1 1
 3v 2  2  2v  3  2v 2  2  3v 2  3
(1)  0  (2)  0  (3)  0  (4)  0 
 2   3   3   2 

3. A stone is dropped from the top of a building. When it crosses a point 5 m below the top, another
stone starts to fall from a point 25 m below the top. Both stones reach the bottom of building
simultaneously. The height of the building is : [JEE MAIN 2021 (FEB)]
(1) 35 m (2) 45m (3) 50 m (4) 25m

4. A scooter accelerates from rest for time t1 at constant rate a1 and then retards at constant rate a2 for
t
time t2 and comes to rest. The correct value of 1 will be : [JEE MAIN 2021 (FEB)]
t2
a a a a a a
(1) 1 2 (2) 2 (3) 1 (4) 1 2
a2 a1 a2 a1

5. The velocity-displacement graph of a particle is shown in the figure.


[JEE MAIN 2021 (MARCH)]

NUCLEUS-92, Rajeev Gandhi Nagar, Kota (Raj.) India 324005, Mob. 9358006181, 97831-97831 1
JEE MAIN 2021
The acceleration-displacement graph of the same particle is represented by:

(1) (2) (3) (4)

6. The position, velocity and acceleration of a particle moving with a constant acceleration can be
represented by: [JEE MAIN 2021 (MARCH)]

(1)

(2)

(3)

(4)

7. The velocity – displacement graph describing the motion of a bicycle is shown in the figure.
[JEE MAIN 2021 (MARCH)]

The acceleration-displacement graph of the bicycle's motion is best described by:

NUCLEUS-92, Rajeev Gandhi Nagar, Kota (Raj.) India 324005, Mob. 9358006181, 97831-97831 2
JEE MAIN 2021

(1) (2)

(3) (4)

8. A mosquito is moving with a velocity v  0.5t 2 î  3tjˆ  9k̂ m/s and accelerating in uniform
conditions. What will be the direction of mosquito after 2s ? [JEE MAIN 2021 (MARCH)]
–1  2  1  2 
(1) tan   from x-axis (2) tan   from y -axis
3 3
5 5
(3) tan 1   from y -axis (4) tan 1   from x -axis
2 2

9. A car accelerates from rest at a constant rate  for some time after which it decelerates at a
constant rate  to come to rest. If the total time elapsed is t seconds, the total distance travelled is:
[JEE MAIN 2021 (MARCH)]
4 2 2 2
(1) t (2) t
     
 
(3) t2 (4) t2
2    4   

10. A number ball is released from a height of 5 m above the floor. It bounces back repeatedly, always
rising to (81/100)of the height through which it falls. Find the average speed of the ball.
(Take g = 10 ms–2) [JEE MAIN 2021 (MARCH)]
(1) 3.0 ms–1 (2) 3.50 ms–1
(3) 2.0 ms–1 (4) 2.50 ms–1

11. The velocity of a particle is v = v0 + gt + Ft2. Its position is x = 0 at t = 0; then its displacement
after time (t = 1) is: [JEE MAIN 2021 (MARCH)]
g F
(1) v0 + g + F (2) v0 + 
2 3
g
(3) v 0   F (4) v0  2g  3F
2

NUCLEUS-92, Rajeev Gandhi Nagar, Kota (Raj.) India 324005, Mob. 9358006181, 97831-97831 3
JEE MAIN 2021
12. A butterfly is flying with a velocity 4 2 m/s in North – East direction. Wind is slowly blowing at
1 m/s from North to South . The resultant displacement of the butterfly in 3 seconds is :
[JEE MAIN 2021 (JULY)]
(1) 3 m (2) 20 m (3) 12 2m (4) 15 m

13. A body having specific charge 8 C / g is resting on a frictionless plane at a distance 10 cm from
the wall (as shown in the figure) . It starts moving towards the wall when a uniform electric field
of 100 V/m is applied horizontally towards the wall. If the collision of the body with wall is
perfectly elastic, then the time period of the motion will be______s. [JEE MAIN 2021 (JULY)]

14. A boy reaches the airport and finds that the escalator is not working. He walks up the stationary
escalator in time t1. If he remains stationary on a moving escalator then the escalator takes him up
in time t2. The time taken by him to walk up on the moving escalator will be:
[JEE MAIN 2021 (JULY)]
tt t t tt
(1) 1 2 (2) 1 2 (3) 1 2 (4) t 2  t1
t 2  t1 2 t 2  t1

15. Water droplets are coming from an open tap at a particular rate. The spacing between a droplet
observed at 4th second after its fall to the next droplet is 34.3 m. At what rate droplets are coming
from the tap? (Take g = 9.8 m/s2) [JEE MAIN 2021 (JULY)]
(1) 3 drops / 2 seconds (2) 2 drops / second
(3) 1 drop / second (4) 1 drop / 7 second

16. A particle of mass ‘m’ is moving in time ‘t’ on a trajectory given by [JEE MAIN 2021 (JULY)]
r  10 t 2ˆi  5  t  5 ˆj

Where  and  are dimensional constants.


The angular momentum of the particle becomes the same as it was for t = 0 at time
t =______seconds.

17. The relation between time t and distance x for a moving body is given as t = mx 2+ nx , where m
and n are constant. The retardation of the motion is : (When v stands for velocity)
[JEE MAIN 2021 (JULY)]
3 3 3
(1) 2 mv (2) 2 mnv (3) 2nv (4) 2N2v3

NUCLEUS-92, Rajeev Gandhi Nagar, Kota (Raj.) India 324005, Mob. 9358006181, 97831-97831 4
JEE MAIN 2021
18. A force F  (40 ˆi  10ˆj) N acts on a body of mass 5 kg. If the body start from rest, its position
vector r̂ at time t = 10 s, will be [JEE MAIN 2021 (JULY)]
(1) (100i  400 j)m (2) (100i  100 j)m (3) (400i  100 j)m (4) (400iˆ  400ˆj)m
ˆ ˆ ˆ ˆ ˆ ˆ

19. The instantaneous velocity of a particle moving in a straight line is given as v  t  t 2 , where

:
 and  are constant. The distance travelled by the particle between 1s and 2s is :
[JEE MAIN 2021 (JULY)]
3 7 3 7
(1) 3  7 (2)   (3) V (4)   
2 3 2 2

20. An automobile of mass 'm' accelerates starting from origin and initially at rest, while the engine
supplies constant power P. The position is given as a function of time by :
[JEE MAIN 2021 (JULY)]
1 1 1 1
3 3 3 3
 9P  2 2  8P  2 2  9m  2 2  8P  2 2
(1)   t (2)   t (3)   t (4)   t
 8m   9m   8P   9m 

21. A particle of mass M originally at rest is subjected to a force whose direction is constant but
magnitude varies with time according to the relation [JEE MAIN 2021 (JULY)]
  t T  
2

F  F0 1    
  T  
Where F0 and T are constant. The force acts only for the time interval 2T. The velocity v of the
particle after time 2T is :
(1) 2F0T/M (2) F0T/2M (3) 4F0T/3M (4) F0T/3M

22. A swimmer wants to cross a river from point A to point B. Line AB makes an angle of 300 with
the flow of river. Magnitude of velocity of the swimmer is same as that of the river. The angle 
with the line AB should be ________ 0, so that the swimmer reaches point B.
[JEE MAIN 2021 (JULY)]

23. The ranges and heights for two projectiles projected with the same initial velocity at angles 42°
and 48° with the horizontal are R1, R2 and H1, H2 respectively. Choose the correct option :
[JEE MAIN 2021 (AUGUST)]
(1) R1> R2 and H1 = H2 (2) R1 = R2 and H1< H2
(3) R1< R2 and H1< H2 (4) R1 = R2 and H1 = H2

NUCLEUS-92, Rajeev Gandhi Nagar, Kota (Raj.) India 324005, Mob. 9358006181, 97831-97831 5
JEE MAIN 2021
-1
24. Car B overtake another Car A at a relative speed of 40 ms . How fast will the image of Car B
appear to move in the mirror of focal length 10 cm fitted in Car A, when the Car B is 1.9 m away
from the Car A? [JEE MAIN 2021 (AUGUST)]
(1) 4 ms–1 (2) 0.2 ms–1 (3) 40 ms–1 (4)0.1 ms–1

25. Water drops are falling from a nozzle of a shower onto the floor, from a height of 9.8 m. The

:
drops fall at a regular interval of time . When the first drop strikes the floor, at that instant , the
third drop begins to fall. Locate the position of second drop from the floor when the first drop
strikes the floor . [JEE MAIN 2021 (AUGUST)]
(1) 4.18 m (2) 2.94 m (3) 2.45 m (4) 7.35 m

26. If the velocity of a body related to displacement x is given by √ m/s, then the
acceleration of the body is .....m/s2. [JEE MAIN 2021 (AUGUST)]

27. A particle is moving with constant acceleration 'a'. Following graph shown v2 versus x
(displacement) plot. The acceleration of the particle is _____m/s2.
[JEE MAIN 2021 (AUGUST)]

ANSWER KEY
1. 2 2. 4 3. 2 4. 2 5. 2 6. 2 7. 1
8. 2 9. 3 10. 4 11. 2 12. 4 13. 1 14. 3
15. 3 16. 50 17. 1 18. 3 19. 2 20. 4 21. 3
22. 30 23. 2 24. 4 25. 4 26. 12 27. 1

NUCLEUS-92, Rajeev Gandhi Nagar, Kota (Raj.) India 324005, Mob. 9358006181, 97831-97831 6
JEE MAIN 2021
SOLUTION
1. (2)
Sol. Slope of v-t graph gives acceleration

 Acceleration will be

2. (4)
Sol. F = –x2
ma = –x2
x 2
a
m
vdv 
  x2
dx m
0 x
 2
v vdv  0  m x dx
0
0 x
 v2    x3 
    
 2  v0 m  3 0
v02  x3

2 m 3
1
 3mv02  3 Option (4) is most suitable option as
x  
 2  (m) is not given in any option
3. (2)
Sol.

NUCLEUS-92, Rajeev Gandhi Nagar, Kota (Raj.) India 324005, Mob. 9358006181, 97831-97831 7
JEE MAIN 2021
Srel 20
Time for particle to meet = t' =   2sec
Srel 10
Time taken by Ist particle to reach ground = 3 sec
1
H  g  3  45 m
2

4. (2)
Sol. Draw v–t curve

v max
tan 1  a1 
t1
v
& tan 2  a 2  max
t2
 above
t1 a 2

t 2 a1

5. (2)
v 
Sol. v  –  0  x  v0
 x0 
vdv
a
dx
 v    v0 
a  –  0  x  v0   – 
  x 0    x 0 
v  v
a   0 x – 0
 x0  x0

6. (2)
Sol. Option (2) represent correct graph for particle moving with constant acceleration, as for constant
acceleration velocity time graph is straight line with positive slope and x-t graph should be an
opening upward parabola.

7. (1)
Sol. For 0  x  200
v  mx  C
1
v x  10
5

NUCLEUS-92, Rajeev Gandhi Nagar, Kota (Raj.) India 324005, Mob. 9358006181, 97831-97831 8
JEE MAIN 2021
vdv  x  1 
a    10  
dx  5  5 

x
a  2  Straight line till x  200
25
for x > 200
v = constant
a0
Hence most appropriate option will be (1).
otherwise it would be BONUS.

8. (2)
Sol. Given:
v  0.5t 2 î  3tjˆ  9k̂
⃗ ̂ ̂ ̂
 Angle made by direction of motion of mosquito will be,
2 117
cos1 ( from x -axis )  tan 1
11 2
6 85
cos 1 ( from y-axis )  tan 1
11 6
9 40
cos 1 ( from z-axis )  tan 1
11 9
None of the option is matching.
Hence this question should be bonus.

9. (3)
Sol. v0  t1 and 0  v0  t 2  v0  t 2
t1 + t2 = t

NUCLEUS-92, Rajeev Gandhi Nagar, Kota (Raj.) India 324005, Mob. 9358006181, 97831-97831 9
JEE MAIN 2021
 1 1 t
v 0     t  v0 
   
Distance = area of v–t graph
1 1 t t 2
  t  v0   t  
2 2    2    

10. (4)
Sol.  v2 = ev1 (v2)2 = (ev1)2⇒ 2gh2 = e22gh1 h2 = e2h1⇒e √(h2/h1) = 0.9
 v2 = ev1 gt2 = egt1 t2 = et1
vav = (Total distance)/(total time) = (h1 + 2h2 + 2h3 + …)/(t1 + 2t2 + 2t3 + …….)
= (h1 + 2e2h1 + 2e4h1 +…..)/(t1 + 2et2 + 2e2t3 +…) = (h1/t1) (1 + 2e2 + 2e4 +…..) (1 + 2e + 2e2 ±…)
= (h1/(2h1/g)) (2 + 2e2 + 2e4 + ….. –1)/(2 + 2e + 2e2 + ……–1) = 2.5 m/s
11. (2)
Sol. (2) v = v0 + gt + Ft2
ds
 v 0  gt  Ft 2
dt
1

 ds    v0  gt  Ft dt
2

 gt 2 Ft 3 
s   v0 t   
 2 3 
g F
s  v0  
2 3

12. (4)

Sol.

VBW  4 2 cos 45iˆ  4 2 sin 45jˆ


 4î  4ˆj
VW   ĵ
VB  VBW  VW  4î  3ĵ
S  V  t  (4iˆ  3j)
B B
ˆ  3  12iˆ  9ˆj

SB  (12) 2  (9) 2  15m

NUCLEUS-92, Rajeev Gandhi Nagar, Kota (Raj.) India 324005, Mob. 9358006181, 97831-97831 10
JEE MAIN 2021
13. (1)

Sol.

F = ma
qE = ma
qE
a=
m
1 2
Now d = at
2
2d
t
a
2d
t
 qE 
 
m
2  0.1 1
t 
 8 10 
6
2
 3  100
 10 
 Time period = 2t = 1 sec

14. (3)
Sol. L = Length of escalator
L
Vb/esc =
t1
When only escalator is moving.
L
Vesc 
t2
When both are moving
Vb/g = Vb/esc + Vesc
L L  L tt 
Vb/g =    t   12 
t1 t 2  Vb/g t1  t 2 

15. (3)
1
Sol. In 4 sec. 1st drop will travel    9.8    4 2  78.4m
2
nd
 2 drop would have travelled
 78.4 – 34.3 = 44.1 m.

NUCLEUS-92, Rajeev Gandhi Nagar, Kota (Raj.) India 324005, Mob. 9358006181, 97831-97831 11
JEE MAIN 2021
nd
Time for 2 drop
1
 9.8  t 2  44.1
2
t = 3 sec
time gap between drops = 1 sec
 1 drop per sec

16. (50)
Sol. r  10t 2 ˆi  5( t  5) ĵ

v  20 ˆi  5 ĵ

L  m(r  v)

 m 10t 2ˆi  5(t  5)ˆj  [20tiˆ  5ĵ]

L  m 50t 2 kˆ  100  t 2  5t  kˆ 

At t  0, L  0

50t 2  100  t 2  5t   0

t – 2 (t – 5) = 0
t = 10 sec

17. (1)
Sol. t = mx2 + nx
1 dt
  2mx  n
v dx
1
v
2mx  n
dv 2m  dx 
 2  
dt (2mx  n)  dt 
a = –(2m) v3

18. (3)
dv F ˆ / s2
Sol.  a   (8î  2j)m
dt m
dr
 v  (8tiˆ  2tj)m
ˆ /s
dt
2
ˆ t m
r  (8iˆ  2j)
2
At t = 10 sec
r  [(8î  2ˆj)50]m  r  (400iˆ  100ˆj)m

NUCLEUS-92, Rajeev Gandhi Nagar, Kota (Raj.) India 324005, Mob. 9358006181, 97831-97831 12
JEE MAIN 2021
19. (2)
Sol. V = t + t2
ds
 t   t 2
dt
S2 2

 ds    t  t 2  dt
S1 1
2
 t 2 t 3 
S2  S1    
 2 3 1
As particle is not changing direction So distance = displacement.
 [4  1] [8  1] 
Distance =  
 2 3 
3 7
 
2 3

20. (4)
Sol. P = const
mv 2dv
P  P  Fv 
dx
x v
p
0 m dx  0 v dv
2

Px v 3

m 3
1/3
 3Px  dx
  v
 m  dt
1/3 t x
 3P 
 dt   x
1/3
  dx
m 0 0
1/2
 8P  3/2
x   t
 9m 

21. (3)
Sol. t  0, u  0
F F dv
a  o  o 2 (t  T) 2 
M MT dt
v 2T
F F 2
0 dv  t0  Mo  MTo 2 (t  T)  dt
2T 2T
F  F  t3 
V   o t   o 2   t 2T  T 2 t 
 M  o MT  3 0
4Fo T
V
3M

NUCLEUS-92, Rajeev Gandhi Nagar, Kota (Raj.) India 324005, Mob. 9358006181, 97831-97831 13
JEE MAIN 2021
22. (30)

Sol.

Both velocity vectors are of same magnitude therefore resultant would pass exactly midway
through them
  30

23. (2)
u 2 sin 2
Sol. Range R = and same for  and 90 – 
g
So same for 42° and 48°
u 2 sin 2 2
Maximum height H =
2g
H is high for higher 
So H for 48° is higher than H for 42°
Option (2)

24. (4)

Sol.
Mirror used is convex mirror (rear – view mirror)
 VI/m  m 2 VO/m
Given,
VO/m  40m / s
f 10 10
m  
f  u 10  190 200
1
 VI/m    40  0.1m / s
400
 Car will appear to move with speed 0.1 m/s.
Hence option (4)

25. (4)

Sol.

NUCLEUS-92, Rajeev Gandhi Nagar, Kota (Raj.) India 324005, Mob. 9358006181, 97831-97831 14
JEE MAIN 2021
1 2
H gt
2
9.8  2 2
t
9.8
t  2 sec
t : time interval between drops
1
h g( 2  t) 2
2
1
0  g( 2  2t) 2
2
1
t 
2
2
1  1  1 1 9.8
h  g 2     9.8    2.45m
2  2 2 2 4
H  h  9.8  2.45 = 7.35 m

26. (12)
Sol. V  5000  24x
dV 1 12
 
dx 2 5000  24x 5000  24x
dV
now a V
dx
12
 5000  24x 
5000  24x

27. (1)
Sol. y = mx +C
20
v2  x  20
10
v2 = 2x + 20
dv
2v 2
dx
dv
a  v 1
dx

NUCLEUS-92, Rajeev Gandhi Nagar, Kota (Raj.) India 324005, Mob. 9358006181, 97831-97831 15
JEE MAIN 2021
KINEMATICS-2D
1. The trajectory of a projectile in a vertical plane is y = x – x2, where  and  are constants and
x & y are respectively the horizontal and vertical distances of the projectile from the point of
projection. The angle of projection  and the maximum height attained H are respectively given by
[JEE MAIN 2021 (FEB)]
2 2 4 2  a
2
(1) tan 1 , (2) tan 1 , (3) tan 1 , (4) tan 1   ,
4 2   

2. A person is swimming with a speed of 10m/s at an angle of 120° with the flow and reaches to a
point directly opposite on the other side of the river. The speed of the flow is 'x' m/s. The value of
'x' to the nearest integer is ________ . [JEE MAIN 2021 (MARCH)]

3. The projectile motion of a particle of mass 5g is shown in the figure.


[JEE MAIN 2021 (MARCH)]

The initial velocity of the particle is 5√2 m/sand the air resistance is assumed to be negligible. The
magnitude of the change in momentum between the point A and B is x × 10–2 kgms–1. The value
of x, to the nearest integer, is ________.

4. A swimmer can swim with velocity of 12km/h in still water. Water flowing in a river has velocity
6km/h. The direction with respect to the direction of flow of river water he should swim in order
to reach the point on the other bank just opposite to his starting point is _ _ _ _ _ _ _ _ _°. (Round
off to the Nearest Integer) (Find the angle in degree) [JEE MAIN 2021 (MARCH)]

5. A player kicks a football with an initial speed of 25 ms-1 at an angle of 45o from the ground. What
are the maximum height and the time taken by the football to reach at the highest point during
motion ?( Take = 10 ms-2) [JEE MAIN 2021 (AUGUST)]
(1) hmax = 10 m T = 2.5 s (2) hmax = 15.625 m T = 3.54 s
(3) hmax = 15.625 m T = 1.77 s (4) hmax = 3.54 m T = 0.125 s

NUCLEUS-92, Rajeev Gandhi Nagar, Kota (Raj.) India 324005, Mob. 9358006181, 97831-97831 16
JEE MAIN 2021
6. A helicopter is flying horizontally with a speed 'v' at an altitude 'h' has to drop a food packet for a
man on the ground. What is the distance of helicopter from the man when the food packet is
dropped? [JEE MAIN 2021 (AUGUST)]

2ghv 2  1 2v 2 h 2gh
(1) (2) 2ghv  h
2 2
(3)  h2 (4) 2
 h2
h2 g v

7. A bomb is dropped by fighter plane flyinghorizontally. To an observer sitting in the plane,the


trajectory of the bomb is a : [JEE MAIN 2021 (AUGUST)]
(1) hyperbola
(2) parabola in the direction of motion of plane
(3) straight line vertically down the plane
(4) parabola in a direction opposite to the motion of plane

ANSWER KEY

1. 1 2. 5 3. 5 4. 120 5. 3 6. 3 7. 3

NUCLEUS-92, Rajeev Gandhi Nagar, Kota (Raj.) India 324005, Mob. 9358006181, 97831-97831 17
JEE MAIN 2021
SOLUTION

1. (1)
Sol. y = x –x2
comparing with trajectory equation
1 gx 2
y  x tan  
2 u 2 cos 2 
tan  =  = tan –1
1 g
  
2 u cos 2 
2

g
 u2  
2 cos 2 
Maximum height : H
u 2 sin 2  g sin 2 
H 
2g 2 cos 2  2g
tan 2   2
H 
4 4

2. (5)

Sol.

10 sin 30° = x
x = 5 m/s

3. (5)

Sol.

| u || v | .......(1)
u  u cos 45iˆ  u sin 45ˆj .....(2)
v  v cos 45iˆ  v sin 45ˆj ......(3)
| ΔP || m(v  u) |
1
ΔP  2mu sin 45  2  5 10 – 3  5 2 
2
= 50 × 10–3 = 5 × 10–2

NUCLEUS-92, Rajeev Gandhi Nagar, Kota (Raj.) India 324005, Mob. 9358006181, 97831-97831 18
JEE MAIN 2021
4. (120)

Sol.

12sin   vr
1
sin  
2
  30
   120

5. (3)
U 2 sin 2 
Sol. H
2g
(25) .(sin 45)2
2

2 10
= 15.625 m
U sin 
T
g
25  sin 45o
 = 2.5 × 0.7 = 1.77 s
10

6. (3)

Sol.

2h
R .v
g
D  R2  h2
2
 2h 
  .v   h 2
 g 
2hv2
D  h2
g
option (3) is correct

NUCLEUS-92, Rajeev Gandhi Nagar, Kota (Raj.) India 324005, Mob. 9358006181, 97831-97831 19
JEE MAIN 2021
7. (3)

Sol.

vB  u 0 î  gtjˆ
vB/P  vB  vP
v B/P  8t̂
Straight line vertically down

NUCLEUS-92, Rajeev Gandhi Nagar, Kota (Raj.) India 324005, Mob. 9358006181, 97831-97831 20
JEE MAIN 2021
BASIC MATHEMATICS-1
1. Inan octagon ABCDEFGH of equal side, what is the sum of [JEE MAIN 2021 (FEB)]
AB  AC  AD  AE  AF  AG  AH '
if, AO  2iˆ  3jˆ  4kˆ

(1) 16iˆ  24ˆj  32kˆ (2) 16iˆ  24ˆj  32kˆ (3) 16iˆ  24ˆj  32kˆ (4) 16iˆ  24ˆj  32kˆ

2. The incident ray, reflected ray and the outward drawn normal are denoted by the unit vectors a, b
and c respectively. Then choose the correct relation for these vectors.[JEE MAIN 2021 (FEB)]
(1) b  a  2c (2) b  2a  c (3) b  a  2  a.c  c (4) b  a  c

3. Suppose you have taken a dilute solution of oleic acid in such a way that its concentration
becomes 0.01 cm3 of oleic acid per cm3 of the solution. Then you make a thin film of this solution
1

(monomolecular thickness) of area 4cm by considering 100 spherical drops of radius 


2 3 3
 10 cm.
–3

 40 
Then the thickness of oleic acid layer will be x × 10–14 m. [JEE MAIN 2021 (FEB)]

4. Match Lish-I With List II. [JEE MAIN 2021 (JULY)]

NUCLEUS-92, Rajeev Gandhi Nagar, Kota (Raj.) India 324005, Mob. 9358006181, 97831-97831 21
JEE MAIN 2021
Choose the correct from the options given below :
(1)  a   iv ,  b  (i),  c   iii  ,  d   ii 
(2)  a   iv ,  b  (iii),  c   i  ,  d   ii 
(3)  a   iii  ,  b   (ii),  c   iv  ,  d   iii 
(4)  a    i  ,  b   iv  ,  c   ii  ,  d   iii 

5. The velocity X and Y have equal magnitude. The magnitude of (X  Y) is n times the magnitude
of (X  Y) . The angle between X and Y is : [JEE MAIN 2021 (JULY)]
 n 2  1   n2 1   n2 1   n2 1 
(1) cos1  2  (2) cos1  2  (3) cos1  2  (4) cos 1  2 
 n 1   n  1   n  1   n 1 

6. Assertion A : If A, B, C, D are four points on a semi-circular arc with centre at 'O' such that
AB = BC = CD , then [JEE MAIN 2021 (JULY)]
AB  AC  AD  4AO  OB  OC
Reason R : Polygon law of vector addition yields
AB  BC  CD  AD  2AO

In the light of the above statements, choose the most appropriate answer from the options given
below :
(1) A is correct but R is not correct.
(2) A is not correct but R is correct.
(3) Both A and R are correct and R is the correct explanation of A.
(4) Both A and R are correct but R is not the correct explanation of A.

7. A ball is thrown up with a certain velocity so that it reaches a height 'h'. Find the ratio of the two
h
different times of the ball reaching in both the directions. [JEE MAIN 2021 (JULY)]
3
2 1 1 3 2 3 1
(1) (2) (3) (4)
2 1 3 3 2 3 1

8. If A and B are two vectors satisfying the relation ⃗ ⃗⃗ |⃗ ⃗⃗ | . Then the value of | ⃗ ⃗⃗ |
will be: [JEE MAIN 2021 (JULY)]
(1) A 2  B2 (2) A2  B2  2AB
(3) A 2  B2  2AB (4) A2  B2  2AB

NUCLEUS-92, Rajeev Gandhi Nagar, Kota (Raj.) India 324005, Mob. 9358006181, 97831-97831 22
JEE MAIN 2021
9. Two vectors P and Q have equal magnitude . If the magnitude of P  Q is n time the
magnitude of P  Q , then angle between P and Q is : [JEE MAIN 2021 (JULY)]
1  n  1  1  n  1 
2 2
 n 1 
1  n 1 
1
(1) sin   (2) cos   (3) sin  2  (4) cos  2 
 n 1   n 1   n 1   n 1 

10. What will be the projection of vector A  ˆi  ˆj  kˆ on vector B  ˆi  ˆj


[JEE MAIN 2021 (JULY)]
(1) 2  ˆi  ˆj  kˆ  (2) 2  ˆi  ˆj  kˆ  (3) 2  ˆi  ˆj (4)  ˆi  ˆj

11. The angle between vector  A  and  A  B  is : [JEE MAIN 2021 (AUGUST)]

 B 
    3B   Bcos  
(1) tan 1  2  (2) tan 1  A  (3) tan 1  (4) tan 1 
   
 3  0.7B   2A  B   A  Bsin  
 AB 
 2 

12. The magnitude of vectors OA, OB and OC in the given figure are equal. The direction of
OA  OB  OC with x- axis will be :- [JEE MAIN 2021 (AUGUST)]

(1  3  2) ( 3  1  2)
(1) tan 1 (2) tan 1
(1  3  2) (1  3  2)
( 3  1  2) (1  3  2)
(3) tan 1 (4) tan 1
(1  3  2) (1  3  2)

13. Two spherical balls having equal masses with radius of 5 cm each are thrown upwards along the
same vertical direction at an interval of 3s with the same initial velocity of 35 m/s, then these
balls collide at a height of ....... m. (Take g = m/s2) [JEE MAIN 2021 (AUGUST)]

NUCLEUS-92, Rajeev Gandhi Nagar, Kota (Raj.) India 324005, Mob. 9358006181, 97831-97831 23
JEE MAIN 2021
14. The resultant of these forces OP, OQ, OR, OS and OT is approximately .......N.
[Take 3  1.7, 2  1.4 Given î and ĵ unit vectors along x, y axis]
[JEE MAIN 2021 (AUGUST)]

(1) 9.25iˆ  5ˆj (2) 3iˆ  15ˆj (3) 2.5iˆ  14.5jˆ (4) 1.5iˆ  15.5jˆ

15. Statement I : [JEE MAIN 2021 (AUGUST)]


   
Two forces P  Q and P  Q where P  Q , when act at an angle 1 to each other, the

magnitude of their resultant is 3(P 2  Q2 ), when they act an angle 2 , the magnitude of their
resultant becomes 2(P2  Q2 ) . This is possible only when 1  2 .
Statement II :
In the situation given above.
1  60 and 2  90
In the light of the above statements, choose the most appropriate answer from the option given
below :-
(1) Statement –I is false but Statement –II is true
(2) Both Statement –I and Statement – II are true
(3) Statement – I is true but statement – II is false
(4) Both statement – I and Statement – II are false

16. Statement :I [JEE MAIN 2021 (AUGUST)]


If three force F1 , F2 and F3 are represented by three sides of a triangle and F  F2  F3 , then these
three forces are concurrent forces and satisfy the condition for equilibrium .
Statement : II
A triangle made up of three ⃗ ⃗ and ⃗ as its sides taken in the same order, satisfy the condition
for translatory equilibrium.
In the light of the above statements, choose the most appropriate answer from the option given
below :
(1) Statement – I is false but Statement – II is true
(2) Statement – I is true but Statement – II is false
(3) Both Statement – I and Statement – II are false
(4) Both Statement – I and Statement – II are true
NUCLEUS-92, Rajeev Gandhi Nagar, Kota (Raj.) India 324005, Mob. 9358006181, 97831-97831 24
JEE MAIN 2021
ANSWER KEY

1. 2 2. 3 3. 25 4. 2 5. 2 6. 4 7. 3
8. 4 9. 4 10. 4 11. 3 12. 1 13. 50 14. 1
15. 2 16. 4

SOLUTION

1. (2)
Sol. We know,
 AB  OB  OC  OD  OE  OF  OG  OH  0
By triangle law of vector addition, we can write
AB  AO  OB ; AC  AO  OC
AD  AO  OD ; AE  AO  OE
AF  AO  OF ; AG  AO  OG
AH  AO  OH
Now
AB  AC  AD  AE  AF  AG  AH
 (7 AO  OB  OC  OD  OE  OF  OG  OH
 (7 AO  0  OA
 (7 AO  AO
 8 AO  AO


 8AO  8 2iˆ  3jˆ  4kˆ 
= 16iˆ  24ˆj  32kˆ

2. (3)
Sol. a  sin ˆj  cos ˆj
b  sin ˆi  cos ˆj

a  2  a.c  c  sin ˆi  cos ˆj

NUCLEUS-92, Rajeev Gandhi Nagar, Kota (Raj.) India 324005, Mob. 9358006181, 97831-97831 25
JEE MAIN 2021
3. (25)
4 4 3
Sol. 4t T  100  r 3  100    10 –9  10 –8 cm 3
3 3 40
t T  25 10 –10 cm
= 25 × 10–12 m
t0 = 0.01 tT = 25 × 10–14 m
= 25

4. (2)
Sol. (a) C  A  B
Option (iv)
(b) A  B  C  C  B
Option (iii)
(c) B  A  C
Option (i)
(d) A  B  C  0
Option (ii)

5. (2)
Sol. Given X = Y
X2  Y2  2  Ycos 
 n X2  Y2  2  Ycos 
Square both sides
2X2 (1 – cos ) n2 .2X2 (1 + cos )
1 – cos = n2 + n2cos
 n 2
 cos  =
n  n2
 2 
 = cos–1  n 2 1 
 n  1

6. (4)
Sol. Polygon law is applicable in both but the equation given in the reason is not useful in explaining
the assertion.

7. (3)
Sol. u = 2gh
Now,
h
S a = –g
3
1
S  ut  at 2
2
h 1
 2ght  (g)t 2
3 2

NUCLEUS-92, Rajeev Gandhi Nagar, Kota (Raj.) India 324005, Mob. 9358006181, 97831-97831 26
JEE MAIN 2021
g h
t 2    2ght   0
2 3
From quadratic equation
h
4g
2 4gh
2gh  2gh  2gh 
t1 , t 2  3 ;
t1
 3  3 2
g t2 4gh 3 2
2gh 
3

8. (4)
Sol. A.B | A  B |
ABcos = ABsin = 45°
|AB| = A2  B2  2ABcos 45
A2  B2  2AB

9. (4)
Sol. | P || Q | x …..(i)
| P  Q | n | P  Q |
P2  Q2  2PQcos   n 2  P2  Q2  2PQcos  
Using (i) in above equation
n2 1
cos  =
1 n2
 2 
 = cos 1  n 2  1 
 n 1

10. (4)

Sol. (A cos )B ˆ  A  A  B  B


ˆ  AB B
ˆ
 AB  B
ˆ ˆ
2  i  j ˆ ˆ
  i j
2  2 

11. (3)

Sol.

NUCLEUS-92, Rajeev Gandhi Nagar, Kota (Raj.) India 324005, Mob. 9358006181, 97831-97831 27
JEE MAIN 2021
Angle between A and B ,  = 60°
Angle between A and A  B
Bsin 
tan  =
A  B cos 
3
B
 2
1
A  B 2
2
3B
tan  
2A  B

12. (1)

Sol.

Let magnitude be equal to  .


 3 ˆ 1 ˆ
OA   cos 300 ˆi  sin 30jˆ     i  j
 
 2 2 

1 3 ˆ
OB   cos 600 ˆi  sin 60ˆj    ˆi  j
 
2 2 

 1 ˆ 1 ˆ
OC   cos 450 (ˆi)  sin 45jˆ      i j
   2 2 
 OA  OB  OC
 3  1 1   1 3 1  ˆ
     ˆi      j
 2 2   2 2 2  
Angles with x-axis
1 3 1 
   
1 2
tan 
2 2   tan 1  2  6  2 
 
 3 1 1   6  2  2
 2 2 
 2
1  3  2 
 tan 1  
 3 1 2 
Hence option (1)

NUCLEUS-92, Rajeev Gandhi Nagar, Kota (Raj.) India 324005, Mob. 9358006181, 97831-97831 28
JEE MAIN 2021
13. (50)

Sol.

When both balls will collied


y1 = y2
1 1
35t  10  t 2  35(t  3)  10  (t  3) 2 ]
2 2
1 1
35t   10  t 2  35t  105  10  t 2
2 2
1 1
  10  32   10  6t
2 2
0 = 150 – 30 t
t = 5 sec
Height at which both balls will collied
1
h  35t  10  t 2
2
1
 35  5   10  52
2
h = 50 m

14. (1)

Sol.

 3 1  1   1   3 ˆ
Fx  10 

 20    20    15    15    i  9.25 ˆi
 2  2   2   2   2 
 1  3 1  1   1 ˆ
Fy  15    20  
  10    15    20    j  5 ˆj
 2  2  
  2  2  2

NUCLEUS-92, Rajeev Gandhi Nagar, Kota (Raj.) India 324005, Mob. 9358006181, 97831-97831 29
JEE MAIN 2021
15. (2)
Sol. A  PQ
B  PQ PQ
A  B  P2  Q2

A  B  2(P 2  Q 2 )(1  cos )

For A  B  3(P 2  Q 2 )
1 = 60°
For A  B  2(P 2  Q 2 )
2 = 90°

16. (4)

Sol.

Here F1  F2  F3  0
F1  F2  F3
Since Fnet  0 (equilibrium)
Both statement correct

NUCLEUS-92, Rajeev Gandhi Nagar, Kota (Raj.) India 324005, Mob. 9358006181, 97831-97831 30
JEE MAIN 2021
UNIT & DIMENSION
x2

1. The work done by a gas molecule in an isolated system is given by, W =  e 2 kT
where x is the
displacement, k is the Boltzmann constant and T is the temperature,  and  are constants. Then
the dimension of  will be : [JEE MAIN 2021 (FEB)]
(1) [M L2 T–2] (2) [M L T–2] (3) [M2 L T2] (4) [M0 L T0]

2. If e is the electronic charge, c is the speed of light in free space and h is Planck's constant, the
1  e |2
quantity has dimensions of : [JEE MAIN 2021 (FEB)]
4 0 hc
(1) [M0 L0 T0] (2) [L C–1] (3) [M L T–1] (4) [M L T0]

3. If 'C' and 'V' represent capacity and voltage respectively then what are the dimensions of , where
C
 [JEE MAIN 2021 (FEB)]
V
(1) [M–2L–3I2T6] (2) [M–3L–4 I3T7] (3) [M–1L–3I–2T–7] (4) [M–2L–4I3T7]

4. The entropy of any system is given by [JEE MAIN 2021 (JULY)]


 kR 
S   2 ln  2  3
 J 
Where  and  are the constant.  , J, k and R no. of moles, mechanical equivalent of heat,
Boltzmann constant and gas constant respectively.
 dQ 
TakeS  T 
Choose the incorrect option from the following :
(1)  and J have the same dimensions .
(2) S,  ,k and R have the same dimensions.
(3) S and  have different dimensions.
(4)  and K have the same dimensions.

5. If time (t), velocity (v), and angular momentum (l) are taken as the fundamental units . Then the
dimension of mass (m) in terms of t,v and l is : [JEE MAIN 2021 (JULY)]
(1) [t-1v 1 l-2] (2) [t-1v2 l-1] (3) [t--2v-1 l1] (4) [t--1v-2 l1]

6. The force is given in terms of time t and displacement x by the equation


F  A cos Bx  C sin Dt [JEE MAIN 2021 (JULY)]
AD
The dimensional formula of is ;
B
(1) [M0 L T-1] (2) [M L2 T-3] (3)[M1 L1 T-2] (4) [M2 L2 T-3]

NUCLEUS-92, Rajeev Gandhi Nagar, Kota (Raj.) India 324005, Mob. 9358006181, 97831-97831 31
JEE MAIN 2021
7. Match List-II with List-II. [JEE MAIN 2021 (JULY)]
LIST I LIST II
1 3 1
(a) Capacitance, C (i) M1LT A
1 3 4 2
(b) Permittivity of free space , 0 (ii) M L T A
(c) Permeability of free space , 0 (iii) M1L2T4A2
1 2 2
(d) Electric field , E (iv) M1LT A
Choose the correct answer from the option given below
(1) (a)  (iii),(b)  (ii),(c)  (iv),(d)  (i)
(2) (a)  (iii),(b)  (iv),(c)  (ii),(d)  (i)
(3) (a)  (iv),(b)  (ii),(c)  (iii),(d)  (i)
(4) (a)  (iv),(b)  (iii),(c)  (ii),(d)  (i)

3
8. A physical quantity 'y' is represented by the formula y  m 2 r 4 g x [JEE MAIN 2021 (JULY)]
2
If the percentage errors found in y, m, r, l and g are 18, 1, 0.5, 4 and P respectively, then find the
value of X and P.
(1) (2) (3) (4)

MgL3
9. A student determined Young's Modulus of elasticity using the formula Y = . The value of g
4bd3
is taken to be 9.8 m/s2, without any significant error, his observation are as following.
[JEE MAIN 2021 (AUGUST)]

Least count of the


Physical Quantity Equipment used for Observed value
measurement
Mass (M) 1g 2 kg
Length of bar (L) 1 mm 1m
Breadth of bar (b) 0.1 mm 4 cm
Thickness of bar (d) 0.01 mm 0.4 cm
Depression () 0.01 mm 5 mm
Then the fractional error in the measurement of Y is :
(1) 0.0083 (2) 0.0155
(3) 0.155 (4) 0.083

10. Two resistors R1 = (4 ± 0.8)  and R2 = (4 ± 0.4) are connected in parallel. The equivalent
resistance of their parallel combination will be : [JEE MAIN 2021 (AUGUST)]
(1) (4 ± 0.4)  (2) (2 ± 0.4) 
 (3) (2 ± 0.3)  (4) (4 ± 0.3) 

NUCLEUS-92, Rajeev Gandhi Nagar, Kota (Raj.) India 324005, Mob. 9358006181, 97831-97831 32
JEE MAIN 2021
11. Match List–I with List–II. [JEE MAIN 2021 (AUGUST)]
List-I List-II
2 –2 –1
(a) Magnetic Induction (i) ML T A
(b) Magnetic Flux (ii) M0L–1A
(c) Magnetic Permeability (iii) MT–2A–1
(d) Magnetization (iv) MLT–2 A–2
Choose the most appropriate answer from the options given below :
(1) (a)-(ii), (b)-(iv), (c)-(i), (d)-(iii) (2) (a)-(ii), (b)-(i), (c)-(iv), (d)-(iii)
(3) (a)-(iii), (b)-(ii), (c)-(iv), (d)-(i) (4) (a)-(iii), (b)-(i), (c)-(iv), (d)-(ii)

12. If E, L, M and G donote the quantities as energy, angular momentum, mass and constant of
gravitation respectively, then the dimensions of P in the formula P = EL2M-5 G-2 are :-
[JEE MAIN 2021 (AUGUST)]
0 1 0 -1 -1 2 1 1 -2
(1) [M L T ] (2) [M L T ] (3) [M L T ] (4) [M0 L0 T0]

13. Match LIST – I with LIST – II [JEE MAIN 2021 (AUGUST)]


LIST- I LIST –II
(a) RH(Rydberg constant) (i) kg m-1 s-1
(b) h(Planck's constant) (ii) kg m2 s-1
(c) B (Magnetic field energy density) (iii) m-1
(d)  (coefficient of viscosity) (iv) kg m-1 s-2
Choose the most appropriate answer from the option given below :
(1) (a) – (ii), (b) – (iii), (c) – (iv), (d) – (i) (2) (a) – (iii), (b) – (ii), (c) – (iv), (d) – (i)
(3) (a) – (iv), (b) – (ii), (c) – (i), (d) – (iii) (4) (a) – (iii), (b) – (ii), (c) – (i), (d) – (iv)

14. If force (F), length (L) and time (T) are taken as the fundamental quantities. Then what will be the
dimension of density : [JEE MAIN 2021 (AUGUST)]
-4 2 -3 2 -5 2
(1) [FT T ] (2) [FL T ] (3) [FL T ] (4) [FL-3 T3]

15. Which of the following is not a dimensionless quantity ? [JEE MAIN 2021 (AUGUST)]
(1) Relative magnetic permeability (r) (2) Power factor
(3) Permeability of free space (0) (4) Quality factor

16. If E and H represents the intensity of electric field and magnetizing field respectively, then the unit
of E/H will be : [JEE MAIN 2021 (AUGUST)]
(1) ohm (2) mho (3) joule (4) Newton

NUCLEUS-92, Rajeev Gandhi Nagar, Kota (Raj.) India 324005, Mob. 9358006181, 97831-97831 33
JEE MAIN 2021
17. If velocity [V], time [T] and force [F] are chosen as the base quantities, the dimensions of the mass
will be : [JEE MAIN 2021 (AUGUST)]
(1) [FT–1V–1 ] (2)[FTV–1] (3) [FT2V] (4) [FVT–1]

18. Match List – I with List – II [JEE MAIN 2021 (AUGUST)]


List – I List – II
(a) Torque (i) MLT-1
(b) Impulse (ii) MT-2
(c) Tension (iii) ML2 T-2
(d) Surface Tension (iv) MLT-2
Choose the most appropriate answer from the option given below :
(1) (a) – (iii), (b) – (i), (c) – (iv), (d) – (ii) (2) (a) – (ii), (b) – (i), (c) – (iv), (d) – (iii)
(3) (a) – (i), (b) – (iii), (c) – (iv), (d) – (ii) (4) (a) – (iii), (b) – (iv), (c) – (i), (d) – (ii)

19. Which of the following equations is dimensionally incorrect? [JEE MAIN 2021 (AUGUST)]
Where t = time, h = height, s = surface tension,   angle,   density a, r = radius, g =
acceleration due to gravity, v = volume, p = pressure, W = work done,   torque ,
 permittivity E = electric field, j = current density, L = length.

pa 4 2s cos  E
(1) v  (2) h  (3) j  (4) W  
8L rg t

ANSWER KEY

1. 2 2. 1 3. 4 4. 4 5. 4 6. 2 7. 1
8. 3 9. 2 10. 3 11. 4 12. 4 13. 2 14. 1
15. 3 16. 1 17. 2 18. 1 19. 1

NUCLEUS-92, Rajeev Gandhi Nagar, Kota (Raj.) India 324005, Mob. 9358006181, 97831-97831 34
JEE MAIN 2021
SOLUTION

1. (2)
x2
Sol.  dimensionless
kT
 2
 [] = x 
L2
= M–1T2
2 2
kT ML T
Now [W] = [] []2
ML2 T 2
[] = = M1L1T–2
M 1T 2

2. (1)
1 e2
Sol. F
4 0 r 2
hc
E

 e 2
1  Fr 2
     M0 L0T0 
 40 hc  E

3. (4)
C Q/V Q
Sol.    2
V V V
work
V
Q
 It 
3
Q3
 
 work   F.s 
2 2

 I3T 3 
=   M 2 L4 I3T 7 
 ML T 
2 2

4. (4)
 KR 
Sol. S   2 n  2  3 
 J 
Q
S   joulek /k
T
2
[ ] = Joule/k
 KR 
PV = nRT  J 2   1
 
Joule
R=
K

NUCLEUS-92, Rajeev Gandhi Nagar, Kota (Raj.) India 324005, Mob. 9358006181, 97831-97831 35
JEE MAIN 2021
Joule Joule
R ,K 
K R
 Joule 
 
 K 
 Joule 
 2   
 K 
 = dimensionless

5. (4)
Sol. m  t a vb c

b c
m  [T]a  LT 1   ML2T 1 
M1L0T0  McLb2cTa bc
Comparing powers
c = 1, b = –2, a = –1
m  t 1v2 1

6. (2)
Sol. [A]  MLT2 
[B] = [L–1]
[D] = [T–1]
 AD   MLT  T 
2 1

 B  
 L1 
 AD 
 B    ML T 
2 3

7. (1)
Sol. q = CV
 q  (A  T)
2
[C]     2 2
 M1L2T4A2
V
  ML T
 F  MLT 2
[E]     = MLT-3 A-1
q  AT
q1q 2
F
4o r 2
[O ]  M 1L3T 4 A 2
1
Speed of light c 
 0 0
1
0 
0 c2
1
[0 ]  1 3 2 4 1 2
 [M1L1T 2 A 2 ]
[M L T A ][LT ]
NUCLEUS-92, Rajeev Gandhi Nagar, Kota (Raj.) India 324005, Mob. 9358006181, 97831-97831 36
JEE MAIN 2021
8. (3)
y 2m 4r xg 3 
Sol.    
y m r g 2
3
18  2(1)  4(0.5)  xp  (4)
2
8 = xp
By checking from option.
16 3
x  ,p  
3 2

9. (2)
MgL3
Sol. y=
4bd3
y M 3L b 3d 
    
y M L b d 
y 103 3 103 102 3 102 102
    
y 2 1 4 4 5
–3
= 10 [0.5 + 3 + 2.5 + 7.5 + 2] = 0.0155
Option (2)

10. (3)
1 1 1
Sol.  
R eq R1 R 2
1 1 1
   Req = 2
R eq 4 4
 R eq R1 R 2
 Also 2
  2
R eq R12 R2
 R eq .8 .4 1.2
  
4 16 16 16
 Re q  0.3
Req = (2  0.3)
Option (3)
11. (4)
Sol. (a) Magnetic Induction = MT–2A–1 (b) Magnetic Flux = ML2T–2A–1
(c) Magnetic Permeability = MLT–2A–2 (d) Magnetization = M0L–1A
12. (4)
Sol. E = ML2T-2
L = ML2T-1
m=M
G = M-1L+3T-2

NUCLEUS-92, Rajeev Gandhi Nagar, Kota (Raj.) India 324005, Mob. 9358006181, 97831-97831 37
JEE MAIN 2021
2
EL
P
M5 G 2
(ML2 T 2 )(M 2 L4T 2 )
[P]  5 2 6 4
 M 0 L0T 0
M (M L T )
Option (4)

13. (2)
Sol. SI unit of Rydberg const. = m-1
SI unit of Plank's const. = kg m2 s-1
SI unit of Magnetic field energy density = kg m-1 s-2
SI unit of coeff. of viscosity = kg m-1 s-1

14. (1)
Sol. Density = [FaLbTc]
[ML-3] = [MaLaT-2aLbTc]
[M1L-3] = [MaLa+bT-2a+c]
a=1; a+b=-3; - 2a + c = 0
1+b=-3 c = 2a
b=-4 c=2
So, density = [F1L-4T2]

15. (3)

Sol. [r ]  1 as  r 
m
[Power factor (cos ) ]= 1
B0
0  (unit  NA 2 ) : Not Dimensionless
H
[0 ]  MLT2 A2 
Energy sotred
quality factor (Q) 
Energy dissipated per cycle
So Q is unitless & dimensionless.

16. (1)
E volt / netre volt
Sol. Unit of is   ohm
H Ampere / metre Ampere

17. (2)
Sol. [M] = K[F]a [T]b [V]c
[M1] = [M1L1T–2]a [T1]b [L1T–1]c
a = 1, b = 1 , c = – 1
[M] = [FTV–1]

NUCLEUS-92, Rajeev Gandhi Nagar, Kota (Raj.) India 324005, Mob. 9358006181, 97831-97831 38
JEE MAIN 2021
18. (1)
Sol. torque   ML2T 2 (III)
Impulse I  MLT 1 (I)
tension force  MLT 2 (IV)
Surface tension  MT 2 (II)
option (1)

19. (1)
pa 4 dv
Sol. (i)  = Volumetric flow rate (poiseuille's law)
8L dt
2s
(ii) hg  cos 
r
1 a 1 q 1
(iii) RHS     
40 r 2  t r 2
I
 2  IL2
L
LHS
I
T   IL2
A
(iv) W  
Option (1)

NUCLEUS-92, Rajeev Gandhi Nagar, Kota (Raj.) India 324005, Mob. 9358006181, 97831-97831 39
JEE MAIN 2021
NEWTON'S LAWS OF MOTION
1. Two solids A and B of mass 1 kg and 2 kg respectively are moving with equal linear momentum.
The ratio of their kinetic energies [JEE MAIN 2021 (FEB)]
A
(K.E.)A : (K.E.)B will be , so the value of A will be ___.
1

2.  
A boy pushes a box of mass 2 kg with a force F  20iˆ  10jˆ N on a frictionless surface. If the box
was initially at rest, then _________ m is displacement along the x – axis after 10 s.
[JEE MAIN 2021 (FEB)]

3. As shown in the figure, a block of mass 3 kg is kept on a horizontal rough surface of coefficient
1
of friction . The critical force to be applied on the vertical surface as shown at an angle 60°
3 3
with horizontal such that it does not move, will be 3x. The value of x will be

[ ] [JEE MAIN 2021 (FEB)]

4. The coefficient of static friction between a wooden block of mass 0.5 kg and a vertical rough wall
is 0.2. The magnitude of horizontal force that should be applied on the block to keep it adhered to
the wall will be ______N. [g = 10 ms–2] [JEE MAIN 2021 (FEB)]

x2
5. An inclined plane is bent in such a way that the vertical cross-section is given by y = where y
4
is in vertical and x in horizontal direction. If the upper surface of this curved plane is rough with
coefficient of friction μ = 0.5, the maximum height in cm at which a stationary block will not slip
downward is ________ cm. [JEE MAIN 2021 (FEB)]

6. An inclined plane making an angle of 30° with the horizontal is placed in a uniform horizontal
N
electric field 200 as shown in the figure. A body of mass 1kg and charge 5 mC is allowed to
C
slide down from rest at a height of 1m. If the coefficient of friction is 0.2, find the time taken by
1 3
the body to reach the bottom.[g = 9.8 m/s2, sin 30° = ; cos 30° = ].
2 2

NUCLEUS-92, Rajeev Gandhi Nagar, Kota (Raj.) India 324005, Mob. 9358006181, 97831-97831 40
JEE MAIN 2021
[JEE MAIN 2021 (FEB)]

(1) 0.92 s (2) 0.46 s (3) 2.3 s (4) 1.3 s

7. A block of mass m slides along a floor whilea force of magnitude F is applied to it at an angle  as
shown in figure. The coefficient of kinetic friction is μK. Then, the block's acceleration 'a' is given
by : (g is acceleration due to gravity): [JEE MAIN 2021 (MARCH)]

(1) 
F  F  F  F 
cos    k  g  sin   (2) cos   k  g  sin  
m  m  m  m 
F  F  F  F 
(3) cos   k  g  sin   (4) cos    k  g  sin  
m  m  m  m 

8. Consider a frame that is made up of two thin massless rods AB and AC as shown in the figure. A
vertical force of magnitude 100N is applied at point A of the frame.
[JEE MAIN 2021 (MARCH)]

Suppose the force is p resolved parallel to the arms AB and AC of the frame. The magnitude of
the resolved component along the arm AC is xN. The value of x, to the nearest integer, is ______.
[Given : sin(35°) = 0.573, cos(35°) = 0.819, sin(110°) = 0.939, cos(110°) = –0.342]

9.  
A body of mass 2kg moves under a force of 2iˆ  3jˆ  5kˆ N. It starts from rest and was at the
origin initially. After 4s, its new coordinates are (8, b, 20). The value of b is _______. (Round off
the Nearest Integer) [JEE MAIN 2021 (MARCH)]

NUCLEUS-92, Rajeev Gandhi Nagar, Kota (Raj.) India 324005, Mob. 9358006181, 97831-97831 41
JEE MAIN 2021
10. A bullet of mass 0.1 kg is fired on a wooden block to pierce through it, but it stops after moving a
distance of 50 cm into it. If the velocity of bullet before hitting the wood is 10 m/s and it slows
down with uniform deceleration, then the magnitude of effective retarding force on the bullet is
''x'N. The value of ''x' to the nearest integer is ________. [JEE MAIN 2021 (MARCH)]

11. A ball of mass 4kg, moving with a velocity of 10ms–1, collides with a spring of length 8m and
force constant 100Nm–1. The length of the compressed spring is x m. The value of x, to the nearest
integer, is _________ . [JEE MAIN 2021 (MARCH)]

12. Two blocks (m = 0.5 kg and M = 4.5 kg) are arranged on a horizontal frictionless table as shown

in figure. The coefficient of static friction between the two blocks is . Then the maximum
7
horizontal force that can be applied on the larger block so that blocks move together is _____ N.
(Round off to the Nearest Integer) [Take g as 9.8 ms–2] [JEE MAIN 2021 (MARCH)]

13. A body of mass 1 kg rests on a horizontal floor with which it has a coefficient of static friction
1
. It is desired to make the body move by applying the minimum possible force F.N. The value
3
of F will be __________. (Round off to the Nearest Integer) [JEE MAIN 2021 (MARCH)]
–2
[Take g = 10 ms ]

14. A boy of mass 4kg is standing on a piece of wood having mass 5kg. If the coefficient of friction
between the wood and the floor is 0.5, the maximum force that the boy can exert on the rope so
that the piece of wood does not move from its place is _________ N. (Round off to the Nearest
Integer) [JEE MAIN 2021 (MARCH)]
–2
[Take g = 10 ms ]

15. The normal reaction 'N' for a vertical of 800 kg mass, negotiating a turn on a 30 0 banked road at
maximum possible speed without skidding is _____________× 103 kg m/s2
[JEE MAIN 2021 (JULY)]
(1) 10.2 (2) 7.2 (3) 12.4 (4) 6.96

NUCLEUS-92, Rajeev Gandhi Nagar, Kota (Raj.) India 324005, Mob. 9358006181, 97831-97831 42
JEE MAIN 2021
16. A steel block of 10 kg rests on a horizontal floor as shown. When three iron cylinders are placed
on it as shown, the block and cylinders go down with an acceleration 0.2 m/s2. The normal
reaction ‘R' by the floor if mass of the iron cylinders are equal and of 20 kg each , is _______ N.
[Take g = 10 m/s2 and s = 0.2 ] [JEE MAIN 2021 (JULY)]

(1) 716 (2) 686 (3) 714 (4) 684

17. A body of mass 'm' is launched up on a rough inclined plane making an angle of 30° with the
x
horizontal. The coefficient of friction between the body and plane is if the time of ascent is
5
half of the time of descent. The value of x is ____. [JEE MAIN 2021 (JULY)]

18. A bullet of '4g' mass is fired from a gun of mass 4kg. If the bullet moves with the muzzle speed of
50 ms–1, the impulse imparted to the gun and velocity of recoil of gun are :
[JEE MAIN 2021 (JULY)]
(1) 0.4 kg ms–1, 0.1 ms–1 (2) 0.2 kg ms–1, 0.05 ms–1
(3) 0.2 kg ms–1, 0.1 ms–1 (4) 0.4 kg ms–1, 0.05 ms–1

19. A block of mass m slides on the wooden wedge, which in turn slides backward on the horizontal
surface. The acceleration of the block with respect to the wedge is : Given m = 8 kg, M = 16
kgAssume all the surfaces shown in the figure to be friction less. [JEE MAIN 2021 (AUGUST)]

4 6 3 2
(1) g (2) g (3) g (4) g
3 5 5 3

20. An object of mass 'm' is being moved with aconstant velocity under the action of an appliedforce
of 2N along a frictionless surface withfollowing surface profile.
[JEE MAIN 2021 (AUGUST)]

The correct applied force vs distance graph will be:

NUCLEUS-92, Rajeev Gandhi Nagar, Kota (Raj.) India 324005, Mob. 9358006181, 97831-97831 43
JEE MAIN 2021

(1) (2)

(3) (4)

21. When a body slides down from rest along a smooth inclined plane making an angle of 30° with the
horizontal, it takes time T. When the same body slides down from the rest along a rough inclined
plane making the same angle and through the same distance, it takes time T, where  is a
constant greater than 1. The co-efficient of friction between the body and the rough plane is
1  2  1 
  where x = ……… [JEE MAIN 2021 (AUGUST)]
x  2 
22. Two blocks of masses 3 kg and 5 kg are connected by a metal wire going over a smooth pulley.
24 –2
The breaking stress of the metal is  102 Nm . What is the minimum radius of the wire?

(Take g = 10–2) [JEE MAIN 2021 (AUGUST)]

(1) 125 cm (2) 1250 cm (3) 12.5 cm (4) 1.25 cm


23. The coefficient of static friction between two blocks is 0.5 and the table is smooth. The maximum
horizontal force that can be applied to move the blocks together is .......N.
(take g = 10 ms–2) [JEE MAIN 2021 (AUGUST)]

NUCLEUS-92, Rajeev Gandhi Nagar, Kota (Raj.) India 324005, Mob. 9358006181, 97831-97831 44
JEE MAIN 2021
24. Inside a Uniform spherical shell: [JEE MAIN 2021 (AUGUST)]
(a) the gravitational field is zero
(b) the gravitational potential is zero
(c) the gravitational field is same everywhere
(d) the gravitation potential is same everywhere
(e) all of the above
Choose the most appropriate answer from the option given below :
(1) (a), (c) and (d) only
(2) (e) only
(3) (a) , (b) and (c) only
(4) (b) (c) and (d) only

25. The initial mass of a rocket is 1000 kg. Calculate at what rate the fuel should be burnt so that the
rocket is given an acceleration of 20 ms-2 . The gases come out at a relative speed of 500 ms-1 with
respect to the rocket :[Use g = 10 m/s2] [JEE MAIN 2021 (AUGUST)]
2 -1 -1 -1
(1) 6.0 × 10 kg s (2) 500 kg s (3) 10 kg s (4) 60 kg s-1

26. The boxes of masses 2 kg and 8 kg are connected by a massless string passing over smooth
pulleys. Calculate the time taken by box of mass 8 kg to strike the ground starting from rest.(use g
= 10 m/s2) [JEE MAIN 2021 (AUGUST)]

(1) 0.34 s (2) 0.2 s (3) 0.25 s (4) 0.4 s

27. A car is moving on a plane inclined at 30° to the horizontal with an acceleration of 10 ms-2 parallel
to the plane upward. A bob is suspended by a string from the roof of the car. The angle in degrees
which the string makes with the vertical is ________.(Take g = 10 ms-2)
[JEE MAIN 2021 (AUGUST)]

ANSWER KEY
1. 2 2. 500 3. 3.33 4. 25 5. 25 6. 4 7. 2
8. 82 9. 12 10. 10 11. 6 12. 21 13. 5 14. 30
15. 1 16. 2 17. 3 18. 2 19. 4 20. 2 21. 3
22. 3 23. 15 24. 1 25. 4 26. 4 27. 30

NUCLEUS-92, Rajeev Gandhi Nagar, Kota (Raj.) India 324005, Mob. 9358006181, 97831-97831 45
JEE MAIN 2021
SOLUTION
1. (2)
p2
Sol. Kinetic energy K  ,  PA  PB 
2m
1
K
m
K A mB 2
 =
K B mA 1

2. (500)
Sol. F  20iˆ  10ˆj
F 20iˆ  10jˆ
a   10iˆ  5jˆ
m 2
s  
1 2 1
2 2
 
at  10iˆ  5ˆj  10   50 10  500m
2

 Ans. 500
3. (3.33)

Sol.

F 1
Fcos 60  N or  N ……………(1)
2 3 3
& N = sin 60° + 3g ……………(2)
From equation (1) & (2)
F 1 F 3 
   3g   F = g = 10 Newton = 3x
2 3 3 2 
10
So x   3.33
3
4. (25)
Sol. F.B.D of the block is shown in the diagram

Since block is at rest therefore


NUCLEUS-92, Rajeev Gandhi Nagar, Kota (Raj.) India 324005, Mob. 9358006181, 97831-97831 46
JEE MAIN 2021
fr – mg = 0 ....(1)
F–N=0 ....(2)
fr  μN
In limiting case
fr = μN = μF ....(3)
Using eq. (1) and (3)
 μF = mg
0.5  10
F= = 25 N
0.2

5. (25)

Sol.

At maximum ht. block will experience maximum friction force. Therefore if at this height slope of
the tangent is tan q, then  = Angle of repose.
dy 2x x
 tan  =    0.5
dx 4 2
x2
 x = 1 and therefore y = = 0.25 m
4
= 25 cm
Answer is 25 cm
(Assuming that x & y in the equation are given in meter)

6. (4)
Sol. FBD

here N = 9.8 cos 30 + 1 sin 30  9N


9.8sin 30  1cos 30  N
so a =
1
a = 2.233 m/s2
1 2
By S  ut  at
2

NUCLEUS-92, Rajeev Gandhi Nagar, Kota (Raj.) India 324005, Mob. 9358006181, 97831-97831 47
JEE MAIN 2021
1
  2.233 t 2
2
( )
t  1.3sec

7. (2)

Sol.

N = mg – f sin 
F cos  – μkN = ma
F cos  – μk (mg – F sin ) = ma
F  F 
a  cos    k  g  sin  
m  m 

8. (82)

Sol.

Component along AC
= 100 cos 35°N
= 100 × 0.819N = 81.9N  ~ 82N

9. (12)
F 2iˆ  3ˆj  5kˆ
Sol. a 
m 2
 ˆi  1.5ˆj  2.5kˆ
1
s  ut  at 2
2
1 ˆ
 0  (i  1.5ˆj  2.5kˆ )(16)  8iˆ  12ˆj  20kˆ
2
b = 12
10. (10)
NUCLEUS-92, Rajeev Gandhi Nagar, Kota (Raj.) India 324005, Mob. 9358006181, 97831-97831 48
JEE MAIN 2021
Sol. v  u  2as
2 2

1
0  (10)2  2(a)  
2
a  100m / s2
F  ma  (0.1)(100)  10N

11. (6)
Sol. Let's say the compression in the spring by : y.
So, by work energy theorem we have
1 1
 mv 2  ky 2
2 2
m
y .v
k
4
y  10
100
 y  2m
 final length of spring
= 8 – 2 = 6m

12. (21)

Sol. amax = g = × 9.8
7
F = (M + m) amax = 5 amax
= 21 Newton

13. (5)

Sol.

F cos = N
F sin  = mg
mg
F
cos    sin 
1
10
mg 3
Fmin   5
1  2 2
3
14. (30)

NUCLEUS-92, Rajeev Gandhi Nagar, Kota (Raj.) India 324005, Mob. 9358006181, 97831-97831 49
JEE MAIN 2021

Sol.

N + T = 90
T = N = 0.5 (90 – T)
1.5 T = 45
T = 30

15. (1)

Sol.

At vmax, f will be limiting in nature.


 Balancing force in vertical direction
 Ncos30° – mg – Ncos60° = 0
 N[cos30° – cos60°] = mg
800 10 3 2
N=  10.2 × 10 kg m/s
 0.87  0.1
Hence option 1.

16. (2)

Sol.

Writing force equation in vertical direction


Mg – N = Ma
 70g – N = 70 × 0.2
 N = 70 [g – 0.2] = 70 × 9.8
 N = 686 Newton
Note : Since there is no compressive normal fromthe sides, hence friction will not act.
Hence option 2.
17. (3)
NUCLEUS-92, Rajeev Gandhi Nagar, Kota (Raj.) India 324005, Mob. 9358006181, 97831-97831 50
JEE MAIN 2021
1
Sol. ta  td
2
2s 1 2s

aa 2 ad
a a  g sin   g cos 
g 3
  g
2 2
g 3
a d  g sin   g cos    g
2 2
3
using the above values of aa and ad and putting in equation (i) we will gate  =
5

18. (2)

Sol.

By momentum conservation
4 × 10–3 (50 – v) – 4v = 0
4 103  50
v=  0.05ms 1
4  4 103
Impulse J = mv = 4 × .05 = 0.2 kgms–1

19. (4)
Sol. Let acceleration of wedge is a1 and acceleration of block w.r.t. wedge is a2

Ncos60° = Ma1 = 16a1  N = 32a1


F.B.D. of block w.r.t wedge

to incline

NUCLEUS-92, Rajeev Gandhi Nagar, Kota (Raj.) India 324005, Mob. 9358006181, 97831-97831 51
JEE MAIN 2021
N = 8g cos 30° – 8a1 sin 30°  32a1 = 4 3 g – 4a1
3
 a1 = g
9
Along incline
8gsin30° + 8a1cos30° = ma2 = 8a2
1 3 3 2g
a2 = g × + g. 
2 9 2 3
Option (4)

20. (2)
Sol. During upward motion

F = 2N = (+ve) constant
During downward motion

 F = 2N = (–ve) constant
 answer is option (2)

21. (3)

Sol.

On smooth incline
a = g sin 30°
1 2
by S = ut + at
2
1g 2 g 2
S= T  T …..(i)
22 4

NUCLEUS-92, Rajeev Gandhi Nagar, Kota (Raj.) India 324005, Mob. 9358006181, 97831-97831 52
JEE MAIN 2021

On rough incline
a = g sin30° – μg cos 30°
1
by S  ut  at 2
2
1
S g(1  3)(T) 2 ……(ii)
4
By (i) and (ii)
1 2 1
gT  g(1  3) 2 T 2
4 4
1  2  1  1
 1  3g   g   2 
2    3
 x = 3.00

22. (3)

Sol.

2m1m2g 2  3  5 10
T 
m1  m2 8
75

2
T
Stress 
A
24 75
 102 
 2  R 2
75 3
R2  
2  24 100 8  24
R = 0.125 m
R = 12.5 cm

23. (15)

NUCLEUS-92, Rajeev Gandhi Nagar, Kota (Raj.) India 324005, Mob. 9358006181, 97831-97831 53
JEE MAIN 2021

Sol.

F = 3a(For system) ……..(i)

fsmax = 1a (for 1kg block)


×1×g=a
5=a
F = 15N

24. (1)
Sol. Inside a spherical shell, gravitational field is zero and hence potential remains same everywhere
hence option (1)

25. (4)

Sol.

 dm 
Fthrust   .Vrel 
 dt 
 dm 
 Vrel  mg   ma
 dt 
 dm 
   500  10 10  10  20
3 3

 dt 
dm
 (60kg / s)
dt
Option (4)

26. (4)

Sol.

(m1g  2T)  m1a  (1)


T  m 2g  m 2 (2a)
2T  2m 2g  4m 2 a  (2)

NUCLEUS-92, Rajeev Gandhi Nagar, Kota (Raj.) India 324005, Mob. 9358006181, 97831-97831 54
JEE MAIN 2021
m1g  2m 2g  (m1  4m 2 )a
(8  4)g 4 g
a  g
(8  8) 16 4
10
a m / s2
4
1 2
S  at
2
0.2  2  4
 t2
10
t = 0.4 sec

27. (30)

Sol.

mg sin 30  ma
tan(30  ) 
mg cos30
5  10 1  2
tan(30  )  
5 3 3
1
tan  
3  3
1
1 tan 
3
3 tan   1  3  3 tan 
2 3 tan   2
1
tan 
3
 = 30°

NUCLEUS-92, Rajeev Gandhi Nagar, Kota (Raj.) India 324005, Mob. 9358006181, 97831-97831 55
JEE MAIN 2021
CENTRE OF MASS
1. A ball with a speed of 9 m/s collides with another identical ball at rest. After the collision, the
direction of each ball makes an angle of 30° with the original direction. The ratio of velocities of
the balls after collision is x : y, where x is ________. [JEE MAIN 2021 (FEB)]

A circular hole of radius   is cut out of a circular disc of radius 'a' as shown in figure. The
a
2.
2
centroid of the remaining circular portion with respect to point 'O' will be :
[JEE MAIN 2021 (FEB)]

1 10 5 2
(1) a (2) a (3) a (4) a
6 11 6 3

3. Given below are two statements : one is labelled as Assertion A and the other is labelled as
Reason R. [JEE MAIN 2021 (FEB)]
Assertion A : Body 'P' having mass M moving with speed 'u' has head-on collision elastically
with another body 'Q' having mass 'm' initially at rest. If m << M, body 'Q' will have a maximum
speed equal to '2u' after collision.
Reason R : During elastic collision, the momentum and kinetic energy are both conserved. In the
light of the above statements, choose the most appropriate answer from the options given below :
(1) A is not correct but R is correct.
(2) Both A and R are correct but R is NOT the correct explanation of A.
(3) Both A and R are correct and R is the correct explanation of A.
(4) A is correct but R is not correct.

4. Two masses A and B, each of mass M are fixed together by a massless spring. A force acts on the
mass B as shown in figure. If the mass A starts moving away from mass B with acceleration 'a',
then the acceleration of mass B will be :- [JEE MAIN 2021 (FEB)]

Ma  F MF F  Ma F  Ma
(1) (2) (3) (4)
M F  Ma M M
5. A ball of mass 10kg moving with a velocity 10 3ms –1 along X-axis, hits another ball of mass
20kg which is at rest. After collision, the first ball comes to rest and the second one disintegrates
into two equal pieces. One of the pieces starts moving along Y-axis at a speed of 10m/s. The
second piece starts moving at a speed of 20m/s at an angle  (degree) with respect to the X-axis.
The configuration of pieces after collision is shown in the figure. The value of  to the nearest
integer is _________ . [JEE MAIN 2021 (MARCH)]

NUCLEUS-92, Rajeev Gandhi Nagar, Kota (Raj.) India 324005, Mob. 9358006181, 97831-97831 56
JEE MAIN 2021

6. A large block of wood of mass M = 5.99kg is hanging from two long massless cords. A bullet of
mass m = 10g is fired into the block and gets embedded in it. The (block + bullet) then swing
upward, their centre of mass rising a vertical distance h = 9.8 cm before the (block - bullet)
pendulum comes momentarily to rest at the end of its arc. The speed of the bullet just before
collision is: (Take g = 9.8ms–2) [JEE MAIN 2021 (MARCH)]

(1) 841.4m/s (2) 811.4m/s


(3) 831.4m/s (4) 821.4m/s

7. Two identical blocks A and B each of mass m resting on the smooth horizontal floor are connected
by a light spring of natural length L and spring constant K. A third block C of mass m moving
with a speed v along the line joining A and B collides with A. The maximum compression in the
spring is [JEE MAIN 2021 (MARCH)]

 mv mv m
(1) v (2) (3) (4)
2k 2k k 2k

8. The disc of mass M with uniform surface mass density  is shown in the figure. The centre of
xa xa
mass of the quarter disc (the shaded area) is at the position , where x is _______.
3 3
(Round off to the Nearest Integer) [JEE MAIN 2021 (MARCH)]
[a is an area as shown in the figure]

NUCLEUS-92, Rajeev Gandhi Nagar, Kota (Raj.) India 324005, Mob. 9358006181, 97831-97831 57
JEE MAIN 2021

9. The position of the centre of mass of a uniform semi-circular wire of radius 'R' placed in x-y plane
 xR 
with its centre at the origin and the line joining its ends as x-axis is given by  0, 
  
Then, the value of |x| is __________. [JEE MAIN 2021 (JULY)]

10. Three particles P, Q and R are moving along the vectors A  ˆi  ˆj, B  ˆj  kˆ and C  ˆi  ˆj
respectively. They strike on a point and start to move in different directions. Now particle P is
moving normal to the plane which contains vector A and B . Similarly particle Q is moving
normal to the plane which contains vector A and B . The angle between the direction of motion of
 1 
P and Q is cos–1   . Then the value of x is ________. [JEE MAIN 2021 (JULY)]
 x
11. Two billiard balls of equal mass 30 g strike a rigid wall with same speed of 108 kmph (as shown)
but at different angles. If the balls get reflected with the same speed then the ratio of the
magnitude of impulses imparted to ball ‘a’ and ball ‘b’ by the wall along ‘X’ direction is :
[JEE MAIN 2021 (JULY)]

(1) 1:1 (2) 2 :1 (3) 2 :1 (4) 1: 2


12. A body of mass 2 kg moving with a speed of 4 m/s. makes an elastic collision with another body
at rest and continues to move in the original direction but with one fourth of its initial speed. The
x
speed of the two body centre of mass is m / s .Then the value of x is_________.
10
[JEE MAIN 2021 (JULY)]
13. Three objects A, B and C are kept in a straight line on a frictionless horizontal surface. The masses
of A, B and C are m, 2m and 2m respectively. A moves towards B with a speed of 9 m/s and
makes an elastic collision with it. Thereafter B makes a completely inelastic collision with C. All
motions occur along same straight line. The final speed of C is : [JEE MAIN 2021 (JULY)]

(1) 6 m/s (2) 9 m/s (3) 4 m/s (4) 3 m/s


NUCLEUS-92, Rajeev Gandhi Nagar, Kota (Raj.) India 324005, Mob. 9358006181, 97831-97831 58
JEE MAIN 2021
14. A bullet of 10 g, moving with velocity  , collides head-on with the stationary bob of a pendulum
and recoils with velocity 100 m/s. The length of the pendulum is 0.5 m and mass of the bob is 1
kg. The minimum value of  = __________m/s so that the pendulum describes a circle. (Assume
the string to be inextensible and g = 10 m/s2) [JEE MAIN 2021 (AUGUST)]

15. A block moving horizontally on a smooth surface with a speed of 40 m/s split into two parts with
masses in the ratio of 1 : 2. If the smaller part moves at 60 m/s in the same direction, then the
fractional change in kinetic energy is :- [JEE MAIN 2021 (AUGUST)]
1 2 1 1
(1) (2) (3) (4)
3 3 8 4

16. A body of mass M moving at speed V0 collides elastically with a mass 'm' at rest. After the
collision, the two masses move at angle 1 and 2 with respect to the initial direction of motion of
the body of mass M. The largest possible value of the ratio M/m, for which the angles 1 and 2
will be equal , is : [JEE MAIN 2021 (AUGUST)]
(1) 4 (2) 1 (3) 3 (4) 2

17. A block moving horizontally on a smooth surface with a speed of 40 ms-1 splits into two equal
parts. If one of the parts moves at 60 ms-1 in the same direction, then the fractional change in the
kinetic energy will be x : 4 where x =_________. [JEE MAIN 2021 (AUGUST)]

18. A ball of mass 10kg moving with a velocity along the x-axis, hits another ball of mass 20kg which
is at rest. After the collision, first ball of mass 20kg which is at rest. After the collision, first ball
comes to rest while the second ball disintegrates into two equal pieces. One piece starts moving
along y-axis with a speed of 10m/s. The second piece starts moving at an angle of 30° with respect
to the x-axis. The velocity of the ball moving at 30° with x-axis is x m/s. The configuration of
pieces after collision is shown in the figure below. The value of x to the nearest integer is
________. [JEE MAIN 2021 (AUGUST)]

19. An object of mass m1 collides with another object of mass m2, which is at rest. After the collision
the objects move with equal speeds in opposite direction. the ratio of the masses m2 : m1 is :
[JEE MAIN 2021 (AUGUST)]
(1) 3 : 1 (2) 2 : 1 (3) 1 : 2 (4) 1 : 1

NUCLEUS-92, Rajeev Gandhi Nagar, Kota (Raj.) India 324005, Mob. 9358006181, 97831-97831 59
JEE MAIN 2021
ANSWER KEY

1. 1 2. 3 3. 3 4. 4 5. 30 6. 3 7. 1
8. 4 9. 2 10. 3 11. 2 12. 25 13. 4 14. 400
15. 3 16. 3 17. 1 18. 20 19. 1

SOLUTION

1. (1)

Sol.

From conservation of momentum along y-axis.


Piy  Pfy
0 + 0 = mv1 sin 30 ĵ + mv2 sin 30°   ĵ
mv2 sin 30° = mv1 sin 30°
v
v2 = v1 or 1  1
v2

2. (3)

Sol.

Let  be the uniform mass density of disc then


 a 2  3a
 
a 2 a     
x COM   4 2
a 2
a 2 
4
3a
a
= 8  5a
1 6
1
4
Option (2) is correct

NUCLEUS-92, Rajeev Gandhi Nagar, Kota (Raj.) India 324005, Mob. 9358006181, 97831-97831 60
JEE MAIN 2021
3. (3)
Sol. For e = 1 & second body at rest
2m1u1 2u  M 
V2   ~ 2u
m1  m 2 M  m
Since M >> m

4. (4)
m1a1  m 2a 2
Sol. a cm 
m1  m 2
F Ma  Ma B

2M 2M
F  Ma
aB 
M
5. (30)
Sol. Before Collision

After Collision

From conservation of momentum along x axis;


Pi  Pf
10 10 3  200cos 
3
cos   =   30
2
6. (3)
Sol. From energy conservation
after bullet embedded till the 
system comes momentrarily at rest 
 
1
 M  m  g h =  M  m  v12
2
[v1 is velocity after collision]  v1  2gh
Applying momentum conservation, (just before and just after collision)
mv = (M + m)v1
Mm 6
v  v1   2  9.8  9.8 10 –2  8314.55m/s
 m  10 10 –3

NUCLEUS-92, Rajeev Gandhi Nagar, Kota (Raj.) India 324005, Mob. 9358006181, 97831-97831 61
JEE MAIN 2021
7. (1)
Sol. For max compression collision has to be elastic
After collisioin, C comes to rest and A moves with v
 vcm of A and B = v/2
Maximum compression will occur when A and B move with same velocity i.e. vcm
By energy conservation, ½ mv2 = ½ 2m vcm2 + ½ kx2
 x = v√(m/2k)

8. (4)
4a 4a
Sol. C.O.M of quarter disc is at ,
3 3
x=4

9. (2)
2R
Sol. COM of semi-circular ring is at

Distance from centre  x = 2

10. (3)
AB ˆi  ˆj  kˆ
Sol. Direction of P vˆ 1   
| AB| 3
AB 2kˆ
Direction of Q vˆ 2      kˆ
| AC | 2
Angle between vˆ 1 and vˆ 2
vˆ 1  vˆ 2 1/ 3 1
 
vˆ 1 vˆ 2 (1)(1) 3
x=3

11. (2)
Sol. Impulse = change in momentum
Ball (a) | Δp | 2mu  J1
Ball (b) | Δp | 2mu cos 45°  J 2
J1 1
  2
J 2 cos 45

NUCLEUS-92, Rajeev Gandhi Nagar, Kota (Raj.) India 324005, Mob. 9358006181, 97831-97831 62
JEE MAIN 2021
12. (25)
Sol. pi = pf
2 × 4 = 2 × 1 + m2 × v2
m2v2 = 6 …..(i)
by coefficient of restitution
v2  1
1=  v2 = 5 m/s
4
by (i)
m2 × 5 = 6
m2 = 1.2 kg
m v  m2 v2
vcm  1 1
m1  m2
2  1  1.2  5 8 25
v cm   
2  1.2 3.2 10
x = 25
13. (4)
Sol. Collision between A and B

m × 9 = mv1 + 2m v2 (from momentum conservation)


v 2  v1
e 1
9
 v2 = 6 m/sec. , v1 = –3 m/sec.
collision between B and C

2m × 6 = 4mv (from momentum conservation)


v = 3 m/s
14. (400)

Sol.

V'  5gr  5 10  0.5


V '  5m / s
m1V  m2  5  m1 100
10 10
V  5  100
1000 1000
V = 400m/s

NUCLEUS-92, Rajeev Gandhi Nagar, Kota (Raj.) India 324005, Mob. 9358006181, 97831-97831 63
JEE MAIN 2021
15. (3)

Sol.

3MV0 = 2MV2 +MV1


3V0 = 2V2 +V1
120  2V2  60  V2  30m / s
1 1 1
MV12  2MV22  3MV02
K.E. 2 2 2

K.E. 1
3MV02
2
V 2  2V22  3V02 3600  1800  4800 1
 1 2  
3V0 4800 8

16. (3)

Sol.

given 1  2  
from momentum conservation
in x-direction MV0  MV1 cos   mV2 cos 
in y-direction 0  MV1 sin   mV2 sin 
Solving above equations
MV1
V2  , V0  2V1 cos 
m
From energy conservation
1 1 1
MV02  MV12  MV22
2 2 2
Substituting value of V2& V0, we will get
M
 1  4 cos 2   4
m
M
3
m
17. (1)

Sol.

Pi  Pf
m m
m  40   v   60
2 2
NUCLEUS-92, Rajeev Gandhi Nagar, Kota (Raj.) India 324005, Mob. 9358006181, 97831-97831 64
JEE MAIN 2021
v
40   30
2
 v  20
1
(K.E.) I  m  (40) 2  800m
2
1m 1 m
(K.E.)f  .(20) 2  . (60) 2  1000m
2 2 2 2
K.E.  1000m  800m  200m
K.E 200m 1 x
  
(K.E.)i 800m 4 4
x=1

18. (20)
Sol. Let velocity of 2nd fragment is v then by conservation of linear momentum
10(10 3)iˆ  (10)(10jˆ)  10v
 v  10 3ˆi  10 ĵ
| v | 300  100  400  20m / s

19. (1)

Sol.

m1v1 = - m1v + m2v

m2
v1 = -v+ v
m1
(v1  v) m 2

v m1
2v
e= =1
v1
v1
v
2
v1 v1 / 2 m 2

v1 / 2 m1
m
3 2
m1

NUCLEUS-92, Rajeev Gandhi Nagar, Kota (Raj.) India 324005, Mob. 9358006181, 97831-97831 65
JEE MAIN 2021
CIRCULAR MOTION
1. The point A moves with a uniform speed along the circumference of a circle of radius 0.36 m and
covers 30° in 0.1 s. The perpendicular projection 'P' from 'A' on the diameter MN represents the
simple harmonic motion of 'P'. The restoration force per unit mass when P touches M will be :
[JEE MAIN 2021 (FEB)]

(1) 100 N (2) 0.49 N (3) 50 N (4) 9.87 N


2. A particle is moving with uniform speed along the circumference of a circle of radius R under the
action of a central fictitious force F which is inversely proportional to R 3. Its time period of
revolution will be given by : [JEE MAIN 2021 (FEB)]
3 5 4
(1) T  R 2 (2) T  R 2 (3) T  R 2 (4) T  R 3

3. A block of 200g mass moves with a uniform speed in a horizontal circular groove, with vertical
side walls of radius 20cm. If the block takes 40 s to complete one round, the normal force by the
side walls of the groove is: [JEE MAIN 2021 (MARCH)]
–2 –3
(1) 0.314 N (2) 9.859 × 10 N (3) 6.28 × 10 N (4) 9.859 × 10–4N

4. Statement I: A cyclist is moving on an unbanked road with a speed of 7 kmh–1 and takes a sharp
circular turn along a path of radius of 2m without reducing the speed. The static friction
coefficient is 0.2. The cyclist will not slip and pass the curve (g = 9.8 m/s2)
Statement II: If the road is banked at an angle of 45°, cyclist can cross the curve of 2m radius
with the speed of 18.5 kmh–1 without slipping. [JEE MAIN 2021 (MARCH)]
In the light of the above statements, choose the correct answer from the options given below.
(1) Statement I is incorrect and statement II is correct.
(2) Statement I is correct and statement II is incorrect.
(3) Both statements I and statement II are false
(4) Both statements I and statement II are true

5. A modern grand-prix racing car of mass m is travelling on a flat track in a circular arc of radius R
with a speed v. If the coefficient of static friction between the tyres and the track is s, then the
magnitude of negative lift F1. acting downwards on the car is: [JEE MAIN 2021 (MARCH)]
(Assume forces on the four tyres are identical and g = acceleration due to gravity)

NUCLEUS-92, Rajeev Gandhi Nagar, Kota (Raj.) India 324005, Mob. 9358006181, 97831-97831 66
JEE MAIN 2021

 v2   v2   v2   v2 
(1) m   g (2) m  – g (3) m  g –  (4) –m  g  
 s R   s R   s R   s R 

6. If the angular velocity of earth's spin is increased such that the bodies at the equator start floating,
the duration of the day would be approximately: [JEE MAIN 2021 (MARCH)]
–2 3
(Take : g = 10ms , the radius of earth, R = 6400 × 10 m, Take  = 3.14)
(1) 60 minutes (2) does not change
(3) 1200 minutes (4) 84 minutes

7. The angular speed of truck wheel is increased from 900 rpm to 2460 rpm in 26 seconds. The
number of revolutions by the truck engine during this time is _______ .
[JEE MAIN 2021 (MARCH)]

8. A body rotating with an angular speed of 600 rpm is uniformly acceleration to 1800 rpm in 10 sec.
The number of rotations made in the process is ___. [JEE MAIN 2021 (JULY)]

9. A solid disc of radius 20 cm and mass 10 kg is rotating with an angular velocity of 600 rpm, about
an axis normal to its circular plane and passing through its centre of mass. The retarding torque
required to bring the disc at rest in 10 s is _____  × 5 10–1Nm [JEE MAIN 2021 (JULY)]

10. A small block slides down from the top of hemisphere of radius R = 3 m as shown in the figure.
The height 'h' at which the block will lose contact with the surface of the sphere is _________m.
(Assume there is no friction between the block and the hemisphere )
[JEE MAIN 2021 (JULY)]

11. Two satellites revolve around a planet in coplanar circular orbits in anticlockwise direction. Their
period of revolutions are 1 hour and 8 hours respectively. The radius of the orbit of nearer satellite
is 2 × 103 km. The angular speed of the farther satellite as observed from the nearer satellite at the

instant when both the satellites are closest is rad h–1 where x is ………
x
[JEE MAIN 2021 (AUGUST)]
NUCLEUS-92, Rajeev Gandhi Nagar, Kota (Raj.) India 324005, Mob. 9358006181, 97831-97831 67
JEE MAIN 2021
12. A particle of mass m is suspended from a ceiling through a string of length L. The particle moves
L
in a horizontal circle of radius r such that r = .The speed of particle will be :
2
[JEE MAIN 2021 (AUGUST)]
rg
(1) rg (2) 2rg (3) 2 rg (4)
2

13. A huge circular arc of length 4.4 ly subtends an angle '4s' at the centre of the circle. How long it
would take for a body to complete 4 revolution if its speed is 8 AU per second ?
Given : 1 1y = 9.46  1015 m [JEE MAIN 2021 (AUGUST)]
1 AU = 1.5  1011 m
(1) 4.1  108 s (2) 4.5  1010 s (3) 3.5  106 s (4) 7.2  108 s

ANSWER KEY
1. 4 2. 1 3. 4 4. 4 5. 2 6. 4 7. 728
8. 200 9. 4 10. 2 11. 3 12. 1 13. 2

NUCLEUS-92, Rajeev Gandhi Nagar, Kota (Raj.) India 324005, Mob. 9358006181, 97831-97831 68
JEE MAIN 2021
SOLUTION

1. (4)

Sol.

30°  0.1 s
360°  1.2 s = T
2  5
 
T 3
F
At M, F = m A   2 A
2

2. (1)
1
Sol. F
R3
K
 2 R
R3
K 1
2   4
m R
2
 2  K 1
    4
T m R
T2  R 4
T  R2

3. (4)
Sol. N = m2 R
 42 
N  m 2 R .......(i)
T 
Given m = 0.2kg. T = 40 S, R = 0.2m
Put value in equation (1)
N = 9.859 × 10–4N

4. (4)
Sol. Statement I:
vmax  μRg  (0.2)  2  9.8
vmax = 1.97m/s
7km/h = 1.944 m/s
NUCLEUS-92, Rajeev Gandhi Nagar, Kota (Raj.) India 324005, Mob. 9358006181, 97831-97831 69
JEE MAIN 2021
Speed is lower than vmax, hence it can take safe turn.
Statement II
 tan θ  μ  1  0.2 
v max  Rg    2  9.8   5.42m / s
1  μ tan θ  1  0.2 
18.5km/h = 5.14 m/s
Speed is lower than vmax, hence it can take safe turn.

5. (2)
mv2
Sol. s N 
R
2
mv
N  mg  FL
s R
mv 2
FL  – mg
s R

6. (4)
Sol. For objects to float
mg = m2R
 = angular velocity of earth
R = Radius of earth
g
 ........(1)
R
Duration of day = T
2
T ........(2)

R
 T  2
g
6400 103
= 2
10
T
  83.775 minutes
60
84 minutes.

7. (728)
   2 
Sol. We know,    1 t
 2 
Let number of revolution be N
 900  2460 
 2N  2    26
 60  2 
N = 728

NUCLEUS-92, Rajeev Gandhi Nagar, Kota (Raj.) India 324005, Mob. 9358006181, 97831-97831 70
JEE MAIN 2021
8. (200)
Sol. f = 0 + t
 = 1200 × 6
1
 = 0t + at2
2
10 1 1
= 600 ×   1200  6 
60 2 36
 = 200

9. (4)
L I  f  i 
Sol.  
t t
2
mR
 [0  ] 10   20 102 2
600  
 2   = 0.4 = 4 × 10–2
t 2 30 10

10. (2)

Sol.

mv2
mg cos   ....(1)
R
h
cos  
R
Energy conservation
1
mg{R  h}  mV 2 ....(2)
2
 h  2mg{R  h}
from (1) & (2)  mg   
R  R
2R
h  2m
3

11. (3)

Sol.

NUCLEUS-92, Rajeev Gandhi Nagar, Kota (Raj.) India 324005, Mob. 9358006181, 97831-97831 71
JEE MAIN 2021
T1 = 1 hour
1 = 2rad/hour

T2 = 8 hours  2  rad / hour
4
R1 = 2 × 103 km
As T2 R3
3 2 2/3
R  T  R 8
 2   2   2   = 4  R2 = 8 × 103 km
 R1   T1  R1  1 

V1 = 1R1 = 4 × 103 km / h
V2 = 2R2 = 2 × 103 km / h
V  V2 2103 
Relative  = 1  = rad/hour
R 2  R1 6 10 3
3
x=3

12. (1)
Sol Conical pendulum

r
2
r 1
sin   
2
 = 45°
mv2
T sin  
r
Tcos = mg
v2
tan    v  rg
rg

NUCLEUS-92, Rajeev Gandhi Nagar, Kota (Raj.) India 324005, Mob. 9358006181, 97831-97831 72
JEE MAIN 2021
13. (2)
Sol. R

4  2R 4  2  
Time    
v v 
Put  4.4  9.46 1015
v  8 1.5 1011
4 
  rad.
3600 180
we get time = 4.5  1010 sec

NUCLEUS-92, Rajeev Gandhi Nagar, Kota (Raj.) India 324005, Mob. 9358006181, 97831-97831 73
JEE MAIN 2021

RBD
1. A uniform thin bar of mass 6 kg and length 2.4 meter is bent to make an equilateral hexagon. The
moment of inertia about an axis passing through the centre of mass and perpendicular to the plane
of hexagon is ____ × 10–1 kg m2. [JEE MAIN 2021 (FEB)]

2. Four identical solid spheres each of mass 'm' and radius 'a' are placed with their centres on the four
corners of a square of side 'b'. The moment of inertia of the system about one side of square where
the axis of rotation is parallel to the plane of the square is : [JEE MAIN 2021 (FEB)]
4 8 8 4
(1) ma 2  2mb 2 (2) ma 2  mb 2 (3) ma 2  2mb 2 (4) ma 2
5 5 5 5

3. A cord is wound round the circumference of wheel of radius r. The axis of the wheel is horizontal
and the moment of inertia about it is I. A weight mg is attached to the cord at the end. The weight
falls from rest. After falling through a distance 'h', the square of angular velocity of wheel will be
[JEE MAIN 2021 (FEB)]
2mgh 2mgh 2gh
(1) (2) (3) 2 g h (4)
I  2mr 2
I  mr 2
I  mr 2

4. A sphere of mass 2kg and radius 0.5 m is rolling with an initial speed of 1ms–1 goes up an inclined
plane which makes an angle of 30° with the horizontal plane, without slipping. How long will the
sphere take to return to the starting point A? [JEE MAIN 2021 (MARCH)]

(1) 0.60 s (2) 0.52 s (3) 0.57 s (4) 0.80 s

5. Four equal masses, m each are placed at the corners of a square of length (l) as shown in the
figure. The moment of inertia of the system about an axis passing through A and parallel to DB
would be : [JEE MAIN 2021 (MARCH)]

(1) ml2 (2) 2ml2 (3) 3ml2 (4) 3 ml2

NUCLEUS-92, Rajeev Gandhi Nagar, Kota (Raj.) India 324005, Mob. 9358006181, 97831-97831 74
JEE MAIN 2021
6. Consider a 20kg uniform circular disk of radius 0.2 m. It is pin supported at its center and is at rest
initially. The disk is acted upon by a constant force F = 20N through a massless string wrapped
around its periphery as shown in the figure. [JEE MAIN 2021 (MARCH)]

Suppose the disk makes n number of revolutions to attain an angular speed of 50 rad s–1. The value
of n, to the nearest integer, is _______. [Given: In on complete revolution, the disk rotates by 6.28
rad] [JEE MAIN 2021 (MARCH)]

7. A force f  4iˆ  3jˆ  4kˆ is applied on an intersection point of x = 2 plane and x-axis. The
magnitude of torque of this force about a point (2,3,4) is _______. (Round off to the Nearest
Integer) [JEE MAIN 2021 (MARCH)]
Official Ans. by NTA (20)

8. A solid disc of radius 'a' and mass 'm' rolls down without slipping on an inclined plane making an
2
angle  with the horizontal. The acceleration of the disc will be g sin where b is ______ .
b
(Round off to the Nearest Integer) [JEE MAIN 2021 (MARCH)]
(g = acceleration due to gravity)
( = angle as shown in figure)

9. A thin circular ring of mass M and radius r is rotating about its axis with an angular speed . Two
particles having mass m each are now attached at diametrically opposite points. The angular speed
of the ring will become: [JEE MAIN 2021 (MARCH)]
M M  2m M M – 2m
(1)  (2)  . (3)  (4) 
Mm M M  2m M  2m

NUCLEUS-92, Rajeev Gandhi Nagar, Kota (Raj.) India 324005, Mob. 9358006181, 97831-97831 75
JEE MAIN 2021
10. A solid cylinder of mass m is wrapped with an inextensible light string and, is placed on a rough
inclined plane as shown in the figure. The frictional force acting between the cylinder and the
inclined plane is: [JEE MAIN 2021 (MARCH)]

[The coefficient of static friction, s, is 0.4]


7 mg
(1) mg (2) 5 mg (3) (4) 0
2 5

11. A triangular plate is shown. A force F  4iˆ – 3jˆ is applied at point P. The torque at point P with
respect to point 'O' and 'Q' are: [JEE MAIN 2021 (MARCH)]

(1) –15 – 20 3,15 – 20 3 (2) 15  20 3,15 – 20 3


(3) 15 – 20 3,15  20 3 (4) –15  20 3,15  20 3

12. A mass M hangs on a massless rod of length l which rotates at a constant angular steady speed in a
circular path of constant radius. Assume that the system is in steady circular motion with constant
angular velocity . The angular momentum of M about point A is LA which lies in the positive z
direction and the angular momentum of M about B is LB. The correct statement for this system is:
[JEE MAIN 2021 (MARCH)]

NUCLEUS-92, Rajeev Gandhi Nagar, Kota (Raj.) India 324005, Mob. 9358006181, 97831-97831 76
JEE MAIN 2021
(1) LA and LB are both constant in magnitude and direction.
(2) LB is constant in direction with varying magnitude
(3) LB is constant, both in magnitude and direction.
(4) LA is constant, both in magnitude and direction.

13. The following bodies,


(1) a ring (2) a disc (3) a solid cylinder (4) a solid sphere,
of same mass 'm' and radius 'R' are allowed to roll down without slipping simultaneously from the
top of the inclined plane. The body which will reach first at the bottom of the inclined plane is
________ . [JEE MAIN 2021 (MARCH)]
[Mark the body as per their respective numbering given in the question]

14. A body rolls down an inclined plane without slipping. The kinetic energy of rotation is 50% of its
translational kinetic energy. The body is : [JEE MAIN 2021 (JULY)]
(1) Solid sphere (2) Sold cylinder (3) Hollow cylinder (4) Ring

15. Two bodies, a ring and a solid cylinder of same material are rolling down without slipping an
inclined plane. The radii of the bodies are same. The ratio of velocity of the centre of mass at the
x
bottom of the inclined plane of the ring to that of the cylinder is . Then, the value of x is ____.
2
[JEE MAIN 2021 (JULY)]

16. Consider a situation in which a ring, a solid cylinder and a solid sphere roll down on the same
inclined plane without slipping. Assume that they start rolling from rest and having identical
diameter. [JEE MAIN 2021 (JULY)]
The correct statement for this situation is :-
(1) The sphere has the greatest and the ring has the least velocity of the centre of mass at the
bottom of the inclined plane.
(2) The ring has the greatest and the cylinder has the least velocity of the centre of mass at the
bottom of the inclined plane.
(3) All of them will have same velocity.
(4) The cylinder has the greatest and the sphere has the least velocity of the centre of mass at the
bottom of the inclined plane.

NUCLEUS-92, Rajeev Gandhi Nagar, Kota (Raj.) India 324005, Mob. 9358006181, 97831-97831 77
JEE MAIN 2021
17. The centre of a wheel rolling on a plane surface moves with a speed v0. A particle on the rim of
the wheel at the same level as the centre will be moving at a speed x 0. Then the value of x is
_____. [JEE MAIN 2021 (JULY)]

18. [JEE MAIN 2021 (JULY)]


List-I List-II
(a) MI of the rod (length L, mass M, about an axis (i) 8 ML2/3
 to the rod passing through the midpoint)
(b) MI of the rod (length L, mass 2M, about an axis (ii) ML2/3
to the rod passing through one of its end)
(c) MI of the rod (length 2L, Mass M, about an (iii) ML2/12
axis  to the rod passing through its midpoint)
(d) MI of the rod (Length 2L, Mass 2M, about an (iv) 2 ML2/3
axis  to the rod passing through one of its end)
Choose of correct answer from the option given below :
(1) (a)–(ii), (b)–(iii), (c)– (i), (d)–(iv) (2) (a)–(ii), (b)–(i), (c)– (iii), (d)–(iv)
(3) (a)–(iii), (b)–(iv), (c)– (ii), (d)–(i) (4) (a)–(iii), (b)–(iv), (c)– (i), (d)–(ii)

19. The figure shows two solid discs with radius R and r respectively. If mass per unit area is same for
both, what is the ratio of MI of bigger disc around axis AB (Which is  to the plane of the disc
and passing through its centre) to MI to smaller disc around one of its diameters lying on its
plane? Given 'M' is the mass of the larger disc. (MI stands for moment of inertia)
[JEE MAIN 2021 (JULY)]

(1) R2 : r2 (2) 2r4 : R4 (3) 2R2 : r2 (4) 2R4 : r4

20. In the given figure, two wheels P and Q are connected by a belt B. The radius of P is three time as
I 
that of Q. In case of same rotational kinetic energy, the ratio of rotational inertias  1  will be
 I2 
x: 1. The value of x will be __________. [JEE MAIN 2021 (JULY)]

NUCLEUS-92, Rajeev Gandhi Nagar, Kota (Raj.) India 324005, Mob. 9358006181, 97831-97831 78
JEE MAIN 2021
21. Given below are two statements : one is labeled as Assertion A and the other is labeled as Reason
R. Assertion A :Moment of inertia of a circular disc of mass ‘M’ and radius ‘R’ about X, Y axes
(passing through its plane) and Z-axis which is perpendicular to its plane were found to be Ix, Iy
and Iz respectively .The respective radii of gyration about all the three axes will be the same.
Reason R : A rigid body making rotational motion has fixed mass and shape. In the light of the
above statements, choose the most appropriate answer from the options given below:
[JEE MAIN 2021 (JULY)]
(1) Both A and R are correct but R is NOT the correct explanation of A.
(2) A is not correct but R is correct.
(3) A is correct but R is not correct.
(4) Both A and R are correct and R is the correct explanation of A.

22. A 2 kg steel rod of length 0.6 m is clamped on a table vertically at its lower end and is free to
rotate in vertical plane. The upper end is pushed so that the rod falls under gravity, Ignoring the
friction due to clamping at its lower end, the speed of the free end of rod when it passes through its
lowest position is ............ms–1. (Take g = 10 ms–2) [JEE MAIN 2021 (AUGUST)]

23. The solid cylinder of length 80 cm and mass M has a radius of 20 cm. Calculate the density of the
material used if the moment of inertia of the cylinder about an axis CD parallel to AB as shown in
figure is 2.7 kg m2. [JEE MAIN 2021 (AUGUST)]

(1) 14.9 kg / m3 (2) 7.5 × 101 kg / m3 (3) 7.5 × 102 kg/m3 (4) 1.49 × 102 kg / m3

24. Consider a badminton racket with length scales as shown in the figure.
[JEE MAIN 2021 (AUGUST)]

If the mass of the linear and circular portions of the badminton racket are same (M) and the mass
of the threads are negligible, the moment of inertia of the racket about an axis perpendicular to the
r
handle and in the plane of the ring at , distance from the end A of the handle will be.....Mr2.
2

NUCLEUS-92, Rajeev Gandhi Nagar, Kota (Raj.) India 324005, Mob. 9358006181, 97831-97831 79
JEE MAIN 2021
25. Two discs have moments of inertia I1 and I2 about their respective axes perpendicular to the plane
and passing through the centre. They are rotating with angular speeds, 1 and  2 respectively and
are brought into contact face to face with their axes of rotation coaxial. The loss in kinetic energy
of the system in the process is given by: [JEE MAIN 2021 (AUGUST)]
I1I2 (I1  I 2 ) 2 12
(1) (1  2 ) 2 (2)
(I1  I 2 ) 2(I1  I 2 )
(I1I 2 ) 2 (1  2 ) 2
(3) (1  2 ) 2 (4)
2(I1  I 2 ) 2(I1  I 2 )

26. Moment of inertia of a square plate of side l about the axis passing through one of the corner and
perpendicular to the plane of square plate is given by : [JEE MAIN 2021 (AUGUST)]
Ml2 Ml2 2 2
(1) (2) Ml2 (3) (4) Ml
6 12 3

27. A system consist of two identical spheres each of mass 1.5 kg and radius 50 cm at the end of light
rod. The distance between the centres of the two spheres is 5 m. What will be the moment of
inertia of the system about an axis perpendicular to the rod passing through its midpoint ?
[JEE MAIN 2021 (AUGUST)]
2 2 2
(1) 18.75 kgm (2) 1.905 10 kgm (3) 19.05 kgm
5
(4) 1.875 105 kgm2

28. A uniform heavy rod of weight 10 kg ms-2 , cross-sectional area 1000 cm2 and length 20 cm is
hanging from a fixed support. Young modules of the material of the rod is 2  1011 Nm-2.
Neglecting the lateral contraction, find the elongation of rod due to its own weight .
[JEE MAIN 2021 (AUGUST)]
(1) 2  10 m
-9
(2) 5  10 m
-8
(3) 4  10 m
-8
(4) 5  10-10 m

29. Angular momentum of a single particle moving with constant speed along circular path :
(1) changes in magnitude but remains same in the direction [JEE MAIN 2021 (AUGUST)]
(2) remains same in magnitude and direction
(3) remains same in magnitude but changes in the direction
(4) is zero

ANSWER KEY
1. 8 2. 3 3. 2 4. 3 5. 3 6. 20 7. 20
8. 3 9. 3 10. 3 11. 1 12. 4 13. 4 14. 2
15. 3 16. 1 17. 2 18. 3 19. 4 20. 9 21. 2
22. 6 23. 4 24. 52 25. 3 26. 4 27. 3 28. 4
29. 2

NUCLEUS-92, Rajeev Gandhi Nagar, Kota (Raj.) India 324005, Mob. 9358006181, 97831-97831 80
JEE MAIN 2021
SOLUTION
1. (8)

Sol.

m = mass of one side of hexagon = 1 kg


6 = 2.4   0.4m
r
sin 60° =

3
r =  sin 60° =
2
m 2 
MOI, I    mr 2  6
 12 
m 2  3 
2

  m    6
 12  2  

= 5 m2
= 5 × 1 × 0.16
= 0.8
I = 8 × 10–1 kg m2
Ans. 8

2. (3)

Sol.

2  2 
I  2   ma 2   2   ma 2  mb 2 
5  5 
8
I ma 2  2mb 2
5
NUCLEUS-92, Rajeev Gandhi Nagar, Kota (Raj.) India 324005, Mob. 9358006181, 97831-97831 81
JEE MAIN 2021
3. (2)
1 2 1
Sol. mgh  I  mv 2
2 2
v  r

1 2 1
mgh = I  m2 r 2
2 2
2mgh
 2
 I  mr2

4. (3)
g sin  5 10 5g
Sol. a   
I 7 2 14
1 2
mR
2v 2 114 28
t 0   = 0.57
a 5g 5  9.8

5. (3)
Sol. Moment of inertia of point mass
= mass × (Perpendicular distance from axis)2

2 2
 l   l 
Moment of Inertia  m(0)  m(l 2)  m 
2 2
  m  = 3ml2
 2  2
6. (20)
 F.R. | 2F
Sol.   2

I mR / 2 mR
2  200
  10rad / s 2
20   0.2 

NUCLEUS-92, Rajeev Gandhi Nagar, Kota (Raj.) India 324005, Mob. 9358006181, 97831-97831 82
JEE MAIN 2021
  0  2
2 2

2500
 50   0 2  2 10     
2

20
  125rad
125
No. of revolution =  20 revolution.
2

7. (20)
Sol. τ  r F
ˆ  (2iˆ  3ˆj  4k)
r  (2i) ˆ  3jˆ  4k̂
&F  4î  3 ĵ  4kˆ
ˆi ˆj k̂
τ  r  F  0 3 4
4 3 4
 ˆi(12  12)  ˆj(0  16)  k̂(0  12)
 –16iˆ  12kˆ
 | τ | 162  122  20

8. (3)
g sin  g sin  2
Sol. a   g sin 
I 1 3
1 2
1
mR 2
b=3

9. (3)
Sol. Using conservation of angular momentum
(Mr2) = (Mr2 + 2r2)'
M
' 
M  2m

10. (3)

Sol.

Let's take solid cylinder is in equilibrium


T + f = mg sin60 .....(i)
NUCLEUS-92, Rajeev Gandhi Nagar, Kota (Raj.) India 324005, Mob. 9358006181, 97831-97831 83
JEE MAIN 2021
TR – fR = 0 ......(ii)
Solving we get
mg sin 
T  f req 
2
But limiting friction < required friction
mg sin 60
mgcos60ׄ° <
2
 Hence cylinder will not remain in equilibrium.
Hence f = kinetic
= kN
mg
= kmgcos 60° =
5

11. (1)
Sol. F  4iˆ  3 ĵ
r1  5iˆ  5 3j&r
ˆ ˆ ˆ
2  5i  5 3j
Torque about 'O'
τo  r1  F  (15  20 3)kˆ  (15  20 3)(k)
ˆ
Torque about 'Q'
τQ  r2  F  (15  20 3)kˆ  (15  20 3)(k)
ˆ

12. (4)
Sol. We know, L  m(r  v)
Now with respect to A, we always get direction of L along +ve z-axis and also constant
magnitude as mvr. But with respect to B, we get constant magnitude but continuously changing
direction.

13. (4)
Sol. Mg sin R = (mk2 + mR2) 
Rg sin  g sin 
 = 2 a
k R 2
k2
1 2
R
2s 2s  k2 
t   1  
a gsin  R 2 
for least time, k should be least & we know k is least for solid sphere.

14. (2)
1 2 1 1
Sol. I   mv 2
2 2 2
1
I  mR 2
2
Body is solid cylinder
NUCLEUS-92, Rajeev Gandhi Nagar, Kota (Raj.) India 324005, Mob. 9358006181, 97831-97831 84
JEE MAIN 2021
15. (3)
Sol. I in both cases is about point of contact
Ring
1
mgh = I2
2
1 v2R
mgh =  2mR  2 2
2 R
vR = gh
Solid cylinder
1
mgh = I2
2
1 3 v2
mgh =  mR 2  C2
22 R
4gh
vC =
3
vR 3

vC 2
16. (1)
g sin 
Sol. a
I
1
mR 2
I ring  Isolid cylinder  Isolid sphere  a ring  a solid cylinder  a solid sphere  v ring  vsolid cylinder  vsolid sphere
17. (2)

Sol.

For no slipping v0 = R
Now vA = vB = v02   R 2 = 2v0  x = 2
18. (3)

Sol.

NUCLEUS-92, Rajeev Gandhi Nagar, Kota (Raj.) India 324005, Mob. 9358006181, 97831-97831 85
JEE MAIN 2021
19. (4)
1
MR 2
Sol. Ratio of moment of inertia = 2
1 2
mr
4
2R 2 R 2 2R 4
  4
r 2 r 2 r

20. (9)

Sol.

1 1
I1 (1 ) 2  I 2 (2 ) 2
2 2
2 2
 v  v
I1    I2  
 3R  R
I1 9

I2 1

21. (2)
Sol. Iz = Ix + Iy (using perpendicular axis theorem) & I = mk2 (K : radius of gyration)
so mKz2 = mKx2 + mKy2
Kz2 = Kx2 + Ky2
So, radius of gyration about axes x, y & z won't be same hence assertion A is not correct reason R
is correct statement (property of a rigid body)

22. (6)

Sol.

1 2 1 m 22 6g
by energy conservation mg  I   =
2 2 3
Speed v = r =  = 6g
v= 6 10  .6 = 6m/s

NUCLEUS-92, Rajeev Gandhi Nagar, Kota (Raj.) India 324005, Mob. 9358006181, 97831-97831 86
JEE MAIN 2021
23. (4)
Sol. Parallel axis theorem
I = ICM + Md2
2
Mr 2
 M  
L
I=
2 2
2
 0.2 2
 M 
0.8 
2.7 = M 
2  2 
 2 16 
2.7  M  
100 100 
M 15
M = 15kg    2  = 0.1492 × 103
r L (0.2)  0.8
2

24. (52)

Sol.

 5   
2
 13r    M(36r ) M(25r )   Mr 169Mr 
2 2 2 2 2
2
I   I1  M  r     I 2  M       
    = 52 Mr
 2
     2    12 4   2 4 
25. (3)
Sol. From conservation of angular momentum we get
I11  I22  (I1  I2 )
I11  I22

I1  I2
1 1
ki  I112  I 2 22
2 2
1
k f  (I1  I 2 )2
2
1 2 (I11  I22 )2 
ki  k f  I  
1 1 2 2I 2
 
2 I1  I2 
Solving above we get
1 I I 
k i  k f   1 2  (1  2 )2
2  I1  I2 
NUCLEUS-92, Rajeev Gandhi Nagar, Kota (Raj.) India 324005, Mob. 9358006181, 97831-97831 87
JEE MAIN 2021
26. (4)
Sol. According to perpendicular Axis theorem.
Ix  i y  Iz

m 2 m 2 2m 2
Iz   
3 3 3

27. (3)

Sol.

5
M = 1.5 kg, r = 0.5 m, d  m
2
2  2
I  2  Mr 2  Md 2  = 19.05 kgm
5 

28. (4)
Sol. We know,
WL
 
2AY
10 1
  100 104  2 1011
25
1
   109  5  10 10 m
2

29. (2)

Sol.

L  mvr
And direction will be upward & remain constant

NUCLEUS-92, Rajeev Gandhi Nagar, Kota (Raj.) India 324005, Mob. 9358006181, 97831-97831 88
JEE MAIN 2021
WORK POWER ENERGY
1. The potential energy (U) of a diatomic molecule is a function dependent on r (interatomic
distance) as [JEE MAIN 2021 (FEB)]
 
U  10  5  3
r r
where,  and  and are positive constants. The equilibrium distance between two atoms will be
a
 2  b
  , where a = ________.
  

2. A boy is rolling a 0.5kg ball on the frictionless floor with the speed of 20ms –1. The ball gets
deflected by an obstacle on the way. After deflection it moves with 5% of its initial kinetic energy.
What is the speed of the ball now? [JEE MAIN 2021 (MARCH)]
–1 –1 –1
(1) 19.0 ms (2) 4.47 ms (3) 14.41 ms (4) 1.00 ms–1

3. A constant power delivering machine has towed a box, which was initially at rest, along a
horizontal straight line. The distance move by the box in time't' is proportional to:-
[JEE MAIN 2021 (MARCH)]
2/3 3/2
(1) t (2) t (3) t (4) t1/2

4. As shown in the figure, a particle of mass 10kg is placed at a point A. When the particle is slightly
displaced to its right, its starts moving and reaches the point B. The speed of the particle at B is x
m/s. (Take g = 10 m/s2) [JEE MAIN 2021 (MARCH)]
The value of 'x' to the nearest integer is ___________.

5. In a spring gun having spring constant 100 N/m a small ball 'B' of mass 100 g is put in its barrel
(as shown in figure) by compressing the spring through 0.05 m. There should be a box placed at a
distance 'd' on the ground so that the ball falls in it. If the ball leaves the gun horizontally at a
height of 2 m above the ground. The value of d is ____ m.(g = 10 m/s2 )
[JEE MAIN 2021 (JULY)]

NUCLEUS-92, Rajeev Gandhi Nagar, Kota (Raj.) India 324005, Mob. 9358006181, 97831-97831 89
JEE MAIN 2021
6. If the kinetic energy of a moving body becomes four times its initial Kinetic energy, then the
percentage change in its momentum will be: [JEE MAIN 2021 (JULY)]
(1) 100% (2) 200% (3) 300% (4) 400%

7. A body at rest is moved along a horizontal straight line by a machine delivering a constant power.
The distance moved by the body in time 't' is proportional to : [JEE MAIN 2021 (JULY)]
3 1 1 3
(1) t 2 (2) t 2 (3) t 4 (4) t 4

8. A porter lifts a heavy suitcase of mass 80 kg and at the destination lowers it down by a distance of
80 cm with a constant velocity. Calculate the workdone by the porter in lowering the suitcase.
(take g = 9.8 ms–2) [JEE MAIN 2021 (JULY)]
(1) –62720.0 J (2) –627.2 J (3) +627.2 J (4) 784.0 J

9. A pendulum bob has a speed of 2 m/s at its lowest position. The pendulum is 50 cm long. The
speed position. The pendulum is 50 cm long. The speed of bob when the length makes an angle of
60° to the vertical will be  g  10 m / s2  _______m/s. [JEE MAIN 2021 (JULY)]

10. A force of F = (5y +20) ĵ N acts on a particle. The work done by this force when the particle is
moved from y = 0 m to y = 10 m is ____J. [JEE MAIN 2021 (JULY)]

11. Given below is the plot of a potential energy function U(x) for a system , in which a particle is in
one dimensional motion, while a conservativ e force F(x) acts on it. Suppose that Emech = 8 J, the
incorrect statement for this system is : [JEE MAIN 2021 (JULY)]

[Where K.E. = kinetic energy]


(1) at X > X4, K.E. is constant throughout the region.
(2) at X < X1, K.E. is smallest and the particle is moving at the slowest speed.
(3) at X = X2, K.E. is greatest and the particle is moving at the fastest speed.
(4) at X = X3, K.E. = 4 J.

12. A body of mass 'm' dropped from a height 'h' reaches the ground with a speed of 0.8 gh . The
value of workdone by the air-friction is : [JEE MAIN 2021 (AUGUST)]
(1) –0.68 mgh (2) mgh (3) 1.64 mgh (4) 0.64 mgh

NUCLEUS-92, Rajeev Gandhi Nagar, Kota (Raj.) India 324005, Mob. 9358006181, 97831-97831 90
JEE MAIN 2021
13. An engine is attached to a wagon through a shock absorber of length 1.5 m. The system with a
total mass of 40,000 kg is moving with a speed of 72 kmh–1 when the brakes are applied to bring it
to rest. In the process of the system being brought to rest, the spring of the shock absorber gets
compressed by 1.0 m. If 90% of energy of the wagon is lost due to friction, the spring constant is
............. × 105 N/m. [JEE MAIN 2021 (AUGUST)]

14. A uniform chain of length 3 meter and mass 3 kg overhangs a smooth table with 2 meter lying on
the table . If k is the kinetic energy of the chain in joule as it completely slips off the table, then the
value of k is ........(Take g = 10 m/s2) [JEE MAIN 2021 (AUGUST)]

15. Two persons A and B perform same amount of work in moving a body through a certain distance
d with application of forces acting at angle 45° and 62° with the direction of displacement
1
respectively. The ratio of force applied by person A to the force applied by person B is . The
x
value of x is ...... . [JEE MAIN 2021 (AUGUST)]

ANSWER KEY
1. 1 2. 2 3. 2 4. 10 5. 1 6. 1 7. 1
8. 2 9. 50 10. 450 11. 2 12. 1 13. 16 14. 40
15. 2

NUCLEUS-92, Rajeev Gandhi Nagar, Kota (Raj.) India 324005, Mob. 9358006181, 97831-97831 91
JEE MAIN 2021
SOLUTION
1. (1)
Sol. For equilibrium
dU
0
dr
10 5
 6 0
r11 r
5 10
 11
r6 r
2
r5 

1
 2  5
r  
  
a=1

2. (2)
Sol. Given, m = 0.5kg and u = 20 m/s
1 1
Initial kinetic energy (ki) = mu 2   0.5  20  20  100J
2 2
After deflection it moves with 5% of ki
5 5
 kf   ki   100  k f  5 J
100 100
Now, let the final speed be 'v' m/s, then :
1
k f  5  mv 2  v2  20  v  20  4.47m / s
2
3. (2)
Sol. P=C
FV = C
dV
M VC
dt
V2
 t1/2
2
dx
 t1/ 2
dx
x  t 3/2
4. (10)
Sol. Using work energy theorem,
Wg = K.E
1
(10) (g) (5) = 10  v 2 – 0
2
v = 10 m/s

NUCLEUS-92, Rajeev Gandhi Nagar, Kota (Raj.) India 324005, Mob. 9358006181, 97831-97831 92
JEE MAIN 2021
5. (1)

Sol.

1 2 1
kx  mv 2
2 2
Kx  mv2
2

k 100
vx  0.05  0.05 10 10
m 0.1
v  0.5 10
1
From h  gt 2
2
2h 2 2 2
t  
g 10 10
2
 d  vt  0.5 10   1m
10
6. (1)
Sol. K2 = 4K1
1 1
mv 22  4 mv12
2 2
v2 = 2v1
P = mv P2 = mv2 = 2mv1
P1 = mv1
ΔP 2mv1  mv1
% change  100  100  100%
P1 mv1
7. (1)
Sol. P = constant
1
mv 2  Pt
2
dx
C t C = constant
dt
by integration.
1
1
t2
x=C
1
1
2
x  t3/2

NUCLEUS-92, Rajeev Gandhi Nagar, Kota (Raj.) India 324005, Mob. 9358006181, 97831-97831 93
JEE MAIN 2021
8. (2)
Sol. Wporter + Wmg = K.E. = 0
WPorter = –Wmg = –mgh
= –80 × 9.8 × .8 = –627.2 J

9. (50)

Sol.

Applying work energy theorem :


w g  w T  K

1 1
 mg11  cos 60   mv 2  mu 2
2 2
v2  u 2  2gl 1  cos 60 
1
v2  9  2 10  0.5  
2
v2 = 4
v = 2 m/s

10. (450)
Sol. F  (5y  20) ĵ
10 10
 5y 2  5
  Fdy   (5y  20)dy    20y    100  20  10 = 250 + 200 = 450 J
0  2 0 2
11. (2)
Sol. Emech = 8J
(A) at x > x4, U = constant = 6J
K = Emech – U = 2J = constant
(B) at x < x1, U = constant = 8J
K = Emech – U = 8 – 8 = 0 J
Particle is at rest
(C) At x = x2, U = 0  Emech = K = 8 J
KE is greatest, and particle is moving at fastest speed
(D) At x = x3. U=4J
U+K=8J
K=4J

NUCLEUS-92, Rajeev Gandhi Nagar, Kota (Raj.) India 324005, Mob. 9358006181, 97831-97831 94
JEE MAIN 2021
12. (1)
Sol. Work done = Change in kinetic Energy
1 1
 
2
Wmg + Wair-friction = m .8 gh  m  0 
2

2 2
.64
Wair-friction = mgh-mgh = –0.68 mgh
2
Option (1)

13. (16)
Sol. Work = K.E.
1
Wfriction  Wspring  0  mv 2
2
90  1 2 1
  mv   WSpring   mv
2

100  2  2
10 1
Wspring    mv 2
100 2
1 1 40000  (20) 2
 kx 2   mv 2  k   16 105
2 20 10  (1) 2

14. (40)

Sol.

From energy conservation


Ki + Ui = kf + Uf
 1  3
0   110    k f   3 10  
 2  2
5  k f  45
kf = 40 J
15. (2)
Sol. Given WA = WB
FAdcos 45° = FBdcos60°
1 1
FA   FB 
2 2
FA 2 1
 
FB 2 2
x2

NUCLEUS-92, Rajeev Gandhi Nagar, Kota (Raj.) India 324005, Mob. 9358006181, 97831-97831 95
JEE MAIN 2021
KTG & THERMODYNAMICS
1. n mole a perfect gas undergoes a cyclic process ABCA (see figure) consisting of the following
processes. [JEE MAIN 2021 (FEB)]
A  B : Isothermal expansion at temperature T so that the volume is doubled from V1 to V2 = 2V1
and pressure changes from P1 to P2.
B  C : Isobaric compression at pressure P2 to initial volume V1.
C  A : Isochoric change leading to change of pressure from P2 to P1.
Total workdone in the complete cycle ABCA is :

(2) nRT  ln 2   (4) nRT  ln 2  


1 1
(1) 0 (3) nRTln2
 2  2

2. On the basis of kinetic theory of gases, the gas exerts pressure because its molecules :
(1) continuously lose their energy till it reaches wall. [JEE MAIN 2021 (FEB)]
(2) are attracted by the walls of container.
(3) continuously stick to the walls of container.
(4) suffer change in momentum when impinge on the walls of container.
3. If one mole of an ideal gas at (P1, V1) is allowed to expand reversibly and isothermally (A to B) its
pressure is reduced to one-half of the original pressure (see figure). This is followed by a constant
volume cooling till its pressure is reduced to one-fourth of the initial value (B  C). Then it is
restored to its initial state by a reversible adiabatic compression (C to A). The net workdone by the
gas is equal to : [JEE MAIN 2021 (FEB)]

 1  RT
(1) RT  ln 2  (2) 
 2    1  2    1
(3) 0 (4) RT ln 2
4. The root mean square speed of molecules of a given mass of a gas at 27°C and 1 atmosphere
pressure is 200 ms–1. The root mean square speed of molecules of the gas at 127°C and 2
x
atmosphere pressure is ms 1 . The value of x will be ______ [JEE MAIN 2021 (FEB)]
3
NUCLEUS-92, Rajeev Gandhi Nagar, Kota (Raj.) India 324005, Mob. 9358006181, 97831-97831 96
JEE MAIN 2021
7 5
5. A diatomic gas, having C p  R and C v  R is heated at constant pressure. The ratio dU : dQ :
2 2
dW : [JEE MAIN 2021 (FEB)]
(1) 5 : 7 : 3 (2) 5 : 7 : 2 (3) 3 : 7 : 2 (4) 3 : 5 : 2

6. In a certain thermodynamical process, the pressure of a gas depends on its volume as kV3.
The work done when the temperature changes from 100°C to 300°C will be ___ nR, where n
denotes number of moles of a gas. [JEE MAIN 2021 (FEB)]

7. A monoatomic gas of mass 4.0 u is kept in an insulated container. Container is moving with
velocity 30 m/s. If container is suddenly stopped then change in temperature of the gas
x
(R = gas constant) is . Value of x is _______ [JEE MAIN 2021 (FEB)]
3R

8. Thermodynamic process is shown below on a P-V diagram for one mole of an ideal gas. If
V2 = 2V1 then the ratio of temperature T2/T1 is : [JEE MAIN 2021 (FEB)]

1 1
(1) (2) 2 (3) 2 (4)
2 2

9. Given below are two statements : [JEE MAIN 2021 (FEB)]


Statement I : In a diatomic molecule, the rotational energy at a given temperature obeys Maxwell's
distribution.
Statement II : In a diatomic molecule, the rotational energy at a given temperature equals the
translational kinetic energy for each molecule.
In the light of the above statements, choose the correct answer from the options given below :
(1) Statement I is false but Statement II is true.
(2) Both Statement I and Statement II are false.
(3) Both Statement I and Statement II are true.
(4) Statement I is true but Statement II is false.

10. A container is divided into two chambers by a partition. The volume of first chamber is 4.5 litre
and second chamber is 5.5 litre. The first chamber contain 3.0 moles of gas at pressure 2.0 atm and
second chamber contain 4.0 moles of gas at pressure 3.0 atm. After the partition is removed and
the mixture attains equilibrium, then, the common equilibrium pressure existing in the mixture is x
× 10–1 atm. Value of x is_. [JEE MAIN 2021 (FEB)]

NUCLEUS-92, Rajeev Gandhi Nagar, Kota (Raj.) India 324005, Mob. 9358006181, 97831-97831 97
JEE MAIN 2021
11. Match List-I with List-II : [JEE MAIN 2021 (FEB)]
List-I List-II
(a) Isothermal (i) Pressure constant
(b) Isochoric (ii) Temperature constant
(c) Adiabatic (iii) Volume constant
(d) Isobaric (iv) Heat content is constant
Choose the correct answer from the options given below :
(1) (a) (i), (b)  (iii), (c)  (ii), (d)  (iv)
(2) (a) (ii), (b)  (iii), (c)  (iv), (d)  (i)
(3) (a) (ii), (b)  (iv), (c)  (iii), (d)  (i)
(4) (a) (iii), (b)  (ii), (c)  (i), (d)  (iv)

12. The internal energy (U), pressure (P) and volume (V) of an ideal gas are related as U = 3PV + 4.
The gas is :- [JEE MAIN 2021 (FEB)]
(1) Diatomic only (2) Polyatomic only
(3) Either monoatomic or diatomic (4) Monoatomic only

13. The volume V of a given mass of monoatomic gas changes with temperature T according to the
relation V = KT2/3. The workdone when temperature changes by 90 K will be xR. The value of x
is [R = universal gas constant] [JEE MAIN 2021 (FEB)]

14. 1 mole of rigid diatomic gas performs a work of Q/5 when heat Q is supplied to it. The molar heat
xR
capacity of the gas during this transformation is , The value of x is …………….
8
[K = universal gas constant] [JEE MAIN 2021 (FEB)]

15. The volume V of an enclosure contains a mixture of three gases, 16 g of oxygen, 28 g of nitrogen
and 44 g of carbon dioxide at absolute temperature T. Consider R as universal gas constant. The
pressure of the mixture of gases is: [JEE MAIN 2021 (MARCH)]
88RT 3RT 5RT 4RT
(1) (2) (3) (4)
V V V V

16. In thermodynamics, heat and work are : [JEE MAIN 2021 (MARCH)]
(1) Path functions (2) Intensive thermodynamic state variables
(3) Extensive thermodynamic state variables (4) Point functions

17. Calculate the value of mean free parth () for oxygen molecules at temperature 27°C and pressure
1.01 × 105 Pa. Assume the molecular diameter 0.3 nm and the gas is ideal.
(k = 1.38 × 10–23 JK–1) [JEE MAIN 2021 (MARCH)]
(1) 58nm (2) 32nm (3) 86 nm (4) 102 nm
18. For an ideal heat engine, the temperature of the source is 127°C. In order to have 60% efficiency
the temperature of the sink should be _ _ _ _ _ _ _ °C. (Round off to the Nearest Integer)
[JEE MAIN 2021 (MARCH)]
19. A polyatomic ideal gas has 24 vibrational modes. What is the value of  ?
[JEE MAIN 2021 (MARCH)]
(1) 1.03 (2) 1.30 (3) 1.37 (4) 10.3
NUCLEUS-92, Rajeev Gandhi Nagar, Kota (Raj.) India 324005, Mob. 9358006181, 97831-97831 98
JEE MAIN 2021
20. A Carnot's engine working between 400k and 800K has a work output of 1200J per cycle. The
amount of heat energy supplied to the engine from the source in each cycle is:
[JEE MAIN 2021 (MARCH)]
(1) 3200 J (2) 1800 J
(3) 1600 J (4) 2400 J

21. Two ideal polyatomic gases at temperatures T1 and T2 are mixed so that there is no loss of energy.
If F1 and F2, m1 and m2, n1 and n2 be the degrees of freedom, masses, number of molecules of the
first and second gas respectively, the temperature of mixture of these two gases is:
[JEE MAIN 2021 (MARCH)]
n T n T n FT  n F T
(1) 1 1 2 2 (2) 1 1 1 2 2 2
n1  n 2 n1F1  n 2 F2
n FT  n F T n FT  n F T
(3) 1 1 1 2 2 2 (4) 1 1 1 2 2 2
F1  F2 n1  n 2

22. If one mole of the polyatomic gas is having two vibration modes and  is the ratio of molar
 C 
specific heats for polyatomic gas    P  then the value of  is : [JEE MAIN 2021 (MARCH)]
 Cv 
(1) 1.02 (2) 1.2 (3) 1.25 (4) 1.35

23. Which one is the correct option for the two different thermodynamic processes?
[JEE MAIN 2021 (MARCH)]

(A) (B)

(C) (D)

(1) (c) and (a) (2) (c) and (d) (3) (a) only (4) (b) and (c)

24. What will be the average value of energy along one degree of freedom for an ideal gas in thermal
equilibrium at a temperature T? (kB is Boltzmann constant) [JEE MAIN 2021 (MARCH)]
1 2
(1) k B T (2) k B T
2 3
3
(3) k B T (4) k BT
2

NUCLEUS-92, Rajeev Gandhi Nagar, Kota (Raj.) India 324005, Mob. 9358006181, 97831-97831 99
JEE MAIN 2021
25. The P-V diagram of a diatomic ideal gas system going under cyclic process as shown in figure
The work done during an adiabatic process CD is (use  = 1.4): [JEE MAIN 2021 (MARCH)]

(1) –500 J (2) –400 J (3) 400 J (4) 200 J

26. For an adiabatic expansion of an ideal gas, the fractional change in its pressure is equal to (where 
is the ratio of specific heats): [JEE MAIN 2021 (MARCH)]
dV V  dv dv
(1) –  (2) –  (3) – (4)
V dV  V V

27. An ideal gas in a cylinder is separated by a piston in such a way that the entropy of one part is S1
and that of the other part is S2. Given that S1> S2. If the piston is removed then the total entropy of
the system will be: [JEE MAIN 2021 (MARCH)]
S1
(1) S1 × S2 (2) S1 – S2 (3) (4) S1 + S2
S2

28. Consider a sample of oxygen behaving like an ideal gas. At 300 K, the ratio of root mean square
(rms) velocity to the average velocity of gas molecule would be: [JEE MAIN 2021 (MARCH)]
(Molecular weight of oxygen is 32 g/mol; R = 8.3 J K–1 mol–1)
3 8 3 8
(1) (2) (3) (4)
3 3 8 8

29. The amount of heat needed to raise the temperature of 4 moles of a rigid diatomic gas from 00C to
500C when no work is done is _________ (R is the universal gas constant)
[JEE MAIN 2021 (JULY)]
(1) 250 R (2) 750 R (3) 175 R (4) 500 R

30. A heat engine has an efficiency of . When the temperature of sink is reduced by 620C, its
6
efficiency gets doubled . The temperature of the source is : [JEE MAIN 2021 (JULY)]
(1) 1240C (2) 370 C (3) 620 C (4) 990 C

31. Consider a mixture of gas molecule of types A, B and C having masses m A< mB< mC. The ratio of
their root mean square speed at normal temperature and pressure is :
[JEE MAIN 2021 (JULY)]
1 1 1 1 1 1
(1) v A  v B  vC  0 (2)   (3) vA  vB  vC (4)  
vA vB vC vA vB vC

NUCLEUS-92, Rajeev Gandhi Nagar, Kota (Raj.) India 324005, Mob. 9358006181, 97831-97831 100
JEE MAIN 2021
32. Which of the following graphs represent the behaviour of an ideal gas ? Symbols have their usual
meaning . [JEE MAIN 2021 (JULY)]

(1) (2)

(3) (4)

33. The correct relation between the degree of freedom  and the ratio of specific heat  is :
[JEE MAIN 2021 (JULY)]
2 2  1 1
(1)   (2)   (3)   (4)  
 1  1 2  1

34. What will be the average value of energy for amonoatomic gas in thermal equilibrium
attemperature T ? [JEE MAIN 2021 (JULY)]
2 3 1
(1) k B T (2) kBT (3) k B T (4) k B T
3 2 2

35. For a gas CP  CV  R in a state P and CP  CV  1.10 R in a state Q, TPA and TQ are the
temperatures in two diffllerent states; P and Q respectively. Then [JEE MAIN 2021 (JULY)]
(1) TP = TQ (2) TP< TQ (3) TP = 0.9 TQ (4) TP> TQ

36. A system consist of two types of gas molecules A and B having same number density 2 × 10 25
/m3. The diameter of A and B are 10 Å and 5 Årespectively. They suffer collision at room
temperature. The ratio of average distance covered by the molecule A to that of B between two
successive collision is______ × 10–2. [JEE MAIN 2021 (JULY)]

37. The number of molecules in one litre of an ideal gas at 300 K and 2 atmospheric pressure with
mean kinetic energy 2 × 10–9J per molecules is : [JEE MAIN 2021 (JULY)]
11 11 11
(1) 0.75 × 10 (2) 3 × 10 (3) 1.5 × 10 (4) 6 × 1011

38. Two carnot engines A and B operate in series such that engine A absorbs heat at T1 and rejects
heat to a sink at temperature T. Engine B absorbs half of the heat rejected by engine A and reject
heat to the sink at T3. When workdone in both the case is equal, the value of T is :
[JEE MAIN 2021 (JULY)]
2 3 1 2 3 1 2 1
(1) T1  T3 (2) T1  T3 (3) T1  T3 (4) T1  T3
3 2 3 3 2 3 3 3

NUCLEUS-92, Rajeev Gandhi Nagar, Kota (Raj.) India 324005, Mob. 9358006181, 97831-97831 101
JEE MAIN 2021
39. A monatomic ideal gas, initially at temperature T1 is enclosed in a cylinder fitted with a
frictionless piston. The gas is allowed to expand adiabatically to a temperature T2 by releasing the
position suddenly. If 1 and  2 are the lengths of the gas column, before and after the expansion
T
respectively, then value of 1 will be : [JEE MAIN 2021 (JULY)]
T2
2 2
  3   3 2 1
(1)  1  (2)  2  (3) (4)
 2   1  1 2

40. Two different metal bodies A and B of equal mass are heated at a uniform rate under similar
conditions. The variation of temperature of the bodies is graphically represented as shown in the
figure. The ratio of specific heat capacities is : [JEE MAIN 2021 (JULY)]

8 3 3 4
(1) (2) (3) (4)
3 8 4 3
41. In the reported figure, there is a cyclic process ABCDA on a sample of 1 mol of a diatomic gas.
The temperature of the gas during the process A B and C D are T1 and T2 (T1 > T2)
respectively. [JEE MAIN 2021 (JULY)]

Choose the correct option out of the following for work done if processes BC and DA are
adiabatic.
(1) WAB = WDC (2) WAD = WBC (3) WBC + WDA> 0 (4) WAB< WCD
42. The temperature of 3.00 mol of an ideal diatomicgas is increased by 40.0 °C without changing the
pressure of the gas. The molecules in the gas rotatebut do not oscillate. If the ratio of change in
x
internal energy of the gas to the amount of work done by the gas is . Then the value of x (round
10
off to the nearest integer) is _______. [JEE MAIN 2021 (AUGUST)]
(Given R = 8.31 J mol–1 K–1)
NUCLEUS-92, Rajeev Gandhi Nagar, Kota (Raj.) India 324005, Mob. 9358006181, 97831-97831 102
JEE MAIN 2021
43. The average translational kinetic energy of N2 gasmolecules at ...........°C becomes equal to the
K.E.of an electron accelerated from rest through apotential difference of 0.1 volt.
(Given kB = 1.38 × 10–23 J/K) [JEE MAIN 2021 (AUGUST)]
(Fill the nearest integer).

44. The temperature of an ideal gas in 3-dimensions is300 K. The corresponding de-Broglie
wavelengthof the electron approximately at 300 K, is : [JEE MAIN 2021 (AUGUST)]
–31
[me = mass of electron = 9 × 10 kg
h = Planck constant = 6.6 × 10–34 Js
kB = Boltzmann constant = 1.38 × 10–23 JK–1]
(1) 6.26 nm (2) 8.46 nm (3) 2.26 nm (4) 3.25 nm

45. A cylindrical container of volume 4.0 × 10–3 m3contains one mole of hydrogen and two moles
ofcarbon dioxide. Assume the temperature of themixture is 400 K. The pressure of the mixture
ofgases is : [JEE MAIN 2021 (AUGUST)]
–1 –1
[Take gas constant as 8.3 J mol K ]
(1) 249 × 101 Pa (2) 24.9 × 103 Pa (3) 24.9 × 105 Pa (4) 24.9 Pa

46. The rms speed of the molecules of Hydrogen, Oxygen and carbon dioxide at the same temperature
are VH , VO and VC respectively then : [JEE MAIN 2021 (AUGUST)]
(1) 263 J/s (2) 298 J/s (3) 350 J/s (4) 35 J/s

47. The rms speed of the molecules of Hydrogen, Oxygen and carbondioxide at the same temperature
are VH , VO and VC respectively then : [JEE MAIN 2021 (AUGUST)]
(1) VH> VO> VC (2) VC> VO> VH (3) VH = VO> VC (4) VH = VO = VC

48. If the rms speed of oxygen molecules at 0°C is 160 m/s, find the rms speed of hydrogen molecules
at 0°C. [JEE MAIN 2021 (AUGUST)]
(1) 640 m/s (2) 40 m/s (3) 80 m/s (4) 332 m/s

49. A heat engine operates between a cold reservoir at temperature T2 = 400 K and a hot reservoir at
temperature T1.It takes 300 J of heat from the hot reservoir and delivers 240 J of heat to the cold
reservoir in a cycle. The minimum temperature of the hot reservoir has to be _________K.
[JEE MAIN 2021 (AUGUST)]

50. The height of victoria falls is 63 m. What is the difference in temperature of water at the top and
at the bottom of fall ? [JEE MAIN 2021 (AUGUST)]
-1o -1
[Given 1 cal = 4.2 J and specific heat of water = 1 cal g C ]
(1) 0.147o C (2) 14.76o C (3) 1.476o (4) 0.014o C

51. A balloon carries a total load of 185 kg at normal pressure and temperature of 27° C. What load
will the balloon carry on rising to a height at which the barometric pressure is 45 cm of Hg and the
temperature is -7°C. Assuming the volume constant ? [JEE MAIN 2021 (AUGUST)]
(1) 181.46 kg (2) 214.15 kg (3) 219.07 kg (4) 123.54 kg

NUCLEUS-92, Rajeev Gandhi Nagar, Kota (Raj.) India 324005, Mob. 9358006181, 97831-97831 103
JEE MAIN 2021
3
52. An ideal gas is expanding such that PT = constant. The coefficient of volume expansion of the
gas is : [JEE MAIN 2021 (AUGUST)]
1 2 4 3
(1) (2) (3) (4)
T T T T

53. A mixture of hydrogen and oxygen has volume 500cm3 , temperature 300 K, pressure 400 kPa and
mass 0.76 g. The ratio of masses of oxygen to hydrogen will be :-
[JEE MAIN 2021 (AUGUST)]
(1) 3 : 8 (2) 3 : 16 (3) 16 : 3 (4) 8 : 3

54. A sample of gas with  = 1.5 is taken through an adiabatic process in which the volume is
compressed from 1200 cm3 to 300 cm3. If the initial pressure is 200 kPa. The absolute value of the
work done by the gas in the process = __________J. [JEE MAIN 2021 (AUGUST)]

1
55. A reversible engine has an efficiency of . If the temperature of the sink is reduced by 58°C, its
4
efficiency becomes double. Calculate the temperature of the sink :
[JEE MAIN 2021 (AUGUST)]
(1) 174° C (2) 280°C (3) 180.4°C (4) 382°C

56. For an ideal gas the instantaneous change in pressure 'p' with volume 'v' is given by the equation
dp
 ap If p = p0 at v = 0 is the given boundary condition, then the maximum temperature one
dv
mole of gas can attain is : [JEE MAIN 2021 (AUGUST)]
(Here R is the gas constant )
p ap
(1) 0 (2) 0 (3) infinity (4) 0°C
aeR eR

57. Consider two separate ideal gases of electrons and protons having same number of particles. The
temperature of both the gases are same. The ratio of the uncertainty in determining the position of
an electron to that of a proportional to :- [JEE MAIN 2021 (AUGUST)]
3/2
 mp  me mp mp
(1)   (2) (3) (4)
 me  mp me me

ANSWER KEY
1. 4 2. 4 3. 1 4. 400 5. 2 6. 50 7. 3600
8. 3 9. 4 10. 25.50 11. 2 12. 2 13. 60 14. 25
15. 3 16. 1 17. 4 18. 113 19. 1 20. 4 21. 2
22. 2 23. 2 24. 1 25. 1 26. 1 27. 4 28. 3
29. 4 30. 4 31. 4 32. 3 33. 1 34. 3 35. 4
36. 25 37. 3 38. 4 39. 2 40. 2 41. 2 42. 25
43. 500 44. 1 45. 3 46. 1 47. 1 48. 1 49. 500
50. 1 51. 4 52. 3 53. 3 54. 480 55. 1 56. 1
57. 3
NUCLEUS-92, Rajeev Gandhi Nagar, Kota (Raj.) India 324005, Mob. 9358006181, 97831-97831 104
JEE MAIN 2021
SOLUTION
1. (4)
v 
Sol. Wisothermal = nRT ln  2 
 v1 
WIsobaric = PV = nRT
WIsochoric = 0

W1 = nRT ln 
 2V  = nRT ln2

 V 
T
W2 = nR   T   nR
T
2  2
W3 = 0
 Wnet = W1 + W2 + W3
Wnet = nRT  ln 2  
1
 2
2. (4)
Sol. From the assumption of KTG, the molecules of gas collide with the walls and suffers momentum
change which results in force on the wall and hence pressure. Hence option (4) is correct

3. (1)
Sol. A – B = isothermal process
 2V 
WAB = P1V1 ln  1  = P1V1 ln(2)
 V1 
B – C  Isochoric process
WBC = 0
C – A  Adiabatic process
P  1
P1V1  1  2V1 P1V1 1  
WCA = 4   2   P1V1
1  1  2 1   
Wnet = WAB + WBC + WCA {P1V1 = RT}
P1V1
= P1V1ln(2) + 0 +
2 1  
 1 
Wnet = RT ln(2) 
 2    1 
Option (1) is correct.

NUCLEUS-92, Rajeev Gandhi Nagar, Kota (Raj.) India 324005, Mob. 9358006181, 97831-97831 105
JEE MAIN 2021
4. (400)
3RT
Sol. v rms 
M
vrms  T
 Vrms 2 T2

 Vrms 1 T1
400

300
2

3
2
 vrms 2   vrms 1
3
2
  200
3
400
 vrms 2  m/s
3
Ans. 400

5. (2)
Sol. dU = nCvdT
dQ = nCpdT
dW = PdV = nRdT (isobaric process)
5R 7R
dU : dQ : dW : Cv : Cp : R  : : 5 : 7 : 2
2 2

6. (50)
Sol. P = kV3
Ti = 100°C & Tf = 300°C
T = 300 – 100
T = 200°C
P = kV3
now PV = nRT
 kV4 = nRT
now 4kV3 dV = nRdT
 PdV = nRdT/4
nRdT nR 200
 Work   PdV    T   nR  50nR
4 4 4

7. (3600)
Sol. Given that mass of gas is 4u hence its molar mass M is 4g/mol
1
 mv 2  nC v T
2

NUCLEUS-92, Rajeev Gandhi Nagar, Kota (Raj.) India 324005, Mob. 9358006181, 97831-97831 106
JEE MAIN 2021
1 m 3R
m   30   
2
 T
2 M 2
3600
 T 
3R

8. (3)

Sol.

PV½ = c
nRT 1/ 2
V c
V
T = c1 V1/2
1/2 1/2
T2  V2   2V 
   1
T1  V1   V1 
T2
 2
T1
9. (4)
Sol. (4) Translational degree of freedom = 3
Rotational degree of freedom = 2

10. (25.50)
Sol. Let common equilibrium pressure of mixture is P atm. then
U1  U 2  U mixture
f f f
P1V1  P2 V2  P  V1  V2 
2 2 2
f f f
 2  4.5    3 5.5   P  4.5  5.5   P = 2.55 = x × 10–1 atmp
2 2 2
So x = 25.5  26 (Nearest integer)

11. (2)
Sol. (a) Isothermal  Temperature constant
(a)  (ii)
(b) Isochoric  Volume constant
(a)  (iii)
(c) Adiabatic  DQ = 0
 Heat content is constant
(c)  (iv)
(d) Isobaric  Pressure constant
(d)  (i)

NUCLEUS-92, Rajeev Gandhi Nagar, Kota (Raj.) India 324005, Mob. 9358006181, 97831-97831 107
JEE MAIN 2021
12. (2)
Sol. U  3PV  4
nf
RT  3PV  4
2
f
PV  3PV  4
2
8
f  6
PV
Since degree of freedom is more than 6 therefore gas is polyatomic
13. (60)
Sol. We know that work done is
W   PdV ……….(1)
nRT
P  ………..(2)
V
nRT
W   dv ………..(3)
V
and V = Kt 2/3 ………..(4)
nRT
w .dv ………..(5)
KT 2/3
T2
2
 from (4) : dv  KT 1/3 dT  W  d  nRT
2/3
2 1
K 1/3 dt
3 T1
KT 3 T

2
W nR   T2  T1  ……….. (6)
3
 T2  T1  90K ……….. (7)
2
 W  nR  90  W = 60 nR
3
Assuming 1 mole of gas
n=1
So W = 60 R
14. (25)
Sol. Q  U  W
Q
Q  U 
5
4Q
U 
5
4
nC  T  nCT
5
5
C  C
4
5f  55
C   R   R
42 42

NUCLEUS-92, Rajeev Gandhi Nagar, Kota (Raj.) India 324005, Mob. 9358006181, 97831-97831 108
JEE MAIN 2021
28
C R
8
x = 25

15. (3)
Sol. PV = (n1 + n2 + n3)RT
 16 28 44 
P  V      RT
 32 28 44 
1 
PV    1  1 RT
2 
5 RT
P
2 V

16. (1)
Sol. Heat and work are treated as path functions in thermodynamics.
Q  U  W
Since work done by gas depends on type of process i.e. path and U depends just on initial and
final states, so Q i.e. heat, also has to depend on process i.e. path.

17. (4)
RT
Sol. 
2d 2 N A P
  102nm

18. (113)
TL
Sol. n = 0.60 = 1 =
TH
TL
 0.4  TL  0.4  400
TH
= 160K
= –113°C

19. (1)
Sol. Since each virbrational mode has 2 degrees of freedom hence total vibrational degrees of freedom
= 48
f = 3 + 3 + 48 = 54
2 28
=1+   1.03
f 27
20. (4)
T2 W
Sol. = 1 –
T1 Q
400 1200 1 1200
1–     Q = 2400 J
800 Q 2 Q

NUCLEUS-92, Rajeev Gandhi Nagar, Kota (Raj.) India 324005, Mob. 9358006181, 97831-97831 109
JEE MAIN 2021
21. (2)
Sol. Let the final temperature of the mixture be T. Since, there is no loss in energy.
U = 0
F F
 1 n1RT  2 n 2 RT  0
2 2
F1 F
 n1R  T1 – T   2 n 2 R  T2 – T   0
2 2
F n RT  F n RT Fn T  F n T
T 1 1 1 2 2 2  1 1 1 2 2 2
F1n1R  F2 n 2 R F1n1  F2 n 2

22. (2)
Sol. (2) f = 4 + 3 + 3 = 10
assuming non linear
C 2 12
  p  1   1.2
Cv ƒ 10

23. (2)
Sol. (2) option (a) is wrong; since in adiabatic process V  constant.
Option (b) is wrong, since in isothermal process T = constant
Option (c) & (d) matches isotherms & adiabatic formula:
T
TV  –1  cons tan t &  –1  cons tan t
p

24. (1)
1
Sol. Energy associated with each degree of freedom per molecule = k BT .
2

25. (1)
Sol. Adiabatic process is from C to D
P V – PV
WD  2 2 1 1
1– 
P V – PC VC
 D D
1– 
200  3 – 100  4 

1–1.4
= –500 J Ans. (1)

26. (1)
Sol. PV  cons tan t
Differentiating
dP P
–
dV V
dP dV
–
P V

NUCLEUS-92, Rajeev Gandhi Nagar, Kota (Raj.) India 324005, Mob. 9358006181, 97831-97831 110
JEE MAIN 2021
27. (4)

Sol.

After piston is removed

28. (3)
3RT
Sol. v rms 
M
8RT RT
v avg 
π M
vrms 3π

vavg 8

29. (4)
Sol. Q = U + W
Here W = 0
Q = U = nCVT
5R
Q  4   50   500R
2
Hence option (4).

30. (4)
TL
Sol.   1  (i)
TH

2  1 
 TL  62   1  TL  62 
62 1 62
   TH  6  62  372K
TH TH TH TH 6 TH
In °C  372 – 273 = 99°C

31. (4)
3RT
Sol. VRMS =
M
1 1 1
mA< mB< mC  VA< VB> VC   
VA VB VC

32. (3)
Sol. PV = nRT
PV T
Straight line with positive slope (nR)
NUCLEUS-92, Rajeev Gandhi Nagar, Kota (Raj.) India 324005, Mob. 9358006181, 97831-97831 111
JEE MAIN 2021
33. (1)
2
Sol.   1
f
2
f
 1

34. (3)
Sol. As per Equi-partition law :
Each degree of freedom contributes
1
k B T Average Energy
2
In monoatomic gas D.O.F. = 3
1 3
Average energy = 3  k BT  k BT
2 2

35. (4)
Sol. CP– CV = R for ideal gas and gas behaves as ideal gas at high temperature
so TP> TQ

36. (25)
Sol.  mean free path
1

2d 2 n
2
1 d 22 n 2  5 
 2     0.25  25 102
 2 d1 n1  10 

37. (3)
3
Sol. KE  kT
2
N
PV  RT
NA
PV
N
kT
= N = 1.5 × 1011

38. (4)

Sol.

Q2 T Q T
WA  1   1  2 
Q1 T1 Q1 T1

NUCLEUS-92, Rajeev Gandhi Nagar, Kota (Raj.) India 324005, Mob. 9358006181, 97831-97831 112
JEE MAIN 2021
Q3 T 2Q3 T3
WB  1   1 3  
(Q2 / 2) T Q2 T
Now , WA = WB
Q 2Q1 2Q3 2T T
Q1  Q 2  2  Q3   3  1  3 3
2 Q2 Q2 T T
2T1 T3
 T
3 3

39. (2)
Sol. PVr = const.
TVr–1 = const.
5
T   3 1  const.
2/3
T1  
 2

T2  
1 

40. (2)
 Q   Q 
Sol.    
 t A  t  B
 T   T 
mSA    mSB  
 t A  t  B
 T 
SA  t A 90 / 6 15 3
   
SB  T  120 / 3 40 8
 
 t B

41. (2)
nR
Sol. Work done in adiabatic process =  Tf  Ti 
 1
nR
 WAD   T2  T1 
 1
nR
and WBC   T2  T1 
 1
 WAD = WBC

42. (25)
Sol. Pressure is not changing  isobaric process
5nRT
U = nCvT =
2
and W = nRT
U 5 x
   x  25.00
W 2 10

NUCLEUS-92, Rajeev Gandhi Nagar, Kota (Raj.) India 324005, Mob. 9358006181, 97831-97831 113
JEE MAIN 2021
43. (500)
Sol. Given
Translation K.E. of N2 = K.E. of electron
3
kT = eV
2
3
× 1.38 × 10–23 T = 1.6 × 10–19 × 0.1
2
 T = 773k
T = 773 – 273 = 500°C

44. (1)
Sol. De-Broglie wavelength
h h
= 
mv 2mE
Where E is kinetic Energy
3kT
E= for gas
2
h 6.6 1034
 
3mkT 3  9 1031 1.38 1023  300
–9
 = 6.26 × 10 m = 6.26 nm
Option (1)

45. (3)
Sol. V = 4 × 10–3 m3
n = 3 moles
T = 400 K
nRT
PV = nRT  P =
V
3  8.3  400
P= 3
= 24.9 × 105 Pa
4  10

46. (1)
dH
TL
Sol.  C. O.P.  dt
TH  TL dW
dt
263 dH
 35 
35 dt
dH
 263 watts
dt

47. (1)
3RT
Sol. VRMS 
MW

NUCLEUS-92, Rajeev Gandhi Nagar, Kota (Raj.) India 324005, Mob. 9358006181, 97831-97831 114
JEE MAIN 2021
1
At the same temperature VRMS 
MW
 VH  VO  VC
Option (1)

48. (1)
3KT
Sol. Vrms 
M
(Vrms ) o2 M H2 2
 
(Vrms ) H 2 M o2 32
(Vrms ) H2  4  (Vrms )o2
 4  160 = 640 m/s

49. (500)
Sol. Qin = 300 J ; Qout = 240 J
Work done = Qin - Qout = 300 – 240 = 60 J
T
Efficiency = 1  2
t1
1 400 400 4
 1  
5 T1 T1 5
T1 = 500 k

50. (1)
Sol. Change in P.E. = Heat energy
gh 10  63
T   = 0.147o C
S 4200J / kgC
51. (4)
Sol. Pm  RT
P T
 1  1 1
P2 2 T2
1 P T  76  266
 1 2   
2 P2T1  45  300
l M 76  266
 1 
2 M 2 45  300
45  300 185
 M2   123.54kg
76  266

52. (3)
Sol. PT3 = constant
 nRT  3
  T  constant
 v 
NUCLEUS-92, Rajeev Gandhi Nagar, Kota (Raj.) India 324005, Mob. 9358006181, 97831-97831 115
JEE MAIN 2021
4 -1
T V = constant
T4 = kV
T V
4  ......(1)
T V
V  V2
comparing (1) and (2)
we get
4

T

53. (3)
Sol. PV = nRT
 25 
400 103  500 106  n   (300)
 3 
2
n
25
n = n1 + n2
2 M1 M 2
 
25 2 32
Also M1 + M2 = 0.76 gm
M 2 16

M1 3

54. (480)
Sol.   1.5
p1v1  p 2 v 2 
(200) (1200)1.5 = P2 (300)1.5
P2 = 200 [4]3/2 = 1600 kPa
p v  p v  480  240 
W.D.  2 2 1 1     480 J
 1  0.5 

55. (1)
Ans. by Nucleus(Bonus)
Sol. T2 = sink temperature
T
  1 2
T1
1 T
 1 2
4 T1
T2 3
 ...(i)
T1 4
1 T  58
 1 2
2 T1

NUCLEUS-92, Rajeev Gandhi Nagar, Kota (Raj.) India 324005, Mob. 9358006181, 97831-97831 116
JEE MAIN 2021
T2 58 1
 
T1 T1 2
3 58 1
 
4 T1 2
1 58
  T1  232
4 T1
3
T2   232
4
T2 = 174 K

56. (1)
p v
dp
Sol.  P  a 0 dv
p0

P
n    av
 P0 
p  P0e av
For temperature maximum p-v product should be maximum
pv p0 ve av
T 
nR R
dT p
dv
 
 0  0 e  av  ve  av (a)
R
 av
p0 e
1  av  0
R
1
v  ,
a
p1 p
T 0  0
Rae Rae
at v  
T=0
Option (1)

57. (3)
h
Sol. x.p 
4
h 3KT
x  v
4mv m
x e mp

x p me

NUCLEUS-92, Rajeev Gandhi Nagar, Kota (Raj.) India 324005, Mob. 9358006181, 97831-97831 117
JEE MAIN 2021

ELASTICITY & HEAT


1. Each side of a box made of metal sheet in cubic shape is 'a' at room temperature 'T', the coefficient
of linear expansion of the metal sheet is ''. The metal sheet is heated uniformly, by a small
temperature T, so that its new temperature is T + T. Calculate the increase in the volume of the
metal box. [JEE MAIN 2021 (FEB)]
4
(1) 3a3T (2) 4a3T (3) 4a3T (4) a3T
3

2. Given below are two statement : one is labeled as Assertion A and the other is labelled as Reason
R. [JEE MAIN 2021 (FEB)]
Assertion A : When a rod lying freely is heated, no thermal stress is developed in it.
Reason R : On heating the length of the rod increases.
In the light of the above statements, choose the correct answer from the options given below:
(1) Both A and R are true but R is NOT the correct explanation of A
(2) A is false but R is true
(3) A is true but R is false
(4) Both A and R are true and R is the correct explanation of A

3. The normal density of a material is  and its bulk modulus of elasticity is K. The magnitude of
increase in density of material, when a pressure P is applied uniformly on all sides, will be :
[JEE MAIN 2021 (FEB)]
K P K PK
(1) (2) (3) (4)
P K P 

4. The temperature  at the junction of two insulating sheets, having thermal resistances R1 and R2 as
well as top and bottom temperatures 1 and 2 (as shown in figure) is given by :
[JEE MAIN 2021 (FEB)]

2 R 2  1R1 1R 2  2 R1 1R 2  2 R1 1R1  2 R 2


(1) (2) (3) (4)
R 2  R1 R 2  R1 R1  R 2 R1  R 2

5. The length of metallic wire is 1 when tension in it is T1. It is 2 when the tension is T2. The
r original length of the wire will be - [JEE MAIN 2021 (FEB)]
 T T T2 1  T1 T T
(1) 1 2 (2) 2 1 1 2
(3) 2
(4) 1 1 2 2
2 T1  T2 T2  T1 T2  T1

NUCLEUS-92, Rajeev Gandhi Nagar, Kota (Raj.) India 324005, Mob. 9358006181, 97831-97831 118
JEE MAIN 2021

:
6. If Y, K and  are the values of Young's modulus, bulk modulus and modulus of rigidity of any
material respectively. Choose the correct relation for these parameters.
[JEE MAIN 2021 (FEB)]
9K 3YK 9K  Y
(1) Y  N / m2 (2)   N / m 2 (3) Y  N / m2 (4) K  N / m2
3K   9K  Y 2  3K 9  3Y

7. A bimetallic strip consists of metals A and B. It is mounted rigidly as shown. The metal A has
higher coefficient of expansion compared to that of metal B. When the bimetallic strip is placed in
a cold bath, it will: [JEE MAIN 2021 (MARCH)]

(1) Bend toward the right (2) Not bend but shrink
(3) Neither bend nor shrink (4) Bend toward the left

8. Two identical metal wires of thermal conductivities k1 and k2 respectively are connected in series.
The effective thermal conductivity of the combination is: [JEE MAIN 2021 (MARCH)]
2K1K 2 K  K2 K  K2 K  K2
(1) (2) 1 (3) 1 (4) 1
K1  K 2 2K1  K 2 K1  K 2 K1  K 2

9. Two separate wires A and B are stretched by 2mm and 4mm respectively, when they are subjected
to a force of 2N. Assume that both the wires are made up of same material and the radius of wire
B is 4 times that of the radius of wire A. The length of the wires A and B are in the ratio of a : b.
Then a/b can be expressed as 1/x where x is ________ . [JEE MAIN 2021 (MARCH)]

V
10. An object is located at 2km beneath the surface of the water. If the fractional compression is
V
1.3696, the ratio of hydraulic stress to the corresponding hydraulic strain will be ___________ .
[Given: density of water is 1000 kg ms–3.] [JEE MAIN 2021 (MARCH)]
7 –2 7 –2
(1) 1.96 × 10 Nm (2) 1.44 × 10 Nm (3) 2.26 × 10 Nm–2 (4) 1.44 × 109 Nm–2
9

11. A stone of mass 20 g is projected from a rubber catapult of length 0.1 m and area of cross section
10–6 m2 stretched by an amount 0.04 m. The velocity of the projected stone is ______ m/s.
(Young's modulus of rubber = 0.5 × 109 N/m2) [JEE MAIN 2021 (JULY)]

12. One mole of an ideal gas is taken through an adiabatic process where the temperature rises from
270C to 370 C. If the ideal gas is composed of polyatomic molecule that has 4 vibrational modes,
Which of the following is true? [JEE MAIN 2021 (JULY)]
[R = 8.314 J mol-1 k-1]
(1) Work done on the gas is close to 332 J (2) Work done on the gas is close to 582 J
(3) Work done by the gas is close to 582 J (4) Work done on the gas is close to 332 J

NUCLEUS-92, Rajeev Gandhi Nagar, Kota (Raj.) India 324005, Mob. 9358006181, 97831-97831 119
JEE MAIN 2021
0
13. One mole of an ideal gas at 27 C is taken from A to B as shown in the given PV indicator
diagram. The work done by the system will be____× 10-1 j. [Given : R = 8.3 J/ mole K,
In2=0.6931] [JEE MAIN 2021 (JULY)]
(Round off to the nearest integer )

Kurt ) '

f-
tested
=

14. The area of cross-section of a railway track is 0.01m2. The temperature variation is 10°C.
Coefficient of linear expansion of material of track is 10–5/°C. The energy stored per meter in the
track is_________J/m. [JEE MAIN 2021 (JULY)]
11 –2
(Young's modulus of material of track is 10 Nm )

15. The value of tension in a long thin metal wire has been changed from T1 to T2 . The lengths of the
metal wire at two different values of tension T1 and T2 are 1 and n 2 respectively. The actual
n -
length of the metal wire is : -
[JEE MAIN 2021 (JULY)]
T  T2 1 T T 
(1) 1 2 (2) 1 1 2 2 (3) 1 2 (4) T1T2 1 2
-

T1  T2 T1  T2 2

16. The length of a metal wire is 1 , when the tension in it is T1 and is 2 when the tension is T2.
The natural length of the wire is : [JEE MAIN 2021 (JULY)]
T  Tl T  2 Tl 
(1) (2) 1 2 2 (3) 1 2 (4) 1 2
1 2
T2  T1 T2  T1 2

17. In the reported figure , heat energy absorbed by a system in going through a cyclic process is ____
J . [JEE MAIN 2021 (JULY)]

NUCLEUS-92, Rajeev Gandhi Nagar, Kota (Raj.) India 324005, Mob. 9358006181, 97831-97831 120
JEE MAIN 2021
18. In 5 minutes, a body cools from 75°C to 65°C at room temperature of 25°C. The temperature of
body at the end of next 5 minutes is _______°C.   [JEE MAIN 2021 (JULY)]

19. Two wires of same length and radius are joined end to end loaded. The Young’s modulii of the
materials of the two wires are Y1 and Y2. The combination behaves as a single wire then its
Young’s modulus is: [JEE MAIN 2021 (JULY)]
2Y1Y2 2Y1Y2 Y1Y2 Y1Y2
(1) Y  (2) Y  (3) Y  (4) Y 
3  Y1  Y2   Y1  Y2  2  Y1  Y2   Y1  Y2 

20. A body takes 4 min. to cool from 61° C to 59°C. If the temperature of the surroundings is 30°C,
the time taken by the body to cool from 51°C to 49° C is : [JEE MAIN 2021 (JULY)]
(1) 4 min. (2) 3 min. (3) 8 min. (4) 6 min.

21. A steel rod with y = 2.0 × 1011 Nm–2 and  = 10–5 °C–1 of length 4 m and area of cross-section 10
cm2 is heated from 0° C to 400°C without being allowed to extend. The tension produced in the
rod is x × 105 N where the value of x is ............. [JEE MAIN 2021 (AUGUST)]

22. The temperature of equal masses of three different liquids x,y and z are 10ºC, 20ºC and 30ºC
respectively. The temperature of mixture when x is mixed with y is 16ºC and that when y is mixed
with z is 26ºC. The temperature of mixture when x and z are mixed will be :
[JEE MAIN 2021 (AUGUST)]
(1) 28.32º C (2) 25.62º C (3) 23.84ºC (4) 20.28ºC

23. Wires W1 and W2 are made of same materials having the breaking stress of 1.25 x 109 N/m2. W1
and W2 have cross-sectional area of 8 x 10-7 m2 and 4 x 10-7 m2, respectively. Masses of 20 kg
hang from them as shown in the figure. The maximum mass that can be placed in the pan without
breaking the wires is _________kg.
(Use g = 10m/s2) [JEE MAIN 2021 (AUGUST)]

24. A rod CD of the thermal resistance 10.0 KW-1 is joined at the middle of an identical rod AB as
shown in figure, the end A, B and D are maintained at 200°C, and 100°C and 125° C respectively.
The heat current in CD is P watt. The value of P is ...... . [JEE MAIN 2021 (AUGUST)]

NUCLEUS-92, Rajeev Gandhi Nagar, Kota (Raj.) India 324005, Mob. 9358006181, 97831-97831 121
JEE MAIN 2021
25. Four identical hollow cylindrical columns of mild steel support a big structure of mass 5  103 kg,
✓ the inner and outer radii of each column are 50 cm and 100 cm respectively. Assuming uniform
local distribution, calculate the compression strain of each column. [Use Y = 2.0  1011 Pa, g =
9.8 m/s2] [JEE MAIN 2021 (AUGUST)]
-8 -7
(1) 3.60  10 (2) 2.60  10 (3) 1.87  10 (4) 7.07  10-4
-3

26. Two thin metallic spherical shells of radii r1 and r2 (r1< r2) are place with their centres coinciding.
A material of thermal conductivity K is filled in the space between the shells. The inner shell is
maintained at temperature 1 and the outer shell at temperature 2 (1  2 ) . The rate at which
heat flows radially through the material is :- [JEE MAIN 2021 (AUGUST)]
4Kr1r2 (2  1 ) r r (   )
(1) (2) 1 2 2 1
r2  r1 r2  r1
K(2  1 ) K(2  1 )(r2  r1 )
(3) (4)
r2  r1 4r1r2

27. When a rubber ball is taken to a depth of _________m in deep sea, its volume decreases by 0.5%
. [JEE MAIN 2021 (AUGUST)]
(The bulk modules of rubber = 9.8  108 nM-2 Density of sea water = 103 kgm-3 g = 9.8 m/s2)

ANSWER KEY

1. 1 2. 1 3. 2 4. 3 5. 3 6. 4 7. 4
8. 1 9. 32 10. 4 11. 20 12. 2 13. 17258 14. 5
15. 1 16. 2 17. 100 18. 57 19. 2 20. 4 21. 8
22. 3 23. 40 24. 2 25. 2 26. 1 27. 500

NUCLEUS-92, Rajeev Gandhi Nagar, Kota (Raj.) India 324005, Mob. 9358006181, 97831-97831 122
JEE MAIN 2021
SOLUTION
1. (1)
Sol. V = VT
V = 3a3T
2. (1)
Sol. A and R are true but R is not the correct explanation of A.

3. (2)
M
Sol. 
V
d dV

 V
P
k
dV
V
dV P
 
V K
d P P
  d 
 k k

4. (3)

Sol.

Heat flow rate will be same through both


     2
 1 
R1 R2
R21 – R2= R1– R12
R   R12
 2 1
R1  R 2
5. (3)
Sol. Assuming Hooke's law to be valid.
T   
T  k  
Let, 0 = natural length (original length)
T  k   0 
T1  T2 1  T1
so, T1  k  1  0  & T2  k  2  0    1 0
 0  2
T2 2  0 T2  T1

NUCLEUS-92, Rajeev Gandhi Nagar, Kota (Raj.) India 324005, Mob. 9358006181, 97831-97831 123
JEE MAIN 2021
6. (4)
Sol. Y – Younge modulus, K – Bulk modulus,
- modulus of rigidity
We know that
y = 3k (1 – 2)
1 y 
 = 1   …..(i)
2  3k 
y = 2 (1 + )
y
= 1 …..(ii)
2
From Eq. (i) and Eq. (ii)
1 Y  y
1    1
2  3k  2
y y
1–  2
3k 
y y
 3
3k 
y 3  y

3k 
y
 3  y
3k
y
k=
9  3y
7. (4)
Sol. A  B
Length of both strips will decrease
LA  LB

8. (1)

Sol.

l l 2l
R eff   
K1A K 2 A K eg A
2K1K 2
K eq 
K1  K 2

NUCLEUS-92, Rajeev Gandhi Nagar, Kota (Raj.) India 324005, Mob. 9358006181, 97831-97831 124
JEE MAIN 2021
9. (32)
E 2mm
Sol. For A y .....(1)
r 2
a
E 4mm
For B y ......(2)
.16r 2
b
 (1)/(2)
2b
16 
4a
a 1

b 32

10. (4)
Sol. P = h g
p 2 103 103  9.8
 
V 1.36 10 –2
V
= 1.44 × 109 N/m2

11. (20)
Sol. By energy conservation
1 YA 2 1
. .x  mv 2
2 L 2
0.5 10 106  (0.04)2
9
20 2
 v
0.1 1000
 v2  400
v  20m / s

12. (2)
Sol. Since, each vibrational mode, corresponds to two degrees of freedom, hence, f = 3 (trans.) + 3
(rot.) + 8 (vib.) = 14
2
&   1
f
2 8
  1 
14 7
nRT
w  582
 1
As W < 0. work is done on the gas

13. (17258)
Sol. Process of isothermal
 V2 
W = nRT n  
 V1 
= 1 × 8.3 × 300 × ln2
= 17258 × 10–1 J
NUCLEUS-92, Rajeev Gandhi Nagar, Kota (Raj.) India 324005, Mob. 9358006181, 97831-97831 125
JEE MAIN 2021
14. (5)
Y
Sol. Elastic energy = (strain)2 × Area × length
2
Y
 Elastic energy per unit length = (strain)2 ×5 Area
2
  
 strain   T  105 10  10 –4 
 
11
10
 
2
  104 102  5J / m
2

15. (1)
FL
Sol. Y
AL
T1 T2
Y 0
 0
A 1  0  A 2  0 
T1   
1 2 0

T2  1 0

T21 – T20 = T12 – T10


(T1 – T2)0 = T12 – T21
T T 
 1 2 2 1
 T1  T2 
0

16. (2)
Sol. T1  k  1  0 
T2  k  2  0

T1 
 1 0
T2 2  0
T1 2  T2 1
 0
T1  T2

17. (100)

Sol.

For complete cyclic process


NUCLEUS-92, Rajeev Gandhi Nagar, Kota (Raj.) India 324005, Mob. 9358006181, 97831-97831 126
JEE MAIN 2021
U = 0
 from Q = U + W
=0+W
Q = W
= Area
= r1·r2
=  × (10 × 103) × (10 × 10–3)
Q = 100 

18. (57)
Sol. By newton’s law of cooling (with approximation)
ΔT
 C  Tavg  Ts 
Δt
10 C
1st  C  70 C  25 C 
5min
2
 C min 1
45
T  65  T  65   2  T  15 
2nd  C   25      
5min  2   45  2 
 9(T – 65) = – (T + 15)
 10T = 570
 T = 57°C
Alternate Solution :
Newton’s law of cooling (without approximation)
TP – TS = (Ti – TS)e–Ct
4
1st 65  25  (75  25)e 5C  e 5C 
5
4
2nd T  25  (65  25)e 5C  40   32
5
T = 57°C

19. (2)
Sol. In series combination l = 1 + 2
F/A F
Y 
 / AY
 D
AY
Equivalent length of rod after joining is = 2
 As, lengths are same and force is also same in series
 = 1 + 2
eq 2 2Y1Y2
     Y 
Yeq Y1 Y2 Y Y1 Y2 Y1  Y2

NUCLEUS-92, Rajeev Gandhi Nagar, Kota (Raj.) India 324005, Mob. 9358006181, 97831-97831 127
JEE MAIN 2021
20. (4)
T
Sol.  K  Tt  TS  Tt = average temp.
t
61  59 61  59
 K  
 30  …….(1)
4  2 
51  49 51  49
 K  
 30  …….(2)
t  2 
Divide (1) & (2)
t 60  30 30
 
4 50  30 20
so, t = 6 minutes

21. (8)
Sol. Thermal force F = Ay T
F = (10 × 10–4) (2 × 1011) (10–5) (400)
F = 8 × 105 N  x = 8
22. (3)
Sol. X Y Z
m1 = m m2 = m m3 = m
T1 = 10°C T3 = 20°C T3 = 30°C
s1 s2 s3
when x & y are mixed, Tf1 = 16°C
m1s1T + m2s2T2 = (m1s1 + m2s2)Tf1
s1 × 10 + s2 × 20 = (s1 + s2) × 16
2
s1 = s2 ……(i)
3
when y & z are mixed, Tf 2 = 26°C
m2s2T + m3s3T3 = (m3s3 + m3s3)Tf2
s2 × 20 + s3 × 30 = (s2 + s3) × 26
3
s3 = s2 …….(ii)
2
when x & z are mixex
m1s1T1 + m3s3T3 = (m1s1 + m3s3)Tf
2 2 2 3
s 2 10  s 2  20   s 2   Tf
3 3 3 2
Tf = 23.84°C

23. (40)
T1max
Sol. B.S1   T1max  8  1.25  100 = 1000 N
8 107
T
B.S2  2 max7  T2 max  4  1.25 1000 = 500 N
4  10
500  100
m  40kg
10
NUCLEUS-92, Rajeev Gandhi Nagar, Kota (Raj.) India 324005, Mob. 9358006181, 97831-97831 128
JEE MAIN 2021
24. (2)

Sol.

Rods are identical so


RAB = RCD = 10 Kw-1
C is mid-points of AB, so
RAC = RCB = 5Kw-1
at point C
200  T T  125 T  100
 
5 10 5
2(200-T) = T – 125 + 2(T – 100 )
400 – 2 T = T – 125 + 2T – 200
725
T  145C
5
145  125 20
Ih  w w
10 10
Ih  2w

25. (2)
mg
Sol. Force on each columns 
4
mg
Strain 
4AY
50 103  9.8

4  (1  0.25)  2 1011
 2.6 107

26. (1)

Sol.

Thermal resistance of spherical sheet of thickness dr and radius r is


dr
dR 
K(42 )

NUCLEUS-92, Rajeev Gandhi Nagar, Kota (Raj.) India 324005, Mob. 9358006181, 97831-97831 129
JEE MAIN 2021
2
r
dr
R
r1
K(4r 2 )
1 1 1 1  r2  r1 
R     
4K  r1 r2  4K  r1r2 
 
Thermal current (i)  2 1
R
4Kr1r2
i (2  1 )
r2  r1

27. (500)
P gh
Sol. B 
 V   V 
   
 V   V 
V
B
 V h
g
9.8 108  0.5
h
100 103  9.8
h = 500

NUCLEUS-92, Rajeev Gandhi Nagar, Kota (Raj.) India 324005, Mob. 9358006181, 97831-97831 130
JEE MAIN 2021
FLUID MECHANICS

1. A hydraulic press can lift 100 kg when a mass 'm' is placed on the smaller piston. It can lift
_______kg when the diameter of the larger piston is increased by 4 times and that of the smaller
piston is decreased by 4 times keeping the same mass 'm' on the smaller piston.
[JEE MAIN 2021 (FEB)]

2. The pressure acting on a submarine is 3 × 105 Pa at a certain depth. If the depth is doubled, the
percentage increase in the pressure acting on the submarine would be : (Assume that atmospheric
pressure is 1 × 105 Pa density of water is 103 kg m–3, g = 10 ms–2): [JEE MAIN 2021 (MARCH)]
200 200 5 3
(1) % (2) % (3) % (4) %
3 5 200 200

3. What will be the nature of flow of water from a circular tap, when its flow rate increased from
0.18 L/min to 0.48 L/min ? The radius of the tap and viscosity of water are 0.5 cm and 10–3Pa s,
respectively.
(Density of water : 103kg/m3) [JEE MAIN 2021 (MARCH)]
(1) Unsteady to steady flow (2) Remains steady flow
(3) Remains turbulent flow (4) Steady flow to unsteady flow

4. Consider a water tank as shown in figure. Its cross-sectional area is 0.4 m2. The tank has an
opening B near the bottom whose cross-section area is 1 cm2. A load of 24kg is applied on the
water at the top when the height of the water level is 40cm above the bottom, the velocity of water
coming out the opening B is v ms–1. The value of v, to the nearest integer, is ______. [Take value
of g to be 10 ms–2] [JEE MAIN 2021 (MARCH)]

NUCLEUS-92, Rajeev Gandhi Nagar, Kota (Raj.) India 324005, Mob. 9358006181, 97831-97831 131
JEE MAIN 2021
5. When two soap bubbles of radii a and b(b > a) coalesce, the radius of curvature of common
surface is: [JEE MAIN 2021 (MARCH)]
ab ab b–a ab
(1) (2) (3) (4)
b–a ab ab ab

6. Two small drops of mercury each of radius R coalesce to form a single large drop. The ratio of
total surface energy before and after the change is : [JEE MAIN 2021 (JULY)]
1 1
(1) 2 :1
3
(2) 1: 2 3 (3) 2 :1 (4) 1: 2

7. Two spherical soap bubbles of radii r1 and r2 in vacuum combine under isothermal conditions.
The resulting bubble has a radius equal to : [JEE MAIN 2021 (JULY)]
r1r2 r1  r2
(1) (2) r1r2 (3) r12  r22 (4)
r1  r2 2

8. A light cylindrical vessel is kept on a horizontal surface. Area of base is A. A hole of cross-
sectional area 'a' is made just at its bottom side. The minimum coefficient of friction necessary to
prevent sliding the vessel due to the impact force of the emerging liquid is (a << A) :
[JEE MAIN 2021 (JULY)]

A 2a a
(1) (2) None of these (3) (4)
2a A A

9. The water is filled upto height of 12 m in a tank having vertical sidewalls. A hole is made in one
of the walls at a depth 'h' below the water level. The value of 'h' for which the emerging stream of
water strikes the ground at the maximum range is _____m. [JEE MAIN 2021 (JULY)]

NUCLEUS-92, Rajeev Gandhi Nagar, Kota (Raj.) India 324005, Mob. 9358006181, 97831-97831 132
JEE MAIN 2021
10. A circular disc reaches from top to bottom of an inclined plane of length 'L' .When it slips down
t2 3
the plane , it takes time 't1'. When it rolls down the plane , it takes time t2. the value of is .
t1 x
The value of x will be _________. [JEE MAIN 2021 (JULY)]

11. A rod of mass M and length L is lying on a horizontal frictionless surface. A particle of mass 'm'
travelling along the surface hits at one end of the rod with a velocity 'u' in a direction
perpendicular to the rod. The collision is completely elastic. After collision , particle comes to rest.
m 1
The ratio of masses   is . The value of 'x' will be ______. [JEE MAIN 2021 (JULY)]
M x

12. A raindrop with radius R = 0.2 mm falls from a cloud at a height h = 2000 m above the ground.
Assume that the drop is spherical throughout its fall and the force of buoyancy may be neglected
then the terminal speed attained by the raindrop is : [Density of water  w  1000 kg m3 and
density of air  a  1.2 kg m3 ,g  10 m / s2 [JEE MAIN 2021 (JULY)]
Coefficient of viscosity of air  1.8 105 Nsm2 ]
(1) 250.6 ms–1 (2) 43.56 ms–1 (3) 4.94 ms–1 (4)14.4 ms–1

13. A soap bubble of radius 3 cm is formed inside the another soap bubble of radius 6 cm . The radius
of an equivalent soap bubble which has the same excess pressure as inside the smaller bubble with
respect to the atmospheric pressure is .... cm . [JEE MAIN 2021 (AUGUST)]

14. In Millikan's oil drop experiment, what is viscous force acting on an unchanged drop of radius
2.0  10-5 m and density 1.2  103 kgm-3 ? Take viscosity of liquid = 1.8  10-5 Nsm-2 .(Neglect
buoyance due to air) [JEE MAIN 2021 (AUGUST)]
(1) 3.8  10-11 N (2) 3.9  10-10 (3) 1.8  10-10 N (4) 5.8  10-10 N

15. A bob of mass 'm' suspended by thread of length l undergoes simple harmonic oscillation with
1
time period T. If the bob is immersed in a liquid that has density times that of the bob and the
4
length of the thread is increased by 1/3rd of the original length, then the time period of the simple
harmonic oscillation will be :- [JEE MAIN 2021 (AUGUST)]
3 3 4
(1) T (2) T (3) T (4) T
2 4 3

16. Two narrow bores of diameter 5.0 mm and 8.0 mm are joined together to form a U-shaped tube
open at both ends. If this U-tube contains water, what is difference in the level of two limbs of the
tube.[The surface tension of water T = 7.3 x 10-2 Nm-1, angle of contact = 0, g = 10ms-2 and
density of water = 1.0 x 103 kg m-3 ] [JEE MAIN 2021 (AUGUST)]
(1) 3.62 mm (2) 2.19 mm (3)5.34 mm (4) 4.97 mm

NUCLEUS-92, Rajeev Gandhi Nagar, Kota (Raj.) India 324005, Mob. 9358006181, 97831-97831 133
JEE MAIN 2021
ANSWER KEY
1. 25600 2. 1 3. 4 4. 3 5. 1 6. 1 7. 3
8. 3 9. 6 10. 2 11. 4 12. 3 13. 2 14. 2
15. 4 16. 2

SOLUTION
1. (25600)
Sol. Using Pascals law

100  g mg
 …..(1)
A2 A1
Let m mass can lift M0 in second case then
M0g mg
 …..(2)
16A 2 A1 /16

d 2
{Since A  }
4
From equation (1) and (2) we get
M0
= 16
16.100
 M0 = 25600 kg

2. (1)
Sol. P1 = gd + P0 = 3 × 105 Pa
gd = 2 × 105 Pa
P2 = 2gd + P0
= 4 × 105 + 105 = 5 × 105 Pa
P2  P1
% increase = 100
P1

5 105  3 105 200


100  %
3 10 5
3
NUCLEUS-92, Rajeev Gandhi Nagar, Kota (Raj.) India 324005, Mob. 9358006181, 97831-97831 134
JEE MAIN 2021
3. (4)
Sol. The nature of flow is determined by Reynolds Number.
ρvD
Re 
η

 ρ  density of fluid; η  coefficient of viscosity 


 v  velocity of flow 
 
 D  Diameter of pipe 

From NCERT
If R e  1000  flow is steady

1000  R e  2000  flow becomes unsteady

R e  2000  flow is turbulent

0.18 103 1102


R einitial  103    382.16
π   0.5 102   60 10
2 3

0.48 103 1102


R efinal  103  
π   0.5 102   60 10
2 3

= 1019.09

4. (3)

Sol.

m = 24 kg
A = 0.4 m2
a = 1 cm2
H = 40cm
Using Bernoulli's equation

NUCLEUS-92, Rajeev Gandhi Nagar, Kota (Raj.) India 324005, Mob. 9358006181, 97831-97831 135
JEE MAIN 2021
 mg  1 2
  P0    gH  V1
 A  2
1
 P0  0  v 2 .......(1)
2
 Neglecting v1
2mg
 v  2gH 
A
 v  8  1.2
 v  3.033m / s
 v  3m / s

5. (1)
Sol. Excess pressure at common surface is given by
 1 1  4T
Pex  4T  –  
a b r
1 1 1
  –
r a b
ab
r
b–a

6. (1)

Sol.

4 4 4
R 3  R 3  R 3
3 3 3
1
R  23 R
Ai  2  4R 2 

Af  4R 2

NUCLEUS-92, Rajeev Gandhi Nagar, Kota (Raj.) India 324005, Mob. 9358006181, 97831-97831 136
JEE MAIN 2021
2
Ui Ai 2R
  2/3 2  21/3
Uf Af 2 R
7. (3)

Sol.

number of moles is conserved


n1 + n2 = n3
P1V1 + P2V2 = P3V
4S  4 3  4S  4 3  4S  4 3 
 r1    r2    r3 
r1  3  r2  3  r3  3 
r12  r22  r32

r3  r12  r22

8. (3)
Sol. For no sliding
f av2
mg av2
Ahg a2gh
2a

A
Option (3)

9. (6)

Sol.

NUCLEUS-92, Rajeev Gandhi Nagar, Kota (Raj.) India 324005, Mob. 9358006181, 97831-97831 137
JEE MAIN 2021
(12  h) 
R  2gh 
g

4h(12  h)  R

For maximum R
dR
0
dh
 h  6m

10. (2)

Sol.

If disk slips on inclined plane, then it's acceleration


a1 = gsin
1 2
L a 1 t1
2

2L
 t1  ……..(i)
a1

If disk rolls on inclined plane, its acceleration,


g sin 
a2 
I
1
mR 2
g sin 
a2 
mR 2
1
2mR 2
2
a2  g sin 
3
1
Now L  a 2  t 22
2

NUCLEUS-92, Rajeev Gandhi Nagar, Kota (Raj.) India 324005, Mob. 9358006181, 97831-97831 138
JEE MAIN 2021
2L
 t2  …….(ii)
a2

t2 a 3
Now  1  x=2
t1 a2 2

11. (4)

Sol.

Just after collision


From momentum conservation, Pi0  Pf
mu = Mv ……(i)
From angular momentum conservation about O,
L ML2
mu   
2 12
6mu
 = ……(ii)
ML
R.V.S
From e =
R.V.A
L
V
1 2
u
L
v u
2
3mu
v u
M
mu 3mu
 u
M M

NUCLEUS-92, Rajeev Gandhi Nagar, Kota (Raj.) India 324005, Mob. 9358006181, 97831-97831 139
JEE MAIN 2021
4mu
u
M
m 1

M 4
X=4

12. (3)
Sol. At terminal speed
a =0
Fnet = 0
mg = Fv = 6Rv
mg
v
6Rv
4 3
w Rg
v 3
6R
2w R 2g 400
  m/s
9 81
= 4.94 m/s

13. (2)

Sol.

excess pressure inside the smaller soap bubble


4S 4S
P   .....(i)
r1 r2
The excess pressure inside the equivalent soap bubble
4S
P  ....(ii)
R eq
From (i)& (ii)
4S 4S 4S
 
R eq r1 r2
1 1 1 1 1
   
R eq r1 r2 6 3
Req = 2 cm

NUCLEUS-92, Rajeev Gandhi Nagar, Kota (Raj.) India 324005, Mob. 9358006181, 97831-97831 140
JEE MAIN 2021
14. (2)
4 
Sol. Viscous force = Wight    r 3  g  3.9 1010
3 

15. (4)
Sol. T  2  / g
When bob is immersed in liquid
mgeff = mg – Buoyant force
mgeff = mg – v  g (  = density of liquid)

 mg  v g
4
mg 3mg
 mg  
4 4
3g
 g eff 
4

1  4 3g
T1  2 1     ,  eff 
g eff 3 3 4
By solving
4
T1  2  / g
3
4T
T1 
3

16. (2)

Sol.

We have PA = PB . [Points A & B at same horizontal level ]


2T 2T
 Patm   g(x  h)  Patm   gx
r1 r2

1 1 
gh  2T     2  7.3 102 
1 1 

 r1 r2   2.5 10
3
4 103 

2  7.3 102 103  1 1 -3


h   2.5  4  = 2.19 x 10 m = 2.19 mm
103 10  
Hence option (2)

NUCLEUS-92, Rajeev Gandhi Nagar, Kota (Raj.) India 324005, Mob. 9358006181, 97831-97831 141
JEE MAIN 2021
SHM
1. When a particle executes SHM, the nature of graphical representation of velocity as a function of
displacement is : [JEE MAIN 2021 (FEB)]
(1) circular (2) elliptical (3) parabolic (4) straight line

2. In the given figure, a body of mass M is held between two massless springs, on a smooth inclined
plane. The free ends of the springs are attached to firm supports. If each spring has spring constant
k, the frequency of oscillation of given body is : [JEE MAIN 2021 (FEB)]

1 k 1 2k 1 2k 1 k
(1) (2) (3) (4)
2 2M 2 Mg sin  2 M 2 Mg sin 

3. Two identical springs of spring constant '2k' are attached to a block of mass m and to fixed support
(see figure).When the mass is displaced from equilibrium position on either side, it executes
simple harmonic motion. The time period of oscillations of this system is :
[JEE MAIN 2021 (FEB)]

m m m m
(1) 2 (2)  (3) 2 (4) 
k 2k 2k k

4. Y = A sin (t + 0) is the time-displacement equation of a SHM. At t = 0 the displacement of the
A
particle is Y = and it is moving along negative x-direction. Then the initial phase angle 0 will
2
be : [JEE MAIN 2021 (FEB)]
  5 2
(1) (2) (3) (4)
6 3 6 3

NUCLEUS-92, Rajeev Gandhi Nagar, Kota (Raj.) India 324005, Mob. 9358006181, 97831-97831 142
JEE MAIN 2021
5. If two similar springs each of spring constant K1 are joined in series, the new spring constant and
time period would be changed by a factor: [JEE MAIN 2021 (FEB)]
1 1 1 1
(1) , 2 (2) , 2 (3) ,2 2 (4) ,2 2
2 4 4 2

6. A particle executes S.H.M., the graph of velocity as a function of displacement is :-


[JEE MAIN 2021 (FEB)]
(1) A circle (2) A parabola (3) An ellipse (4) A helix

7. Time period of a simple pendulum is T. The time taken to complete 5/8 oscillations starting from

mean position is T . The value of  is ………….. [JEE MAIN 2021 (FEB)]

8. In the given figure, a mass M is attached to a horizontal spring which is fixed on one side to a
rigid support. The spring constant of the spring is k. The mass oscillates on a frictionless surface
with time period T and amplitude A. When the mass is in equilibrium position, as shown in the
figure, another mass m is gently fixed upon it. The new amplitude of oscillation will be :
[JEE MAIN 2021 (FEB)]

Mm M Mm M
(1) a (2) A (3) A (4) A
M Mm M Mm

9. For what value of displacement the kinetic energy and potential energy of a simple harmonic
oscillation become equal? [JEE MAIN 2021 (FEB)]
(1) x = 0 (2) x =  A
A A
(3) x   (4) x 
2 2

NUCLEUS-92, Rajeev Gandhi Nagar, Kota (Raj.) India 324005, Mob. 9358006181, 97831-97831 143
JEE MAIN 2021
10. Time period of a simple pendulum is T inside a lift when the lift is stationary. If the lift moves
upwards with an acceleration g/2, the time period of pendulum will be :
[JEE MAIN 2021 (MARCH)]

T 3 2
(1) 3T (2) (3) T (4) T
3 2 3

11. For what value of displacement the kinetic energy and potential energy of a simple harmonic
oscillation become equal? [JEE MAIN 2021 (MARCH)]
(1) x = 0 (2) x =  A
A A
(3) x   (4) x 
2 2

12. Consider two identical springs each of spring constant k and negligible mass compared to the mass
M as shown. Fig .1 shows one of them and Fig. 2 shows their series combination. The ratios of
Tb
time period of oscillation of two SHM is  x , where value of x is _______.
Ta
(Round off to the Nearest Integer) [JEE MAIN 2021 (MARCH)]

13. A block of mass 1 kg attached to a spring is made to oscillate with initial amplitude of 12cm. After
2 minutes the amplitude decreases to 6cm. Determine the value of the damping constant for this
motion. (take ln 2 = 0.693) [JEE MAIN 2021 (MARCH)]
(1) 0.69 × 102 kg s–1 (2) 3.3 × 102 kg s–1
(3) 1.16 × 102 kg s–1 (4) 5.7 × 10–3 kg s–1

NUCLEUS-92, Rajeev Gandhi Nagar, Kota (Raj.) India 324005, Mob. 9358006181, 97831-97831 144
JEE MAIN 2021
14. Two particles A and B of equal masses are suspended from two massless springs of spring
constant K1 and K2 respectively. If the maximum velocities during oscillations are equal, the ratio
of the amplitude of A and B is [JEE MAIN 2021 (MARCH)]
k2 k1 k1 k2
(1) (2) (3) (4)
k1 k2 k2 k1

15. A particle performs simple harmonic motion with a period of 2 second. The time taken by the
1
particle to cover a displacement equal to half of its amplitude from the mean position is s. The
a
value of 'a' to the nearest integer is ________ . [JEE MAIN 2021 (MARCH)]


16. The function of time representing a simple harmonic motion with a period of is:

[JEE MAIN 2021 (MARCH)]
(1) sin  t   cos  t  (2) cos  t   cos  2t   cos 3t 
 
(3) sin 2  t  (4) 3cos  – 2t 
4 

17. A particle is making simple harmonic motion along the X-axis. If at a distance x1and x2 from the
mean position the velocities of the particle are v1 and v2 respectively. The time period of its
oscillation is given as. [JEE MAIN 2021 (JULY)]
2 2 2 2

(1) T  2 x x2
2
1
2
(2) T  2 x x2
2
1
2
v v1 2 v v
1 2

2 2 2 2

(3) T  2 x x 2
2
1
2
(4) T  2 x x2
2
1
2
v v1 2 v v1 2

18. Amplitude of a mass-spring system, which is executing simple harmonic motion decreases with
time. If mass = 500g, Decay constant = 20g/s then how much time is required for the amplitude of
the system to drop to half of its initial value? (In 2 = 0.693) [JEE MAIN 2021 (MARCH)]
(1) 34.65s (2) 17.32s (3) 0.034 s (4) 15.01 s

19. T0 is the time period of a sample pendulum at a place. If the length of the pendulum is reduced to
1
times of its initial value, the modified time period is : [JEE MAIN 2021 (JULY)]
16
1
(1) T0 (2) 8T0 (3) 4T0 (4) T0
4

NUCLEUS-92, Rajeev Gandhi Nagar, Kota (Raj.) India 324005, Mob. 9358006181, 97831-97831 145
JEE MAIN 2021
20. The motion of a mass on a spring, with spring constant K is as shown in figure.
[JEE MAIN 2021 (JULY)]

K
The equation of motion is given by x(t) = Asint + Bcost with  =
m
Suppose that at time t = 0, the position of mass is x(0) and velocity v (0), then its displacement can
also be represented as x(t) = C cos(t – ), where C and  are :

2v(0)2  v(0) 
(1) C   x(0)2 ,   tan 1  
 2
 x(0) 

2v(0)2  x(0) 
(2) C   x(0)2 ,   tan 1  
 2
 2v(0) 

v(0)2  x(0) 
(3) C   x(0)2 ,   tan 1  
 2
 v(0) 
v(0)2  v(0) 
(4) C   x(0)2 ,   tan 1  
 2
 x(0) 

21. In the reported figure, two bodies A and B of masses 200 g and 800 g are attached with the system
of springs. Springs are kept in a stretched position with some extension when the system is
released. The horizontal surface is assumed to be frictionless. The angular frequency will
be________rad/s when k = 20 N/m. [JEE MAIN 2021 (JULY)]

22. A particle of mass 1 mg and charge q is lying at the mid-point of two stationary particles kept at a
distance ‘2 m’ when each is carrying same charge ‘q’ If the free charged particle is displaced from
its equilibrium position through distance ‘x’   1m .The particle executes SHM. Its angular

frequency of oscillation will be ________  105 rad/s if q 2  10 C2 . [JEE MAIN 2021 (JULY)]

NUCLEUS-92, Rajeev Gandhi Nagar, Kota (Raj.) India 324005, Mob. 9358006181, 97831-97831 146
JEE MAIN 2021
23. In a simple harmonic oscillation , what fraction of total mechanical energy is in the form of kinetic
energy, when the particle is midway between mean and extreme position.
[JEE MAIN 2021 (JULY)]
1 3 1 1
(1) (2) (3) (4)
2 4 3 4

24. A particle starts executing simple harmonic motion (SHM) of amplitude 'a' and total energy E. At
3E
any instant, its kinetic energy is then its displacement 'y' is given by :
4
[JEE MAIN 2021 (JULY)]
a a 3 a
(1) y = a (2) y  (3) y  (4) y 
2 2 2

25. An object of mass 0.5 kg is executing simple harmonic motion. Its amplitude is 5 cm and time
T
period (T) is 0.2 s. What will be the potential energy of the object at an instant t  s starting
4
from mean position, Assume that the initial phase of the oscillation is zero.
[JEE MAIN 2021 (JULY)]
-3 3
(1) 0.62 J (2) 6.2 × 10 J (3) 1.2 × 10 J (4) 6.2 × 103 J

26. A particle execute simple harmonic motion represented by displacement function as


x(t)  A sin (t  )
If the position and velocity of the particle at t = 0 s are 2 cm and 2  cm s-1 respectively, then its
amplitude is x 2 cm where the value of x is _______. [JEE MAIN 2021 (JULY)]

27. A mass of 5 kg is connected to a spring. The potential energy curve of the simple harmonic motion
executed by the system is shown in the figure. A simple pendulum of length 4 m has the same
period of oscillation as the spring system. What is the value of acceleration due to gravity on the
planet where these experiments are performed? [JEE MAIN 2021 (AUGUST)]

(1) 10 m/s2 (2) 5 m/s2 (3) 4 m/s2 (4) 9.8 m/s2

NUCLEUS-92, Rajeev Gandhi Nagar, Kota (Raj.) India 324005, Mob. 9358006181, 97831-97831 147
JEE MAIN 2021
28. If the length of the pendulum in pendulum clock increases by 0.1%, then the error in time per day
is: [JEE MAIN 2021 (AUGUST)]
(1) 86.4 s (2) 4.32 s (3) 43.2 s (4) 8.64 s

29. Two simple harmonic motions are represented by the equations


 
x1  5sin  2t   and x 2  5 2(sin 2t  cos 2t) [JEE MAIN 2021 (AUGUST)]
 4
The amplitude of second motion is ……… times the amplitude in first motion.

 
30. Two simple harmonic motion, are represented by the equations y1 = 10 sin  3t  
 3
y2 = 5 (sin 3t  3 cos 3t) ratio of amplitude of y1 to y2 = x :1. The value of x is _______.
[JEE MAIN 2021 (AUGUST)]

31. The variation of displacement with time of a particle executing free simple harmonic motion is
shown in the figure. [JEE MAIN 2021 (AUGUST)]

The potential energy U(x) versus time (t) plot of the particle is correctly shown in figure :

(1) (2)

(3) (4)

32. For a body executing S.H.M.: [JEE MAIN 2021 (AUGUST)]


(a) Potential energy is always equal to its K.E.
(b) Average potential and kinetic energy over any given time interval are always equal.
(c) Sum of the kinetic and potential energy at any point of time is constant.
(d) Average K.E. in one time period is equal to average potential energy in one time period.
Choose the most appropriate option from the option given below :
(1) (c) and (d) (2) only (c) (3) (b) and (c) (4) only (b)
NUCLEUS-92, Rajeev Gandhi Nagar, Kota (Raj.) India 324005, Mob. 9358006181, 97831-97831 148
JEE MAIN 2021
-1
33. A particle of mass 1 kg is hanging from a spring of force constant 100 Nm . The mass is pulled
slightly downward and released so that it executes free simple harmonic motion with time period
T
T. The time when the kinetic energy and potential energy of the system will become equal, is .
x
The value of x is ___________. [JEE MAIN 2021 (AUGUST)]

ANSWER KEY
1. 2 2. 3 3. 4 4. 3 5. 1 6. 3 7. 7
8. 2 9. 3 10. 4 11. 3 12. 2 13. Bonus 14. 4
15. 6 16. 4 17. 4 18. 1 19. 4 20. 4 21. 10
22. 6 23. 2 24. 4 25. 1 26. 2 27. 3 28. 3
29. 2 30. 1 31. 4 32. 1 33. 8

NUCLEUS-92, Rajeev Gandhi Nagar, Kota (Raj.) India 324005, Mob. 9358006181, 97831-97831 149
JEE MAIN 2021
SOLUTION

1. (2)
Sol. For a particle executing SHM,
x = Asin(t + )
v = A cos(t + )
v2 x2
  1
2 A 2 A 2
 equation of ellipse between v and x
Hence option (2)

2. (3)

Sol.

Keq = K1 + K2 = K + K = 2K
m m
T = 2  2
K eq 2K
1 1 2K
f=  (Option 3) is correct
T 2 m

3. (4)
Sol. For parallel combination keq= k1 + k2
keq=4 K
m
T  2
k eq

4. (3)

Sol.

  5
Initial phase  
2 3 6

NUCLEUS-92, Rajeev Gandhi Nagar, Kota (Raj.) India 324005, Mob. 9358006181, 97831-97831 150
JEE MAIN 2021
5. (1)
1 1 1
Sol.  
k eq k1 k 2

1 1 1 k
   k eq 
k eq k k 2

k
k'
2

M
T  2
k
M M
T '  2  T '  2  2
k' k
T  2T

6. (3)
Sol. v2  2  A2  x 2 
v2 x2
 1
 A 
2
A2
This is an equation of an ellipse.

7. (7)
th
5  1 1
Sol. th of oscillation =    of oscillation
8  2 8

    t
  2 
  t
6  T 
7   2 
  t
6  T 
7T
t
12

NUCLEUS-92, Rajeev Gandhi Nagar, Kota (Raj.) India 324005, Mob. 9358006181, 97831-97831 151
JEE MAIN 2021
8. (2)

Sol.

Momentum of system remains conserved.


pi = pf
MA = (m + M) ''

k k
MA  m  M A '
M mM

M
A' = A
Mm

9. (3)
Sol. KE = PE
1 1
m2  A 2 – x 2   m2 x 2
2 2
A2 – x2  x2
2x2 = A2
A
x
2

10. (4)
Sol. When lift is stationary

L
T  2
g
When lift is moving upward  Pseudo force acts downwards
g 3g
 g eff  g    New time period
2 2

L 2L
T '  2  2
geff 3g

2
T'  T
3

NUCLEUS-92, Rajeev Gandhi Nagar, Kota (Raj.) India 324005, Mob. 9358006181, 97831-97831 152
JEE MAIN 2021
11. (3)
Sol. KE = PE
1 1
m2  A 2 – x 2   m2 x 2
2 2
A2 – x2  x2
2x2 = A2
A
x
2

12. (2)
M
Sol. Ta  2
K
M
Tb  2
K/2
Tb
 2  x x2
Ta

13. Official Ans. by NTA (NA)


Official Ans. by Nucleus (Bonus)
Sol. A  A 0 e  γt
b
ln2  120
2m
0.693  2 1
b
120
1.16 × 10–2 kg/sec.

14. (4)
Sol. (4) A11 = A22
k k
A1 1  A 2 2
m m

15. (6)
Sol.

2 1
t 
12 6
 Correct answer = 6.00

NUCLEUS-92, Rajeev Gandhi Nagar, Kota (Raj.) India 324005, Mob. 9358006181, 97831-97831 153
JEE MAIN 2021
16. (4)
2
Sol. Time period T 
'
 2

 '
 '  2  Angular Frequency of SHM
Option (3)
1 1
sin2t =
2
 2sin 2 t   1 – cos2t 
2
1 1 
Angular frequency of  – cos 2t  is 2
2 2 
Options (4)
Angular frequency of SHM
 
3cos  – 2t  is 2.
4 
So option (3) & (4) both have angular frequency 2 but option (4) is direct answer.

17. (4)
Sol. v2  2  A2  x 2 

v12 v22
A  x  2  x2  2
2 2
1
2

 
v 22  v12
  2
2

x1  x 22
x12  x 22
T  2
v22  v12

18. (1)
bt

 γt
Sol. A  A0 e  A0 e 2m

bt
A0
 A0e 2m
2
bt
 ln 2 `
2m
2m 2  500  0.693
t ln 2   t  34.65sec
b 20

NUCLEUS-92, Rajeev Gandhi Nagar, Kota (Raj.) India 324005, Mob. 9358006181, 97831-97831 154
JEE MAIN 2021
19. (4)

Sol. T0  2
g
/16 2
New time period T = 2 
g 4 g
T0
T=
4

20. (4)
Sol x = A sin t + Bcos t
dx
v= = A cost – B sint
dt
At t = 0, x (0) = B
v (0) = A
 x = A sin t + B sin (t + 90°)

A net  A 2  B2
B A
tan    cot  
A B
 x  A2  B2 sin(t  )
 x  A2  B2 cos(t  (90  ))
x  Ccos(t  )
 C  A 2  B2
[v(0)]2
C  [x(0)]2
 2

  90  
A
tan   cos  
B
v  0
 tan =
x  0  .

1  v  0  
 = tan  
 x 0  

NUCLEUS-92, Rajeev Gandhi Nagar, Kota (Raj.) India 324005, Mob. 9358006181, 97831-97831 155
JEE MAIN 2021
21. (10)

k eq
Sol. 

 = reduced mass
springs are in series connection.
k 1k 2
keq =
k1  k 2
k  4k 4k
k eq  
5k 5
4  20
k eq  N / m  16N / m
5
m1m2 0.2  0.8
   0.16kg
m1  m2 0.2  0.8

16
  100  10
0.16

22. (6)

Sol.

Net force on free charged particle


kd 2 kq 2
F= 
 d  x 2  d  x 2

 
 4dx 
F  kq 2

  d 2  x 2 2 
 
4kq 2d  x 
a  
m  d4 

 4kq 2 
a   3 
x
 md 
So, angular frequency

4kq 2

md 3

NUCLEUS-92, Rajeev Gandhi Nagar, Kota (Raj.) India 324005, Mob. 9358006181, 97831-97831 156
JEE MAIN 2021
4  9  109  10

1 10 6  13

 = 6 × 108rad/sec

23. (2)
1
Sol. K m2  A 2  x 2 
2

1  A2 
 m2  A2  
2  4 

2  3A 
2
1
 m  
2  4 
31 
K   m2 A 2 
42 

24. (4)
1 2
Sol. E Ka
2
3E 1
 K  a 2  y2 
4 2
3 1 2 1
 Ka  K  a 2  y 2 
4 2 2
2
3a
y2  a 2 
4
a
y=
2

25. (1)
T
Sol. If it starts from MP, after it is at extreme position
4
T
 PE at = KE at t = 0
4
2
1 2 1 2 2 1 2  2 
  mv = m A  = mA  
2 2 2  T 

1 1 4 2
=   .05 2   0.62J
2 2  0.2 2

NUCLEUS-92, Rajeev Gandhi Nagar, Kota (Raj.) India 324005, Mob. 9358006181, 97831-97831 157
JEE MAIN 2021
26. (2)
Sol. x(t)  Asin(t  )
v(t)  Acos(t  )
2  Asin  ...(1)
2  Acos  ...(2)
From (1) and (2)
tan   1
  45
Putting value of  in equation (1)
 1 
2  A 
 2
A2 2
x=2

27. (3)
Sol. From potential energy curve
1 1
Umax = kA 2  = k  2 2
2 2
k=5
Now Tspring = Tpendulum
5 4
2  2
5 g
4
1=  g = 4 on planet
g
Option (3)

28. (3)

Sol. T  2
g
T 1 

T 2
1 0.1
T    24  3600
2 100
T = 43.2

29. (2)
 1 1  
Sol. x2  5 2  sin 2t  cos 2t  2  10sin  2t  
 2 2   4

A 2 10
  2
A1 5

NUCLEUS-92, Rajeev Gandhi Nagar, Kota (Raj.) India 324005, Mob. 9358006181, 97831-97831 158
JEE MAIN 2021
30. (1)
 
Sol. y1  10sin  3t   Amplitude = 10
 3
y2  5(sin3t  3 cos3t)
1 3 
y 2  10  sin 3t  cos 3t 
2 2 
1  
y2  10  sin 3t  sin cos 3 
2 3 
 
y2  10sin  3t    Amplitude = 10
 3
10
So ratio of amplitude   1
10

31. (4)
Sol. Potential energy is maximum at maximum distance from mean position.

32. (1)
1
Sol. In S.H.M. total mechanical energy remains constant and also <K.E.> = < P.E. > = KA2
4
(for 1 time period)

33. (8)

Sol.

KE = PE
A
y  A sin t
2

T T
t 
8 x
x=8

NUCLEUS-92, Rajeev Gandhi Nagar, Kota (Raj.) India 324005, Mob. 9358006181, 97831-97831 159
JEE MAIN 2021
ELECTROSTATICS
1. A cube of side 'a' has point charges +Q located at each of its vertices except at the origin where the
charge is –Q. The electric field at the centre of cube is : [JEE MAIN 2021 (FEB)]

Q 2Q
(1)  xˆ  yˆ  zˆ  (2)  xˆ  yˆ  zˆ 
3 30a 2 3 30a 2
2Q Q
(3)  xˆ  yˆ  zˆ  (4)  xˆ  yˆ  zˆ 
3 30a 2 3 30a 2

2. Two electrons each are fixed at a distance '2d'. A third charge proton placed at the midpoint is
displaced slightly by a distance x (x << d) perpendicular to the line joining the two fixed charges.
-

Proton will execute simple harmonic motion having angular frequency : (m = mass of charged
particle) [JEE MAIN 2021 (FEB)]
1 1 1 1
 2q 2  2   md3  2  q2 2  20 md3  2
(1)  3 
(2)  0 2  (3)  3 
(4)  
 0 md   2q   20 md   q2 

3. A point charge of +12 C is at a distance 6 cm vertically above the centre of a square of side 12
cm as shown in figure. The magnitude of the electric flux through the square will be ______ × 103
Nm2/C. [JEE MAIN 2021 (FEB)]

NUCLEUS-92, Rajeev Gandhi Nagar, Kota (Raj.) India 324005, Mob. 9358006181, 97831-97831 160
JEE MAIN 2021
4. A charge 'q' is placed at one corner of a cube as shown in figure. The flux of electrostatic field E
through the shaded area is : [JEE MAIN 2021 (FEB)]

q q q q
(1) (2) (3) (4)
4 0 24 0 48 0 8 0

5. An electron with kinetic energy K1 enters between parallel plates of a capacitor at an angle '' with
the plates. It leaves the plates at angle '' with kinetic energy K2. Then the ratio of kinetic energies
K1 : K2 will be : [JEE MAIN 2021 (FEB)]
sin 2  cos2  cos  cos 
(1) (2) (3) (4)
cos2  cos2  cos  sin 

6. The peak electric field produced by the radiation coming from the 8 W bulb at a distance of 10 m

x 0c v
is . The efficiency of the bulb is 10 % and it is a point source. The value of x is
10  m
______ [JEE MAIN 2021 (FEB)]

7. Two small spheres each of mass 10 mg are suspended from a point by threads 0.5 m long. They
are equally charged and repel each other to a distance of 0.20 m. The charge on each of the sphere
a
is 108 C . The value of 'a' will be ______. [JEE MAIN 2021 (FEB)]
21
[Given g = 10 ms–2]

8. In an electrical circuit, a battery is connected to pass 20 C of charge through it in a certain given


time. The potential difference between two plates of the battery is maintained at 15 V. The work
done by the battery is ______J. [JEE MAIN 2021 (FEB)]

NUCLEUS-92, Rajeev Gandhi Nagar, Kota (Raj.) India 324005, Mob. 9358006181, 97831-97831 161
JEE MAIN 2021
3 4 N
9. The electric field in a region is given E   E 0ˆi  E 0ˆj  . The ratio of flux of reported field
5 5 C
through the rectangular surface of area 0.2 m2 (parallel to y – z plane) to that of the surface of area
0.3 m2 (parallel to x – z plane) is a : b, where a = _____. [JEE MAIN 2021 (FEB)]
[Here ˆi, ˆj and k̂ are unit vectors along x, y and z-axes respectively]

10. Find the electric field at point P (as shown in figure) on the perpendicular bisector of a uniformly
charged thin wire of length L carrying a charge Q. The distance of the point P from the centre of

3
the rod is a  L. [JEE MAIN 2021 (FEB)]
2

3Q Q Q Q
(1) (2) (3) (4)
40 L2 3 0 L2 2 30 L2 40 L2

11. In an electrical circuit, a battery is connected to pass 20 C of charge through it in a certain given
time. The potential difference between two plates of the battery is maintained at 15 V. The work
done by the battery is ______J. [JEE MAIN 2021 (FEB)]

12. Find out the surface charge density at the intersection of point x = 3 m plane and x-axis, in the
region of uniform line charge of 8nC/m lying along the z-axis in free space.
[JEE MAIN 2021 (MARCH)]
(1) 0.424 nCm–2 (2) 47.88C/m (3) 0.07nC m–2 (4) 4.0nC m–2

2 ˆ 3 ˆ N
13. The electric field in a region is given by E  E 0 i  E 0 j with E 0  4.0 103 . The flux of this
5 5 C
field through a rectangular surface area 0.4m2 parallel to the Y – Z plane is _______ Nm2C–1.
[JEE MAIN 2021 (MARCH)]
NUCLEUS-92, Rajeev Gandhi Nagar, Kota (Raj.) India 324005, Mob. 9358006181, 97831-97831 162
JEE MAIN 2021
–3
14. An oil drop of radius 2mm with a density 3g cm is held stationary under a constant electric field
3.55 × 105 Vm–1 in the Millikan's oil drop experiment. What is the number of excess electrons that
the oil drop will possess? [JEE MAIN 2021 (MARCH)]
(consider g = 9.81 m/s2)
(1) 48.8 × 1011 (2) 1.73 × 1010 (3) 17.3 × 1010 (4) 1.73 × 1012

15. An infinite number of point changes, each carrying 1C charge, are placed along the y-axis at y =
1 m, 2 m, 4 m, 8 m .............. [JEE MAIN 2021 (MARCH)]
The total force on a 1 C point charge, placed at the origin, is x × 103N. The value of x, to the
nearest integer, is _________ .
1
[Take  9 109 Nm 2 / C2 ]
4 0
16. A certain charge Q is divided into two parts q and (Q-q). How should the charge Q and q be
divided so that q and (Q-q) placed at a certain distance apart experience maximum electrostatic
repulsion ? [JEE MAIN 2021 (JULY)]
q
(1) Q  (2) Q  2q (3) Q  4q (4) Q  3q
2

17. An electric dipole is placed on x-axis in proximity to a line charge of linear charge density 3.0 ×
10–6 C/m. Line charge is placed on z-axis and positive and negative charge of dipole is at a
distance of 10 mm and 12 mm from the origin respectively. If total force of 4 N is exerted on the
dipole, find out the amount of positive or negative charge of the dipole.
[JEE MAIN 2021 (JULY)]
(1) 815.1 C (2) 8.8 μC (3) 0.485 C (4) 4.44 μC

18. Two ideal electric diploes A and B , having their dipole moment p1 and p2 respectively are placed
on a plane with their centers at O as shown in the figure . At point C on the axis of dipole A, the
resultant electric field is making an angle of 370 with the axis . The Ratio of the dipole moment of
P 3
A and B , 1 is : (take sin 370 = ) [JEE MAIN 2021 (JULY)]
P2 5

3 3 2 4
(1) (2) (3) (4)
8 2 3 3

NUCLEUS-92, Rajeev Gandhi Nagar, Kota (Raj.) India 324005, Mob. 9358006181, 97831-97831 163
:
JEE MAIN 2021
19. Two identical tennis balls each having mass 'm' and charge 'q' are suspended from a fixed point by
threads of length 'l'. What is the equilibrium separation when each thread makes a small angle '''
with the vertical ? [JEE MAIN 2021 (JULY)]
1 1 1 1
 q 2l  2  q 2l  3  q 2l 2  3  q 2l 2  3
(1) x    (2) x    (3) x   2 
(4) x   2 2 
 20 mg   20 mg   20 m g   20 m g 

20. What will be the magnitude of electric field at point O as shown in the figure ? Each side of the
figure is l and perpendicular to each other ? [JEE MAIN 2021 (JULY)]

(1)
1 q
40  
(2)
1 q

40 (2  )
2 2 1  (3)
q
40 (2 ) 2
(4)
1 2q
40 2 2
 2
21. The total charge enclosed in an incremental volume of 2 × 10–9 m3 located at the origin is _______
nC, if electric flux density of its field is found as D = e–x siny î  e  x cosy ˆj  2zkˆ C/m2.
[JEE MAIN 2021 (JULY)]

22. A cube is placed insidean electric field, E = 150y2 ĵ . The side of cube is 0.5 m and is placed in the
field as shown in the given figure. The charge inside the cube is : [JEE MAIN 2021 (JULY)]

(1) 3.8 × 10–11 C (2) 8.3 × 10–11 C (3) 3.8 × 10–12 C (4) 8.3 × 10–12 C

23. The two thin coaxial rings, each of radius 'a' and having charges +Q and –Q respectively are
separated by a distance of 's'. The potential difference between the centres of the two rings is
[JEE MAIN 2021 (AUGUST)]
Q 1 1  Q 1 1 
(1)   2  (2)   
20  a s  a2  40  a s2  a 2 
Q 1 1  Q 1

1 
(3)   2  (4)  
40  a s  a2  20  a s2  a 2 

NUCLEUS-92, Rajeev Gandhi Nagar, Kota (Raj.) India 324005, Mob. 9358006181, 97831-97831 164
JEE MAIN 2021
24. A solid metal sphere of radius R having charge q is enclosed inside the concentric spherical shell
of inner radius a and outer radius b as shown in the figure. The approximate variation electric field
E as a function of distance r from centre O is given by [JEE MAIN 2021 (AUGUST)]

(1)

(2)

(3)

(4)

25. two short magnetic dipoles m1 and m2 each having magnetic moment of 1 Am2 are placed at point
O and P respectively. The distance between OP is 1 meter . The torque experienced by the
magnetic dipole m2 due to the presence of m1 is ....... × 10–7 Nm.
[JEE MAIN 2021 (AUGUST)]

NUCLEUS-92, Rajeev Gandhi Nagar, Kota (Raj.) India 324005, Mob. 9358006181, 97831-97831 165
JEE MAIN 2021
o
26. Figure shows a rod AB, which is bent in a 120 circular arc of radius R . A charge (- Q ) is
uniformly distributed over rod AB. What is the electric field E at the centre of curvature O ?
[JEE MAIN 2021 (AUGUST)]

(1)
3 3Q
80 R 2

î (2)
3 3Q
820 R 2


(3)
3 3Q
1620 R 2
î  (4)
3 3Q
820 R 2


27. A uniformly charged disc of radius R having surface charge density  is placed in the xy plane
with its centre at the origin. Find the electric field intensity along the z-axis at a distance Z from
origin:- [JEE MAIN 2021 (AUGUST)]
  Z    Z 
(1) E  1  2 2 1/2 
(2) E  1  2 2 1/2 
20  (Z  R )  20  (Z  R ) 

2 0  1    1 1 
(3) E   2  Z (4) E   2  2
  (Z  R )
2 1/2
 20  (Z  R )
2 1/2
Z 

28. Choose the incorrect statement : [JEE MAIN 2021 (AUGUST)]


(a) The electric lines of force entering into a Gaussian surface provide negative flux.
(b) A charge 'q' is placed at the centre of a cube. The flux through all the faces will be the same.
(c) In a uniform electric field net flux through a closed Gaussian surface containing no net charge,
is zero.
(d) When electric field is parallel to a Gaussian surface, it provides a finite non-zero flux.
Choose the most appropriate answer from the option given below
(1) (c) and (d) only (2) (b) and (d) only
(3) (d) only (4) (a) and (c) only

NUCLEUS-92, Rajeev Gandhi Nagar, Kota (Raj.) India 324005, Mob. 9358006181, 97831-97831 166
JEE MAIN 2021
29. Two particles A and B having charges 20 C and 5 C respectively are held with a separation of
5 cm . At what position a third charged particle should be placed so that it does not experience a
net electric force ? [JEE MAIN 2021 (AUGUST)]

(1) At 5 cm from 20 C on the left side of system


(2) At 5 cm from -5 C on the right side
(3) At 1.25 cm from – 5 C between two charges
(4) At midpoint between two charges

30. Which of the following statements are correct  [JEE MAIN 2021 (AUGUST)]
(A) Electric monopoles do not exist whereas magnetic monopoles exist.
(B) Magnetic field lines due to a solenoid at its ends and outside cannot be completely straight and
confined.
(C) Magnetic field lines are completely confined with a toroid.
(D) Magnetic field lines inside a bar magnet are not parallel.
(E)  = - 1 is the condition for a perfect diamagnetic susceptibility.
Choose the correct answer from the option given below :
(1) (C) and (E) only (2) (B) and (D) only
(3) (A) and (B) only (4) (B) and (C) only

ANSWER KEY
1. 2 2. 3 3. 226 4. 2 5. 2 6. 2 7. 20
8. 300 9. 1 10. 3 11. 300 12. 1 13. 640 14. 2
15. 12 16. 2 17. 4 18. 3 19. 2 20. 2 21. 4
22. 2 23. 4 24. 1 25. 1 26. 2 27. 1 28. 3
29. 2 30. 1

NUCLEUS-92, Rajeev Gandhi Nagar, Kota (Raj.) India 324005, Mob. 9358006181, 97831-97831 167
JEE MAIN 2021
SOLUTION
1. (2)
Sol. We can replace –Q charge at origin by +Q and –2Q . Now due to +Q charge at every corner of
cube. Electric field at center of cube is zero so now net electric field at center is only due to –2Q
charge at origin.
a
kqr
1 2Q   xˆ  yˆ  zˆ 
E 3  2
40  
r a
3
2 
2Q  xˆ  yˆ  zˆ 
E
3 3a 20

2. (3)
Sol. From the given condition, we have

Fnetq   2Fq/q cos 

1 q2 x
Fnetq = 2. . .
40  d 2  x 2  2
d  x2
2

q2 x
=
20  d  x 2 3/2
2

For x << d,
q2
Fnet q =  x
2 0 d 3

q2
a= x
20 .md 3
Comparing with equation of SHM (a = –2x)
q
=
20 .md 3
Hence option (3) is correct.

NUCLEUS-92, Rajeev Gandhi Nagar, Kota (Raj.) India 324005, Mob. 9358006181, 97831-97831 168
JEE MAIN 2021
3. (226)
1 q  12 106 3 Nm
2
3 Nm
2
Sol. From symmetry      = 225.98 × 10  226 10
6  0  6  8.85 1012 s C

4. (2)

Sol.

q
flux through cube =
80
flux through surfaces ABEH, ADGH, ABCD will be zero
1 q  q
 (EFGH) =  (DCFG) =  (EBCF) =   
3  8 0  24 0

5. (2)

Sol.

velocity along the plate will not change.


 v1 cos  = v2 cos 
K1 v12 cos 2 
 2 
K2 v 2 cos 2 

6. (2)
1
Sol. I c 0 E 02
2
8 1 1 1
  c  E02
410 2 4
2
0 c 2

2 C
E0   0 x2
10 

NUCLEUS-92, Rajeev Gandhi Nagar, Kota (Raj.) India 324005, Mob. 9358006181, 97831-97831 169
JEE MAIN 2021
7. (20)

Sol.

T cos  = mg = 10 × 10–6 × 10 = 10–4


9 109  q 2
T sin   F
0.04
0.1 F
tan   
0.24 mg
2 10
q 108
3 24
a
0.95 × 10–8 =  10 8
21
a = 20

8. (300)
Sol. Work done by battery = Q (V)

9. (1)
 3E 4E  N
Sol. E    0 ˆi  0 ˆj 
 5 5 C
A1 = 0.2 m2 [parallel to y – z plane]
 A1  0.2m2 ˆi
A2 = 0.3 m2 [parallel to x – z plane]
A2  0.3m2ˆj
 3E 4E  3  0.2
Now, a   0 ˆi  0 ˆj . 0.2iˆ   E0
 5 5  5
 3E 4E  3  0.3
& b   0 ˆi  0 ˆj . 0.3jˆ   E0
 5 5  5
a 0.6 1 a
Now    a:b=1:2
b 1.2 2 b
Q a=1
NUCLEUS-92, Rajeev Gandhi Nagar, Kota (Raj.) India 324005, Mob. 9358006181, 97831-97831 170
JEE MAIN 2021
10. (3)
k
Sol. E  sin 1  sin 2 
a
1 Q 1
E     2sin  
40 L  3L 
 
 2 
L/2 1
tan   
3L 3
2

sin  
2
1 2Q  1
E   2 
40 2
3L  2
Q
E
2 30 L2

11. (300)
Sol. Work done by battery = Q (V)

12. (1)
2K 
Sol.  (x = 3m)
r 0
C
  0.424  10 9
m2

13. (640)
2
Sol.   ExA   4 103  0.4  640
5

14. (2)
Sol. qE = Mg
4 
neE =   r 3   g
3 
n × 1.6 × 10–9 × 3.55 × 105
4
 3  103     2  10 –3   9.81
3

3
n = 173 × 10(3 – 9 – 5 + 19)
n = 1.73 × 1010

NUCLEUS-92, Rajeev Gandhi Nagar, Kota (Raj.) India 324005, Mob. 9358006181, 97831-97831 171
JEE MAIN 2021
15. (12)

Sol.

 1 1 1 
F  k 1C 1C  1  2  2  2  .....
 2 4 8 

 
 1 
 9 103   12 103 N
1
1 – 
 4
16. (2)

Sol.

kq  Q  q  k
Fq   2  qQ  q 2 
L2 L
dF
 0 when force is maximum
dq
dF k
 Q  2q   0
dq L2
 Q – 2q = 0  Q = 2q

17. (4)

Sol.

r = 10 mm, x = 2,
2k
Fq  q
r
2k 2kq 2kq
F q  q  Fnet  
rx r rx

NUCLEUS-92, Rajeev Gandhi Nagar, Kota (Raj.) India 324005, Mob. 9358006181, 97831-97831 172
JEE MAIN 2021
2kq  x
Fnet 
r(r  x)

2  9 109  3 106  q  2mm


4  q = 4.44 C
10mm 12mm

18. (3)

Sol.

kP2
3 3 P 3
tan 37   r  2 
4 2kP1 2P1 4
r3
P2 3

P1 2
P1 2

P2 3

19. (2)

Sol.

Tcos = mg

kq 2
Tsin =
x2
kq 2
tan  =
x 2 mg
x
as tan  sin 
2L

NUCLEUS-92, Rajeev Gandhi Nagar, Kota (Raj.) India 324005, Mob. 9358006181, 97831-97831 173
JEE MAIN 2021
2
x Kq
 2
2L x mg
1/3
 q2L 
x  
 20 mg 

20. (2)
kq
Sol. E1  2
 E2

kq kq
E3   2
( 2 ) 2
2

2kq kq kq
E 2
 2
 2 (2 2  1)
2 2

21. (4)
Sol. Electric flux density

ch arg e Q  Q 
(D)   rˆ  rˆ   0  rˆ
2 
Area 4r 2
 40 r 

D e x sin yiˆ  e x cos yˆj  2zkˆ


E 
0 0
Also by Gauss's law
   ˆ  ˆ  ˆ      D
 i j  k   E   ˆi  ˆj  kˆ  
0  x y z   x y z   0

 x  

x
 e sin y    e x cos y   (2z)
y z

  e x sin y  e x sin y  2

NUCLEUS-92, Rajeev Gandhi Nagar, Kota (Raj.) India 324005, Mob. 9358006181, 97831-97831 174
JEE MAIN 2021
–0 –0
At origin  = –e sin 0 + e sin 0 + 2
 = 2C/m3
Charge =  × volume = 2 × 2 × 10–9 = 4 × 10–9 = 4nC

22. (2)
Sol. As electric field is in y-direction so electric flux is only due to top and bottom surface
Bottom surface y = 0
E=0
 = 0
Top surface y = 0.5 m
150
 E = 150(.5)2 =
4
150 150
Now flux  = EA = .5 2 
4 16
Q in
By Gauss's law  =
0

150
Qin =  8.85  10 12 = 8.3 × 10–11 C
16
Option (2)

23. (4)

Sol.

KQ KQ
VA  
a a 2  s2
KQ KQ
VB  
a a 2  s2
2KQ 2KQ Q 1 1 
VA  VB      2 2
a a s
2 2 20  a s  a 

NUCLEUS-92, Rajeev Gandhi Nagar, Kota (Raj.) India 324005, Mob. 9358006181, 97831-97831 175
JEE MAIN 2021
24. (1)
Official Ans. by Nucleus (1 or 2)
Sol. Considering outer spherical shell is non conducting
Electric field inside a metal sphere is zero.
rRE0
kQ
rRE
r2

Option (2)
Considering outer spherical shell is conducting

r  R, E  0
kQ
Rra E
r2
a  r  b, e0
kQ
eb E
r2

Option (1)

NUCLEUS-92, Rajeev Gandhi Nagar, Kota (Raj.) India 324005, Mob. 9358006181, 97831-97831 176
JEE MAIN 2021
25. (1)

Sol.

  M2  B1

  M 2 B1 sin 900

0 M1
 1 1
4 (1)3
 107 n.M
26. (2)
2k    ˆ
Sol.  sin   (i)
R 2
 
 Q   Q 
    2 
 R   R. 
 3 
3Q

2R

3 3Q
 (ˆi)
8 o R
2 2

27. (1)
Sol. Consider a small ring of radius r and thickness dr on disc.

area of elemental ring on disc


dA  2rdr
charge on this ring dq  dA

NUCLEUS-92, Rajeev Gandhi Nagar, Kota (Raj.) India 324005, Mob. 9358006181, 97831-97831 177
JEE MAIN 2021
kdqz
dEz 
(z  r 2 )3/2
2

R
  z 
E   dE  1  
0
2 0  R 2  z2 

28. (3)
Sol. Since   E.A  EA cos 

  90
  0
29. (2)

Sol.

Null point is possible only right side of -5 C

k(5C) k(20C)
EN    0
x2 (5  x)2
x= 5 cm
 option (2) is correct

30. (1)
Sol. Statement (C) is correct because, the magnetic field outside the toroid is zero and they form closed
loops inside the toroid itself.
Statement (E) is correct because we know that super conductors are materials inside which the net
magnetic field is always zero and they are perfect diamagnetic.
 = 1+ 
=-1
 = 0
For superconductors.

NUCLEUS-92, Rajeev Gandhi Nagar, Kota (Raj.) India 324005, Mob. 9358006181, 97831-97831 178
JEE MAIN 2021
GRAVITATION
1. Two stars of masses m and 2m at a distance d rotate about their common centre of mass in free
space. The period of revolution is : [JEE MAIN 2021 (FEB)]
1 d3 d3
(1) (2) 2
2 3Gm 3Gm
1 3Gm 3Gm
(3) (4) 2
2 d 3 d3

2. Four identical particles of equal masses 1kg made to move along the circumference of a circle of
radius 1 m under the action of their own mutual gravitational attraction. The speed of each particle
will be : [JEE MAIN 2021 (FEB)]
G
(1) 1  2 2  (2) G 1  2 2 
2
G 1  2 2  G
(3)  2 2  1 (4)
2 2

3. Consider two satellites S1 and S2 with periods of revolution 1 hr. and 8hr. respectively revolving
around a planet in circular orbits. The ratio of angular velocity of satellite S 1 to the angular
velocity of satellites S2 is : [JEE MAIN 2021 (FEB)]
(1) 8 : 1 (2) 1 : 4 (3) 2 : 1 (4) 1 : 8

4. A body weighs 49 N on a spring balance at the north pole. What will be its weight recorded on the
same weighing machine, if it is shifted to the equator ? [JEE MAIN 2021 (FEB)]
GM
(Use g = 2
= 9.8 ms–2 and radius of earth, R = 6400 km)
R
(1) 49 N (2) 48.83 N (3) 49.83 N (4) 49.17 N

5. Two satellites A and B of masses 200kg and 400kg are revolving round the earth at height of 600
km and 1600 km respectively. If TA and TB are the time periods of A and B respectively then the
value of TB – TA : [JEE MAIN 2021 (FEB)]

[Given : radius of earth = 6400km, mass of earth = 6 × 1024 kg]


(1) 1.33 × 103 s (2) 3.33 × 102 s (3) 4.24 × 103 s (4) 4.24 × 102 s

NUCLEUS-92, Rajeev Gandhi Nagar, Kota (Raj.) India 324005, Mob. 9358006181, 97831-97831 179
JEE MAIN 2021
6. A solid sphere of radius R gravitationally attracts a particle placed at 3R from its centre with a
R
force F1. Now a spherical cavity of radius   is made in the sphere (as shown in figure) and the
2
force becomes F2. The value of F1 : F2 is : [JEE MAIN 2021 (FEB)]

(1) 25 : 36 (2) 36 : 25 (3) 50 : 41 (4) 41 : 50

7. Given below are two statements : one is labeled as Assertion A and the other is labelled Reason
R. [JEE MAIN 2021 (FEB)]
Assertion A : The escape velocities of planet A and B are same. But A and B are of unequal mass.
Reason R : The product of their mass and radius must be same, M1R1 = M2R2
In the light of the above statements, choose the most appropriate answer from the options given
below :
(1) Both A and R are correct but R is NOT the correct explanation of A
(2) A is correct but R is not correct
(3) Both A and R are correct and R is the correct explanation of A
(4) A is not correct but R is correct

8. The initial velocity vi required to project a body vertically upward from the surface of the earth to
reach a height of 10R, where R is the radius of the earth, may be described in terms of escape
x
velocity ve such that vi =  ve . The value of x will be ________.
y
[JEE MAIN 2021 (FEB)]

9. Find the gravitational force of attraction between the ring and sphere as shown in the diagram,
where the plane of the ring is perpendicular to the line joining the centres. If 8 R is the distance
between the centres of a ring (of mass 'm') and a sphere (mass 'M') where both have equal radius
'R'. [JEE MAIN 2021 (FEB)]

8 GmM 2 2 GmM 1 GmM 8 GmM


(1) . (2) . 2 (3) . 2
(4) .
9 R 3 R 3 8 R 27 R 2

NUCLEUS-92, Rajeev Gandhi Nagar, Kota (Raj.) India 324005, Mob. 9358006181, 97831-97831 180
JEE MAIN 2021
10. Assume that a tunnel is dug along a chord of the earth, at a perpendicular distance (R/2) from the
earth's centre, where 'R' is the radius of the Earth. The wall of the tunnel is frictionless. If a particle
is released in this tunnel, it will execute a simple harmonic motion with a time period :
[JEE MAIN 2021 (FEB)]
2R g 1 g R
(1) (2) (3) (4) 2
g 2 R 2 R g

11. A planet revolving in elliptical orbit has : [JEE MAIN 2021 (FEB)]
(A) a constant velocity of revolution.
(B) has the least velocity when it is nearest to the sun.
(C) its areal velocity is directly proportional to its velocity.
(D) areal velocity is inversely proportional to its velocity.
(E) to follow a trajectory such that the areal velocity is constant.
Choose the correct answer from the options given below :
(1) A only (2) D only (3) C only (4) E only

12. In the reported figure of earth, the value of acceleration due to gravity is same at point A and C but
it is smaller than that of its value at point B (surface of the earth). The value of OA : AB will be x :
y. The value of x is .......... [JEE MAIN 2021 (FEB)]

13. The maximum and minimum distances of a comet from the sun are 1.6 × 1012 m and 8.0 × 1010 in
respectively. If the speed of the comet at the nearest point is 6 × 104 ms–1, the speed at the farthest
point is: [JEE MAIN 2021 (MARCH)]
3 3 3
(1) 1.5 × 10 m/s (2) 6.0 × 10 m/s (3) 3.0 × 10 m/s (4) 4.5 × 103 m/s

14. If one wants to remove all the mass of the earth to infinity in order to break it up completely. The
x GM 2
amount of energy that needs to be supplied will be where x is ______(Round off to the
5 R
Nearest Integer) [JEE MAIN 2021 (MARCH)]
(M is the mass of earth, R is the radius of earth G is the gravitational constant)

15. The radius in kilometer to which the present radius of earth (R = 6400 km) to be compressed so
that the escape velocity is increased 10 time is ________. [JEE MAIN 2021 (MARCH)]

NUCLEUS-92, Rajeev Gandhi Nagar, Kota (Raj.) India 324005, Mob. 9358006181, 97831-97831 181
JEE MAIN 2021
16. A geostationary satellite is orbiting around an arbitrary planet 'P' at a height of 11R above the
surface of 'P', R being the radius of 'P'. The time period of another satellite in hours at a height of
2R from the surface of 'P' is ________. 'P' has the time period of 24 hours.
[JEE MAIN 2021 (MARCH)]
6
(1) 6 2 (2) (3) 3 (4) 5
2

17. The time period of a satellite in a circular orbit of radius R is T. The period of another satellite in a
circular orbit of radius 9R is: [JEE MAIN 2021 (MARCH)]
(1) 9 T (2) 27 T (3) 12 T (4) 3 T

18. The angular momentum of a planet of mass M moving around the sun in an elliptical orbit is L .
The magnitude of the areal velocity of the planet is: [JEE MAIN 2021 (MARCH)]
4L L 2L L
(1) (2) (3) (4)
M M M 2M

19. A person whose mass is 100 kg travels from Earth to Mars in a spaceship. Neglect all other in sky
and the acceleration due to gravity on the surface of the Earth and Mars as 10 m/s2 and 4 m/s2
respectively. Identify from the below figures, the curve that fits best for the weight of the
passenger as a function of time. [JEE MAIN 2021 (JULY)]

(1) (c) (2) (a) (3) (d) (4) (b)

20. A satellite is launched into a circular orbit of radius R around earth , while a second satellite is
launched into a circular orbit of radius 1.02 R. The percentage difference in the time period of the
two satellite is : [JEE MAIN 2021 (JULY)]
(1) 1.5 (2) 2.0 (3) 0.7 (4) 3.0

21. Consider a binary star system of star A and star B with masses mA and mB revolving in a circular
orbit of radii rA and rB, respectively. If TA and TB are the time period of star A and Star B ,
respectively , [JEE MAIN 2021 (JULY)]
then :
3
T  r 2
(1) A   A  (2) TA  TB
TB  rB 
(3) TA  TB (if mA  mB ) (4) TA  TB (if rA  rB )

NUCLEUS-92, Rajeev Gandhi Nagar, Kota (Raj.) India 324005, Mob. 9358006181, 97831-97831 182
JEE MAIN 2021
22. A body is projected vertically upwards from the surface of earth with a velocity sufficient enough
to carry it to infinity. The time taken by it to reach height h is ______ S.
[JEE MAIN 2021 (JULY)]
R e   2R e  
3/2 3/2
h  h 
(1)  1    1 (2)   1    1
2g  R e   g  R e  
  
1 R e   1 2R e  
3/2 3/2
h  h 
(3)  1    1 (4)   1    1
3 2g  R e   3 g  R e  
  

23. Consider a planet in some solar system which has a mass double the mass of earth and density
equal to the average density of earth. If the weight of an object on earth is W , the weight of the
same object on that planet will be : [JEE MAIN 2021 (JULY)]
1
(1) 2W (2) W (3) 2 W
3
(4) 2W

24. Suppose two planets (spherical in shape) of radii R and 2R, but mass M and 9M respectively have a centre
to centre separation 8 R as shown in the figure. A satellite of mass 'm' is projected from the surface of the
planet of mass 'M' directly towards the centre of the second planet. The minimum speed 'v' required for the
a GM
satellite to reach the surface of the second planet is then the value of 'a' is ______.
7 R
[Given : The two planets are fixed in their position] [JEE MAIN 2021 (JULY)]

25. The planet Mars has two moons , if one of them has a period 7 hours, 30 minutes and an orbital
radius of 9.0 x 103 km. Find the mass of Mars. [JEE MAIN 2021 (JULY)]
 4 2

Given  6x1011 n 1m2 kg 2 
 G 
19
(1) 5.96 × 10 kg (2) 3.25 × 1021 kg (3) 7.02 × 1025 kg (4) 6.00 × 1023 kg
26. Two identical particle of mass 1 kg each go round a circle of radius R, under the action of their
mutual gravitational attraction. The angular speed of each particle is :
[JEE MAIN 2021 (JULY)]
G 1 G 1 1 2G
(1) 3
(2) 3
(3) (4)
2R 2 R 2R G R3

27. The minimum and maximum distances of a planet revolving around the Sun are x 1 and x2 . If the
minimum speed of the planet on its trajectory is V0 then its maximum speed will be :
[JEE MAIN 2021 (JULY)]
2 2
VX VX VX VX
(1) 0 2 1 (2) 0 2 2 (3) 0 1 (4) 0 2
x2 x1 x2 x1
NUCLEUS-92, Rajeev Gandhi Nagar, Kota (Raj.) India 324005, Mob. 9358006181, 97831-97831 183
JEE MAIN 2021
28. Four particles each of mass M, move along a circle of radius R under the action of their mutual
gravitational attraction as shown in figure. The speed of each particle is :
[JEE MAIN 2021 (AUGUST)]

1 GM 1 GM
(1) (2)  2 2  1
2 R  2 2  1 2 R
1 GM GM
(3)  2 2  1 (4)
2 R R
29. A mass of 50 kg is placed at the centre of a uniform spherical shell of mass 100 kg and radius 50
m. If the gravitational potential at a point, 25 m from the centre is V kg/m. The value of V is :
[JEE MAIN 2021 (AUGUST)]
(1) – 60 G (2) + 2 G (3) – 20 G (4) – 4 G
30. A body of mass (2M) splits into four masses [m, M – m , m, m – m ], which are rearranged to
M
from a square as shown in figure. The ratio of for which , the gravitational potential energy of
m
the system becomes maximum is x : 1. The value of x is ....... . [JEE MAIN 2021 (AUGUST)]

31. If RE be the radius of earth, then the ratio between the acceleration due to gravity at a depth 'r'
below and a height 'r' above the earth surface is : [JEE MAIN 2021 (AUGUST)]
(Given : r < RE)
r r2 r3 r r2 r3
(1) 1    (2) 1   
R E R E2 R 3E R E R E2 R 3E
r r2 r3 r r2 r3
(3) 1    (4) 1   
R E R E2 R 3E R E R E2 R 3E
32. The masses and radii of the earth and moon are (M1,R1) and (M2, R2) respectively. Their centres
are at distance 'r' apart. Find the minimum escape velocity for a particle of mass 'm' to be projected
from the middle of these two masses : [JEE MAIN 2021 (AUGUST)]
1 4G(M1  M 2 ) 4G(M1  M 2 )
(1) V  (2) V 
2 r r
1 2G(M1  M 2 ) 2G (M1  M 2 )
(3) V  (4) V 
2 r r

NUCLEUS-92, Rajeev Gandhi Nagar, Kota (Raj.) India 324005, Mob. 9358006181, 97831-97831 184
JEE MAIN 2021
ANSWER KEY
1. 2 2. 4 3. 1 4. 2 5. 1 6. 3 7. 2
8. 10 9. 4 10. 4 11. 4 12. 4 13. 3 14. 3
15. 64 16. 3 17. 2 18. 4 19. 1 20. 4 21. 2
22. 4 23. 3 24. 4 25. 4 26. 2 27. 4 28. 2
29. 4 30. 2 31. 4 32. 2

SOLUTION
1. (2)

Sol.

G  2m  m
F= 2
= (2m)2(d/3)
d
Gm d
2
 2
d 3
3Gm
 2 =
d3
3Gm
=
d3
2 d3
T=  2
 3Gm

2. (4)
Sol.

Gmm Gm 2
F1  
 2R 2 4R 2
Gmm Gm 2
F2  
 2R 2 4R 2
Gmm Gm 2
F3  
 2R  2R
2 2

NUCLEUS-92, Rajeev Gandhi Nagar, Kota (Raj.) India 324005, Mob. 9358006181, 97831-97831 185
JEE MAIN 2021
 Fnet = F1 + F2 cos 45° + F3 cos 45°
Gm 2 Gm 2 1 Gm 2 1 Gm 2  1 1 1 
   =  
4R 2
2R 2
2 2R
2
2 R  4 2 2 2 2 
2 

Gm 2  1 1  Gm 2
2 1  2 2 
=  
R 2  4 2  4R
Gm2 mv2
Fnet = 1  2 2  
4R 2 R
G 1  2 2 
v=
2
3. (1)
T1 1
Sol. 
T2 8
2 / 1 1

2 / 2 8
1 8

2 1

4. (2)
Sol. Weight of pole = mg = 49 N
At equator due to rotation = ge = g – R2 so W = mge = m(g – R2)
 Wp > We Wp = 49N
So, We = 48.83 N. We < 49 N
Option (2) is correct.

5. (1)
r3
Sol. T  2
GM

TA  2
 6400  600 103
GM
73
TA  2109
GM
83
TB  2 109
GM
2109 
TB  TA  8 8  7 7 
GM 
= 314 × 4.107
= 1289.64
= 1.289 × 103 s

NUCLEUS-92, Rajeev Gandhi Nagar, Kota (Raj.) India 324005, Mob. 9358006181, 97831-97831 186
JEE MAIN 2021
6. (3)
Sol. Let initial mass of sphere is m'. Hence mass of removed portion will be m'/8
m.Gm '
F1  m.E. 
9R 2

 G.m ' G.m '/ 8 


F2  m   
  3R   5R / 2  
2 2

Gm ' Gm ' 4  1 1  Gm '


     2
9R 2 8  25  9 50  R
41 Gm '
F2  .
50  9 R 2
F1 1 50  9 50
  
F2 9 41 41

7. (2)
2GM
Sol. Vc 
R
M1 M 2

R1 R 2
M1 R2 = M2 R1
Hence reason R is not correct

8. (10)
GMm GMm 1 2
Sol.   mv
11R R 2
20GM
v=
11R

9. (4)
Gmx
Sol. Gravitational field of ring 
R 
3/2
2
 x2

Force between sphere & ring 


GmM  8R  
GmM

8
R 
3/ 2 2
2
 8R 2 R 27

NUCLEUS-92, Rajeev Gandhi Nagar, Kota (Raj.) India 324005, Mob. 9358006181, 97831-97831 187
JEE MAIN 2021
10. (4)

Sol.

Force along the tunnel


 GMmr 
F   3  cos 
 R 
gm  GM 
F x  2  g, r cos   x 
R  R 
g
a x
R
g R
2  T  2
R g
11. (4)
Sol. As per Keppler's 2nd law, Areal velocity is constant.

12. (4)
GM  r 
Sol. gA 
R3
GM
gC  2
 R
R  
 2

gA  gC
r 1

R 3
9
R2
4
4R
r
9
NUCLEUS-92, Rajeev Gandhi Nagar, Kota (Raj.) India 324005, Mob. 9358006181, 97831-97831 188
JEE MAIN 2021
4R 5R
so OA  ; AB  R  r 
9 9
OA : AB = 4 : 5

13. (3)
Sol. By angular momentum conservation:
mv1r1  mv 2 r2
48 1014
v1   3000m / sec
1.6 1012
= 3 × 103 m/sec.

14. (3)
Sol. Energy given = Uf – Ui
 3 GM 2 
= 0–– 
 5 R 
3 GM 2

5 R
x=3

15. (64)
2Gm
Sol. Ve  ......(1)
R
2Gm
10Ve  ......(2)
R'
R
10=
R'
R 6400
 R'   64 km
100 100
16. (3)
Sol. (3) T  R3/2
3/2
24  12R 
   T  3hr
T  3R 

17. (2)
Sol. T2  R3
2 3
 T   9R 
   
T  R 
T2 = T2 × 93
T'= T × 33
T'= 27 T

NUCLEUS-92, Rajeev Gandhi Nagar, Kota (Raj.) India 324005, Mob. 9358006181, 97831-97831 189
JEE MAIN 2021
18. (4)

Sol.

For small displacement ds of the planet its area can be written as

1 1
dA  rdl  rds sin 
2 2
dA 1 ds Vr sin 
A. vel =  r sin  
dt 2 dt 2
dA 1 mVr sin  L
 
dt 2 m 2m

19. (1)
Sol. At neutral point g = 0 so graph (C) is correct
Hence option (1).

20. (4)
Sol T2  R3
T = kR3/2
dT 3 dR

T 2 R
3
 × 0.02 = 0.03
2
% Change = 3%

21. (2)
Sol. TA = TB (since A = B)

22. (4)

Sol.

Applying energy conservation from (1) to (2)

NUCLEUS-92, Rajeev Gandhi Nagar, Kota (Raj.) India 324005, Mob. 9358006181, 97831-97831 190
JEE MAIN 2021
1  2GM  GMm 1 GMm
m   mv2 
2  Re  Re 2 Rr
1 GMm
 mv 2 
2 Rr
2GM dr
 v 
R  r dt
t Re h

 2GM  dt   ( R  r )dr
0 Re

2 Re h
2GM  t  (R  r)3/ 2 
3 Re

2 R 3e  
3/2
h 
t  1    1 
3 2GM  R e  

GM
g
R e2
1 2R e  
3/2
h 
t  1    1
3 g  R e  

23. (3)
Sol. Density is same
4 4
M  R 3, 2m  R '3
3 3
R' = 21/3R
GMm

R2
G2Mm
2 =
R '2
2 = 21/3 W

24. (4)

Sol.

Acceleration due to gravity will be zero at P therefore,


GM G9M

x 2
(8R  x)2
8R  x  3x
NUCLEUS-92, Rajeev Gandhi Nagar, Kota (Raj.) India 324005, Mob. 9358006181, 97831-97831 191
JEE MAIN 2021
x = 2R
Apply conservation of energy and consider velocity at P is zero.
1 GMm G9Mm Gmm G9Mm
mv 2    0 
2 R 7R 2R 6R
4 GM
V 
7 R

25. (4)
Sol. Option D is correct
42 3
T2  .r
GM
4 2 r 3
M .
G T2
by putting values
m  6 1023

26. (2)

Sol.

Gm 2
F 2
 mR2
(2R)
1 G

2 R3

27. (4)
Sol. Angular momentum conservation equation
v0x2 = v1x1
vx
v1 = 0 2
x1
28. (2)
Sol.

NUCLEUS-92, Rajeev Gandhi Nagar, Kota (Raj.) India 324005, Mob. 9358006181, 97831-97831 192
JEE MAIN 2021
2
MV
Fnet =
R
MV 2
2F  F1 
R
GMM GMM MV 2
2  
 2R   2R 
2 2
R
GM  4  2 
V
2

R  4 2 
GM  2 2  1
V
R
1 GM  2 2  1
V=
2 R
Option (2)

29. (4)

Sol.

 GM1 GM 2 
VA    
 r R 
 50 100 
  G  G = - 4G
 25 50 

30. (2)
M
Sol. Energy is maximum when mass is split equally so =2
m
31. (4)
g
Sol. g up  2
 r 
1  
 R
 r 
g down  g 1  
 R
2
g down  r  r 
 1  1  
g up  R  R 
 r   2r r 2  r r 2 r3
 1   1   2   1   2  3
 R  R R  R R R

NUCLEUS-92, Rajeev Gandhi Nagar, Kota (Raj.) India 324005, Mob. 9358006181, 97831-97831 193
JEE MAIN 2021
32. (2)

Sol.

1 GM1m GM 2 m
mV 2   0
2 r/2 r/2
1 2Gm
mV 2  (M1  M 2 )
2 r
4G(M1  M 2 )
V
r
option (2)

NUCLEUS-92, Rajeev Gandhi Nagar, Kota (Raj.) India 324005, Mob. 9358006181, 97831-97831 194
JEE MAIN 2021
CURRENT ELECTRICITY
1. A current through a wire depends on time as i = 0t + t2 where 0 = 20 A/s and  = 8 As–2. Find
the charge crossed through a section of the wire in 15 s. [JEE MAIN 2021 (FEB)]
(1) 2250 C (2) 11250 C (3) 2100 C (4) 260 C

2. A cell E1 of emf 6V and internal resistance 2is connected with another cell E2 of emf 4V and
internal resistance 8 (as shown in the figure). The potential difference across points X and Y is :
[JEE MAIN 2021 (FEB)]

(1) 10.0 V (2) 3.6 V (3) 5.6V (4) 2.0 V

3. A uniform metallic wire is elongated by 0.04 m when subjected to a linear force F. The
elongation, if its length and diameter is doubled and subjected to the same force will be _____ cm.
[JEE MAIN 2021 (FEB)]

4. A cylindrical wire of radius 0.5 mm and conductivity 5 × 107 S/m is subjected to an electric field
of 10 mV/m. The expected value of current in the wire will be x3 mA. The value of x is ____.
[JEE MAIN 2021 (FEB)]

5. In the given circuit of potentiometer, the potential difference E across AB (10m length) is larger
than E1 and E2 as well. For key K1 (closed), the jockey is adjusted to touch the wire at point J1 so
that there is no deflection in the galvanometer. Now the first battery (E1) is replaced by second
battery (E2) for working by making K1 open and K2 closed. The galvanometer gives then null
E a
deflection at J2. The value of 1 is , where a = _______. [JEE MAIN 2021 (FEB)]
E2 b

NUCLEUS-92, Rajeev Gandhi Nagar, Kota (Raj.) India 324005, Mob. 9358006181, 97831-97831 195
JEE MAIN 2021
6. The four arms of a Wheatstone bridge have resistances as shown in the figure. A galvanometer of
15 resistance is connected across BD. Calculate the current through the galvanometer when a
potential difference of 10V is maintained across AC. [JEE MAIN 2021 (MARCH)]

(1) 2.44 A (2) 2.44 mA


(3) 4.87 mA (3) 4.87 A

7. A conducting wire of length 'l', area of cross-section A and electric resistivity  is connected
between the terminals of a battery. A potential difference V is developed between its ends, causing
an electric current.
If the length of the wire of the same material is doubled and the area of cross-section is halved, the
resultant current would be : [JEE MAIN 2021 (MARCH)]
1 VA 3 VA
(1 (2)
4 ρl 4 ρl
1 ρl VA
(3) (4) 4
4 VA ρl

8. In the figure given, the electric current flowing through the 5k resistor is 'x' mA.
[JEE MAIN 2021 (MARCH)]

The value of x to the nearest integer is _________.

9. A resistor develops 500 J of thermal energy in 20s when a current of 1.5 A is passed through it. if
the current is increased from 1.5A to 3A, what will be the energy developed in 20s.
[JEE MAIN 2021 (MARCH)]
(1) 1500J (2) 1000J (3) 500J (4) 2000J

10. The energy dissipated by a resistor is 10mJ in 1s when an electric current of 2mA flows through it.
The resistance is __________ . [JEE MAIN 2021 (MARCH)]
(Round off to the nearest Integer).
Official Ans. by NTA (2500)

NUCLEUS-92, Rajeev Gandhi Nagar, Kota (Raj.) India 324005, Mob. 9358006181, 97831-97831 196
JEE MAIN 2021
11. A current of 10A exists in a wire of cross-sectional area of 5 mm with a drift velocity of 2 × 10–3
2

ms–1. The number of free electrons in each cubic meter of the wire is ________ .
[JEE MAIN 2021 (MARCH)]
6 25
(1) 2 × 10 (2) 625 × 10
25
(3) 2 × 10 (4) 1 × 1023

12. Two cells of emf 2E and E with internal resistance r1 and r2 respectively are connected in series to
an external resistor R (see figure). The value of R, at which the potential difference across the
terminals of the first cells becomes zero, is [JEE MAIN 2021 (MARCH)]

r1
(1) r1 + r2 (2) – r2
2
r1
(3) – r2 (4) r1 – r2
2

13. The voltage across the 10 resistor in the given circuit is x volt. [JEE MAIN 2021 (MARCH)]

The value of 'x' to the nearest integer is ________.

14. Consider a 72cm long wire AB as shown in the figure. The galvanometer jockey is placed P on
AB at a distance x cm from A. The galvanometer shows zero deflection.
[JEE MAIN 2021 (MARCH)]

The value of x, to the nearest integer, is

NUCLEUS-92, Rajeev Gandhi Nagar, Kota (Raj.) India 324005, Mob. 9358006181, 97831-97831 197
JEE MAIN 2021
15. Two wires of same length and thickness having specific resistances 6 cm and 3 cm
respectively are connected in parallel. The effective resistivity is  cm. The value of  , to the
nearest integer is _________. [JEE MAIN 2021 (MARCH)]

16.

The value of current in the 6 resistance is: [JEE MAIN 2021 (JULY)]
(1) 4A ` (2) 8A (3) 10A (4) 6A

17. A current of 5 A is passing through a non-linear magnesium wire of cross –section 0.04m2. At
every point the direction of current density is at an angle of 600 with the unit vector of area of
cross-section. The magnitude of electric field at every point of the conductor is :
(Resistivity of magnesium   44 x108 m ) [JEE MAIN 2021 (JULY)]
(1) 11 x 10-2 V/m (2) 11 x 10-7 V/m (3)11 x 10-5 V/m (4)11 x 10-3 V/m

18. In the given figure switches S1 and S2 are in open condition. the resistance across ab when the
switches S1 and S2 are closed is _________  . . [JEE MAIN 2021 (JULY)]

19. A Copper (Cu) rod of length 25 cm and cross-sectional area mm2 us joined with a similar
Aluminium (Al) rod as shown in figure. Find the resistance of the combination between the ends
A and B. [JEE MAIN 2021 (JULY)]
(Take Resistivity of Copper = 1.7 × 10–8m Resistivity of Aluminium = 2.6 × 10–8 m)

(1) 2.170 m (2) 1.420 m  (3) 0.0858 m (4) 0.858 m

NUCLEUS-92, Rajeev Gandhi Nagar, Kota (Raj.) India 324005, Mob. 9358006181, 97831-97831 198
JEE MAIN 2021
20. In an electric circuit, a cell of certain emf provides a potential difference of 1.25V across a load
resistance of 5. However, it provides a potential difference of 1V across a load resistance of 2.
x
The emf of the cell is given by V. Then the value of x is _______. [JEE MAIN 2021 (JULY)]
10

21. In the given figure, there is a circuit of potentiometer of length AB = 10 m. The resistance per unit
length is 0.1  per cm. Across AB, a battery of emf E and internal resistance ‘r’ is connected. The
maximum value of emf measured by this potentiometer is : [JEE MAIN 2021 (JULY)]

(1) 5V (2) 2.25 V (3) 6V (4) 2.75 V

22. An electric bulb rated as 200 W at 100 V is used in a circuit having 200 V supply. The resistance
‘R’ that must be put in series with the bulb so that the bulb delivers the same power is ______ 

[JEE MAIN 2021 (JULY)]

23. In the given potentiometer circuit arrangement , the balancing length AC is measured to be 250
cm. When the galvanometer connection is shifted from point (1) to point (2) in the given diagram,
1
the balancing length becomes 400 cm. the ratio of the emf of two cells, is :
2
[JEE MAIN 2021 (JULY)]

5 8 4 3
(1) (2) (3) (4)
3 5 3 2

NUCLEUS-92, Rajeev Gandhi Nagar, Kota (Raj.) India 324005, Mob. 9358006181, 97831-97831 199
JEE MAIN 2021
24. The given potentiometer has its wire of resistance 10 . When the sliding contact is in the middle
of the potentiometer wire, the potential drop across 2 resistor is : [JEE MAIN 2021 (JULY)]

40 40
(1) 10 V (2) 5 V (3) V (4) V
9 11

25. A 16  wire is bent to form a square loop. A 9V supply having internal resistance of 1  is
connected across one of its sides. The potential drop across the diagonals of the square loop
is______ × 10–1 V. [JEE MAIN 2021 (JULY)]

26. In the given figure, a battery of emf E is connected across a conductor PQ of length 'l' and
different area of cross-sections having radii r1 and r2 (r2 < r1) [JEE MAIN 2021 (JULY)]
r2

Choose the correct option as one moves from P to Q :


(1) Drift velocity of electron increases (2) Electric field decreases.
(3) Electron current decreases. (4) All of these

27.

A capacitor of capacitance C=1 F is suddenly connected to a battery of 100 volt through a


resistance R = 100  . The time taken for the capacitor to be charged to get 50 V is :
[Take ln 2 = 0.69] [JEE MAIN 2021 (JULY)]
–4 –4 –4
(1) 1.44 × 10 s (2) 3.33 × 10 s (3) 0.69 × 10 s (4) 0.30 × 10–4 s

28. In Bohr's atomic model, the electron is assumed to revolve in a circular orbit 0.5 Å. If the speed of
electron is 2.2 × 166 m/s, then the current associated with the electron will be _______ × 10–2 mA.
22
[Take  as ] [JEE MAIN 2021 (JULY)]
7

NUCLEUS-92, Rajeev Gandhi Nagar, Kota (Raj.) India 324005, Mob. 9358006181, 97831-97831 200
JEE MAIN 2021
0 0
29. The resistance of a conductor at 15 C is 16  and at 100 C is 20  . What will be the
temperature coefficient of resistance of the conductor? [JEE MAIN 2021 (JULY)]
0 -1 0 -1 0 -1
(1) 0.010 C (2) 0.033 C (3) 0.003 C (4) 0.0420C-1

30. For the circuit shown, the value of current at time t = 3.2 s will be ________A.
[JEE MAIN 2021 (JULY)]

[Voltage distribution V(t) is shown by Fig. (1) and the circuit is shown in Fig. (2)]
31. Due to cold weather a 1 m water pipe of cross–sectional area 1 cm2 is filled with ice at –10°C.
Resistive heating is used to melt the ice. Current of 0.5 A is passed through 4 kresistance.
Assuming that all the heat produced is used for melting, what is the minimum time required ?
(Given latent heat of fusion for water/ice = 3.33 × 105 J kg–1, specific heat of ice = 2 × 103 J kg–1
and density of ice = 103 kg / m3 [JEE MAIN 2021 (AUGUST)]
(1) 0.353 s (2) 35.3 s (3) 3.53 s (4) 70.6 s

32. A capacitor is connected to a 20 V battery through a resistance of 10. It is found that the
potential difference across the capacitor rises to 2 V in 1 μs. The capacitance of the capacitor is
....................μF.      [JEE MAIN 2021 (AUGUST)]

Given : In    0.105
10
9
(1) 9.52 (2) 0.95 (3) 0.105 (4) 1.85

33. A uniform heating wire of resistance 36 is connected across a potential difference of 240 V. The
wire is then cut into half and potential difference of 240 V is applied across each half separately.
The ratio of power dissipation in first case to the total power dissipation in the second case would
be 1 : x, where x is........... [JEE MAIN 2021 (AUGUST)]

34. If you are provided a set of resistance 2, 4, 6 and 8. Connect these resistance so as to
46
obtain an equivalent resistance of . [JEE MAIN 2021 (AUGUST)]
3
(1) 4 and 6 are in parallel with 2 and 8 in series
(2) 6 and 8 are in parallel with 2 and 4 in series
(3) 2 and 6 are in parallel with 4 and 8 in series
(4) 2 and 4 are in parallel with 6 and 8 in series

NUCLEUS-92, Rajeev Gandhi Nagar, Kota (Raj.) India 324005, Mob. 9358006181, 97831-97831 201
JEE MAIN 2021
35. An electric bulb of 500 watt at 100 volt is used in a circuit having a 200 V supply. Calculate the
resistance R to be connected in series with the bulb so that the power delivered by the bulb is 500 W.
[JEE MAIN 2021 (AUGUST)]
(1) 20  (2) 30  (3) 5  (4) 10 

36. An electric appliance supplies 6000 J/min heat to the system. If the system delivers a power of 90
W. How long it would take to increase the internal energy by 2.5 103 J ?
[JEE MAIN 2021 (AUGUST)]
(1) 2.5 × 102s (2) 4.1 × 101s (3) 2.4 × 103s (4) 2.5 × 101s

37. In the given figure, the emf of the cell is 2.2 V and if internal resistance is 0.6 . Calculate the
power dissipated in the whole circuit : [JEE MAIN 2021 (AUGUST)]

(1) 1.32 W (2) 0.65 W (3) 2.2 W (4) 4.4 W

38. What equal length of an iron wire and a copper – nickel alloy wire, each of 2 mm diameter
connected parallel to give an equivalent resistance of 3 ? [JEE MAIN 2021 (AUGUST)]
(Given resistivities of iron and copper – nickel alloy wire are 12 cm and 51 cm
respectively)
(1) 82 m (2) 97 m (3) 110 m (4) 90 m

39. The color coding on a carbon resistor is shown in the given figure. The resistance value of the
given resistor is : [JEE MAIN 2021 (AUGUST)]

(1) (5700  285) (2) (7500  750)


(3) (5700  375) (4) (7500  375)

40. For full scale deflection of total 50 divisions, 50 mV voltage is required in galvanometer. The
resistance of galvanometer if its current sensitivity is 2 div/mA will be :
[JEE MAIN 2021 (AUGUST)]
(1) 1 (2) 5 (3) 4 (4) 2

NUCLEUS-92, Rajeev Gandhi Nagar, Kota (Raj.) India 324005, Mob. 9358006181, 97831-97831 202
JEE MAIN 2021
41. The ratio of the equivalent resistance of the network (shown in figure) between the points a and b
when switch is open and switch is closed is x : 8. The value of x is _________.
[JEE MAIN 2021 (AUGUST)]

42. Five identical cells each of internal resistance 1  and emf 5V are connected in series and in
parallel with an external resistance 'R'. For what value of 'R', current in series and parallel
combination will remain the same ? [JEE MAIN 2021 (AUGUST)]
(1) 1  (2) 25  (3) 5  (4)10 

43. First, a set of n equal resistor of 10  each are connected in series to a battery of emf 20V and
internal resistance 10  . A current I is observed to flow. Then, the n resistor are connected in
parallel to the same battery. It is observed that the current is increased 20 times. then the value of n
is _______ . [JEE MAIN 2021 (AUGUST)]
44. The equivalent resistance of the given circuit between the terminals A and B is :
[JEE MAIN 2021 (AUGUST)]

9
(1) 0  (2) 3  (3)  (4) 1 
2

45. A resistor dissipates 192 J of energy in 1 s when a current of 4A is passed through it. Now, when
the current is doubled, the amount of thermal energy dissipated in 5 s in ________J.
[JEE MAIN 2021 (AUGUST)]

46. Consider a galvanometer shunted with 5  resistance and 2% of current passes through it.
What is the resistance of the given galvanometer ? [JEE MAIN 2021 (AUGUST)]
(1)300  (2)344  (3) 245  (4) 226 

47. A square shaped wire with resistance of each side 3  is bent to form a complete circle. The
resistance between two diametrically opposite points of the circle in unit of  will be ______.
[JEE MAIN 2021 (AUGUST)]

NUCLEUS-92, Rajeev Gandhi Nagar, Kota (Raj.) India 324005, Mob. 9358006181, 97831-97831 203
JEE MAIN 2021
48. The voltage drop across 15  resistance in the given figure will be _______V.
[JEE MAIN 2021 (AUGUST)]

ANSWER KEY
1. 2 2. 3 3. 2 4. 5 5. 1 6. 3 7. 1
8. 3 9. 4 10. 2500 11. 2 12. 2 13. 70 14. 48
15. 4 16. 3 17. 3 18. 10 19. 4 20. 15 21. 1
22. 50 23. 1 24. 3 25. 45 26. 1 27. 3 28. 112
29. 3 30. 1 31. 2 32. 2 33. 4 34. 4 35. 1
36. 1 37. 3 38. 2 39. 4 40. 4 41. 9 42. 1
43. 20 44. 4 45. 3840 46. 3 47. 3 48. 6

NUCLEUS-92, Rajeev Gandhi Nagar, Kota (Raj.) India 324005, Mob. 9358006181, 97831-97831 204
JEE MAIN 2021
SOLUTION
1. (2)
dq
Sol. i=   dq   idt
dt
15
q=   20t  8t
0
2
 dt
15
 2 3

q =  20t  8t 
 2 3 0
8 153
q = 10 × (15)2 +
3
q = 2250 + 9000
q = 11250 C

2. (3)
64 1
Sol. I=  A
10 5
1
Vx + 4 + 8 × – Vy = 0
5
Vx – Vy = –5.6  [Vx – Vy] = 5.6 V

3. (2)

Sol.


F  Y.A.

F
  .
Y.A.
F.
 
Y. r 2

 
r2
2
   r1 
2
 2
 
 1  1  r2 

2
1
=  2  
2
 2 1

 1 2

NUCLEUS-92, Rajeev Gandhi Nagar, Kota (Raj.) India 324005, Mob. 9358006181, 97831-97831 205
JEE MAIN 2021
 1
 2 
2
0.04

2
= 0.02 m
 2  2cm
Ans.  2

4. (5)
Sol. Conductivity  = 5 × 107 S/m
Radius r = 0.5 mm = 5 × 10–4 m
V
E = 10×10–3
m
J = E = 10 × 10–3 × 5 ×107
J = 5 × 105
i
 5  105
A
i = 5 × 105 × r2
= 5 × 105 ×  × (5 × 10–4)2
= 125 × 10–3 Amp
i = 125  mA
x 5
Ans. 5

5. (1)
Sol. Length of AB = 10 m
For battery E1, balancing length is l1
l1 = 380 cm [from end A]
For battery E2, balancing length is l2
l2 = 760 cm [from end A]
E l
Now, we know that 1  1
E 2 l2
E1 380 1 a
   
E 2 760 2 b
 a = 1 & b = 2.
a=1

NUCLEUS-92, Rajeev Gandhi Nagar, Kota (Raj.) India 324005, Mob. 9358006181, 97831-97831 206
JEE MAIN 2021
6. (3)

Sol.

x –10 x – y x– 0
  0
100 15 10
53x – 20y = 30 .......(1)
y –10 y – x y – 0
  0
60 15 5
17y – 4x = 10 ......(2)
on solving (1) & (2)
x = 0.865
y = 0.792
V = 0.073 R = 15
i = 4.87 mA
7. (1)
Sol. As per the question

ρ(2l ) 4 ρl V VA
Resistance    Current  
(A / 2) A R 4 ρl
8. (3)

Sol.

21
I  3mA
5 11

NUCLEUS-92, Rajeev Gandhi Nagar, Kota (Raj.) India 324005, Mob. 9358006181, 97831-97831 207
JEE MAIN 2021
9. (4)
Sol. 500 = (1.5)2 × R × 20
E = (3)2 × R × 20
E = 2000 J

10. (2500)
Sol. Q  i 2 RT
Q 10 103
R 2   2500Ω
i t 4 106 1

11. (2)
Sol. i = 10A, A = 5 mm2 = 5 × 10–6 m2
and vd = 2 × 10–3 m/s
We know, i = neAVd
 10 = n × 1.6 × 10–19 × 5 × 10–6 × 2 × 10–3
 n = 0.625 × 1028 = 625 × 1025

12. (2)

Sol.

3E
i
R  r1  r2
TPD = 2E – ir1 = 0
3E  r1
2E 
R  r1  r2
2R + 2r1 + 2r2 = 3r1
r1
R= – r2
2

13. (70)
50  20 100
Sol. R e q1  
70 7

NUCLEUS-92, Rajeev Gandhi Nagar, Kota (Raj.) India 324005, Mob. 9358006181, 97831-97831 208
JEE MAIN 2021
170
R eq 
7
 
 170 
v1   10  70v
170 
 
 7 

14. (48)
Sol. In Balanced conditions
12 x

6 72 – x
x = 48 cm

15. (4)
Sol.  in parallel
R 1R 2
R net 
R1  R 2

1 2

 A A
2A   
1 2
A A
 63
 2
2 63
4

16. (3)
Sol. Applying KCL at point P,
V  0 V  90 V  140
  0
6 5 20
 10V + 12V – 1080 + 3V – 420 = 0  V = 60
V0
 current in 6 = = 10A
6
Hence option 3.

17. (3)
Sol. I = J.A = JAcos()
 4 
5  J   cos  60 
 100 
J = 5 × 50 = 250 A/m2
Now, E  .J
= 44 × 10–8 × 250 = 11 × 10–5 V/m

NUCLEUS-92, Rajeev Gandhi Nagar, Kota (Raj.) India 324005, Mob. 9358006181, 97831-97831 209
JEE MAIN 2021
18. (10)
Sol. When switch S1 and S2 are closed

12  6 6  12
2
12  6 6  12
72 72
2  4  2  4  10
18 18

19. (4)
R1R 2 
Sol. R  . 1 2
R1  R 2 A 1  2
25 102 1.7  2.6 1016
R= 
3 106 4.3 108
R = 0.858 m

20. (15)

Sol.

ER
Terminal voltage v = iR =
Rr
E 5
1st  1.25 = ….(i)
5 r
E  2
2nd  1 = ….(ii)
2r
By (i) and (ii)
3 15
r  1, E  V  volt
2 10
 x = 15

21. (1)
Sol. Max. voltage that can be measured by this potentiometer will be equal to potential drop across AB
RAB = 10 × 0.1 × 100 = 100 ohm.
6 100
VAB =  10  6   5V
20  100 120

NUCLEUS-92, Rajeev Gandhi Nagar, Kota (Raj.) India 324005, Mob. 9358006181, 97831-97831 210
JEE MAIN 2021
22. (50)
V2
Sol. Power, P 
RB

V 2 100 100
RB  
P 200
RB = 50

To produce same power, same voltage (i.e. 100 V) should be across the bulb
Hence, R = RB
R = 50

23. (1)
Sol. E1 = k1 ……(i)
E1 + E2 = k2 ……(ii)
E1 250 5
 1  
E1  E 2 2 400 8
8E1 = 5E1 + 5E2
3E1 = 5E2
E1 5

E2 3

24. (3)

Sol.

20  V0 0  V0 20  V0
  0
5 5 2
2V0 V0
4  10  
5 2
4V  5V0
14  0
10
140
V0  Volt
9
NUCLEUS-92, Rajeev Gandhi Nagar, Kota (Raj.) India 324005, Mob. 9358006181, 97831-97831 211
JEE MAIN 2021
Potential difference across 2resistor is 20 – V0
 140 
That is  20   Volt
 9 
 40 
Hence answer is   Volt
 9 

25. (45)
Sol. Here assume current as

By KVL in outer loop


9 – 12i – 4 I = 0
16i = 9
9
8i = = 4.5
2
= 45 × 10–1

26. (1)

Sol.

Current is constant in conductor i = constant


dx
Resistance of element dR = 2
r
idx
dV = idR  2
r
dV i
E 
dx r 2
eE
&Vd 
m
 Vd E
1
 E 2
r
if r decreases, E will increase  Vd will increase

NUCLEUS-92, Rajeev Gandhi Nagar, Kota (Raj.) India 324005, Mob. 9358006181, 97831-97831 212
JEE MAIN 2021
27. (3)
 
t

Sol. V  V0 1  e RC 
 
 
t

50  100 1  e RC 
 
–4
t = 0.69 × 10 sec.

28. (112)
e e eV
Sol. I  
T 2 2r
1.6 1019  2.2 106  7
I
2  22  0.5 1010
= 1.12 mA
112 102 mA

29. (3)
Sol. 16  R o [1  (15  To )]
20  R o [1  (100  To )]
16 1   15
Assuming To = 0°C, as a general convention.      0.003C1
20 1   100

30. (1)
Sol. From graph voltage at t = 3.2 sec is 6 volt.

65
i
1
i=1A

31. (2)
Sol. mass of ice m = A = 103 × 10–4 × 1 = 10–1 kg
Energy required to melt the ice
Q = msT + mL
= 10–1 (2 × 103 × 10 + 3.33 × 105) = 3.53 × 104 J
2
1
Q = i RT  3.53 × 10 =   (4 × 103) (t)
2 4
2
Time = 35.3 sec
Option (2)

NUCLEUS-92, Rajeev Gandhi Nagar, Kota (Raj.) India 324005, Mob. 9358006181, 97831-97831 213
JEE MAIN 2021
32. (2)
Sol. V = V0 1  e t /RC 
2 = 20 1  e t /RC 
1
 e  t /RC
10
9
e–t/RC =
10
9
et/RC =
10
t  10  t
 n   C =
R n  
RC 9 10
9
6
10
C= = .95F
10  .105

33. (4)
V 2 (240)2
Sol. First case P1  
R 36
Second case Resistance of each half = 18 
(240)2 (240)2 (240) 2
P2   
18 18 9
P1 1

P2 4
x = 4.00

34. (4)

Sol.

35. (1)
Sol. 500 watt at 100 v

NUCLEUS-92, Rajeev Gandhi Nagar, Kota (Raj.) India 324005, Mob. 9358006181, 97831-97831 214
JEE MAIN 2021
P = Vi
500 = Vi
i = 5 Amp
V=i×R
R = 20

36. (1)
Sol. Q  U  W
Q U W
 
t t t
6000 J 2.5 103
  90
60 sec t
t  250 sec
option (1)

37. (3)

Sol.

1 1 1 1 1 6  3  2  4 15
     
R eq 4 8 12 6 24 24
24
R eq   1.6  R T  1.6  0.6  2.2
15
V 2 (2.2)
P   2.2W
RT 2.2
Option (3)

NUCLEUS-92, Rajeev Gandhi Nagar, Kota (Raj.) India 324005, Mob. 9358006181, 97831-97831 215
JEE MAIN 2021
38. (2)
R1R 2
Sol. 3
R1  R 2
(12 106 102 )  4 (51106 102 )  4

(2) 2 106 (2) 2 106
63 106 102   4
(2)2 106
  97m
Option (2)

39. (4)
Sol. R  75 102  5% of 7500
R  (7500  375)

40. (4)
50
Sol. I m ax 
 25mkA
2
V 50mV
R   2
I 25mA

41. (9)
3R
Sol. R eq open 
2
R  2R 4R
` R eq closed  2  
3R 3
R eq open 3R 3 9
  
R eq closed 2 4R 8
x  9

42. (1)
25 5
Sol. i1  i2 
5R 1
R
5
 1
i1  i 2  5  R    5  R
 5
4R = 4
R  1

NUCLEUS-92, Rajeev Gandhi Nagar, Kota (Raj.) India 324005, Mob. 9358006181, 97831-97831 216
JEE MAIN 2021
43. (20)
Sol. In series
Req = nR = 10 n
20 2
is = 
10  10n 1  n
in parallel
10
Req 
n
20 2n
` ip  
10
 10 1  n
n
ip
 20
is
 2n 
 
 1  n   20
 2 
 
 1 n 
n = 20

44. (4)

Sol.

3 3 / 2 9 / 2
R eq    1
33/ 2 9 / 2

NUCLEUS-92, Rajeev Gandhi Nagar, Kota (Raj.) India 324005, Mob. 9358006181, 97831-97831 217
JEE MAIN 2021
45. (3840)
Sol. E = i2 Rt
192 = 16 (R) (1)
R = 12 
E1 = (8)2 (12) (5)
= 3840 J

46. (3)

Sol.

0.02i Rg = 0.98i  5
Rg =245 
option (3)

47. (3)

Sol.

R eq  3

48. (6)

Sol.

 effective circuit diagram will be

NUCLEUS-92, Rajeev Gandhi Nagar, Kota (Raj.) India 324005, Mob. 9358006181, 97831-97831 218
JEE MAIN 2021

Potential drop across 6   1 6  6  VAB


 Hence potential drop across 15   6 volt = VAB

NUCLEUS-92, Rajeev Gandhi Nagar, Kota (Raj.) India 324005, Mob. 9358006181, 97831-97831 219
JEE MAIN 2021
CAPACITANCE
1. Two equal capacitors are first connected in series and then in parallel. The ratio of the equivalent
capacities in the two cases will be: [JEE MAIN 2021 (FEB)]
(1) 4 : 1 (2) 2 : 1 (3) 1 : 4 (4) 1 : 2
2. For changing the capacitance of a given parallel plate capacitor, a dielectric material of dielectric
constant K is used, which has the same area as the plates of the capacitor. The thickness of the
3
dielectric slab is d, where 'd' is the separation between the plates of parallel plate capacitor. The
4
new capacitance (C') in terms of original capacitance (C0) is given by the following relation :
[JEE MAIN 2021 (MARCH)]
3 K 4K 4K 4
(1) C ' C0 (2) C ' C0 (3) C ' C0 (4) C ' C0
4K 3 K3 3 K
3. In a parallel plate capacitor set up, the plate area of capacitor is 2m2 and the plates are separated by
1m. If the space between the plates are filled with a dielectric material of thickness 0.5m and area
2m2 (see fig.) the capacitance of the set-up will be ______ 0.
(Dielectric constant of the material = 3.2) [JEE MAIN 2021 (MARCH)]
(Round of to the Nearest Integer)

4. The equivalent resistance of series combination of two resistors is 's'. When they are connected in
parallel, the equivalent resistance is 'p'. If s = np, then minimum value for n is ______.
(Round off to the Nearest Integer) [JEE MAIN 2021 (MARCH)]
3
5. Four identical rectangular plates with length, l = 2 cm and breadth, b  cm are arranged as
2
x 0
shown in figure. The equivalent capacitance between A and C is . The value of x is _____.
d
(Round off to the Nearest Integer) [JEE MAIN 2021 (MARCH)]

NUCLEUS-92, Rajeev Gandhi Nagar, Kota (Raj.) India 324005, Mob. 9358006181, 97831-97831 220
JEE MAIN 2021
6. A parallel plate capacitor whose capacitance C is 14pF is charged by a battery to a potential
difference V = 12V between its plates. The charging battery is now disconnected and a porcelain
plate with k = 7 is inserted between the plates, then the plate would oscillate back and forth
between the plates with a constant mechanical energy of ______ pJ.
[JEE MAIN 2021 (MARCH)]
(Assume no friction)

7. A parallel plate capacitor has plate area 100m2 and plate separation of 10 m. The space between
the plates is filled up to thickness 5 m with a material of dielectric constant of 10. The resultant
capacitance of the system is 'x' pF. [JEE MAIN 2021 (MARCH)]
The value of 0 = 8.85 × 10–12 F.m–1.
The value of 'x' to the nearest integer is _______ .

8. A 2F capacitor C1 is first charged to a potential difference of 10V using a battery. Then the
battery is removed and the capacitor is connected to an uncharged capacitor C 2 of 8F. The charge
in C2 on equilibrium condition is _______ C. (Round off to the Nearest Integer)
[JEE MAIN 2021 (MARCH)]

9. A parallel plate capacitor with plate area ‘A’ and distance of separation ‘d’ is filled with a
dielectric. What is the capacity of the capacitor when permittivity of the dielectric varies as:
[JEE MAIN 2021 (JULY)]
 d
  x   0  kx, for  0  x  
 2
d 
  x   0  k  d  x  , for   x  d 
2 
2/kA

(1)  0  
kd kA
(2)
 2   2  kd 
2 ln  0 
 20 
kA  20 
(3) 0 (4) ln  
2  20  kd 

10. A balloon was moving upwards with a uniform velocity of 10 m/s. An object of finite mass is
dropped from the balloon when it was at a height of 75 m from the ground level. The height of the
balloon from the ground when object strikes the ground was around:
(take the value of g as 10 m/s2) [JEE MAIN 2021 (JULY)]
(1) 300 m (2) 200 m (3) 125 m (4)250 m

NUCLEUS-92, Rajeev Gandhi Nagar, Kota (Raj.) India 324005, Mob. 9358006181, 97831-97831 221
JEE MAIN 2021
11. If qf is the free charge on the capacitor plates and qb is the bound charge on the dielectric slab of
dielectric constant k placed between the capacitor plates , then bound charge qb can be expressed
as : [JEE MAIN 2021 (JULY)]
 1   1  1   1
(1) q b  q f  1   (2) q b  q f 1   (3) q b  q f  1   (4) q b  q f 1  
 k  k  k  k

12. In the reported figure, a capacitor is formed by placing a compound dielectric between the plates
of parallel plate capacitor. The expression for the capacity of the said capacitor will be :
(Given area of plate = A) [JEE MAIN 2021 (JULY)]

15 K0 A 15 K 0 A 25 K0 A 9 K 0 A
(1) (2) (3) (4)
34 d 6 d 6 d 6 d

13. Two capacitors of capacities 2C and C are joined in parallel and charged up to potential V. The
battery is removed and the capacitor of capacity C is filled completely with a medium of dielectric
constant K. The potential difference across the capacitors will now be :
[JEE MAIN 2021 (JULY)]
V V 3V 3V
(1) (2) (3) (4)
K2 K K2 K

14. A simple pendulum of mass 'm' length 'l' and charge '+q' suspended in the electric field produced
by two conducting parallel plates as shown. The value of deflection of pendulum in equilibrium
position will be. [JEE MAIN 2021 (JULY)]

 q C1  V2  V1    q C2  V2  V1  
(1) tan 1  x  (2) tan 1  x 
 mg  C1  C2  d  t    mg  C1  C2  d  t  
 q C2  V1  V2    q C1  V1  V2  
(3) tan 1  x  (4) tan 1  x 
 mg  C1  C2  d  t    mg  C1  C2  d  t  

NUCLEUS-92, Rajeev Gandhi Nagar, Kota (Raj.) India 324005, Mob. 9358006181, 97831-97831 222
JEE MAIN 2021
15. A parallel - plate capacitor with plate area A has separation d between the plates. Two dielectric
slabs of dielectric constant K1 and K2 of same area A/2 and thickness d/2 are inserted in the space
between the plates. The capacitance of the capacitor will be given by :
[JEE MAIN 2021 (AUGUST)]

0 A  1 K1K 2  ε0 A  1 K1K 2 
(1)    (2)   
d  2 K1  K 2  d  2 2  K1  K 2  
0 A  1 K1  K 2  0 A  1 2  K1  K 2  
(3)    (4)   
d 2 K1K 2  d 2 K1K 2 

16. The material field between the plates of a parallel plate capacitor has resistivity 200 m . The
value of capacitance of the capacitor is 2 pF. If a potential difference of 40 V is applied across the
plates of the capacitor, then the value of leakage current flowing out of the capacitor is : (given the
value of relative permittivity of material is 50) [JEE MAIN 2021 (AUGUST)]
(1) 9.0 A (2) 9.0 mA (3) 0.9 mA (4) 0.9 A

17. Three capacitors C1  2F, C2  6 F and C3  12 F are connected as shown in figure. Find the
ratio of the charges on capacitors C1 , C2, and C3 respectively :
[JEE MAIN 2021 (AUGUST)]

(1) 2 : 1 : 1 (2) 2 : 3 : 3 (3) 1 : 2 : 2 (4) 3 : 4 : 4

18. calculate the amount of charge on capacitor 4 F . The internal resistance of battery is 1  :
[JEE MAIN 2021 (AUGUST)]

(1) 8 C (2) Zero (3)16 C (4) 4 C

NUCLEUS-92, Rajeev Gandhi Nagar, Kota (Raj.) India 324005, Mob. 9358006181, 97831-97831 223
JEE MAIN 2021
19. A parallel plate capacitor of capacitance 200 F is connected to a battery of 200 V. A dielectric
slab of dielectric constant 2 is now inserted into the space between plates of capacitor while the
battery remain connected. The change in the electrostatic energy in the capacitor will be _____J.
[JEE MAIN 2021 (AUGUST)]

20. A capacitor of 50 F is connected in a circuit as shown in figure. The charge on the upper plate of
the capacitor is _______ C . [JEE MAIN 2021 (AUGUST)]

21. The circuit shown in the figure consists of a charged capacitor of capacity 3F and a charge of
30C. At time t = 0, when the key is closed, the value of current flowing through the 5 resistor is
'x' –A. The value of 'x to the nearest integer is ________. [JEE MAIN 2021 (AUGUST)]

ANSWER KEY
1. 3 2. 3 3. 3 4. 4 5. 2 6. 864 7. 161
8. 16 9. 2 10. 3 11. 2 12. 1 13. 3 14. 3
15. 1 16. 3 17. 3 18. 1 19. 4 20. 100 21. 2

NUCLEUS-92, Rajeev Gandhi Nagar, Kota (Raj.) India 324005, Mob. 9358006181, 97831-97831 224
JEE MAIN 2021
SOLUTION
1. (3)
Sol. For series combination

1 1 1 C
   C eq1 
C eq1 C C 2
For parallel combination

C eq 2 = C + C  C eq 2 = 2C
C
Ceq1   1
   2    1: 4
Ceq2 2C 4

2. (3)
Sol.

0 A
C0 
d
C' = C1 and C2 in series.
1 1 1
i.e.  
C ' C1 C2
1  3d / 4  d / 4
 
C ' 0 KA 0 A
1 d  3 K 
 
C ' 4 0 A  K 
4KC0
C' 
3  K 
NUCLEUS-92, Rajeev Gandhi Nagar, Kota (Raj.) India 324005, Mob. 9358006181, 97831-97831 225
JEE MAIN 2021
3. (3)
ε0 A 2ε A *
Sol. C  0
d d d
 d
2K 2 K
2  2ε0 4  3.2
  ε0
1 4.2
1
3.2
= 3.04 0

4. (4)
Sol. R1  R 2  s .....(1)
R 1R 2
 p ......(2)
R1  R 2
R1R2 = sp
R1R2 = np2
nR1R 2
R1 + R2 =
 R1  R 2 
R 
2
 R2
1
n
R 1R 2
for minimum value of n
R1 = R2 = R
 2R 
2

n = 4
R2

5. (2)
Sol.

2C0 2 0 A
Ceq  
3 3 d
2 0  3  3
Ceq    2    2  A = 1b=2× 
3d  2  2

NUCLEUS-92, Rajeev Gandhi Nagar, Kota (Raj.) India 324005, Mob. 9358006181, 97831-97831 226
JEE MAIN 2021
6. (864)
1  1 
Sol. Ui  14 12 12pJ  U  CV 2 
2  2 
= 1008 pJ
1008
Uf  pJ=144pJ  C m  kC 0 
7
Mechanical energy = U = 1008 – 144 = 864 pJ

7. (161)

Sol.

A = 100m2
k 0 A
Using C 
d
10 0 100 
C1  = 2000
5
 100 
C2  0  20 0
5
C1C2 4000 0
C1 & C2 are in series so Ceqv.   = 160.9 × 10–2 161pF
C1  C2 220

8. (16)
Sol 20 = (C1 + C2) V  V = 2 volt.
Q2 = C2V = 16C = 16

9. (2)

Sol.

Taking an element of width dx at a distance


x(x < d/2) from left plate

NUCLEUS-92, Rajeev Gandhi Nagar, Kota (Raj.) India 324005, Mob. 9358006181, 97831-97831 227
JEE MAIN 2021

dc =
 0  kx  A
dx
Capacitance of half of the capacitor
d/2 d/2
1 1 1 dx
   
C 0 dc A 0 0  kx
1 1  0  kd / 2 
 ln  
C kA  0 
Capacitance of second half will be same
C kA
Ceq  
2  2  kd 
2 ln  0 
 20 

10. (3)

Sol.

Object is projected as shown so as per motion under gravity


1
S = ut + at2
2
1
75  10t   10  t 2  t = 5 sec
2
Object takes t = 5 s to fall on ground Height of balloon from ground
H = 75 + ut
= 75 + 10 × 5 = 125 m

11. (2)

Sol.

When a dielectric is inserted in a capacitor


Due to free charge E  E0 only
E
After dielectric E' = 0
k
q B  qf 1  
1
 k

NUCLEUS-92, Rajeev Gandhi Nagar, Kota (Raj.) India 324005, Mob. 9358006181, 97831-97831 228
JEE MAIN 2021
12. (1)
1 d 2d 3d
Sol.   
Ceff K 0 A 3K 0 A 5K 0 A
15K 0 A
Ceff 
34d

13. (3)

Sol.

2CV  CV 3V
VC  
KC  2C K2

14. (3)

Sol.

Let E be electric field in air


Tsin   qE
Tcos   mg
qE
tan  
mg

NUCLEUS-92, Rajeev Gandhi Nagar, Kota (Raj.) India 324005, Mob. 9358006181, 97831-97831 229
JEE MAIN 2021
 CC 
Q   1 2  [V1  V2 ]
 C1  C2 
Q  C C  [V  V2 ]
E  1 2  1
A o  C1  C2  A o
 A C2 [V1  V2 ]
C1  o  E 
dt (C1  C2 )(d  t)
 q.E 
Now   tan 1  
 mg 
 q C2 (V1  V2 ) 
  tan 1   
 mg (C1  C2 )(d  t) 

15. (1)
A
0 A
2 K1K 2
Sol. Ceq   0
d d K1  K 2
A0  1 K1K 2 
   
d  2 K1  K 2 

16. (3)
Sol.   200m
C  2 1012 F
V = 40 V
K= 56
t
q q
i  0 e k0
k0 k0
2 1012  40
i max 
200  50  8.85 1012
80
 4  903A  0.9mA
10  8.85
Option (3)

NUCLEUS-92, Rajeev Gandhi Nagar, Kota (Raj.) India 324005, Mob. 9358006181, 97831-97831 230
JEE MAIN 2021
17. (3)

Sol.

(VD  V)C2  (VD  0)C3  0


(VD  V)6  (VD  0)12  0
VD  V  2VD  0
V
VD 
3
 V
q 2  (V  VD )C2   V   (6F)
 3
q 2  (4V)F
V
Q3  (VD  0)C3  12F  4VF
3
q1  (V  0)C1  V(2F)
q1 : q 2 : q3  2 : 4 : 4
q1 : q 2 : q3  1: 2 : 2

18. (1)
Sol. On simplifying circuit we get

No current in upper wire.


5
 VAB   4  4v.
4 1
  (Ceq )v  2  4  8C

NUCLEUS-92, Rajeev Gandhi Nagar, Kota (Raj.) India 324005, Mob. 9358006181, 97831-97831 231
JEE MAIN 2021
19. (4)
1
Sol. U  ( C)V 2
2
1
U  (KC  C)V 2
2
1
U  (2  1)CV 2
2
1
U   200 106  200  200
2
U  4J

20. (100)

Sol.

Pot. Diff. across each resistor = 2V


q = CV  50 106  2  100 106  100C

21. (2)
V 30 / 3
Sol. i0    2  10 –6
R 5  10 6

NUCLEUS-92, Rajeev Gandhi Nagar, Kota (Raj.) India 324005, Mob. 9358006181, 97831-97831 232
JEE MAIN 2021
MEC
1. A soft ferromagnetic material is placed in an external magnetic field. The magnetic domains :
(1) increase in size but no change in orientation. [JEE MAIN 2021 (FEB)]
(2) have no relation with external magnetic field.
(3) decrease in size and changes orientation.
(4) may increase or decrease in size and change its orientation.

2. In a ferromagnetic material, below the curie temperature, a domain is defined as :


[JEE MAIN 2021 (FEB)]
(1) a macroscopic region with zero magnetization.
(2) a macroscopic region with consecutive magnetic dipoles oriented in opposite direction.
(3) a macroscopic region with randomly oriented magnetic dipoles.
(4) a macroscopic region with saturation magnetization.

3. Magnetic fields at two points on the axis of a circular coil at a distance of 0.05m and 0.2 m from
the centre are in the ratio 8 : 1. The radius of coil is ____. [JEE MAIN 2021 (FEB)]
(1) 0.2 m (2) 0.1 m (3) 0.15 m (4) 1.0 m
4. A bar magnet of length 14 cm is placed in the magnetic meridian with its north pole pointing
towards the geographic north pole. A neutral point is obtained at a distance of 18cm from the
center of the magnet. If BH = 0.4 G, the magnetic moment of the magnet is (1G = 10–4 T)
[JEE MAIN 2021 (MARCH)]
3 –1 2 –1 –1
(1) 2.880 × 10 J T (2) 2.880 × 10 J T (3) 2.880 J T (4) 28.80 J T–1

5. A charge Q is moving⃗⃗⃗⃗ distance magnetic field ⃗ . Find the value of work done by ⃗ .
[JEE MAIN 2021 (MARCH)]
(1) 1 (2) Infinite (3) Zero (4) –1
6. A solenoid of 1000 turns per meter has a core with relative permeability 500. Insulated winding of
the solenoid carry an electric current of 5A. The magnetic flux density produced by the solenoid
is: [JEE MAIN 2021 (MARCH)]
(permeability of free space = 4 × 10–7 H/m)

(1) T (2) 2 × 10–3 T (3) T (4) 10–4T
5
7. A hairpin like shape as shown in figure is made by bending a long current carrying wire. What is
the magnitude of a magnetic field at a point P which lies on the centre of the semicircle?
[JEE MAIN 2021 (MARCH)]

0I 0I 0I 0I


(1)  2 –  (2)  2   (3)  2   (4)  2 – 
4r 4r 2r 2r

NUCLEUS-92, Rajeev Gandhi Nagar, Kota (Raj.) India 324005, Mob. 9358006181, 97831-97831 233
JEE MAIN 2021
8. Four identical long solenoids A,B,C and D are connected to each other as shown in the figure If
the magnetic field at the centre of A is 3T. the field at the center of C would be: (Assume that the
magnetic field is confined within the volume respective solenoid). [JEE MAIN 2021 (MARCH)]

(1) 12T (2) 6T (3) 9T (4) 1T

9. A loop of flexible wire of irregular shape carrying current is placed in an external magnetic field.
Identify the effect to the field on the wire. [JEE MAIN 2021 (MARCH)]
(1) Loop assumes circular shape with its plane normal to the field
(2) Loop assumes circular shape with its plane parallel to the field
(3) Wire gets stretched to become straight.
(4) Shape of the loop remains unchanged.

10. A proton and an -particle, having kinetic energies Kp and K, respectively, enter into a magnetic
field at right angles. [JEE MAIN 2021 (MARCH)]
The ratio of the radii of trajectory of proton to that of a-particle is 2 : 1. The ratio of KP : K is:
(1) 1 : 8 (2) 8 : 1 (3) 1 : 4 (4) 4 : 1
11. Figure A and B show two long straight wires of circular cross-section (a and b with a < b),
carrying current I Which is uniformly distributed across the cross-section. The magnitude of
magnetic field B varies with radius r and can be represented as :
[JEE MAIN 2021 (MARCH)]

(1) (2) (3) (4)

12. Choose the correct option : [JEE MAIN 2021 (JULY)]


(1) True dip is not mathematically related to apparent dip.
(2) True dip is less than apparent dip.
(3) True dip is always greater than the apparent dip.
(4) True dip is always equal to apparent dip.

NUCLEUS-92, Rajeev Gandhi Nagar, Kota (Raj.) India 324005, Mob. 9358006181, 97831-97831 234
JEE MAIN 2021
13. A deuteron and an alpha particle having equal kinetic energy enter perpendicular into a magnetic
r
field. Let rd and r be their respective radii of circular path. The value of d is equal to :
r
[JEE MAIN 2021 (JULY)]
1
(1) (2) 2 (3) 1 (4) 2
2

14. Two ions having same mass have charges in the ratio 1 : 2. They are projected normally in a
uniform magnetic field with their speed in the ratio 2 :3. The ratio of the radii of their circular
trajectories is : [JEE MAIN 2021 (JULY)]
(1) 1:4 (2) 4: 3 (3) 3 : 1 (4) 2 : 3

15. In a uniform magnetic field, the magnetic needle has a magnetic moment 9.85 × 10 –2 A/m2 and
moment of inertia 5 × 10–6 kg m2. If it performs 10 complete oscillations in 5 seconds then the
magnitude of the magnetic field is ________ mT. [Take 2 as 9.85] [JEE MAIN 2021 (JULY)]

16. There are two infinitely long straight current carrying conductors and they are held at right angles
to each other so that their common ends meet at the origin as shown in the figure given below. The
ratio of current in both conductor is 1 : 1. The magnetic field at point P is ____.
[JEE MAIN 2021 (AUGUST)]

0 I 0 I
(1)  x 2  y2   x  y  (2)  x 2  y2   x  y 
4xy   4xy  
 Ixy  Ixy
(3) 0  x 2  y2   x  y  (4) 0  x 2  y2   x  y 
4   4  

17. A coil in the shape of an equilateral triangle of side 10 cm lies in a vertical plane between the pole
pieces of permanent magnet producing a horizontal magnetic field 20 mT. The torque acting on
the coil when a current of 0.2 A is passed through it and its plane becomes parallel to the magnetic
field will be x 105 Nm. The value of x is ……… [JEE MAIN 2021 (AUGUST)]

18. If the maximum value of accelerating potential provided by a ratio frequency oscillator is 12 kV.
The number of revolution made by a proton in a cyclotron to achieve one sixth of the speed of
light is………… [JEE MAIN 2021 (AUGUST)]
[mp = 1.67 × 10 kg, e = 1.6 × 10 C, Speed of light = 3 × 108 m/s]
–27 –19

NUCLEUS-92, Rajeev Gandhi Nagar, Kota (Raj.) India 324005, Mob. 9358006181, 97831-97831 235
JEE MAIN 2021
19. The fractional change in the magnetic field intensity at a distance 'r' from centre on the axis of
current carrying coil of radius 'a' to the magnetic field intensity at the centre of the same coil is :
(Take r < < a) [JEE MAIN 2021 (AUGUST)]
2 2 2
3a 2a 2r 3 r2
(1) (2) (3) (4)
2 r2 3 r2 3 a2 2 a2

20. A coaxial cable consists of an inner wire of radius 'a' surrounded by an outer shell of inner and
outer radii 'b' and 'c' respectively. The inner wire carries an electric current i0 , which is distributed
uniformly across cross-sectional area. The outer shell carries an equal current in opposite direction
and distributed uniformly. What will be the ratio of the magnetic field at a distance x from the
axis when (i) x < a and (ii) a < x < b? [JEE MAIN 2021 (AUGUST)]
x 2
a 2
x 2
b2  a 2
(1) (2) (3) 2 2 (4)
a2 x2 b a x2

21. Two ions of masses 4 amu and 16 amu have charge +2e and + 3e respectively. These ions pass
through the region of constant perpendicular magnetic field. The kinetic energy of both ions is
same. Then : [JEE MAIN 2021 (AUGUST)]
(1) lighter ion will be deflected less than heavier ion
(2) lighter ion will be deflected more than heavier ion
(3) both ion will be deflected equally
(4) no ion will be deflected

22. A uniform conducting wire of length 24a, and resistance R is wound up as a current carrying coil
in theshape of an equilateral triangle of side 'a'and then in the form of a square of side 'a'. The coil
is connected to a voltage source V0.The ratio of magnetic moment of the coil in case of equilateral
triangle to that for square 1: y where y is ......... [JEE MAIN 2021 (AUGUST)]

23. A current of 1.5 A is flowing through a triangle, of side 9 cm each. The magnetic field at the
centroid of the triangle is : [JEE MAIN 2021 (AUGUST)]
(Assume that the current is flowing in the clockwise direction)
(1) 3  10-7 T, outside the plane of triangle
(2) 2 3 107 T , outside the plane of triangle
(3) 2 3 105 T , inside the plane of triangle
(4) 3105 T, inside the plane of triangle

ˆi  ˆj
24. The magnetic field vector of an electromagnetic wave is given by B  Bo cos(kz  t); where
2
ˆi, ˆj represent unit vector along x and y-axis respectively. At t = 0 s, two electric charges q1 of 4 

coulomb and q2 of 2  coulomb located at ( ) and ( ), respectively, have the same


velocity of 0.5 c î , (where c is the velocity of light). The ratio of the force acting on charge q1 to q2
is :- [JEE MAIN 2021 (AUGUST)]
(1) 2 2 :1 (2) 1: 2 (3) 2 :1 (4) 2 :1

NUCLEUS-92, Rajeev Gandhi Nagar, Kota (Raj.) India 324005, Mob. 9358006181, 97831-97831 236
JEE MAIN 2021
25. A long solenoid with 1000 turns/m has a core material with relative permeability 500 and volume
103 cm3. If the core material is replaced by another material having relative permeability of 750
with same volume maintaining same current of 0.75 A in the solenoid the fractional change in the
magnetic moment of the core would be approximately 
x 
 . Find the value of x.
 499 
[JEE MAIN 2021 (AUGUST)]

26. A coil having N turns is wound tightly in the form of a spiral with inner and outer radii 'a' and 'b'
respectively. Find the magnetic field at centre, when a current I passes through coil :
[JEE MAIN 2021 (AUGUST)]
0 IN I ab
(1) b
log e   (2) 0 log e  
2(b  a) a 8 ab
0 I  1 1   I a b
(3)   (4) 0 

4(a  b)  a b  8  a  b 

27. A small square loop of side 'a' and one turn is placed inside a larger square loop of side b and one
turn (b > > a). The two loops are coplanar with their centres coinciding. If a current I is passed in
the square loop of side 'b', then the coefficient of mutual inductance between the two loops is :
[JEE MAIN 2021 (AUGUST)]
 a 2
 8 2  b 2
 8 2
(1) 0 8 2 (2) 0 (3) 0 8 2 (4) 0
4 b 4 a 4 a 4 b

ANSWER KEY
1. 4 2. 4 3. 2 4. 3 5. 3 6. 1 7. 2
8. 4 9. 1 10. 4 11. 3 12. 2 13. 2 14. 2
15. 8 16. 1 17. 3 18. 543 19. 4 20. 1 21. 2
22. 3 23. 4 24. 3 25. 250 26. 1 27. 1

NUCLEUS-92, Rajeev Gandhi Nagar, Kota (Raj.) India 324005, Mob. 9358006181, 97831-97831 237
JEE MAIN 2021
SOLUTION
1. (4)
Sol. Soft ferromagnetic materials are materials which can be easily magnetised and demagnetised by
external magnetic field. When external field is applied, the domains experiences a net torque
hence change its orientation. Hence option (4) is correct

2. (4)
Sol. (4) conceptual

3. (2)
Sol. We know, the magnetic field on the axis of a current carrying circular ring is given by
0 2NIA
B

4 R 2  X 2 
3/ 2

3/2
B1 8  R   0.2  
2 2

   
B2 1  R 2   0.05 2 
 
4[R + (0.05) ] = [R + (0.2)2]
2 2 2

4R2 – R2 = (0.2)2 – 4 × (0.05)2


4R2 – R2 = (0.2)2 – (0.1)2
3R2 = 0.3 × 0.1
R2 = (0.1)2  R = 0.1

4. (3)

Sol.

2 0 m 7
i.e.   0.4  10 –4
4 r 2
r
m7
 2 10 –7  104
7 2
 18 
2 3/2

–4
= 0.4 × 10
4  10 – 2   373
3/2

m=
14
0.04   373
3/2
14 14
M = m × 14cm = m ×   = 4 × 10–4 × 7203.82 = 2.88J/t
100 14 100

NUCLEUS-92, Rajeev Gandhi Nagar, Kota (Raj.) India 324005, Mob. 9358006181, 97831-97831 238
JEE MAIN 2021
5. (3)
Sol. Since force on a point charge by magnetic field is always perpendicular to v[F  qV  B]
 Work by magnetic force on the point charge is zero.

6. (1)
Sol. B = nI = 0rnI
B = 4 × 10–7 × 500 × 1000 × 5
B =  Tesla

7. (2)
Sol. B = 2 × Bst.wire + B1oop
 i  i  
B  2 0  0  
4r 2r  2 
 0i
B  2  
4r

8. (4)

Sol.

 ~ i
 Bi
3
so, field at centre of C   1T
3

9. (1)
Sol. Every part (dl) of the wire is pulled by force i(dl) B acting perpendicular to current & magnetic
field giving it a shape of circle.

10. (4)
mv p mα
Sol. r  4
qB qB mp
rp pp q α 2
 
rα q p pα 1
pp 2q p 1
  2 
pα qα 2

NUCLEUS-92, Rajeev Gandhi Nagar, Kota (Raj.) India 324005, Mob. 9358006181, 97831-97831 239
JEE MAIN 2021
pp
1

Kp pp2 m
  1 4 
K pp mp

11. (3)
Sol. Graph for wire of radius R :

As b > a
Ba > Bb
i
Ba  0
2 a
i
Bb  0
2b

12. (2)
Sol. If apparent dip circle is at angle  with true dip circle then

Hence true dip () is less than apparent dip (')

NUCLEUS-92, Rajeev Gandhi Nagar, Kota (Raj.) India 324005, Mob. 9358006181, 97831-97831 240
JEE MAIN 2021
13. (2)
mv 2mk
Sol. r= 
qB qB
rd md q  2 2
    2
r m q d 4 1
Hence option (2).

14. (2)
mv1
mv R qB v q q v
Sol. R  1  1  1 2  2 1
qB R 2 mv 2 q1 v 2 q1 v 2
q2B
2 2 4
   
1 3 3

15. (8)
I
Sol. T  2
MB
B  8 104  8mT

16. (1)

Sol.

0 I
Bdue to wire (1) = sin 90  sin 1 
4y
0 I 1  x 
=  2 
………(1)
4 y x y 
2

 I
Bdue to wire (2) = 0  sin 90  sin 2 
4 y
0I 1  x 
=  2 
……..(2)
4y x y 
2

Total magnetic field


B = B1 + B2

NUCLEUS-92, Rajeev Gandhi Nagar, Kota (Raj.) India 324005, Mob. 9358006181, 97831-97831 241
JEE MAIN 2021

0 I  1 x 1 y 
B     
4  y y x 2  y 2 x x x 2  y 2 
 
0 I  x  y x 2  y2 
B   
4  xy xy x 2
 y 2


0 I  x  y x 2  y2 
B   
4  xy xy 
 
I
B  0  x 2  y2  (x  y) 
4xy  
Option (1)

17. (3)

Sol.

  M  B  MBsin 90
2
i 3
 MB  B
4
 3 105 N  m

18. (543)
Sol. V = 12 kV
Number of revolution = n
1
n  2  q P  V   m P  v 2P
2
n  2 1.6 1019 12 103
2
1  3 108 
 1.67 1027   
2  6 
n(38.4 × 10–16) = 0.2087 × 10–11
n = 543.4

19. (4)
0iR 2
Sol. Baxis 
2(R 2  X 2 )3/2
i
Bcentre  0
2R

NUCLEUS-92, Rajeev Gandhi Nagar, Kota (Raj.) India 324005, Mob. 9358006181, 97831-97831 242
JEE MAIN 2021
 0i
 Bcentre 
2a
0ia 2
 Baxis 
2(a 2  r 2 )3/2
 fractional change in magnetic field =
 0i  0ia 2

2a 2(a 2  r 2 )3/2 1
 1
 0i   r 2 
3/2

2a 1   2  
  a 
 3 r2  3 r2
 1  1  2
 2
 2a  2a
3/2
 r2   3 r2 
Note : 1  2   1  2 
 a   2a 
[True only if r < < a]
Hence option (4) is the most suitable option

20. (1)

Sol.

when x < a
 i 
B (2x)      x 2
1  2 
 a 
 i x2
B(2x)   
a2
 i x
B   ....(i)
1
2a 2
when a < x < b
B (2x)   i
2 
 i
B   ....(2)
2 2x

NUCLEUS-92, Rajeev Gandhi Nagar, Kota (Raj.) India 324005, Mob. 9358006181, 97831-97831 243
JEE MAIN 2021
x
i
B
1 
  2a 2 x2

B i a2
2 
2x

21. (2)
P 2mk
Sol. r 
qB qB
Given they have same kinetic energy
m
r 
q
r1 4 3 3
  
r2 2 16 4
4r1
r2  (r2 is for heavier ion and r1 is for lighter ion )
3

d
sin  
R
  Deflection
1

R
(R  Radius of path)
R2 > R1  2  1
22. (3)
24a
Sol. In triangle shape N t  8
3a
24a
In square N s  6
4a
M t N t IA t
 [ I will be same in both]
M3 Nsia S
3 2
8 a
 4
6 a2
NUCLEUS-92, Rajeev Gandhi Nagar, Kota (Raj.) India 324005, Mob. 9358006181, 97831-97831 244
JEE MAIN 2021
Mt 1

Ms 3
y3

23. (4)

Sol.

 i 
B  3  0 (sin 60  sin 60) 
 4r 
/2
tan 60 
r
2
9 10
where r  M
2 3
 B  3105 T
Current is flowing in clockwise direction so, B is inside plane of triangle by right hand rule.

24. (3)
Sol. F1  q(V  B)
  ˆi  ˆj    
F1  4 0.5iˆ  B0   cos  K.  0  
 2   K  
  
  ˆi  ˆj   3 
F1  2 0.5iˆ  B0   cos  K.  0  
 2   K  
  
cos   1, cos 3  1
F
 1 2
F2

25. (250)
M  250 1
Sol.   
M  500 2
1 x
  x ~ 250
2 499

NUCLEUS-92, Rajeev Gandhi Nagar, Kota (Raj.) India 324005, Mob. 9358006181, 97831-97831 245
JEE MAIN 2021
26. (1)

Sol.

N
No. of turns in dx width  dx
ba
b
 N  0i
 dB   
a  b  a
 dx
 2x
N0i b
B n 
2(b  a)  a 
option (1)

27. (1)

Sol.

 I 
B 0  2sin 45  4
 4 b / 2 
0 I 2
2 2 a
 b
 2 20a 2 0 a2
M   8 2
I b 4 b
option (1)

NUCLEUS-92, Rajeev Gandhi Nagar, Kota (Raj.) India 324005, Mob. 9358006181, 97831-97831 246
JEE MAIN 2021
EMI
1. The current (i) at time t = 0 and t =  respectively for the given circuit is :
[JEE MAIN 2021 (FEB)]

18E 5E 10E 5E 5E 18E 5E 10E


(1) , (2) , (3) , (4) ,
55 18 33 18 18 55 18 33

2. Figure shows a circuit that contains four identical resistors with resistance R = 2.0 , two identical
inductors with inductance L = 2.0 mH and an ideal battery with emf E = 9 V. The current 'i' just
after the switch 'S' is closed will be : [JEE MAIN 2021 (FEB)]

(1) 2.25 A (2) 3.0 A (3) 3.37 A (4) 9 A

3. A coil of inductance 2H having negligible resistance is connected to a source of supply whose


voltage is given by V = 3t volt. (where t is in second). If the voltage is applied when t = 0, then
the energy stored in the coil after 4s is _____ J. [JEE MAIN 2021 (FEB)]

4. A conducting bar of length L is free to slide on two parallel conducting rails as shown in the figure
[JEE MAIN 2021 (MARCH)]

Two resistors R1 and R2 are connected across the ends of the rails. There is a uniform magnetic
field ⃗ pointing into the page. An external agent pulls the bar to the left at a constant speed v.
The correct statement about the direction of induced currents I1 and I2 flowing through R1 and R2
respectively is:
NUCLEUS-92, Rajeev Gandhi Nagar, Kota (Raj.) India 324005, Mob. 9358006181, 97831-97831 247
JEE MAIN 2021
(1) Both I1 and I2 are in anticlockwise direction.
(2) Both I1 and I2 are in clockwise direction.
(3) I1 is in clockwise direction and I2 is in anticlockwise direction.
(4) I1 is in anticlockwise direction and I2 is in clockwise direction.

x
5. The magnetic field in a region is given by B  B0   kˆ A square loop of side d is placed with its
a
edges along the x and y axes. The loop is moved with a constant velocity v  v0î . The emf induced
in the loop is: [JEE MAIN 2021 (MARCH)]

B0 v02d B0 v 0 d B0 v0d 2 B0 v0d 2


(1) (2) (3) (4)
2a 2a a 2a

6. In a series LCR resonance circuit, if we change the resistance only, from a lower to higher value:
(1) The bandwidth of resonance circuit will increase. [JEE MAIN 2021 (MARCH)]
(2) The resonance frequency will increase.
(3) The quality factor will increase.
(4) The quality factor and the resonance frequency will remain constant.
7. The time taken for the magnetic energy to reach 25% of its maximum value, when a solenoid of
resistance R, inductance L is connected to a battery, is: [JEE MAIN 2021 (MARCH)]
L L L
(1) n5 (2) infinite (3) n2 (4) n10
R R R
8. The arm PQ of a rectangular conductor is moving from x = 0 to x = 2b outwards and then
inwards from x = 2b to x = 0 as shown in figure .A uniform magnetic field perpendicular to the
plane is acting from x = 0 to x = b. Indentify the graph showing the variation of different
quantities with distance : [JEE MAIN 2021 (JULY)]

(1) A-Flux, B-Power dissipated , C-EMF (2) A-Power dissipated, B-Flux, C-EMF
(3) A-Flux, B-EMF, C-Power dissipated (4) A-EMF, B-Power dissipated , C-Flux

NUCLEUS-92, Rajeev Gandhi Nagar, Kota (Raj.) India 324005, Mob. 9358006181, 97831-97831 248
JEE MAIN 2021
9. An inductor of 10 mH is connected to a 20 V battery through a resistor of 10 k and a switch
After a long time, when maximum current is set up in the circuit, the current is switched off. The
x
current in the circuit after 1 s is mA .Then x is equal to___________.  Take e1  0.37 
100
[JEE MAIN 2021 (JULY)]

10. A circular conducting coil of radius 1 m is being heated by the change of magnetic field B
passing perpendicular to the plane in which the coil is laid. The resistance of the coil is 2 
.The magnetic field is slowly switched off such that its magnitude changes in time as
4  t 
B  103 T 1  
  100 
The energy dissipated by the coil before the magnetic field is switched off completely is E =
_______mJ. [JEE MAIN 2021 (JULY)]

11. Consider an electrical circuit containing a two way switch 'S'. Initially S is open and then T 1 is connected to
T2. As the current in R = 6 attains a maximum value of steady state level, T1 is disconnected from T2 and
immediately connected to T3. Potential drop across r = 3  resistor immediately after T1 is connected to T3
is______V.
(Round off to the Nearest Integer) [JEE MAIN 2021 (JULY)]

12. In the given figure the magnetic flux through the loop increases according to the relation
B (t)  10t 2  20t, where  B is in milliwebers and t is in seconds . [JEE MAIN 2021 (JULY)]
The magnitude of current through R = 2 resistor at t = 5 s is ____ mA.

NUCLEUS-92, Rajeev Gandhi Nagar, Kota (Raj.) India 324005, Mob. 9358006181, 97831-97831 249
JEE MAIN 2021
13. A square loop of side 20 cm and resistance 1 is moved towards right with a constant speed v0.
The right arm of the loop is in a uniform magnetic field of 5T. The field is perpendicular to the
plane of the loop and is going into it. The loop is connected to a network of resistors each of value
4. What should be the value of v0 so that a steady current of 2 mA flows in the loop ?
[JEE MAIN 2021 (AUGUST)]

(1) 1 m/s (2) 1 cm/s (3) 102 m/s (4) 10–2 cm/s

14. For the given circuit the current i through the battery when the key in closed and the steady state
has been reached is_________. [JEE MAIN 2021 (AUGUST)]

(1) 6 A (2) 25 A (3) 10 A (4) 0 A

15. A circular coil of radius 0.8 cm and 20 turns is rotated about its vertical diameter with an angular
speed of 50 rad s–1 in a uniform horizontal magnetic field of 3.0 × 10–2 T. The maximum emf
induced in the coil will be …….. × 10–2 volt (rounded off to the nearest integer)
[JEE MAIN 2021 (AUGUST)]

16. An inductor coil stores 64 J of magnetic field energy and dissipates energy at the rate of 640 W
when a current of 8A is passed through it. If this coil is joined across an ideal battery, find the time
constant of the circuit in seconds : [JEE MAIN 2021 (AUGUST)]
(1) 0.4 (2) 0.8 (3) 0.125 (4) 0.2

17. A constant magnetic field of 1 T is applied in the x > 0 region. A metallic circular ring of radius 1
m is moving with a constant velocity of 1 m/s along the x-axis. At t = 0s , the centre O of the ring
is at x = - 1m. What will be the value of the induced emf in the ring at t = 1s? (Assume the
velocity of the ring does not change.) [JEE MAIN 2021 (AUGUST)]

NUCLEUS-92, Rajeev Gandhi Nagar, Kota (Raj.) India 324005, Mob. 9358006181, 97831-97831 250
JEE MAIN 2021

(1) 1 V (2) 2V (3) 2 V (4) 0 V

18. A bar magnet is passing through a conducting loop of radius R with velocity  . The radius of the
bar magnet is such that is just passes through the loop. The induced e.m.f. in the loop can be
represented by the approximate curve: [JEE MAIN 2021 (AUGUST)]

(1) (2)

(3) (4)

19. A coil is placed in a magnetic field B as shown below: [JEE MAIN 2021 (AUGUST)]

A current is induced in the coil because B is :


(1) Outward and decreasing with time .
(2) parallel to the plane of coil and decreasing with time.
(3) Outward and increasing with time.
(4) parallel to the plane of coil and increasing with time.

NUCLEUS-92, Rajeev Gandhi Nagar, Kota (Raj.) India 324005, Mob. 9358006181, 97831-97831 251
JEE MAIN 2021
ANSWER KEY
1. 4 2. 1 3. 144 4. 3 5. 3 6. 1 7. 3
8. 3 9. 74 10. 80 11. 3 12. 60 13. 2 14. 3
15. 60 16. 4 17. 3 18. 3 19. 1

SOLUTION
1. (4)
Sol. At t = 0 current through inductor is zero.
18
hence R eq   5  1 ||  5  4  
5
E 5E
i1  
18 / 5 18
At t = , inductor becomes a simple wire and now the circuit will be as shown in figure
33
hence R eq   5 || 5    4 ||1  ; || parallel 
10
E 10E
i2  
33 /10 33

2. (1)
Sol. Just after the switch is closed, inductor will behave like infinite resistance (open circuit) so the
circuit will look like

9 9
i   2.25
RR 4
Option (1) is correct

3. (144)
LdI
Sol. 
dt
4 I
3 t dt 2 dI 
0 0

NUCLEUS-92, Rajeev Gandhi Nagar, Kota (Raj.) India 324005, Mob. 9358006181, 97831-97831 252
JEE MAIN 2021

3
16  2I
2
I = 12
1 1
V  LI 2   2 12   144 J
2

2 2

4. (3)

Sol.

5. (3)
B0 (x  d)
Sol. E1  v0d
a
B (x)
E2  0 v0d
a
E net  E1  E 2
B0 v0d 2
E net 
a

6. (1)
Sol. Bandwidth = R/L
Bandwidth  R
So bandwidth will increase

NUCLEUS-92, Rajeev Gandhi Nagar, Kota (Raj.) India 324005, Mob. 9358006181, 97831-97831 253
JEE MAIN 2021
7. (3)
1 2
Sol. Magnetic energy = Li  25%
2
i0
ME  25%  i 
2
–Rt\L
i = i0(1 – R ) for charging
L
t n2
R

8. (3)
Sol. As rod moves in field area increases upto x = b then field is absent and again flux is generated on
return journey from x = b to x = 0. Thus plot A for flux.
d
e=– curve B for emf
dt
 Power dissipated = vi  curve C for power dissipated

9. (74)
V 20V
Sol. I max    2mA
R 10K
For LR – decay circuit
I = Imaxe–Rt/L
10103 1106
3
I  2mAe 1010
I = 2mA e–1
I = 2 × 0.37 mA
74
I= mA
100
x = 74

10. (80)
Sol.   BS
4  t 
  103 1    R
2

  100 
 t 
  4 103  (1) 2 1  
 100 
 d

dt
d   t 
  4  103 1  
dt   100  
 1 
  4 103  5
  4 10 V
 100 
When B = 0

NUCLEUS-92, Rajeev Gandhi Nagar, Kota (Raj.) India 324005, Mob. 9358006181, 97831-97831 254
JEE MAIN 2021
t
1– =0
100
t = 100 sec
2
Heat = t
R

Heat 
 4 10 5 2

100J
2 106
16 1010 100
Heat  J
2 106
Heat = 0.08J
Heat = 80 mJ

11. (3)
6
Sol. When T1 and T2 are connected, then the steady state current in the inductor I = = 1A
6
When T1 and T3 are connected then current through inductor remains same. So potential difference across
3
V  Ir  1 3  3 volt

12. (60)
d
Sol.   2t  20 mV
dt

i   10t  10 mA
R
at t = 5
i  60 mA

13. (2)
Sol. Equivalent circuit

V0 B 5  2mA 
i=  V0 = = 10–2 m/s = 1 cm/s
4 1 5  .2
option (2)

NUCLEUS-92, Rajeev Gandhi Nagar, Kota (Raj.) India 324005, Mob. 9358006181, 97831-97831 255
JEE MAIN 2021
14. (3)
Sol. In steady state, inductor behaves as a conducting wire.
So, equivalent circuit becomes

1 1 1 1
   1
R eq 3 3 3
 Req = 1
 Circuit becomes

30
i= = 10A
3

15. (60)
Sol. Maximum emf  = N AB
N = 20,  = 50, B = 3 ×10–2T
 = 20 × 50 ×  × (0.08)2 × 3 × 10–2 = 60.28 × 10–2
Rounded off to nearest integer = 60

16. (4)
1 2
Sol. U Li  64  L  2
2
i2 R  640
640
R  2  10
(8)
L 1
    0.2
R 5
Option (4)

17. (3)
Sol. emf = BLV
= 1. (2R).1
=2 V

NUCLEUS-92, Rajeev Gandhi Nagar, Kota (Raj.) India 324005, Mob. 9358006181, 97831-97831 256
JEE MAIN 2021
18. (3)

Sol.

 When magnet passes through centre region of solenoids, no current / Emf is induced in loop.
 while entering flux increases so negative induced emf
 While leaving flux decreases so positive induced emf.

19. (1)
Sol. B must not be parallel to the plane of coil for non zero flux and according to lenz low if B is
outward it should be decreasing for anticlockwise induced current.

NUCLEUS-92, Rajeev Gandhi Nagar, Kota (Raj.) India 324005, Mob. 9358006181, 97831-97831 257
JEE MAIN 2021
ALTERNATING CURRENT
1. A resonance circuit having inductance and resistance 2 × 10–4 H and 6.28  respectively oscillates
at 10 MHz frequency. The value of quality factor of this resonator is _______.
[ = 3.14] [JEE MAIN 2021 (FEB)]

2. A common transistor radio set requires 12V (D.C.) for its operation. The D.C. source is
constructed by using a transformer and a rectifier circuit, which are operated at 220 V (A.C.) on
standard domestic A.C. supply. The number of turns of secondary coil are 24, then the number of
turns of primary are _______. [JEE MAIN 2021 (FEB)]

3. A series LCR circuit is designed to resonate at an angular frequency 0 = 105 rad/s. The circuit
draws 16 W power from 120 V source at resonance. The value of resistance 'R' in the circuit is
___. [JEE MAIN 2021 (FEB)]

4. The angular frequency of alternating current in a L-C-R circuit is 100 rad/s. The components
connected are shown in the figure. Find the value of inductance of the coil and capacity of
condenser. [JEE MAIN 2021 (FEB)]

(1) 0.8 H and 150 F (2) 0.8 H and 250 F


(3) 1.33 H and 250 F (4) 1.33 H and 150 F

5. A transmitting station releases waves of wavelength 960 m. A capacitor of 2.56 F is used in the
resonant circuit. The self inductance of coil necessary for resonance is______× 10—8 H .
[JEE MAIN 2021 (FEB)]

6. An alternating current is given by the equation i= i1 sint + i2 cost. The rms current will be
[JEE MAIN 2021 (FEB)]
1
1 1 1 2 2 1
(1) 1
(2)  i1  i 2 
2
(3)
2

i1  i 2  2 (4)  i1  i2 

2 i12  i 
2 2
2
2 2

7. In a series LCR resonant circuit, the quality factor is measured as 100. If the inductance is
increased by two fold and resistance is decreased by two fold, then the quality factor after this
change will be_____. [JEE MAIN 2021 (FEB)]

NUCLEUS-92, Rajeev Gandhi Nagar, Kota (Raj.) India 324005, Mob. 9358006181, 97831-97831 258
JEE MAIN 2021
8. Find the peak current and resonant frequency of the following circuit (as shown in figure)
[JEE MAIN 2021 (FEB)]

(1) 0.2 A and 50 Hz (2) 0.2 A and 100 Hz (3) 2 A and 100 Hz (4) 2A and 50 Hz

9. An RC circuit as shown in the figure is driven by an AC source generating a square wave. The
output wave pattern monitored by CRO would look close to: [JEE MAIN 2021 (MARCH)]

(1) (2)

(3) (4)

10. A sinusoidal voltage of peak value 250 V is applied to a series LCR circuit, in which R = 8, L =
24 mH and C = 60F. The value of power dissipated at resonant condition is 'x' kW. The value of
x to the nearest integer is _______ . [JEE MAIN 2021 (MARCH)]
11. For the given circuit, comment on the type of transformer used :

(1) Auxilliary transformer (2) Auto transformer


(3) Step-up transformer (4) Step down transformer

12. An AC current is given by I = I1 sint + I2cost. A hot wire ammeter will give a reading:
[JEE MAIN 2021 (MARCH)]
I12 – I 22 I12  I 22 I1  I 2 I1  I 2
(1) (2) (3) (4)
2 2 2 2 2
13. Match List – I with List – II [JEE MAIN 2021 (MARCH)]
NUCLEUS-92, Rajeev Gandhi Nagar, Kota (Raj.) India 324005, Mob. 9358006181, 97831-97831 259
JEE MAIN 2021
List – I List – II

(a) Phase difference (i) ; current leads
2
between current and voltage
voltage in a purely
resistance AC circuit
(b) Phase difference (ii) zero
between current and voltage
in a pure inductive AC circuit

(c) Phase difference (iii) ; current lags
2
between current and voltage in voltage
a pure capacitive ac circuit
 X – XL 
(d) Phase difference (iv) tan –1  c 
 R 
between current and voltage in
an LCR series circuit
Choose the most appropriate answer from the options given below:
(1) (a) – (i), (b) – (iii), (c) – (iv), (d) – (ii)
(2) (a) – (ii), (b) – (iv),(c) – (iii),(d) – (i)
(3) (a) – (ii), (b) – (iii), (c) – (iv), (d) – (i)
(4) (a) – (ii), (b) – (iii), (c) – (i), (d) – (iv)

14. What happens to the inductive reactance and the current in a purely inductive circuit if the
frequency is halved? [JEE MAIN 2021 (MARCH)]
(1) Both, inductive reactance and current will be halved
(2) Inductive reactance will be halved and current will de doubled.
(3) Inductive reactance will be doubled and current will be halved.
(4) Both, inducting reactance and current will be doubled.

15. An AC source rated 220V, 50HZ is connected to a resistor. The time taken by the current to the
change from its maximum to the rms value is: [JEE MAIN 2021 (MARCH)]
(1) 2.5 ms (2) 25ms (3) 2.5s (4) 0.25 ms

16. In a series LCR circuit, the inductive reactance (XL) is 10 and the capacitive reactance (XC) is
4. The resistance (R) in the circuit is 6. The power factor of the circuit is:
[JEE MAIN 2021 (MARCH)]
1 1 1 3
(1) (2) (3) (4)
2 2 2 2 2
17. In an LCR series circuit , an inductor 30 mH and a resistor 1  are connected to an AC source of
angular frequency 300 rad/s. The value of capacitance for which, the current leads the voltage by
1
450 is   F. Then the value of x is _____. [JEE MAIN 2021 (JULY)]
x
NUCLEUS-92, Rajeev Gandhi Nagar, Kota (Raj.) India 324005, Mob. 9358006181, 97831-97831 260
JEE MAIN 2021
18. For a series LCR circuit with R = 100  L = 0.5 mH and C = 0.1 pF connected across 220 V-50
Hz AC supply, the phase angle between current and supplied voltage and the nature of the circuit
is : [JEE MAIN 2021 (JULY)]
0
(1) 0 , resistive circuit (2)  90 , predominantly inductive circuit
0

0
(3) 0 , resonance circuit (4)  900 , predominantly capacitive circuit

19. A series LCR circuit of R  5, L  20 mH and C  0.5 F is connected across an AC supply of
250 V, having variable frequency. The power dissipated at resonance condition is_____× 102 W.
[JEE MAIN 2021 (JULY)]

20. In a circuit consisting of a capacitance and a generator with alternating emf Eg = Eg0 sint , VC
and IC are the voltage and current. Correct phasor diagram for such circuit is :
[JEE MAIN 2021 (JULY)]

(1) (2)

(3) (4)

21. Match List-I with List-II : [JEE MAIN 2021 (JULY)]


List-I List-II
(a) 1 (i) Current is in phase with emf
L >
C
(b) 1 (ii) Current lags behind the applied emf
L =
C
(c) 1 (iii) Maximum current occurs
L <
C
(d) Resonant frequency (iv) Current leads the emf
Choose the correct answer from the options given below :
(1) (a) – (ii) ; (b) – (i) ; (c) - (iv) ; (d) – (iii) (2) (a) – (ii) ; (b) – (i) ; (c) - (iii) ; (d) – (iv)
(3) (a) – (iii) ; (b) – (i) ; (c) - (iv) ; (d) – (ii) (4) (a) – (iv) ; (b) – (iii) ; (c) - (ii) ; (d) – (i)

NUCLEUS-92, Rajeev Gandhi Nagar, Kota (Raj.) India 324005, Mob. 9358006181, 97831-97831 261
JEE MAIN 2021
22. A 10 resistance is connected across 220 V – 50Hz AC apply. The time taken by the current to
change from its maximum value to the rms value is : [JEE MAIN 2021 (JULY)]
(1) 2.5 ms (2) 1.5 ms (3) 3.0 ms (4) 4.5 ms

23. Two circuits are shown in the figure (a) & (b). At a frequency of ________ rad/s the average
power dissipated in one cycle will be same in both the circuits. [JEE MAIN 2021 (JULY)]

24. A 0.07 H inductor and a 12  resistor are connected in series to a 220 V, 50 Hz ac source. The
approximate current in the circuit and the phase angle between current and source voltage are
22
respectively. [take  as ] [JEE MAIN 2021 (JULY)]
7
 11   11 
(1) 8.8 A and tan–1   (2) 88 A and tan–1  
6 6
 11   11 
(3) 0.88 A and tan–1   (4) 8.8 A and tan–1  
6 6

25. A 100  resistance , a 0.1 F capacitor and an indicator are connected in series across a 250 V
supply at variable frequency. Calculate the value of inductance of inductor at which resonance will
occur. Given that the resonant frequency is 60 Hz. [JEE MAIN 2021 (JULY)]
(1) 0.70 H (2) 70.3 mH (3) 7.03 × 10–5 H (4) 70.3 H
26. In the given circuit the AC source has  = 100 rad s–1. Considering the inductor and capacitor to
be ideal, what will be the current I flowing through the circuit? [JEE MAIN 2021 (JULY)]

(1) 5.9 A (2) 4.24 A (3) 0.94 A (4) 6 A

27. A series LCR circuit driven by 300 V at a frequency of 50 Hz contains a resistance R  3k, an
inductor of inductive reactance X L  250  and an unknown capacitor. The value of capacitance
to maximize the average power should be : (Take   10)
2
[JEE MAIN 2021 (AUGUST)]
(1) 4 F (2) 25 F (3) 400 F (4) 40 F

NUCLEUS-92, Rajeev Gandhi Nagar, Kota (Raj.) India 324005, Mob. 9358006181, 97831-97831 262
JEE MAIN 2021
28. An ac circuit has an inductor and a resistor of resistance R in series, such that XL = 3R. Now, a
capacitor is added in series such that XC = 2R. The ratio of new power factor with the old power
factor of the circuit is 5 : x . The value of x is ___________. [JEE MAIN 2021 (AUGUST)]

29. The alternating current is given by [JEE MAIN 2021 (AUGUST)]


  2  
i   42 sin  t   10  A
  T  
The r.m.s. value of this current is ........A .

30. At very high frequencies, the effective impendence of the given circuit will be __________  .
[JEE MAIN 2021 (AUGUST)]

31. In an ac circuit, an inductor, a capacitor and a resistor are connected in series with XL = R = XC.
Impedance of this circuit is : [JEE MAIN 2021 (AUGUST)]
2
(1) 2R (2) Zero (3) R (4) R 2

ANSWER KEY
1. 2000 2. 440 3. 900 4. 2 5. 10.00 6. 1 7. 282.84
8. 1 9. 3 10. 4 11. 3 12. 2 13. 4 14. 2
15. 1 16. 3 17. 3 18. 4 19. 125 20. 3 21. 1
22. 1 23. 500 24. 1 25. 4 26. 2 27. 1 28. 1
29. 11 30. 2 31. 3

NUCLEUS-92, Rajeev Gandhi Nagar, Kota (Raj.) India 324005, Mob. 9358006181, 97831-97831 263
JEE MAIN 2021
SOLUTION
1. (2000)
Sol. Given : L = 2 × 10–4 H
R = 6.25 
f = 10 MHz = 107 Hz
Since quality factor,
L L
Q = 0  2f
R R
2 104
 Q = 2 × 107 ×
6.28
3
Q = 2 × 10 = 2000
 Ans. is 2000 

2. (440)
N P VP
Sol. 
NS VS
N P 220

24 12
220  24
NP =
12
NP = 440

3. (900)
Sol. At resonance
V2
P
R
V 2 100 
2

R 
P 16
= 900 

4. (2)
15 1
Sol. Current through 60 resistance  A
60 4
1
thus capacitor current = A
4
VC = I XC
1 1
10  
4 C
1 1
C    250 F
40 4000

NUCLEUS-92, Rajeev Gandhi Nagar, Kota (Raj.) India 324005, Mob. 9358006181, 97831-97831 264
JEE MAIN 2021
Now,
20 1
current through 40  resistance =  A
40 2
1 1 1
thus current through inductor =   A
2 4 4
1
VL  IX L  L
4
1
20   100  L
4
 L = 0.8 H

5. (10.00)
Sol.  = 960 m
C = 2.56 F = 2.56 × 10–6 F
c = 3 × 108 m/s
L=?
1
Now at resonance, 0 
LC
[Resoant frequency]
1
2f0 
LC
c c 1
On substituting f 0  , we have 2 
  LC
c2 1
Squaring both sides : 42 
 2
LC
 
2
4 10  3 108 1

 960  L  2.56 106
2

1 4 10  9 1016  2.56 106


 
L 960  960
–8
 L =10 ×10 H

6. (1)
Sol. i = i1 sint + i2sin(t + 90)
i  i12  i 22 sin  t   
i0 i2  i2
i rms   1 2
2 2

NUCLEUS-92, Rajeev Gandhi Nagar, Kota (Raj.) India 324005, Mob. 9358006181, 97831-97831 265
JEE MAIN 2021
7. (282.84)
X L 1 L L
Sol. Q L   
R R LC R R C
2L
Q'   2 2Q  2 2 100  = 282.84
R
  C
2
8. (1)

Sol.

as given z = z  x L 
 x C  R2
x L  L  100 100  103  0
1 1
xC    10
C 100 100 106

10 100
2
z  R 2  902  1202  30  5  150
v 30 1
i peak    amp  0.2amp & For resonant frequency
z 150 5
1 1 1
 L   2  
C LC LC
1 1 100 10 100
&f     50Hz
2 LC 2 100 103 100 106 2 2
as 10  

9. (3)

Sol.

For t1 – t2 Charging graph


t2 - t3 Discharging graph

10. (4)
Sol. At resonance power (P)
 250 / 2 
2
 Vrms 
2

P P  3906.25W
R 8
 4kW

NUCLEUS-92, Rajeev Gandhi Nagar, Kota (Raj.) India 324005, Mob. 9358006181, 97831-97831 266
JEE MAIN 2021
11. (3)
P 60
Sol. VS    545.45
i 0.11
Vp = 220
Vs > VP
 Step up transformer

12. (2)
Sol. I = I1 sin t + I2 cos t = Iu sin(wt +  )
Where I0  I12  I22

I0 I2  I2
 Irms   1 2
2 2

13. (4)

Sol.

VL – Vc X L – X C
(d) tan   
Vr R

14. (2)
Sol. X L  L
v
i 0
L

15. (1)
Sol. i  i0 cos(t)
i  i 0 at t  0
i 
i  0 at t 
2 4
  1
t  
4 4(2f ) 8f
1
t  2.5ms
400

NUCLEUS-92, Rajeev Gandhi Nagar, Kota (Raj.) India 324005, Mob. 9358006181, 97831-97831 267
JEE MAIN 2021
16. (3)

Sol.

We know that power factor is cos  .


R
cos   ......(1)
Z

Z  R 2   X L – XC 
2
.......(2)
(L –1/C)

6
 Z  62  10 – 4
2
 Z  6 2 | cos  
6 2
1
cos  
2
17. (3)
xC  xL
Sol. tan  
R
x  xL
tan 45  C
R
xC  xL  R
1
 L  R
C
1
 300  0.03  1
C
1 1 1
 10 ; C 
C 10 10  300
1
C   10 3
3
X=3

18. (4)
Sol. R = 100
1 1011
X L  L  50103 XC  
C 100
NUCLEUS-92, Rajeev Gandhi Nagar, Kota (Raj.) India 324005, Mob. 9358006181, 97831-97831 268
JEE MAIN 2021
XC  XL
& XC  XL  R

19. (125)
Sol. XL = XC (due to resonance)
V V
Z  R so i mm  
Z R
V 2
250  250
  125 102 W
R 5

20. (3)

Sol. In capacitor, current leads voltage by
2

21. (1)
Sol. (a) For xL > xC , voltage leads the current
(ii)
(b) For xL = xC, voltage & current are in same phase
(i)
(c) For xL < xC, current leads the voltage
(iv)
(d) For resonant frequency xL = xC, current is maximum
(iii)

22. (1)

Sol.

V = 220V/50Hz
 i = i0sint
When i = i0

i0 = i0 sin t1  t1 = …..(i)
2
i0
When i =
2
i0 
 i0 sin t 2  t 2   (ii)
2 4
Time taken by current from maximum value to rms value
    1 1
  t1  t 2        sec = 2.5 ms
2 4 4 4  2f 8  50 400

NUCLEUS-92, Rajeev Gandhi Nagar, Kota (Raj.) India 324005, Mob. 9358006181, 97831-97831 269
JEE MAIN 2021
23. (500)
Sol. For figure (a)
v2mms
Pavg 
R
2 2
vms vmss
 R  1
Z2 R
R2 = Z2
25    xC  xL 
2
 52 
25   x c  x L   25
2

1
xc  xL   L
C
1 106
 
2

LC 0.1 40
 = 500

24. (1)
1  X 
Sol.  = tan  L  XL = L
 R 
22
XL = 2 × × 50 × 0.07 = 22
7
 = tan–1  
22
R = 12 
 12 
 11 
  = tan 1  
6
Z= X 2L  R 2  22.059
V 220
I=   8.77A
Z 25.059
25. (4)
Sol. C  0.1 F  107 F
Resonant frequency = 60 Hz
1
o 
LC
1 1
2f o  L 2 2
LC 4 f 0 C
by putting value L 70.3Hz
26. (2)
By Nucleus (Bonus)
2
 1 
Sol. ZC    R
2

 C 

NUCLEUS-92, Rajeev Gandhi Nagar, Kota (Raj.) India 324005, Mob. 9358006181, 97831-97831 270
JEE MAIN 2021
2
 1 
  6 
 1002
 100 100 10 
ZC  (100)2  (100)2
 100 2
ZL  (L)2  R 2

(100  0.5)2  502


 50 2
200 200
iC    2
z C 100 2
200 200
iL   2 2
z L 50 2
100 1
cos 1    1  45
10 2 2
50 1
cos 2    2  45
50 2 2

I  IC2  IL2  2  8  10
I = 3.16 A

27. (1)
Sol. For maximum average power
XL = XC
1
250 
2(50)C
C  4 106
Option (1)

28. (1)

Sol.

NUCLEUS-92, Rajeev Gandhi Nagar, Kota (Raj.) India 324005, Mob. 9358006181, 97831-97831 271
JEE MAIN 2021
R R
cos   cos  
R 2  3R 2 R2  R2
1 1
 
10 2
cos  ' 10 5
  c  1
cos 2 1

29. (11)
Sol. 2
f rms  f1rms
2
 f 2rms
2

2
 42 
    10
2

 2 
 121  f rms  11 A

30. (2)
Sol. X L  2fL
f is very large
 XL is very large hence open circuit.
1
Xc 
2fC
f is very large.
 Xc is very small, hence short circuit.
Final circuit

2 2
Zeq  1  2
22

31. (3)
Sol. Z  (XL  XC )2  R 2  R XL  XC
option (3)

NUCLEUS-92, Rajeev Gandhi Nagar, Kota (Raj.) India 324005, Mob. 9358006181, 97831-97831 272
JEE MAIN 2021
GEOMETRICAL OPTICS
1. A short straight object of height 100 cm lies before the central axis of a spherical mirror whose
focal length has absolute value |f| = 40cm. The image of object produced by the mirror is of
height 25 cm and has the same orientation of the object. One may conclude from the information :
[JEE MAIN 2021 (FEB)]
(1) Image is real, same side of concave mirror.
(2) Image is virtual, opposite side of concave mirror.
(3) Image is real, same side of convex mirror.
(4) Image is virtual, opposite side of convex mirror.
2. Given below are two statements : one is labelled as Assertion A and the other is labelled as
Reason R. [JEE MAIN 2021 (FEB)]
Assertion A : For a simple microscope, the angular size of the object equals the angular size of
the image.
Reason R : Magnification is achieved as the small object can be kept much closer to the eye than
25 cm and hence it subtends a large angle. In the light of the above statements, choose the most
appropriate answer from the options given below :
(1) A is true but R is false
(2) Both A and R are true but R is NOT the correct explanation of A.
(3) Both A and R are true and R is the correct explanation of A
(4) A is false but R is true
3. A point source of light S, placed at a distance 60 cm infront of the centre of a plane mirror of
width 50 cm, hangs vertically on a wall. A man walks infront of the mirror along a line parallel to
the mirror at a distance 1.2 m from it (see in the figure). The distance between the extreme points
where he can see the image of the light source in the mirror is .......... cm.
[JEE MAIN 2021 (FEB)]

4. Three rays of light, namely red (R), green (G) and blue (B) are incident on the face PQ of a right
angled prism PQR as shown in figure. [JEE MAIN 2021 (MARCH)]

The refractive indices of the material of the prism for red, green and blue wavelength are 1.27,
1.42 and 1.49 respectively. The colour of the ray(s) emerging out of the face PR is:
(1) green (2) red (3) blue and green (4) blue

NUCLEUS-92, Rajeev Gandhi Nagar, Kota (Raj.) India 324005, Mob. 9358006181, 97831-97831 273
JEE MAIN 2021
5. The angle of deviation through a prism is minimum when : [JEE MAIN 2021 (MARCH)]

(A) Incident ray and emergent ray are symmetric to the prism
(B) The refracted ray inside the prism becomes parallel to its base
(C) Angle of incidence is equal to that of the angle of emergence
(D) When angle of emergence is double the angle of incidence
Choose the correct answer from the options given below :
(1) Statements (A), (B) and (C) are true (2) Only statement (D) is true
(3) Only statements (A) and (B) are true (4) Statements (B) and (C) are true

6. The refractive index of a converging lens is 1.4. what will be the focal length of this lens if it is
placed in a medium of same refractive index ? (Assume the radii of curvature of the faces of lens
are R1 and R2 respectively) [JEE MAIN 2021 (MARCH)]
R 1R 2
(1) 1 (2) Infinite (3) (4) Zero
R1 – R 2

7. A deviation of 2° is produced in the yellow ray when prism of crown and flint glass are
achromatically combined. Taking dispersive powers of crown and flint glass as 0.02 and 0.03
respectively and refractive index for yellow light for these glasses are 1.5 and 1.6 respectively.
The refracting angles for crown glass prism will be ________° (in degree)
(Round off to the Nearest Integer) [JEE MAIN 2021 (MARCH)]

8. The thickness at the centre of a plano convex lens is 3 mm and the diameter is 6 cm. If the speed
of light in the material of the lens is 2 × 108 ms–1. The focal length of the lens is _______ .
[JEE MAIN 2021 (MARCH)]
(1) 0.30 cm (2) 15cm (3) 1.5 cm (4) 30cm

9. The image of an object placed in air formed by a convex refracting surface is at a distance of 10m
2rd
behind the surface. The image is real and is at of the distance of the object from the surface.
3
2
The wavelength of light inside the surface is times the wavelength in air. The radius of the
3
x
curved surface is m the value of 'x' is _________. [JEE MAIN 2021 (MARCH)]
13

10. The relative permittivity of distilled water is 81. The velocity of light in it will be :
(Given r = 1) [JEE MAIN 2021 (JULY)]
(1) 4.33 × 107 m/s (2) 2.33 × 107 m/s (3) 3.33 × 107 m/s (4) 5.33 × 107 m/s

NUCLEUS-92, Rajeev Gandhi Nagar, Kota (Raj.) India 324005, Mob. 9358006181, 97831-97831 274
JEE MAIN 2021
11. Region I and II are separated by a spherical surface of radius 25 cm. An object is kept in region I
at a distance of 40 cm from the surface. The distance of the image from the surface is :
[JEE MAIN 2021 (JULY)]

(1) 55.44 cm (2) 9.52 cm (3)18.23 cm (4) 37.58 cm

12. A ray of light passes from a denser medium to a rarer medium at an angle of incidence i. The
reflected and refracted rays make an angle of 90° with each other. The angle of reflection and
refraction are respectively r and r'. The critical angle is given by : [JEE MAIN 2021 (JULY)]

(1) sin–1(cotr) (2) tan–1(sini) (3) sin–1(tanr') (4) sin–1(tanr)

13. A ray of light passing through a prism    3  suffers minimum deviation. It is found that the
angle of incidence is double the angle of refraction within the prism. Then, the angle of prism is
______ (in degree) [JEE MAIN 2021 (JULY)]

14. A ray of laser of a wavelength 630 nm is incident at an angle of 30o at the diamond-air interface. It
is going from diamond to air. The refractive index of diamond is 2.42 and that of air is 1. Choose
the correct option. [JEE MAIN 2021 (JULY)]
o
(1) angle of refraction is 24.41 (2) angle of refraction is 30o
(3) refraction is not possible (4) angle of refraction is 53.4o

15. A prism of refractive index  and angle of prism A is placed in the position of minimum angle of
deviation. If minimum angle of deviation is also A , then in terms of refractive index. value of A
is. [JEE MAIN 2021 (JULY)]
    1  
(1) 2 cos 1   (2) sin 1   (3) sin 1   (4) cos 1  
2 2  2  2

NUCLEUS-92, Rajeev Gandhi Nagar, Kota (Raj.) India 324005, Mob. 9358006181, 97831-97831 275
JEE MAIN 2021
4
16. A ray of light entering from air into a denser medium of refractive index . as shown in the
3
figure. The light ray suffers total internal reflection at the adjacent surface as shown. The
maximum value of angle  should be equal to : [JEE MAIN 2021 (JULY)]

7 5 7 5
(1) sin 1 (2) sin 1 (3) sin 1 (4) sin 1
3 4 4 3

17. The expected graphical representation of the variation of angle of deviation '  ' with angle of
incidence 'i' in a prism is : [JEE MAIN 2021 (JULY)]

(1) (2) (3) (4)

18. A prism of refractive index n1 and another prism of refractive index n2 are stuck together (as
shown in the figure). n1 and n2 depend on , the wavelength of light, according to the relation
10.8 1014 1.8 1014
n1 = 1.2 + and n 2 = 1.45 + The wavelength for which rays incident at any
2 2
angle on the interface BC pass through without bending at that interface will be ______ nm.
[JEE MAIN 2021 (JULY)]

NUCLEUS-92, Rajeev Gandhi Nagar, Kota (Raj.) India 324005, Mob. 9358006181, 97831-97831 276
JEE MAIN 2021
19. A glass tumbler having inner depth of 17.5 cm is kept on a table. A student starts pouring water(μ
= 4/3) into it while looking at the surface of water from the above. When he feels that the tumbler
is half filled, he stops pouring water. Up to what height, the tumbler is actually filled ?
[JEE MAIN 2021 (AUGUST)]
(1) 11.7 cm (2) 10 cm (3) 7.5 cm (4) 8.75 cm

20. An object is placed at a distance of 12 cm from a convex lens. A convex mirror of focal length15
cm is placed on other side of lens at 8 cm as shown in the figure. Image of object coincides with
the object. [JEE MAIN 2021 (AUGUST)]

When the convex mirror is removed, a real and inverted image is formed at a position. The
distance of the image from the object will be …… (cm)

21. Curved surface of a plano-convex lens of reflective index 1 and a plano-concave


lens of reflective index 2 have equal radius of curvature as shown in the figure.
Find the ratio of radius of curvature to the focal length of the combined lenses.
[JEE MAIN 2021 (AUGUST)]

1 1
(1) (2) 1   2 (3) (4) 2  1
 2  1 1   2

22. An object is placed beyond the centre of curvature C of the given concave mirror. If the distance
of the object is d1 from C and the distance of the image formed is d2 from C, the radius of
curvature of this mirror is : [JEE MAIN 2021 (AUGUST)]
2d d 2d1d 2 dd dd
(1) 1 2 (2) (3) 1 2 (4) 1 2
d1  d 2 d1  d 2 d1  d 2 d1  d 2

23. Find the distance of the image from object O, formed by the combination of lenses in the figure :
  10cm  cm   30cm [JEE MAIN 2021 (AUGUST)]

(1) 75 cm (2) 10 cm (3) 20 cm (4) infinity

NUCLEUS-92, Rajeev Gandhi Nagar, Kota (Raj.) India 324005, Mob. 9358006181, 97831-97831 277
JEE MAIN 2021
24. Cross- section view of a prism is the equilateral triangle ABC in the figure. The minimum
deviation is observed using this prism when the angle of incidence is equal to the prism angle. The
time taken by light to travel from P (midpoint of BC) to A is __________ × 10– 10 s.
3
(Given, speed of light in vacuum = 3  108m/s and cos30°  ) [JEE MAIN 2021 (AUGUST)]
2

25. An object is placed at the focus of concave lens having focal length  . What is the magnification
and distance of the image from the optical centre of the lens ? [JEE MAIN 2021 (AUGUST)]
1  1 
(1)  (2) Very high,  (3) , (4) ,
2 2 4 4

26. Two plane mirrors M1 and M2 are at right angle to each other shown. A point source 'P' is placed
at 'a' and '2a' meter away from M1 and M2 respectively. The shortest distance between the images
thus formed is : (Take 5  2.3 ) [JEE MAIN 2021 (AUGUST)]

(1) 3a (2) 4.6 a (3) 2.3 a (4) 2 10 a

ANSWER KEY
1. 4 2. 2 3. 150 4. 2 5. 1 6. 2 7. 12
8. 4 9. 30 10. 3 11. 4 12. 4 13. 60 14. 3
15. 1 16. 1 17. 2 18. 600 19. 2 20. 50 21. 2
22. 1 23. 1 24. 5 25. 3 26. 2

NUCLEUS-92, Rajeev Gandhi Nagar, Kota (Raj.) India 324005, Mob. 9358006181, 97831-97831 278
JEE MAIN 2021
SOLUTION

1. (4)
Sol. Since orientation is same image is virtual. Since image is smaller the mirror has to be convex

2. (2)

Sol.

h ' D
' ;  ' is same for both object and image m  
u0  0
u0  D
Hence m > 1

3. (150)

Sol.

25 x
tan   
60 180
x = 75 cm
so distance between extreme point = 2 x = 2 × 75 = 150 cm

NUCLEUS-92, Rajeev Gandhi Nagar, Kota (Raj.) India 324005, Mob. 9358006181, 97831-97831 279
JEE MAIN 2021
4. (2)

Sol.

Assuming that the right angled prism is an isosceles prism, so the other angles will be 45° each.
 Each incident ray will make an angle of 45° with the normal at face PR.
 The wavelength corresponding to which the incidence angle is less than the critical angle. will
pass through PR.
1
 c  critical angle  c  sin –1  

the light ray will pas
 1 
  c Red  sin –1    51.94
 1.27 
Red will pass.
 1 
  c Green  sin –1    44.76
 1.42 
Green will not pass
 1 
  c Blue  sin –1    42.15
 1.49 
Blue will not pass  So only red will pass through PR.

5. (1)
Sol. Deviation is minimum in a prism when :i = e, r1 = r2 and ray (2) is parallel to base of prism.

6. (2)
1  μL   1 1 
Sol.    1   
F  μS   R1 R 2 
1
If μL  μS   0  F  
F

NUCLEUS-92, Rajeev Gandhi Nagar, Kota (Raj.) India 324005, Mob. 9358006181, 97831-97831 280
JEE MAIN 2021
7. (12)
Sol. 1  0.02; 1  1.5; 2  0.03;  2  1.6
Achromatic combination
net  0
1  2  0
1  2
11  22
&net  1  2  2
11
1  2
2
  
1 1  1   2
 2 
1  6
1  1 1 A1
6  (1.5  1)A1
A1  12

8. (4)
Sol. R2 = r2 + (R – t)2
R2 = r2 + R2 + t2 – 2Rt
Neglecting t2, we get
r2
R
2t
1  1 1   –1
    –1  –  
f R  R

f
R

r2

 3 10 
–2 2


9 10 –4
2
 –1 2r   –1 3  6 10 –3 1
2  3 10 –3   –1
2 
f = 0.3 m = 30 cm

9. (30)
a 3
Sol. m  
 2
 1  1
 
v u R
3
1
3 1 2
 
2 10 15 R
30
R = 30
13

NUCLEUS-92, Rajeev Gandhi Nagar, Kota (Raj.) India 324005, Mob. 9358006181, 97831-97831 281
JEE MAIN 2021
10. (3)
c
Sol. V = 3.33 × 107 m/sec
r r

11. (4)
 2 1  2  1
Sol.  
v u R
1.4 1.25 1.4  1.25
 
v 40 25
1.4 0.15 1.25
 
v 25 40
v = –37.58 cm
Hence option (4)

12. (4)
Sol. r  r  90  180  r  90  r  90  i
n1 sin i  n 2 sin r  n 2 sin(90  i)
n2
n1 sin i  n 2 cos i  tan i 
n1
n2
Now sin C = = tan i
n1
 C = sin–1 (tan i) = sin–1 (tan r)

13. (60)
A
Sol. At minimum deviation r1 = r2 =
2
Also given I = 2r1 = A
Now 1.sini = 3 sin r1
A
1sinA = 3 sin
2
A A A
 2sin cos  3 sin
2 2 2
A 3 A
 cos    30
2 2 2
 A = 60°

14. (3)
1 1
Sol. sin C    sin C
 2 2
sin > sin C
> C
Total internal reflection will happen
NUCLEUS-92, Rajeev Gandhi Nagar, Kota (Raj.) India 324005, Mob. 9358006181, 97831-97831 282
JEE MAIN 2021
15. (1)
 A  min 
sin  
 2 
Sol. 
A
sin  
2
AA
sin  
  2 
A
sin  
2
sin A A
  2 cos
A 2
sin
2

A  2 cos 1  
2

16. (1)

Sol.

At maximum angle  ray at point B goes in grazing emergence, at all less values of , TIR occurs.
At point
4
 sin   1 sin 90
3
3  
  sin 1        
4 2 
At point A
4
1 sin    sin 
3
4  
sin    sin    
3 2 
4  7
sin   cos cos 1 
3  4 

4 7  7
sin      sin 1  
3 4  3 

NUCLEUS-92, Rajeev Gandhi Nagar, Kota (Raj.) India 324005, Mob. 9358006181, 97831-97831 283
JEE MAIN 2021
17. (2)
Sol. Standard graph between angle of deviation and incident angle.

18. (600)
Sol. For no bending , n1 =n2
10.8 1014 1.8 104
1.2   1.45 
2 2
On solving,
9 1014  252
  6 107
  600 nm

19. (2)
Sol.

Height of water observed by observer


H H 3H
  
 w  4 / 3 4
Height of air observer = 17.5 – H According to question, both height observed by observer is
same.
3H
= 17.5 – H
4
 H = 10 cm
Option (2)

20. (50)

Sol.

For the object to coincide with image, the light must fall perpendicularly to mirror. Which means
that the light will have to converge at C of mirror. Without the mirror also, the light would
coverage at C.
So the distance is : 12 + 8 + 30 = 50 cm

NUCLEUS-92, Rajeev Gandhi Nagar, Kota (Raj.) India 324005, Mob. 9358006181, 97831-97831 284
JEE MAIN 2021
21. (2)
1 1
Sol.  (1  1)  
f1 R
1  1
 ( 2  1)   
f2  R
1 1 1 (1  1)  ( 2  1)
  
f1 f 2 f eq R
1 (1   2 )

f eq R
R
 (1   2 )
f eq

22. (1)
Sol. Using Newton's formula
(f + d1) (f – d2) = f2
f2 + fd1 – fd2 – d1d2 = f2
dd
f 1 2
d1  d 2
2d1d 2
R 
d1  d 2
23. (1)
1 1 1
Sol.  
V1 30 10
1 20
  V1  15 cm
V1 30
1 1 1
 
V2 10 10
1
0 V2  
V2
OV3 = 75 cm

24. (5)
Sol. i = A = 60°
  2i  A = 2  60° - 60° = 60°
 min

 A
sin 1  min 
 2   3

A
sin 1  
2
3  108
Vprism 
3

NUCLEUS-92, Rajeev Gandhi Nagar, Kota (Raj.) India 324005, Mob. 9358006181, 97831-97831 285
JEE MAIN 2021
3
AP  10 102 
2
5 102
time   3  3 = 5  10-10 sec
3 108

25. (3)

Sol. U=-f
1 1 1 1 2
   
V U f V f
f
V
2
V 1
M 
U 2
f
distance 
2

26. (2)

Sol.

Shortest distance is 2a between I1& I 3


But answer given is for I1& I2
(4a)2  (2a)2
a 20
4.47 a
option (2)

NUCLEUS-92, Rajeev Gandhi Nagar, Kota (Raj.) India 324005, Mob. 9358006181, 97831-97831 286
JEE MAIN 2021
WAVE ON STRING
1. A sound wave of frequency 245 Hz travels with the speed of 300 ms–1 along the positive x-axis.
Each point of the wave moves to and fro through a total distance of 6 cm. What will be the
mathematical expression of this travelling wave? [JEE MAIN 2021 (MARCH)]
(1) Y(x,t) = 0.03 sin [5.1 x – (0.2 × 103)t]
(2) Y(x,t) = 0.06 sin [5.1 x – (1.5 × 103)t]
(3) Y(x,t) = 0.06 sin [0.8 x – (0.5 × 103)t]
(4) Y(x,t) = 0.03 sin [5.1x – (1.5 × 103)t]

1
2. The amplitude of wave disturbance propagating in the positive x-direction is given by y 
(1  x)2
at t = 1s, where x and y are in metes . The shape of wave does, not change during the propagation.
The velocity of the wave will be ____m/s. [JEE MAIN 2021 (JULY)]

3. Two waves are simultaneously passing through a string and their equation are :
y1 = A1 sin k(x–vt), y2 = A2 sin k(x – vt + x0). Given amplitudes A1 = 12 mm and A2 = 5 mm, x0 =
3.5 cm and wave number k = 6.28 cm–1. The amplitude of resulting wave will be……..mm.
[JEE MAIN 2021 (AUGUST)]

4. Two travelling waves produces a standing wave represented by equation,


y = 1.0 mm cos(1.57 cm-1) x sin(78.5 s-1)t. [JEE MAIN 2021 (AUGUST)]
The node closest to the origin in the region x > 0 will be at x = .........cm.

5. A wire having a linear mass density 9.0  10-4 kg/m is stretched between into rigid supports with a
tension of 900 N. The wire resonates at a frequencyof 500Hz.The next higher frequency at which
the same wire resonates is 550 Hz.
The length of the wire is ______m. [JEE MAIN 2021 (AUGUST)]

ANSWER KEY

1. 4 2. 2 3. 7 4. 1 5. 10

NUCLEUS-92, Rajeev Gandhi Nagar, Kota (Raj.) India 324005, Mob. 9358006181, 97831-97831 287
JEE MAIN 2021
SOLUTION
1. (4)
Sol. (4)  = 2f
= 1.5 × 103
6
A =  3cm  0.03m
2

2. (2)
1
Sol. t  0, y 
1 x2
1
At time t  t, y 
1  (x  vt) 2
1
At t  1, y   (i)
1  (x  v) 2
1
At t  1, y   (ii)
1  (x  2)2
Comparing (i) & (ii)
v = 2m/s

3. (7)
Sol. y1 = A1sink(x –vt)
y1 = 12 sin 6.28 (x – vt)
y2 = 5 sin 6.28 (x – vt + 3.5)
2
   x 

= K(x)
7
 6.28  3.5   2  7 
2
A net  A12  A 22  2A1A 2 cos 

Anet  (12)2  (5)2  2(12)(5) cos(7)


 144  25  120

4. (1)
Sol. For node
Cos(1.57cm-1)x = 0

(1.57cm 1 )x 
2

x cm  1cm
2(1.57)
Ans. 1.00
NUCLEUS-92, Rajeev Gandhi Nagar, Kota (Raj.) India 324005, Mob. 9358006181, 97831-97831 288
JEE MAIN 2021
5. (10)
kg
Sol.   9.0  104
m
T = 900 N
T 900
V   1000m / s
 9 104
f1 = 500 Hz
f = 550
nV (n  1)V
 500.....(i)  500 ....(ii)
2 2
V
(ii) (i)  50
2
1000
  10
2  50

NUCLEUS-92, Rajeev Gandhi Nagar, Kota (Raj.) India 324005, Mob. 9358006181, 97831-97831 289
JEE MAIN 2021
OPTICAL INSTRUMENTS

1. Your friend is having eye sight problem. She is not able to see clearly a distant uniform window
mesh and it appears to her as non-uniform and distorted. The doctor diagnosed the problem as:
[JEE MAIN 2021 (FEB)]
(1) Astigmatism (2) Myopia with Astigmatism
(3) Presbyopia with Astigmatism (4) Myopia and hypermetropia

2. An object viewed from a near point distance of 25 cm, using a microscopic lens with
magnification '6', give an unresolved image. A resolved image is observed at infinite distance with
a total magnification double the earlier using an eyepiece along with the given lens and a tube of
length 0.6 m, if the focal length of the eyepiece is equal to ________cm.
[JEE MAIN 2021 (JULY)]

ANSWER KEY

1. 2 2. 25

NUCLEUS-92, Rajeev Gandhi Nagar, Kota (Raj.) India 324005, Mob. 9358006181, 97831-97831 290
JEE MAIN 2021
SOLUTION

1. (2)
Sol. If distant objects are blurry then problem is Myopia.
If objects are distorted then problem is Astigmatism.

2. (25)
Sol. For simple microscope,
D
m  1
f0
D
6  1
f0
25
5
f0
f0 = 5 cm
For compound microscope,
D
m
f0  fe
60  25
12 
5  fe
fe = 25 cm

NUCLEUS-92, Rajeev Gandhi Nagar, Kota (Raj.) India 324005, Mob. 9358006181, 97831-97831 291
JEE MAIN 2021
SOUND WAVE
1. A signal of 0.1 kW is transmitted in a cable. The attenuation of cable is –5 dB per km and cable
length is 20 km. The power received at receiver is 10–x W. The value of x is ______.
P 
[Gain in dB = 10 log10  0  ] [JEE MAIN 2021 (FEB)]
 Pi 

2. Two cars are approaching each other at an equal speed of 7.2 km/hr. When they see each other,
both blow horns having frequency of 676 Hz. The beat frequency heard by each driver will be
_____ Hz. [Velocity of sound in air is 340 m/s.] [JEE MAIN 2021 (FEB)]

3. A student is performing the experiment of resonance column. The diameter of the column tube is 6
cm. The frequency of the tuning fork is 504 Hz. Speed of the sound at the given temperature is
336 m/s. The zero of the meter scale coincides with the top end of the resonance column tube. The
reading of the water level in the column when the first resonance occurs is:
[JEE MAIN 2021 (FEB)]
(1) 13 cm (2) 16.6 cm (3) 18.4 cm (4) 14.8 cm

4. A closed organ pipe of length L and an open organ pipe contain gases of densities 1 and 2
respectively. The compressibility of gasses are equal in both the pipes. Both the pipes are vibrating
x 
in their first overtone with same frequency. The length of the open pipe is L 1 where x is
3 2
______ (Round off to the Nearest Integer). [JEE MAIN 2021 (MARCH)]

5. The frequency of a car horn encountered a change from 400 Hz to 500 Hz. When the car
approaches a vertical wall. If the speed of sound is 330 m/s. Then the speed of car is ____ km/h.
[JEE MAIN 2021 (JULY)]

6. With what speed should a galaxy move outward with respect to earth so that the sodium –D line
at wavelength 5896 Å is observed at 5890 Å? [JEE MAIN 2021 (JULY)]
(1) 306 km/sec. (2) 322 km/sec. (3) 296 km/sec (4) 336 km/sec

7. A source and a detector move away from each other in absence of wind with a speed of 20 m/s
with respect to the ground. If the detector detects a frequency of 1800 Hz of the sound coming
from the source , then the original frequency of source considering speed of sound in air 340 m/s
will be .... Hz. [JEE MAIN 2021 (AUGUST)]

8. A tuning fork is vibrating at 250 Hz. The length of the shortest closed pipe that will resonate with
the tuning fork will be __________cm. [JEE MAIN 2021 (AUGUST)]
(Take speed of sound air as 340 ms-1)

9. Two cars X and Y are approaching each other with velocities 36km/h and 72 km/h respectively.
The frequency of a whistle sound as emitted by a passenger in car X, heard by the passenger in
car Y is 1320 Hz. If the velocity of sound in air is 340m/s, the actual frequency of the whistle
sound produced is ........Hz. [JEE MAIN 2021 (AUGUST)]
NUCLEUS-92, Rajeev Gandhi Nagar, Kota (Raj.) India 324005, Mob. 9358006181, 97831-97831 292
JEE MAIN 2021
–1
10. A galaxy is moving away from the earth at a speed of 286kms . The shift in the wavelength of a
red line at 630 nm is x × 10–10 m. The value of x, to the nearest integer, is _________.
[JEE MAIN 2021 (AUGUST)]
8 –1
[Take the value of speed of light c, as 3 × 10 ms ]

NUCLEUS-92, Rajeev Gandhi Nagar, Kota (Raj.) India 324005, Mob. 9358006181, 97831-97831 293
JEE MAIN 2021
SOLUTION
1. (8)
Sol. Sound level decreases by 5dB every km so sound level decreased in 20 km = 100 dB
I2
2  1  10 log10
I1

I2 I
–100 = 10log10  1  1010
I1 I2

I2 = 10–10 I1  P2 = 10–10 P1 = 10–8 W

x=8 Ans. 8.

2. (8)

Sol.

Frequency of sound heard by car-1, which comes by reflection from car-2


2
 340  2  340  2   342 
f1  f 0     f0  
 340  2  340  2   338 
Frequency of sound coming directly from car-2
 340  2 
f2 = f0  
 340  2 
 342  342 
 f1  f 2  f 0    1  8.09 8
 338  338 

3. (4)
Sol. d = 6cm, f = 504, v = 336 m/s
e = 0.3d
 V
l e  
4 4f
l = 16.66 – 0.3 × 6
l = 14.866 cm
l = 14.8 cm

NUCLEUS-92, Rajeev Gandhi Nagar, Kota (Raj.) India 324005, Mob. 9358006181, 97831-97831 294
JEE MAIN 2021
4. (4)

Sol.

fc  f0
3VC 2V0

4L 2L
3VC V0
 
4L L
4L V0 4L B 1
L   (B is bulk modulus)
3 VC 3 2  B

4L 1
3 2
x=4

5. (132)

Sol.

Wall as an observer
Frequency received by wall
 C 
f1  f 0  
CV
Again wall as a source
Frequency received by observer on car
 CV 
f 2  f1  
 C 
 CV
f 2  f0  
 CV 
 CV
500  400  
 CV 

NUCLEUS-92, Rajeev Gandhi Nagar, Kota (Raj.) India 324005, Mob. 9358006181, 97831-97831 295
JEE MAIN 2021
5 CV

4 CV
C = 9V
C 330
V  m/s
9 9
330 18
V   132km / hr
9 5

6. (1)
1  v
Sol. f  f0 
1  c

f 1 
f0 1 
2
 f  1
1    (1  )(1  )
 f0 
is small compared to 1
 2f 
1    (1  2)
 f0 
f v
 
f0 c
c
v  6  305.6km / s
5890

7. (2025)

Sol.

 C  V0 
f 'f 
 C  Vs 
 340  20 
1800  f  
 340  20 
f = 2025 Hz
Ans. 2025

NUCLEUS-92, Rajeev Gandhi Nagar, Kota (Raj.) India 324005, Mob. 9358006181, 97831-97831 296
JEE MAIN 2021
8. (34)

Sol.


 4
4
V V
f 
 4
340 34
 250     0.3m
4 4  25
 34cm

9. (1210)

Sol.

Vx = 36 km/hr = 10 m/s
Vy = 72 km/hr = 20 m/s
by Doppler's effect
 V  V0 
F'  F0  
 V  Vs 
 340  20 
1320  F0    F0  1210Hz
 340  10 

10. (6)

Sol. cv

v 286
     630  10 9  6  10 10
c 3  10 5

NUCLEUS-92, Rajeev Gandhi Nagar, Kota (Raj.) India 324005, Mob. 9358006181, 97831-97831 297
JEE MAIN 2021
WAVE OPTICS
1. In a Young's double slit experiment, the width of the one of the slit is three times the other slit.
The amplitude of the light coming from a slit is proportional to the slit-width. Find the ratio of the
maximum to the minimum intensity in the interference pattern. [JEE MAIN 2021 (FEB)]
(1) 1 : 4 (2) 3 : 1 (3) 4 : 1 (4) 2 : 1

2. An unpolarized light beam is incident on the polarizer of a polarization experiment and the
intensity of light beam emerging from the analyzer is measured as 100 Lumens. Now, if the
analyzer is rotated around the horizontal axis (direction of light) by 30° in clockwise direction, the
intensity of emerging light will be _______ Lumens. [JEE MAIN 2021 (FEB)]

3. Which of the following equations represents a travelling wave ? [JEE MAIN 2021 (FEB)]
2
x
(1) y = Asin(15x – 2t) (2) y = Ae (vt + )
(3) y = Aexcos(t – ) (4) y = Asinx cost

4. Consider the diffraction pattern obtained from the sunlight incident on a pinhole of diameter
0.1m. If the diameter of the pinhole is slightly increased, it will affect the diffraction pattern such
that : [JEE MAIN 2021 (FEB)]
(1) its size decreases, and intensity decreases
(2) its size increases, and intensity increases
(3) its size increases, but intensity decreases
(4) its size decreases, but intensity increases

5. In a Young's double slit experiment two slits are separated by 2 mm and the screen is placed one
meter away. When a light of wavelength 500 nm is used, the fringe separation will be:
[JEE MAIN 2021 (FEB)]
(1) 0.25 mm (2) 0.50 mm (3) 0.75 mm (4) 1 mm

6. Given below are two statements : one is labeled as Assertion A and the other is labelled as Reason
R. [JEE MAIN 2021 (FEB)]
Assertion A : An electron microscope can achieve better resolving power than an optical
microscope.
Reason R : The de Broglie's wavelength of the electrons emitted from an electron gun is much
less than wavelength of visible light. In the light of the above statements, choose the correct
answer from the options given below:
(1) A is true but R is false.
(2) Both A and R are true and R is the correct explanation of A.
(3) Both A and R are true but R is NOT the correct explanation of A.
(4) A is false but R is true.

NUCLEUS-92, Rajeev Gandhi Nagar, Kota (Raj.) India 324005, Mob. 9358006181, 97831-97831 298
JEE MAIN 2021
7. The mass per unit length of a uniform wire is 0.135 g/cm. A transverse wave of the form y = –0.21
sin (x + 30t) is produced in it, where x is in meter and t is in second. Then, the expected value of
tension in the wire is x × 10–2 N. Value of x is . (Round-off to the nearest integer)
[JEE MAIN 2021 (FEB)]

8. If the source of light used in a Young's double slit experiment is changed from red to violet :
[JEE MAIN 2021 (FEB)]
(1) consecutive fringe lines will come closer.
(2) the central bright fringe will become a dark fringe.
(3) the fringes will become brighter.
(4) the intensity of minima will increase.

9. A fringe width of 6mm was produced for two slits separated by 1 mm apart. The screen is placed
10m away. The wavelength of light used is 'x' nm. The value of 'x' to the nearest integer is _____.
[JEE MAIN 2021 (MARCH)]

10. In Young's double slit arrangement, slits are separated by a gap of 0.5m mm, and the screen is
placed at a distance of 0.5m from them. The distance between the first and the third bright fringe
0
formed when the slits are illuminated by a monochromatic light of 5890 A is:-
[JEE MAIN 2021 (MARCH)]
–9 –6
(1) 1178 × 10 m (2) 1178 × 10 m
–12
(3) 1178 × 10 m (3) 5890 × 10–7m

11. In the Young’s double slit experiment, the distance between the slits varies in time as
d  t   d0  a 0 sin t ; where d 0,  and a 0 are constants. The difference between the largest fringe
width and the smallest fringe width obtained over time is given as :
[JEE MAIN 2021 (JULY)]
2 D  d 0  2Da 0 D D
(1) (2) (3) a0 (4)
 d0  a 0 
2 2
 d0  a 0 
2 2
D02
d0  a 0

12. In Young's double slit experiment, if the source of light changes from orange to blue then :
[JEE MAIN 2021 (JULY)]
(1) the central bright fringe will become a dark fringe.
(2) the distance between consecutive fringes will decrease.
(3) the distance between consecutive fringes will increase.
(4) the intensity of the minima will increase.

13. The difference in the number of waves when yellow light propagates through air and vacuum
columns of the same thickness is one. The thickness of the air column is _______ mm.
[Refractive index of air = 1.0003, wavelength of yellow light in vacuum = 6000 Å]
[JEE MAIN 2021 (JULY)]

NUCLEUS-92, Rajeev Gandhi Nagar, Kota (Raj.) India 324005, Mob. 9358006181, 97831-97831 299
JEE MAIN 2021
14. The width of one of the two slits in a Young's double slit experiment is three times the other slit. If
the amplitude of the light coming from a slit is proportional to the slit-width, the ratio of minimum
to maximum intensity in the interference pattern is x : 4 where x is _______.
[JEE MAIN 2021 (AUGUST)]

15. White light passed through a double slit and interference is observed on a screen 1.5 m away. The
separation between the slits is 0.3 mm. The first violet and red fringes are formed 2.0 mm and 3.5
mm away from the central white fringes. The difference in wavelength of red and violet light is
....nm. [JEE MAIN 2021 (AUGUST)]

16. The light waves from two coherent sources have same intensity I1 = I2 = I0. In interference pattern
the intensity of light at minima is zero. What will be the intensity of light at maxima ?
[JEE MAIN 2021 (AUGUST)]
(1) I0 (2) 2 I0 (3) 5 I0 (4) 4 I0

17. In a Young's double slit experiment, the slits are separated by 0.3 mm and the screen is 1.5 m
away from the plane of slits. Distance between fourth bright fringes on both side of central bright
is 2.4 cm. The frequency of light used is ____________  1014 Hz.
[JEE MAIN 2021 (AUGUST)]

ANSWER KEY
1. 3 2. 75 3. 1 4. 4 5. 1 6. 2 7. 1215
8. 1 9. 600 10. 2 11. 2 12. 2 13. 2 14. 1
15. 300 16. 4 17. 5

NUCLEUS-92, Rajeev Gandhi Nagar, Kota (Raj.) India 324005, Mob. 9358006181, 97831-97831 300
JEE MAIN 2021
SOLUTION
1. (3)
Sol. Amplitude  Width of slit
 A2 = 3A1
2
Imax  I1  I2 
 
Imin  | I1  I2 | 

 Intensity I  A2
2
I  A  A2 
 max   1
Imin  | A1  A2 | 
2
 A  3A1 
= 1 
 | A1  3A1 | 
2
 4A 
=  1   4 :1
 2A1 

2. (75)

Sol.

Assuming initial axis of polarizer and Analyzer are parallel

I0
Now emerging intensity = cos2 30°
2
 3 3
= 100    100  4  75
 2 

3. (1)
Sol. y = F(x, t)
For travelling wave y should be linear function of x and t and they must exist as (x ± vt) y =
Asin(15x – 2t)  linear function in x and t Option (1) is correct.

4. (4)
m
Sol. sin  
a
when “a” increases, decreases, width decreases so intensity will increase

NUCLEUS-92, Rajeev Gandhi Nagar, Kota (Raj.) India 324005, Mob. 9358006181, 97831-97831 301
JEE MAIN 2021
5. (1)
D 500 109 1
Sol.  
d 2 103
5
  104 m  2.5  10 1 mm
2
b = 0.25 mm

6. (2)
1
Sol. Resolving power 

Since wavelength of electron is much less than visible light, its resolving power will be much
more.

7. (1215)
Sol.  =0.135 gm / cm = 0.0135 kg/ m
y = –0.21 sin (x + 30 t)
(x in meter & t in sec)
 30
v   30m / s
k 1
T
 T  v2   30   0.0135
2
v

= 12.15
= x × 10–2 N
 x = 1215

8. (1)
.D
Sol. 
d
R >  V
 D  D
R = R and V = V
d d
R >  V
Fringe pattern will shrink.
Option (1) is correct.

9. (600)
D
Sol. 
d
d

D
6 10–3 10–3
. 
10
  6 10–7 m  600 10–9 m
  600nm

NUCLEUS-92, Rajeev Gandhi Nagar, Kota (Raj.) India 324005, Mob. 9358006181, 97831-97831 302
JEE MAIN 2021
10. (2)
D 5890 10–10  0.5
Sol.  
d 0.5 10–3
= 589 × 10–6 m
Distance between first and third bright fringe
is 2 = 2 × 589 × 10–6 m
= 1178 × 10–6 m Ans. (2)

11. (2)
D
Sol. Fringe Width,  
d
max  d min and min  d max
d  d 0  a 0 sin t
d max  d0  a 0 and d min  d 0  a 0
D D
min  andmax 
d0  a 0 d0  a 0
D D 2Da
max  min    2 02
d0  a 0 d0  a 0 d0  a 0

12. (2)
Sol. Fringe width = D/d
as decreases , fringe width also decreases

13. (2)
Sol. Thickness t  n
So, n  vac  (n  1) air

n   (n  1)
air
1 104
n 
 air  1 3
t  n
104
  6000 Å = 2 mm
3.

14. (1)
Sol. Given amplitude  slit width
Also intensity  (Amplitude)2 (Slit width)2
2
I1  3 
    9  I1  9 I 2
I2  1 
2
I min  I1  I 2   3  1  1 x
2

       x = 1.00
I max  I1  I 2   3  1  4 4

NUCLEUS-92, Rajeev Gandhi Nagar, Kota (Raj.) India 324005, Mob. 9358006181, 97831-97831 303
JEE MAIN 2021
15. (300)
D
Sol. Position of bright fringe y  n
d
D r
y1 of red   3.5mm
d
d
 r  3.5  103
D
d
Similarly  v  2  103
D
 0.3 103 
 r   v  (1.5 103 )  
 1.5 
 3 107  300 nm

16. (4)
 
2
Sol. Imax  I1  I2 = 4Io

17. (5)
Sol. 8  2.4 cm
8
 2.4 cm
d
8  1.5  c
 2.4  10 2
0.3  10 3  f
f  5 1014 Hz

NUCLEUS-92, Rajeev Gandhi Nagar, Kota (Raj.) India 324005, Mob. 9358006181, 97831-97831 304
JEE MAIN 2021
MODERN PHYSICS-1
1. Given below are two statements : [JEE MAIN 2021 (FEB)]
Statement-I : Two photons having equal linear momenta have equal wavelengths.
Statement-II : If the wavelength of photon is decreased, then the momentum and energy of a
photon will also decrease.
In the light of the above statements, choose the correct answer from the options given below.
(1) Both Statement I and Statement II are true
(2) Statement I is false but Statement II is true
(3) Both Statement I and Statement II are false
(4) Statement I is true but Statement II is false

2. In the given figure, the energy levels of hydrogen atom have been shown along with some
transitions marked A, B, C, D and E. The transitions A, B and C respectively represent :
[JEE MAIN 2021 (FEB)]

(1) The ionization potential of hydrogen, second member of Balmer series and third member of
Paschen series.
(2) The first member of the Lyman series, third member of Balmer series and second member of
Paschen series.
(3) The series limit of Lyman series, third member of Balmer series and second member of
Paschen series.
(4) The series limit of Lyman series, second member of Balmer series and second member of
Paschen series.

3. The de Broglie wavelength of a proton and -particle are equal. The ratio of their velocities is :
[JEE MAIN 2021 (FEB)]
(1) 4 : 3 (2) 4 : 1 (3) 4 : 2 (4) 1 : 4

4. The stopping potential for electrons emitted from a photosensitive surface illuminated by light of
wavelength 491 nm is 0.710 V. When the incident wavelength is changed to a new value, the
stopping potential is 1.43 V. The new wavelength is : [JEE MAIN 2021 (FEB)]
(1) 329 nm (2) 309 nm (3) 382 nm (4) 400 nm
NUCLEUS-92, Rajeev Gandhi Nagar, Kota (Raj.) India 324005, Mob. 9358006181, 97831-97831 305
JEE MAIN 2021
5. The wavelength of an X-ray beam is 10Å. The mass of a fictitious particle having the same energy
x
as that of the X-ray photons is h kg. The value of x is_________. [JEE MAIN 2021 (FEB)]
3
(h = Planck's constant)

6. The recoil speed of a hydrogen atom after it emits a photon in going from n = 5 state to n = 1 state
will be :- [JEE MAIN 2021 (FEB)]
(1) 4.17 m/s (2) 2.19 m/s (3) 3.25 m/s (4) 4.34 m/s

7. Two radioactive substances X and Y originally have N1 and N2 nuclei respectively. Half life of X
is half of the half life of Y. After three half lives of Y, number of nuclei of both are equal. The
N
ratio 1 will be equal to : [JEE MAIN 2021 (FEB)]
N2
1 3 8 1
(1) (2) (3) (4)
8 1 1 3

8. According to Bohr atom model, in which of the following transitions will the frequency be
maximum ? [JEE MAIN 2021 (FEB)]
(1) n = 4 to n = 3 (2) n = 2 to n = 1 (3) n = 5 to n = 4 (4) n = 3 to n = 2

9. A particle is travelling 4 times as fast as an electron. Assuming the ratio of de-Broglie wavelength
of a particle to that of electron is 2 : 1, the mass of the particle is:-
[JEE MAIN 2021 (AUGUST)]
1
(1) times the mass of e– (2) 8 times the mass of e–
16
1
(3) 16 times the mass of e– (4) times the mass of e–
8

(1)
10. A radioactive sample disintegrates via two independent decay processes having half lives T1/2 and
T1/(2)2 respectively. The effective half-life T1/2 of the nuclei is: [JEE MAIN 2021 (MARCH)]
( ) ( )
( ) ( )
(1) ( ) ( ) (2)
( ) ( )
(3) ( ) ( ) (4) None of the above

11. The speed of electrons in a scanning electron microscope is 1 × 107 ms–1. If the protons having the
same speed are used instead of electrons, then the resolving power of scanning proton microscope
will be changed by a factor of: [JEE MAIN 2021 (MARCH)]
1
(1) 1837 (2)
1837
1
(3) 1837 (4)
1837
NUCLEUS-92, Rajeev Gandhi Nagar, Kota (Raj.) India 324005, Mob. 9358006181, 97831-97831 306
JEE MAIN 2021
12. Two identical photocathodes receive the light of frequencies f1 and f2 respectively. If the velocities
of the photo-electrons coming out are v1 and v2 respectively, then [JEE MAIN 2021 (MARCH)]
2h 2h
(1) v1 – v 2 
2 2
f1 – f 2  (2) v1 – v 2 
2 2
f1  f 2 
m m
1 1/2
 2h 
(3) v1  v 2    f1  f 2  
2h
(4) v1  v2    f1  f 2 
2

m  m 

13. The stopping potential in the context of photoelectric effect depends on the following property of
incident electromagnetic radiation: [JEE MAIN 2021 (MARCH)]
(1) Phase (2) Intensity (3) Amplitude (4) Frequency

14. The first three spectral lines H-atom in the Balmer series are given 1,2,3 considering the Bohr
 
atomic mode, the wave lengths of first and third spectral lines  1  are related by a factor of
 3 
–1
approximately 'x' × 10 . The value of x, to the nearest integer, is _______ .
[JEE MAIN 2021 (MARCH)]

15. The de-Broglie wavelength associated with an electron and a proton were calculated by
accelerating them through same potential of 100V. What should nearly be the ratio of their
wavelengths? (mp = 1.00727 u,me = 0.00055u) [JEE MAIN 2021 (MARCH)]
2
(1) 1860 : 1 (2) (1860) : 1 (3) 41.4 : 1 (4) 43 : 1

16. If an electron is moving in the nth orbit of the hydrogen atom, then its velocity (vn) for the nth orbit
is given as: [JEE MAIN 2021 (MARCH)]
1
(1) vn  n (2) v n 
n
1
. (3) v n  n 2 (4) v n  2
n

17. Which level of the single ionized carbon has the same energy as the ground state energy of
hydrogen atom? [JEE MAIN 2021 (MARCH)]
(1) 1 (2) 6
(3) 4 (4) 8

18. The atomic hydrogen emits a line spectrum consisting of various series. Which series of hydrogen
atomic spectra is lying in the visible region? [JEE MAIN 2021 (MARCH)]
(1) Brackett series (2) Paschen series
(3) Lyman series (4) Balmer series

19. A particle of mass m moves in a circular orbit in a central potential field U(r) = U 0r4. If Bohr's
quantization conditions are applied, radii of possible orbital rn vary with n1/, where  is _______ .
[JEE MAIN 2021 (MARCH)]

NUCLEUS-92, Rajeev Gandhi Nagar, Kota (Raj.) India 324005, Mob. 9358006181, 97831-97831 307
JEE MAIN 2021
20. Imagine that the electron in a hydrogen atom is replaced by a muon (). The mass of muon
particle is 207 times that of an electron charge is equal to the charge of an electron. The ionization
potential of this hydrogen atom will be:- [JEE MAIN 2021 (MARCH)]
(1) 13.6 eV (2) 2815.2eV
(3) 331.2 eV (4) 27.2 eV

–C
21. A particle mass m moves in a circular orbit under the central potential field, U  r  
, where C
r
is a positive constant. [JEE MAIN 2021 (MARCH)]
The correct radius – velocity graph of the particle's motion is:

(1) (2)

(3) (4)

22. An electron of mass m and a photon have same energy E. The ratio of wavelength of electron to
that of photon is: (c being the velocity of light) [JEE MAIN 2021 (MARCH)]
1/2 1/2 1/2
1  2m  1 E   E 
(1)   (2)   (3)   (4) c(2mE)1/2
c E  c  2m   2m 

23. If 2.5 × 10–6 N average force is exerted by a light wave on a non-reflecting surface of 30cm2 area
during 40 minutes of time span, the energy flux of light just before it falls on the surface is
_______ W/cm2. [JEE MAIN 2021 (MARCH)]
(Round off to the Nearest Integer)
(Assume complete absorption and normal incidence conditions are there)

24. The radiation corresponding to 3  2 transition of a hydrogen atom falls on a gold surface to
generate photoelectrons. These electrons are passed through a magnetic field of 5 x 10 -4 T.
Assume that the radius of the largest circular path followed by these electrons is 7 mm, the work
function of the metal is : [JEE MAIN 2021 (JULY)]
-31
(Mass of electrons = 9.1 × 10 kg)
(1) 1.36 eV (2)1.88 eV (3) 0.16 eV (4) 0.82 eV

NUCLEUS-92, Rajeev Gandhi Nagar, Kota (Raj.) India 324005, Mob. 9358006181, 97831-97831 308
JEE MAIN 2021
25. An electron and proton are separated by a large distance. The electron starts approaching the
proton with energy 3 eV. The proton capture the electrons and forms a hydrogen atom in second
excited state. The resulting photon is incident on a photosensitive metal of threshold wavelength
4000 A. What is the maximum kinetic energy of the emitted photoelectron ?
[JEE MAIN 2021 (JULY)]
(1) 7.61 eV (2) 1.41 eV
(3) 3.3 eV (4) No photoelectron would be emitted

26. The K  X-ray of molybdenum has wavelength 0.071 nm. If the energy of a molybdenum atom
with a K electron knocked out is 27.5 keV , The energy of this atom when an L electron is
knocked out will be ______ keV. (Round off to the nearest integer ) [JEE MAIN 2021 (JULY)]
[h  4.14 x 1015 eVs, c  3 x 108 ms 1 ]

27. An electron having de-Broglie wavelength  is incident on a target in a X- ray tube. Cut off
wavelength of emitted X-ray is : [JEE MAIN 2021 (JULY)]
2m 2C2  2 2mc 2 hc
(1) 0 (2) (3) (4)
h2 h mc

28. A certain metallic surface is illuminated by monochromatic radiation of wavelength . The


stopping potential for photoelectric current for this radiation is 3V0. If the same surface is
illuminated with a radiation of wavelength 2. , the stopping potential is V0. The threshold
wavelength of this surface for photoelectric effect is _______ . [JEE MAIN 2021 (JULY)]

29. An electron of mass me and a proton of mass mp are accelerated through the same potential
difference. The ratio of the de-Broglie wavelength associated with the electron to that with the
proton is :- [JEE MAIN 2021 (JULY)]
mp mp mp
(1) (2) 1 (3) (4)
me me me

30. What should be the order of arrangement of de-Broglie wavelength of electron  e  , an  -


particle  a  and proton  p  given that all have the same kinetic energy?
[JEE MAIN 2021 (JULY)]
(1)  e   p   a (2)  e   p   a (3)  e   p   a (4)  e   p   a

31. A particle of mass 4M at rest disintegrates into two particles of mass M and 3M respectively
having non zero velocities. The ratio of de-Broglie wavelength of particle of mass M to that of
mass 3M will be : [JEE MAIN 2021 (JULY)]
(1) 1: 3 (2) 3 :1 (3) 1: 3 (4) 1:1

NUCLEUS-92, Rajeev Gandhi Nagar, Kota (Raj.) India 324005, Mob. 9358006181, 97831-97831 309
JEE MAIN 2021
32. An electron moving with speed v and a photon moving with speed c, have same D-Broglie
wavelength. The ratio of kinetic energy of electron to that of photon is :
[JEE MAIN 2021 (JULY)]
3c v v 2c
(1) (2) (3) (3)
v 3c 2c v

33. When radiation of wavelength  is incident on a metallic surface , the stopping potential of
ejected photoelectrons is 4.8 V. If the same surface is illuminated by radiation of double the
previous wavelength , then the stopping potential becomes 1.6 V . The threshold wavelength of
the metal is : [JEE MAIN 2021 (JULY)]
(1) 2  (2) 4  (3) 8  (4) 6 

34. A light beam of wavelength 500 nm is incident on a metal having work function of 1.25 eV,
placed in a magnetic field of intensity B. The electrons emitted perpendicular to the magnetic field
B, with maximum kinetic energy are bent into circular arc of radius 30 cm. The value of B is
_____× 10–7T. [JEE MAIN 2021 (JULY)]
–26 –31
Given hc = 20 × 10 J-m, mass of electron = 9 × 10 kg

35. A particle of mass 9.1 × 10–31 kg travels in a medium with a speed of 106 m/s and a photon of a
radiation of linear momentum 10–27 kg m/s travels in vacuum. The wavelength of photon is
_______ times the wavelength of the particle. [JEE MAIN 2021 (JULY)]

36. Consider the following statements : [JEE MAIN 2021 (JULY)]


A. Atoms of each element emit characteristics spectrum.
B. According to Bohr's postulate, an electron in a hydrogen atom, revolves in certain stationary
orbit.
C. The density of nuclear matter depends on the size of nucleus.
D. A free neutron is stable but a free proton decay is possible .
E. radioactivity is an indication of the instability of nuclei.
Choose the correct answer from the option given below.
(1) A, B, C, D and E (2) A, B and E only
(3) B and D only (4) A, C and E only

37. The de-Broglie wavelength of a particle having kinetic energy E is . How much extra energy
must be given to this particle so that the de-Broglie wavelength reduces to 75% of the initial
value? [JEE MAIN 2021 (AUGUST)]
1 7 16
(1) E (2) E (3) E (4) E
9 9 9

NUCLEUS-92, Rajeev Gandhi Nagar, Kota (Raj.) India 324005, Mob. 9358006181, 97831-97831 310
JEE MAIN 2021
15
38. A particular hydrogen like ion emits radiation of frequency 2.92 x 10 Hz when it makes
transition from n = 3 to n = 1. The frequency in Hz of radiation emitted in transition from n = 2 to
n = 1 will be : [JEE MAIN 2021 (AUGUST)]
15 15 15
(1) 0.44 × 10 (2) 6.57 × 10 (3) 4.38 × 10 (4) 2.46 × 1015

39. In a photoelectronic experiment ultraviolet light of wavelength 280 nm is used with lithium
cathode having work function   2.5eV . If the wavelength of incident light is switched to 400
nm, find out the change in the stopping potential.(h = 6.63 x 10-34 Js, c = 3 x 108 ms-1)
[JEE MAIN 2021 (AUGUST)]
(1) 1.3 V (2) 1.1 V (3) 1.9 V (4) 0.6 V

40. A monochromatic neon lamp with wavelength of 670.5 nm illuminates a photo-sensitive material
which has a stopping voltage of 4.48 V. What will be the stopping voltage if the source light is
changed with another source of wavelength of 476.6 nm? [JEE MAIN 2021 (AUGUST)]
(1) 0.96 V (2) 1.25 V (3) 0.24 V (4)1.5V

41. X different wavelengths may be observed in the spectrum from a hydrogen sample if the atoms are
excited to states with principal quantum number n = 6 ? The value of X is __________.
[JEE MAIN 2021 (AUGUST)]

42. In a photoelectric experiment, increasing the intensity of incident light :


[JEE MAIN 2021 (AUGUST)]
(1) increases the number of photons incident and also increases the K.E. of the ejected electrons
(2) increases the frequency of photons incident and increases The K.E. of the ejected electrons.
(3) increases the frequency of photons incident and the K.E. of the ejected electrons remains
unchanged.
(4) increases the number of photons incident and the K. E. of the ejected electrons remains .
unchanged

43. A free electron of 2.6 eV energy collides with a H+ ion. This results in the formation of a hydrogen
atom in the first excited state and a photon is released. Find the frequency of the emitted
photon. [JEE MAIN 2021 (AUGUST)]
(h = 6.6  10 Js)
-34

(1) 1.45  1016 MHz (2) 0.19  1015 MHz


(3) 1.45  109 MHz (4) 9.0  1027 MHz

44. A moving proton and electron have the same de-Broglie wavelength. If K and P denote the K.E.
and momentum respectively. Then choose the correct option :
[JEE MAIN 2021 (AUGUST)]
(1) Kp > Ke and Pp = Pe (2) Kp = Ke and Pp = pe
(3) Kp < Ke and Pp < Pe (4) Kp > Ke and Pp = Pe

NUCLEUS-92, Rajeev Gandhi Nagar, Kota (Raj.) India 324005, Mob. 9358006181, 97831-97831 311
JEE MAIN 2021
ANSWER KEY
1. 4 2. 3 3. 2 4. 3 5. 10 6. 1 7. 3
8. 2 9. 4 10. 3 11. 1 12. 1 13. 4 14. 15
15. 4 16. 2 17. 2 18. 4 19. 3 20. 2 21. 1
22. 2 23. 25 24. 4 25. 2 26. 10 27. 3 28. 4
29. 3 30. 3 31. 4 32. 3 33. 2 34. 125 35. 910
36. 2 37. 2 38. 4 39. 1 40. 2 41. 15 42. 4
43. 3 44. 1

SOLUTION
1. (4)
Sol. If linear momentum are equal then wavelength also equal
h hc
p = ,E 
 
On decreasing wavelength, momentum and energy of photon increases.

2. (3)
Sol. A  Series limit of Lymen series.
B  Third member of Balmer series.
C  Second member of Paschen series.

3. (2)
h
Sol. 
mv
P = 
mPvP = mv
mPvP = 4mp v (m = 4mp)
vp
4 (Option 2) is correct
v

4. (3)
hc
Sol.    eVs

1240
   0.71 ………..(1)
491
1240
   1.43 ………..(1)

   382 nm

5. (10)
hc
Sol.  mc 2

h
m
c
NUCLEUS-92, Rajeev Gandhi Nagar, Kota (Raj.) India 324005, Mob. 9358006181, 97831-97831 312
JEE MAIN 2021
6. (1)

Sol.

(E) Releases when photon going from n = 5 to n = E = (13.6 –0.54) eV = 13.06 eV.

Pi = Pf (By linear momentum conservation)


h h
   Mv  VRecoil  …………….(i)
 M
hc hc
& E    M  McVRecoil
 M
E 13.06 1.6 1019
VRecoil    4.17m / sec
Mc 1.67 1027  3 108

7. (3)
Sol. Tx = t : Ty = 2t
3Ty = 6t,
N1' = N2'
N1e –  6t  N 2 e 
1 2 6t

1 1 
N1 ln 2  6t
 e 1 2 6t  e  t 2t   e ln 23  eln8  8
N2
N1 8

N2 1
8. (2)
1 1
Sol. E = 13.6  2  2  = h
 n1 n 2 
It is maximum if n1 = 1 and n2 = 2
n = 5 ............................................ –0.544 eV
n = 4 ............................................ –0.850 eV
n = 3 ............................................ –1.511 eV
n = 2 ............................................ –3.4 eV
n = 1 ............................................ –13.6 eV
Option (2) is correct.
NUCLEUS-92, Rajeev Gandhi Nagar, Kota (Raj.) India 324005, Mob. 9358006181, 97831-97831 313
JEE MAIN 2021
9. (4)
h
Sol. 
p
p pe me ve
 
e pp mp vp
me  v e 
2  
mp  4ve 
m
 mp  e Ans. (4)
8

10. (3)
Sol.  eq  1   2
1 1 1
 (1)  (2)
T1/2 T1/2 T1/2
(1) (2)
T1/2 T1/2
T1/2 
T1/2  T1/2
(1) (2)

11. (1)
1
Sol. Resolving power (RP) 

h h
 
P mv
mv
So (RP) 
h
PR  P ; RP  mv ; RP  m

12. (1)
1 2
Sol. (1) mv1  hf1 – 
2
1 2
mv 2  hf 2 – 
2
2h
v12 – v 22   f1 – f 2 
m

13. (4)
Sol. Stopping potential changes linearly with frequency of incident radiation.

14. (15)
Sol. For 1st line
1 1 1
 Rz 2  2 – 2 
1 2 3 

NUCLEUS-92, Rajeev Gandhi Nagar, Kota (Raj.) India 324005, Mob. 9358006181, 97831-97831 314
JEE MAIN 2021
1 5
 Rz 2 .......(i)
1 36
For 3rd line
1 1 1
 Rz 2  2 – 2 
3 2 5 
1 21
 Rz 2 ........(ii)
3 100
(ii) + (i)
1 21 36
   1.512  15.12 10 –1
3 100 5
x | 15

15. (4)
h h h
Sol. λ  
mv 2mK 2mqV
λ1 m2

λ2 m1
λe mP
  1831.4  42.79
λP me

16. (2)
Sol. We know velocity of electron in nth shell of hydrogen atom is given by
2kZe2
v
nh
1
v 
n
17. (2)
Sol. Energy of H-atom is E = –13.6Z2/n2
for H-atom Z = 1 & for ground state, n = 1
12
 E  –13.6  2  –13.6eV
1
Now for carbon atom (single ionized), Z = 6
Z2
E  –13.6  2  –13.6 (given)
n
 n 2  62  n  6

18. (4)
Sol. Assuming visible spectrum to range from 4000 Å ~ 7000 Å,
Energy range of photon = 13400/4000 ~ 13400/7000 = 3.3 eV ~ 1.9 eV
Energy levels in Hydrogen atoms are 13.6 eV, 3.4 eV, 1.9 eV, 0.85 eV
For the energy range to be in 3.3 eV ~ 1.9 eV, transitions have to be to 2nd shell from higher shells.
 Balmer series

NUCLEUS-92, Rajeev Gandhi Nagar, Kota (Raj.) India 324005, Mob. 9358006181, 97831-97831 315
JEE MAIN 2021
19. (3)
–dU mv2
Sol. F  –4U0 r 3 
dr r
2 4
mv = 4U0r
vr 2
nh
mvr 
2
r n
3
1/3
rn
=3

20. (2)
1 1
Sol. E r
r m
Em

Ionization potential  13.6 


 Mass  eV
μ
 13.6  207eV  2815.2eV
 Mass e 

21. (1)
C
Sol. U
r
dU C
F– – 2
dr r
mv 2
F
r
C mv 2 1
 ; v2 
r2 r r

22. (2)
h
Sol. 1 
2mE
hc
2 
E
1/2
1 1  E 
  
 2 c  2m 

23. (25)
IA
Sol. F
C
FC 2.5 10–6  3 108
I   25W / cm2
A 30

NUCLEUS-92, Rajeev Gandhi Nagar, Kota (Raj.) India 324005, Mob. 9358006181, 97831-97831 316
JEE MAIN 2021
24. (4)

Sol.

3  2  1.89 eV
5 × 10–4 T r = 7mm
mv P2 (qRB)2
r=  mv = qrB  E  
qB 2m 2m
1.6 10  7 103  5 104 
19 2
3136 1052
  eV
2  9.11031 Joule 18.2 1031 1.6 1019
= 1.077 eV
We know work function = energy incident – (KE)electron
 = 1.89 – 1.077 = 0.813 eV

25. (2)
Sol. Initially, energy of electron = + 3eV
Finally, in 2nd excited state,
(13.6eV)
Energy of electron  
32
= - 1.5eV
Loss in energy is emitted as photon ,
hc
So, photon energy  4.51eV

Now, photoelectronic effect equation
hc  hc 
KE max     4.51   
   th 
o
12400 eV A
 4.51 eV  o
4000 A
= 1.41 eV

26. (10)
Sol. Ek = Ek – EL
hc
 Ek  EL
k 
hc
EL  Ek 
 k
12.42 107 eVm
 27.5 keV 
0.071109 m
E L  (27.5  17.5)keV = 10 keV

NUCLEUS-92, Rajeev Gandhi Nagar, Kota (Raj.) India 324005, Mob. 9358006181, 97831-97831 317
JEE MAIN 2021
27. (3)
h
Sol. =
mv
P2 h2 hc
kinetic energy,  
2m 2m 2
c
2m 2c
c =
h

28. (4)
hc
Sol. KE   hc

hc
e  3V0    ….(i)
0
hc
eV0   ….(ii)
2 0
Using (i) & (ii)
hc hc
= 
4 0  t
t = 40

29. (3)
Sol. KE = eV
h
e 
2me (eV)
h  mp
P   e 
2mp (eV) p me

30. (3)
h h 1
Sol. =  
p 2mE m
m > mp > me
so e > p > 

31. (4)
h
Sol. =
p
both the particles will move with momentum same in magnitude & opposite in direction.
So De-Broglie wavelength of both will be same i.e. ratio 1 : 1

NUCLEUS-92, Rajeev Gandhi Nagar, Kota (Raj.) India 324005, Mob. 9358006181, 97831-97831 318
JEE MAIN 2021
32. (3)
Sol. e = Ph
h h

pe p ph
E ph
2mk e 
c
E 
2
ph
2mk e 
c2
k e E pa  1 
 2  
E ph c  2m 
p ph  1  pe  1  mv 1 v
       
c  2m  c  2m  c 2m 2c

33. (2)
Sol. VS = h – 
hc
4.8   …….(i)

hc
1.6   …….(ii)
2
Using above equation (i) – (ii)
hc hc
3.2  
 2
hc
3.2  …….(iii)
2
 hc 
  6.4 
Put in equation (ii)
 = 1.6
hc
 1.6
 th
hc
th =
1.6
= 
hc 
  4  4
 6.4 

34. (125)
Sol. By photoelectric equation
hc
   k max

1240
k max   1.25  1.25
500

NUCLEUS-92, Rajeev Gandhi Nagar, Kota (Raj.) India 324005, Mob. 9358006181, 97831-97831 319
JEE MAIN 2021
2mk
r
eB
2mk
B
er
= 125 × 10–7T

35. (910)
h 6.6 1034
Sol. For photon 1  
P 1027
h 6.6 1034
For particle  2  
mv 9.11031 106

 1  910
2

36. (2)
Sol. (A) True, atom of each elements emits characteristics spectrum
nh
(B) True, according to Bohr's postulates mvr  and hence electron resides into orbit of specific
2
radius called stationary orbits.
(C) False, density of nucleus is constant
(D) False, A free neutron is unstable decay into proton and electron and antineutrino.
(E) True unstable nucleus show radioactivity.

37. (2)
h h
Sol.   , mv  2mE
mv 2mE
1

E
2 E1 3
  ,  2  0.751
1 E2 4
2
E1  3 
 
E2  4 
16 16
E 2  E1  E  E1  E 
9 9
16 7
Extra energy given = E–E= E
9 9

38. (4)
1 1 
Sol. nf1  k   2 
1 3 

NUCLEUS-92, Rajeev Gandhi Nagar, Kota (Raj.) India 324005, Mob. 9358006181, 97831-97831 320
JEE MAIN 2021
 1 
nf 2  k 1  2 
 2 
f1 8 / 9
  f 2  2.46 1015
f2 3 / 4
Option (4)

39. (1)
hc 1240
Sol. KE max  eVS     eVS   2.5  1.93eV
 280
 VS1  1.93V...(i)
1240
 eVS2   2.5  0.6eV
400
 VS2  0.6V...(ii)
V  VS1  VS2  1.93  0.6  1.33V
Option (1)

40. (2)
hc
Sol. kE max  
i
hc
or eV0   
i
when i  670.5nm; Vo  0.48
when i  474.6nm; Vo  ?
1240
so, e(0.48)   ....(i)
670.5
1240
e(V0 )   ....(ii)
474.6
(2) – (1)
 1 1 
e(Vo  0.48)  1240    eV
 474.6 670.5 
 670.5  474.6 
Vo  0.48  1240   volts
 474.6  670.5 
Vo  0.48  0.76
Vo  1.24 V ~ 1.25V

41. (15)
n(n  1)
Sol. No. of different wavelengths 
2
6  (6  1) 6  5
   15
2 2

NUCLEUS-92, Rajeev Gandhi Nagar, Kota (Raj.) India 324005, Mob. 9358006181, 97831-97831 321
JEE MAIN 2021
42. (4)
Sol.  Increasing intensity means number of incident photons are increased.
 Kinetic energy of ejected electrons depends on the frequency of incident photons, not the
intensity.

43. (3)
Sol. For every large distance P.E. = 0
& total energy = 2.6 + 0 = 2.6 eV
Finally in first excited state of H atom total energy
= – 3 .4 eV
Loss in total energy = 2.6 – (– 3.4)
=6eV
It is emitted as photon
1240
  206 nm
6
3 108
f  1.45 1015 Hz
206 109
 1.45 109 Hz

44. (1)
h h
Sol. P  e 
PP Pe
 P   e  PP  Pe
PP2
(K) P 
2m P
Pe2
(K)e 
2me
KP < Ke as mP > me
Option (1)

NUCLEUS-92, Rajeev Gandhi Nagar, Kota (Raj.) India 324005, Mob. 9358006181, 97831-97831 322
JEE MAIN 2021
MODERN PHYSICS-2
1. A radioactive sample is undergoing a decay. At any time t1. its activity is A and another time t2
A
the activity is . What is the average life time for the sample ? [JEE MAIN 2021 (FEB)]
5
n5 t t t t n  t 2  t1 
(1) (2) 1 2 (3) 2 1 (4)
t 2  t1 n5 n5 2

2. The half-life of Au198 is 2.7 days. The activity of 1.50 mg of Au198 if its atomic weight is 198g
mol–1 is, (NA = 6 × 1023/mol) [JEE MAIN 2021 (MARCH)]
(1) 240 Ci (2) 357Ci (3) 535 Ci (4) 252Ci

3. Calculate the time interval between 33% decay and 67% decay if half-life of a substance is 20
minutes. [JEE MAIN 2021 (MARCH)]
(1) 60 minutes (2) 20 minutes
(3) 40 minutes (4) 13 minutes

4. The half-life of Au198 is 2.7 days. The activity of 1.50 mg of Au198 if its atomic weight is 198g
mol–1 is, (NA = 6 × 1023/mol) [JEE MAIN 2021 (MARCH)]
(1) 240 Ci (2) 357Ci (3) 535 Ci (4) 252Ci

5. A radioactive material decay by simultaneous emission of two particles with half lives of 1400
years and 700 years respectively. What will be the time after which one third of the material
remains ? (Take ln 3 = 1.1) [JEE MAIN 2021 (JULY)]
(1) 1110 years (2) 700 years (3) 340 years (4) 740 years

6. A nucleus of mass M emits   ray photon of frequency 'v'. The loss of internal energy by the
nucleus is : [JEE MAIN 2021 (JULY)]
 hv   hv 
(1) hv (2) 0 (3) 1  2
(4) hv 1 
 2Mc 
2
 2Mc 
7. For a certain radioactivity process the graph between ln R and t (sec) is obtained as shown in the
figure. Then the value of half life for the unknown radioactivity material is approximately:
[JEE MAIN 2021 (JULY)]

(1) 9.15 sec (2) 6.93 sec (3) 2.62 sec (4) 4.62 sec
NUCLEUS-92, Rajeev Gandhi Nagar, Kota (Raj.) India 324005, Mob. 9358006181, 97831-97831 323
JEE MAIN 2021
th
1
8. A radioactive substance decay to   of its initial activity in 80 days. The half life of the
 16 
radioactive substance expressed in days is ____. [JEE MAIN 2021 (JULY)]

9. A nucleus with mass number 184 initially at rest emits an -particle. If the Q value of the reaction
is 5.5 MeV, calculate the kinetic energy of the -particle. [JEE MAIN 2021 (JULY)]
(1) 5.0 MeV (2) 5.5 MeV (3) 0.12 MeV (4) 5.38 MeV

10. The half-life of198 Au is 3 days. If atomic weight of 198


Au is 198 g/mol then the activity of 2 mg
198
of Au is [in disintegration/second]: [JEE MAIN 2021 (JULY)]
(1) 2.67 1012 (2) 6.06 1018 (3) 32.36 1012 (4) 16.18 1012

27
11. From the given data, the amount of energy required to break the nucleus of aluminium 13 Al

is_____ x 103 J. [JEE MAIN 2021 (JULY)]


Mass of neutron = 1.00866 u
Mass of proton = 1.00726 u
Mass of Aluminium nucleus = 27.18846 u
(Assume l u corresponds to x J of energy)
(Round off to the nearest integer)
th
1
12. The nuclear activity of a radioactive element becomes   of its initial value in 30 years. The
8
half-life of radioactive element is________ years. [JEE MAIN 2021 (JULY)]

13. If 'ƒ' denotes the ratio of the number of nuclei decayed (Nd) to the number of nuclei at t = 0 (N0)
then for a collection of radioactive nuclei, the rate of change of 'ƒ' with respect to time is given as :
[ is the radioactive decay constant] [JEE MAIN 2021 (JULY)]
(1) – (1 – e–t) (2) (1 – e–t) (3) e–t (4) –e–t

14. A radioactive sample has an average life of 30 ms and is decaying. A capacitor of capacitance 200
F is first charged and later connected with resistor 'R'. If the ratio of charge on capacitor to the
activity of radioactive sample is fixed with respect to time then the value of 'R' should be
______. [JEE MAIN 2021 (JULY)]

NUCLEUS-92, Rajeev Gandhi Nagar, Kota (Raj.) India 324005, Mob. 9358006181, 97831-97831 324
JEE MAIN 2021
15. Some nuclei of a radioactive material are undergoing radioactive decay. The time gap between the
instances when a quarter of the nuclei have decayed and when half of the nuclei have decayed is
given as : [JEE MAIN 2021 (JULY)]
(where  is the decay constant)
3
1 ln2
ln
ln2 2ln2 2
(1) (2) (3) (4)
2    

16. The half life period of radioactive element x is same as the mean life time of another radioactive
element y. Initially they have the same number of atoms. Then :
(1) x–will decay faster than y. [JEE MAIN 2021 (AUGUST)]
(2) y– will decay faster than x.
(3) x and y have same decay rate initially and later on different decay rate.
(4) x and y decay at the same rate always.

17. At time t = 0, a material is composed of two radioactive atoms A and B, where N A(0) = 2NB(0).
The decay constant of both kind of radioactive atoms is . However, A disintegrates to B and B
disintegrates to C. Which of the following figures represents the evolution of NB(t) / NB(0) with
respect to time t ? [JEE MAIN 2021 (AUGUST)]
 N A (0)  No. of A atoms at t  0
 N (0)  No. of B atoms at t  0 
 B 

(1) (2)

(3) (4)

18. The are 1010 radioactive nuclei in a given radioactive element, Its half-life time is 1 minute. How
many nuclei will remain after 30 seconds? [JEE MAIN 2021 (AUGUST)]
 2  1.414 
(1) 2 1010 (2) 7 109 (3) 105 (4) 4 1010
NUCLEUS-92, Rajeev Gandhi Nagar, Kota (Raj.) India 324005, Mob. 9358006181, 97831-97831 325
JEE MAIN 2021
19. A sample of radioactivity nucleus A disintegrates to another radioactivity nucleus B, which in turn
disintegrates to same other stable nucleus C. Plot of graph showing the variation of number of
atoms of nucleus B versus time is : [JEE MAIN 2021 (AUGUST)]
(Assume that at t = 0, there are no B atoms in the sample)

(1) (2) (3) (4)

20. The decay of a proton to neutron is: [JEE MAIN 2021 (AUGUST)]
(1) not possible as proton mass is less than the neutron mass
(2) possible only inside the nucleus
(3) not possible but neutron to proton conversion is possible
(4) always possible as it is associated only with + decay.

ANSWER KEY
1. 3 2. 2 3. 2 4. 2 5. 4 6. 4 7. 4
8. 20 9. 4 10. 4 11. 27 12. 10 13. 3 14. 150
15. 4 16. 2 17. 3 18. 2 19. 2 20. 2

NUCLEUS-92, Rajeev Gandhi Nagar, Kota (Raj.) India 324005, Mob. 9358006181, 97831-97831 326
JEE MAIN 2021
SOLUTION
1. (3)
Sol. Let initial activity be A0
A  A 0 e t1 ……….(i)
A
 A 0 e t 2 ……….(ii)
5
(i)  (ii)
5  e t 2  t1 
n5 1
 
t 2  t1 
t t
 2 1
n5

2. (2)
 ln 2 
Sol. A  λN  t1/2  
  
N  nN A
 1.5 103 
N   NA
 198 
1 Curie = 3.7 × 1010 Bq
A = 365 Bq
3. (2)
Sol. N1  N 0 e – t1
N1
 e – t1
N0
0.67  e– t1
ln  0.67  –t1
N 2  N 0 e – t 2
N2
 e – t 2
N0
0.33  e– t 2
ln(0.33) = –t2
ln(0.67) – ln(0.33) = t1 – t2
 0.67 
  t1 – t 2   ln  
 0.33 
  t1 – t 2   ln 2
t1 – t 2
ln 2
t1 – t 2  t1/2

Half life = t1/2 = 20 minutes.
NUCLEUS-92, Rajeev Gandhi Nagar, Kota (Raj.) India 324005, Mob. 9358006181, 97831-97831 327
JEE MAIN 2021
4. (2)
 ln 2 
Sol. A  λN  t1/2  
  
N  nN A
 1.5 103 
N   NA
 198 
1 Curie = 3.7 × 1010 Bq
A = 365 Bq

5. (4)

Sol.

ln 2 n2
Given 1 = 1  /year, 2 = / year
700 1400
 net = 1 + 2 = n 2 
1 1 

 700 1400 
3 n2
= /year
1400
Now, Let initial number of radioactive nuclei be No.
N
 0  N 0 e net t
3
1 3  0.693
 n   net t  1.1  t  t  740 years
3 1400
Hence option 4

6. (4)
Sol. Energy of  ray [E] = h
h h
Momentum of  ray [P] = 
 C
P  PNu  0
Where PNu = Momentum of decayed nuclei
h
 P = PNu   PNu  K.E. of nuclei
C
1  P  1  hv  2
 Mv 2  Nu 
2 2M 2M  C 
Loss in internal energy = E + K.ENu
2
1  hv 
= hv 1 
hv 
= hv   
2M  C   2MC2 

NUCLEUS-92, Rajeev Gandhi Nagar, Kota (Raj.) India 324005, Mob. 9358006181, 97831-97831 328
JEE MAIN 2021
7. (4)
Sol. R = R0e–t
nR = nR0 – t]
 – isslope of straight line
3
 =
20
n2
t1/2 = = 4.62

8. (20)
t1 t t t
N0 1
N 1
N 1
N
Sol. N0 
2
 
2
 0 
2
 0 
2
 0
2 4 8 16
4 × t1/2 = 80
t1/2 = 20 days

9. (4)

Sol.

2
1 1  4v 
(4m)v2  (180m)    5.5Mev
2 2  180 
1   4  
2
5.5
 4mv 1  45 
2
   5.5MeV  K.E α  2
MeV
2   180    4 
1  45   
 180 
5.5
K.E = 2
MeV
1  45. 
4 

 180 

10. (4)
Sol. A = N
ln 2 ln 2
  sec1  2.67 106 sec 1
t1/2 3  24  60  60
N = Number of atoms in 2 mg Au
2 103
  6 1023  6.06 1015
198
A = N = 1.618 × 1013 = 16.18 × 1012 dps

NUCLEUS-92, Rajeev Gandhi Nagar, Kota (Raj.) India 324005, Mob. 9358006181, 97831-97831 329
JEE MAIN 2021
11. (27)
Sol. m = (ZmP + (A – Z)mn) – MA
= (13 ×1.00726 + 14 × 1.00866) – 27.18846 = 27.21562 – 27.18846 = 0.02716 u
E = 27.16 x × 10–3 J

12. (10)
Sol. A = A0e–t
A0
 A 0 e t  t  ln 8
8
t = 3ln2
ln 2 t 30
   10 years
 3 3

13. (3)
Sol. N = N0e–t
Nd = N0 – N
Nd = N0 (1 – e–t)
Nd
 f  1  et
N0
df
 e t
dt

14. (150)
Sol. Tm = 30 ms
C = 200 F
 1 
q Q0e t/RC Q0 t   RC 
  e
N N0et N0
Since q/N is constant hence
1

RC
1 Tm 30 103
R    150
C C 200 106

15. (4)
3N 0
Sol.  N 0 e t1
4
N0
 N 0 e t 2
2
ln(3/4) = –t1 .....(i)
ln(1/2) = –t2 .....(i)
ln(3/4) – ln(1/2) = t2–t1) .....(i)
ln  3 / 2 
t =

NUCLEUS-92, Rajeev Gandhi Nagar, Kota (Raj.) India 324005, Mob. 9358006181, 97831-97831 330
JEE MAIN 2021
16. (2)
n2 1
Sol.  t1/2 x   y    x = 0.693 y
x y
Also initially Nx = Ny = N0
Activity A = N
As x < y  Ax < Ay  y will decay faster than x
Option (2)

17. (3)
Sol. A  B, B  C
dN B
 N A  N B
dt
dN B
 2N Bo e t  N B
dt
 dN 
et  B  N B   2N B0 et  et
 dt 
d
dt
 N Bet   2N B0 , on integrating

NBet  2tNB0  NB0


NB  NB0 [1  2t]et
dN B
 0 at [1  2t)e t  2e t  0
dt
1
N Bmax at t 
2

18. (2)
t
N  1  t1/2
Sol.  
N0  2 
30 1
10
N  1  60  1  2 10
10
    N  1010
    ~ 7 109
10 2 2 2

19. (2)
Sol. A   B   C (stable)
Initially no, of atoms of B = 0 after t = 0, no. of atoms of B will starts increasing & reaches
maximum value when rate of decay of B = rate of formation of B .
After that maximum value, no. of atoms will starts decreasing as growth & decay both are
exponential functions, so best possible graph is (2)
option (2)

20. (2)
Sol. It is possible only inside the nucleus and not otherwise.
NUCLEUS-92, Rajeev Gandhi Nagar, Kota (Raj.) India 324005, Mob. 9358006181, 97831-97831 331
JEE MAIN 2021
SEMICONDUCTOR
1. If an emitter current is changed by 4 mA, the collector current changes by 3.5 mA. The value of 
will be : [JEE MAIN 2021 (FEB)]
(1) 7 (2) 0.5 (3) 0.875 (4) 3.5

2.

The logic circuit shown above is equivalent to : [JEE MAIN 2021 (FEB)]

(A) (B)

(C) (D)

3. Given below are two statements :


Statement I : PN junction diodes can be used to function as transistor, simply by connecting two
diodes, back to back, which acts as the base terminal. [JEE MAIN 2021 (FEB)]
Statement II : In the study of transistor, the amplification factor  indicates ratio of the collector
current to the base current. In the light of the above statements, choose the correct answer from the
options given below :
(1) Statement I is false but Statement II is true
(2) Both Statement I and Statement II are true
(3) Both Statement I and Statement II are false
(4) Statement I is true but Statement II is false

4. Zener breakdown occurs in a p-n junction having p and n both : [JEE MAIN 2021 (FEB)]
(1) lightly doped and have wide depletion layer.
(2) heavily doped and have narrow depletion layer.
(3) lightly doped and have narrow depletion layer.
(4) heavily doped and have wide depletion layer.

5. For extrinsic semiconductors; when doping level is increased; [JEE MAIN 2021 (FEB)]
(1) Fermi-level of p-type semiconductor will go upward and Fermi-level of n-type semiconductors
will go downward.
(2) Fermi-level of p-type semiconductors will go downward and Fermi-level of n-type
semiconductor will go upward.
(3) Fermi-level of both p-type and n-type semiconductors will go upward for T > TF K and
downward for T < TF K, where TF is Fermi temperature.
(4) Fermi-level of p and n-type semiconductors will not be affected.

NUCLEUS-92, Rajeev Gandhi Nagar, Kota (Raj.) India 324005, Mob. 9358006181, 97831-97831 332
JEE MAIN 2021
6. The truth table for the following logic circuit is : [JEE MAIN 2021 (FEB)]

(1) (2)

(3) (4)

7. Match List I with List II. [JEE MAIN 2021 (FEB)]


List – I List – II
(a) Rectifier (i) Used either for stepping up or stepping down the
a. c. voltage
(b) Stabilizer (ii) Used to convert a. c. voltage into d. c. voltage
(c) Transformer (iii) Used to remove any ripple in the rectified output voltage
(d) Filter (iv) Used for constant output voltage even when the input
voltage or load current change
Choose the correct answer from the options given below :
(1) (a)–(ii), (b)–(iv), (c)–(i), (d)–(iii) (2) (a)–(iii), (b)–(iv), (c)–(i), (d)–(ii)
(3) (a)–(ii), (b)–(i), (c)–(iv), (d)–(iii) (4) (a)–(ii), (b)–(i), (c)–(iii), (d)–(iv)

8. LED is constructed from Ga-As-P semiconducting material. The energy gap of this LED is 1.9 eV.
Calculate the wavelength of light emitted and its colour. [JEE MAIN 2021 (FEB)]
–34 8 –1
[h = 6.63 × 10 Js and c = 3 × 10 ms ]
(1) 1046 nm and red colour (2) 654 nm and orange colour
(3) 1046 nm and blue colour (4) 654 nm and red colour

9. The zener diode has a Vz = 30 V. The current passing through the diode for the following circuit is
......... mA. [JEE MAIN 2021 (FEB)]

NUCLEUS-92, Rajeev Gandhi Nagar, Kota (Raj.) India 324005, Mob. 9358006181, 97831-97831 333
JEE MAIN 2021
10. The value of power dissipated across the zener diode (Vz = 15V) connected in the circuit as shown
in the figure is x × 10–1 watt. [JEE MAIN 2021 (MARCH)]

The value of x, to the nearest integer, is ________ .

11. In the logic circuit shown in the figure, if input A and B are 0 to 1 respectively, the output at Y
would be 'x'. The value of x is ________ . [JEE MAIN 2021 (MARCH)]

12. The following logic gate is equivalent to: [JEE MAIN 2021 (MARCH)]

(1) NOR Gate (2) OR Gate


(3) AND Gate (4) NAND Gate

13. The output of the given combination gates represents. [JEE MAIN 2021 (MARCH)]

(1) XOR Gate (2) NAND Gate


(3) AND Gate (4) NOR Gate

14. Which one of the following will be the output of the given circuit? [JEE MAIN 2021 (MARCH)]

(1) NOR Gate (2) NAND GATE (3) AND Gate (4) XOR Gate

15. An npn transistor operates as a common emitter amplifier with a power gain of 106. The input
circuit resistance is 100 and the output load resistance is 10k. The common emitter current
gain '' will be ________. (Round off to the Nearest Integer) [JEE MAIN 2021 (MARCH)]

NUCLEUS-92, Rajeev Gandhi Nagar, Kota (Raj.) India 324005, Mob. 9358006181, 97831-97831 334
JEE MAIN 2021
16. The correct relation between a (ratio of collector current to emitter current) and b (ratio of
collector current to base current) of a transistor is: [JEE MAIN 2021 (MARCH)]
  1 
(1)   (2)   (3)   (4)  
1  1–  1–  1 

17. The typical output characteristic curve for a transistor working in the common-emitter
configuration is shown in the figure. [JEE MAIN 2021 (MARCH)]

The estimated current gain from the figure is ?

18. For the circuit shown below, calculate the value of Iz : [JEE MAIN 2021 (JULY)]

(1) 25 mA (2) 0.15 A (3) 0.1 A (4) 0.05 A

19. A zener diode having zener voltage 8 V and power dissipation rating of 0.5 is connected across a
potential divider arranged with maximum potential drop across zener diode is as shown in the
diagram. The value of protective resistance RP is ............... [JEE MAIN 2021 (JULY)]

NUCLEUS-92, Rajeev Gandhi Nagar, Kota (Raj.) India 324005, Mob. 9358006181, 97831-97831 335
JEE MAIN 2021
20. For the forward biased diode characteristics shown in the figure , the dynamic resistance at ID =3
mA will be_______  . . [JEE MAIN 2021 (JULY)]

21. Consider a situation in which reverse biased current of a particular P-N junction increases when it
is exposed to a light of wavelength  621 nm. During this process, enhancement in carrier
concentration takes place due to generation of hole-electron pairs. The value of band gap is nearly.
[JEE MAIN 2021 (JULY)]
(1) 2 eV (2) 4 eV (3) 1 eV (4) 0.5 eV

22. In a given circuit diagram, a 5 V zener diode along with a series resistance is connected across a
50 V power supply. The minimum value of the resistance required, if the maximum zener current
is 90 mA will be _____ . [JEE MAIN 2021 (JULY)]

23. Identify the logic operation carried out. [JEE MAIN 2021 (JULY)]

(1) OR (2) AND (3) NOR (4) NAND

24. In a semiconductor, the number density of intrinsic charge carriers at 27oC is 1.5 × 1016/m3. If the
semiconductor is doped with impurity atom, the hole density increases to 4.5 × 1022/m3. The
electron density in the doped semiconductor is________ × 109/m3.
[JEE MAIN 2021 (JULY)]

NUCLEUS-92, Rajeev Gandhi Nagar, Kota (Raj.) India 324005, Mob. 9358006181, 97831-97831 336
JEE MAIN 2021
25. A transistor is connected in common emitter circuit configuration, the collector supply voltage is
10 V and the voltage drop across a resistor of 1000 in the collector circuit is 0.6 V. If the current
gain factor () is 24, then the base current is _______ A. (Round off to the Nearest Integer)
[JEE MAIN 2021 (JULY)]

26. Find the truth table for the function Y of A and B represented in the following figure.
[JEE MAIN 2021 (JULY)]

(1) (2) (3) (4)

27. In the given figure, each diode has a forward bias resistance of 30and infinite resistance in
reverse bias. The current I1 will be : [JEE MAIN 2021 (AUGUST)]

(1) 3.75 A (2) 2.35 A (3) 2 A (4) 2.73 A

28. Four NOR gates are connected as shown in figure. The truth table for the given figure is :
[JEE MAIN 2021 (AUGUST)]

A B Y A B Y A B Y A B Y
0 0 1 0 0 0 0 0 0 0 0 1
(1) 0 1 0 (2) 0 1 1 (3) 0 1 1 (4) 0 1 0
1 0 1 1 0 1 1 0 0 1 0 0
1 1 0 1 1 0 1 1 1 1 1 1

NUCLEUS-92, Rajeev Gandhi Nagar, Kota (Raj.) India 324005, Mob. 9358006181, 97831-97831 337
JEE MAIN 2021
29. For the given circuit, the power across Zener diode is…….mW.
[JEE MAIN 2021 (AUGUST)]

30. Identify the logic operation carried out by the given circuit :-
[JEE MAIN 2021 (AUGUST)]

(1) OR (2) AND (3) NOR (4) NAND

31. Statement – I : By doping silicon semiconductor with pentavalent material , the electron density
increases. [JEE MAIN 2021 (AUGUST)]
Statement – II : The n- type semiconductor has net negative charge.
In the light of the above statements, choose the most appropriate answer from the option given
below :
(1) Statement – I is true but statement – II is false
(2) Statement – I is false but Statement – II is true
(3) Both statement – I and Statement – II are true
(4) Both Statement – I and Statement – II are false .

IC 1 
32. For a transistor  and  are given as   and   , Then the correct relation between 
IE 
and  will be : [JEE MAIN 2021 (AUGUST)]
1   
(1)   (2)   (3)   1 (4)  
1  1 
33. A zener diode of power rating 2W is to be used as a voltage regulator. If the zener diode has a
breakdown of 10 V and it has to regulate voltage fluctuated between 6 V and 14 V, the value of R s
for safe operation should be _________  . [JEE MAIN 2021 (AUGUST)]

34. For a transistor in CE mode to be used as an amplifier, it must be operated in :


[JEE MAIN 2021 (AUGUST)]
(1) Both cut-off and Saturation (2) Saturation region only
(3) Cut-off region only (4) The active region only

NUCLEUS-92, Rajeev Gandhi Nagar, Kota (Raj.) India 324005, Mob. 9358006181, 97831-97831 338
JEE MAIN 2021
35. A circuit is arranged as shown in figure. The output voltage V0 is equal to ___________V.
[JEE MAIN 2021 (AUGUST)]

36. If VA and VB ate the input voltage (either 5V or 0V) and Vo is the output voltage then the two
gates represented in the following circuit (A) and (B) are :- [JEE MAIN 2021 (AUGUST)]

(1) AND and OR Gate (2) OR and NOT Gate


(3) NAND and NOR Gate (4) AND and NOT Gate

37. Statement – I : [JEE MAIN 2021 (AUGUST)]


To get a steady dc output from the pulsating voltage received from a full wave rectifier we can
connect a capacitor across the output parallel to the load RL.
Statement – II :
To get a steady dc output from the pulsating voltage received from a full wave rectifier we can
connect an inductor in series with RL.
In the light of the above statements, choose the most appropriate answer from the option given
below :
(1) Statement I is true but statement II is false
(2) Statement – I is false but Statement – II is true
(3) Both Statement – I and Statement – II are false
(4) Both Statement –I and Statement- II are true

38. In the following logic circuit the sequence of the inputs A, B are (0, 0), (0, 1), (1, 0) and (1, 1).
The output Y for this sequence will be : [JEE MAIN 2021 (AUGUST)]

(1) 1, 0, 1, 0 (2) 0, 1, 0, 1 (3) 1, 1, 1, 0 (4) 0, 0, 1, 1

NUCLEUS-92, Rajeev Gandhi Nagar, Kota (Raj.) India 324005, Mob. 9358006181, 97831-97831 339
JEE MAIN 2021
39. Choose the correct waveform that can represent the voltage across R of the following circuit,
assuming the diode is ideal one : [JEE MAIN 2021 (AUGUST)]

(1) (2)

(3) (4)

ANSWER KEY
1. 1 2. 4 3. 1 4. 2 5. 2 6. 2 7. 1
8. 4 9. 9 10. 5 11. 0 12. 1 13. 2 14. 4
15. 100 16. 4 17. 200 18. 1 19. 192 20. 25 21. 1
22. 500 23. 2 24. 5 25. 25 26. 2 27. 3 28. 4
29. 120 30. 3 31. 1 32. 2 33. 20 34. 4 35. 5
36. 2 37. 4 38. 3 39. 3

NUCLEUS-92, Rajeev Gandhi Nagar, Kota (Raj.) India 324005, Mob. 9358006181, 97831-97831 340
JEE MAIN 2021
SOLUTION
1. (1)
Sol. I = IC + IB
 I = IC + IB
 IB = 0.5 mA
I
= C
I B
3.5
=
0.5
=7

2. (4)
Sol. Truth table of the given gate :
A B C
0 0 0
0 1 1
1 0 0
1 1 0
Truth table of option (1)
A B C
0 0 1
0 1 1
1 0 0
1 1 1
Truth table of option (2)
A B C
0 0 1
0 1 0
1 0 1
1 1 1
Truth table of option (3)
A B C
0 0 1
0 1 0
1 0 0
1 1 0
Truth table of option (4)
A B C
0 0 0
0 1 1
1 0 0
1 1 0

NUCLEUS-92, Rajeev Gandhi Nagar, Kota (Raj.) India 324005, Mob. 9358006181, 97831-97831 341
JEE MAIN 2021
Since option (1) has same truth table, hence answer is option (4) is correct.
Alternative solution :
Given Boolean expression can be written as A  B = C
 C = A. B  A.B
Hence option (4) is correct

3. (1)
Sol. Back to back diode will not the make a transistor
i
 c
ib

4. (2)
Sol. Zener diode is heavily doped and have narrow depletion layer.
Option (2) is correct.

5. (2)
Sol. conceptual

6. (2)
Sol. 
y = AB  AB 
y = AB . AB
 
y = A  B .  A  B

y  A.A  AB  A.B  BB
y = AB  AB

7. (1)
Sol. (a) Rectifier AC to DC
(b) Stabilizer  used for constant output voltage even when input voltage or current change.
(c) Transformer  Step - up or step - down ac voltage.
(d) Filter  used to remove any ripple in the rectified output voltage.

NUCLEUS-92, Rajeev Gandhi Nagar, Kota (Raj.) India 324005, Mob. 9358006181, 97831-97831 342
JEE MAIN 2021
8. (4)
hc 6.6 1034  3 108
Sol.    6.54 107
E 1.9 1.6 1019
= 654 nm
Red color

9. (9)

Sol.

60
i A
4000
30
i1  A
5000
60 30 9
i  i1    A
4000 5000 1000
current from zener diode
iz = i– i1 = 9mA

10. (5)

Sol.

Voltage across Rs = 22 – 15 = 7V
7 1
Current through Rs = I =  A
35 5
15 1
Current through 90 = I2 =  A
90 6
1 1 1
Current trough zener = –  A
5 6 30
Power through zener diode
P = VI
1
P = 15   0.5watt
30
NUCLEUS-92, Rajeev Gandhi Nagar, Kota (Raj.) India 324005, Mob. 9358006181, 97831-97831 343
JEE MAIN 2021
11. (0)

Sol.

12. (1)
Sol. Truth table for the given logic gate:

The truth table is similar to that of a NOR gate.

13. (2)
Sol. By De Morgan's theorem, we have

14. (4)
Sol. Conceptual

15. (100)
R0
Sol. 106  2 
Ri
104
106  2 
102
2  104    100

16. (4)
IC I
Sol.  ,  C
IE IB
I E  I B  IC
IC 1 1
  
I B  IC I B  1 1
1
IC 


1 

NUCLEUS-92, Rajeev Gandhi Nagar, Kota (Raj.) India 324005, Mob. 9358006181, 97831-97831 344
JEE MAIN 2021
17. (200)
IC 2  103 1
Sol.      103
I B 10  106 5
  2 102
  200

18. (1)
50
Sol. I=  50mA
1000
R = 1000

50
I  25mA
2000
I Z  I1000  I 2000 = 50 – 25 = 25mA

19. (192)
Sol. P = Vi
0.5 = 8i
1
i= A
16
E = 20 = 8 + I RP
RP = 12 × 16 = 192

20. (25)
dV 1 1
Sol. Rd   
di di 5  1103
dv 0.75  0.65
100
 25
4

21. (1)
hc
Sol. Band gap 
0
0 ; threshold wavelength
1242ev  nm
Band gap =  2eV
621nm

NUCLEUS-92, Rajeev Gandhi Nagar, Kota (Raj.) India 324005, Mob. 9358006181, 97831-97831 345
JEE MAIN 2021
22. (500)

Sol.

Voltage across RL = 5V
5
 i2 =
RL
Also voltage across R = 50 – 5 = 45 volt
v 45
By v  iR  R  
i ii  i 2
45
R
5
90mA 
RL
Current in Zener diode is maximum when RL  
 (i2  0 and i1 = i)
45
So R =  500
90mA
23. (2)

Sol.

24. (5)
Sol. nenh = ni2
n i2 1.5 1016  1.5 1.5 1032
2

ne   =
nh 4.5 1022 4.5 1022
5 × 109/m3
25. (25)
IC
Sol.   24; R C  1000
IB
V  0.6
0.6
IC 
1000
IC  6 104
IC 6 104
IB    25A
 24

NUCLEUS-92, Rajeev Gandhi Nagar, Kota (Raj.) India 324005, Mob. 9358006181, 97831-97831 346
JEE MAIN 2021
26. (2)

Sol.

Y  A.B  B

27. (3)

Sol.

As per diagram,
Diode D1 & D2 are in forward bias i.e. R = 30whereas diode D3 is in reverse bias i.e. R =
infinite
 Equivalent circuit will be
Applying KVL starting from point A

  1   30   1  × 130 – I1 × 20 + 200 = 0
I I
2 2
 – 100 I1 + 200 = 0
I1 = 2
Option (3)

NUCLEUS-92, Rajeev Gandhi Nagar, Kota (Raj.) India 324005, Mob. 9358006181, 97831-97831 347
JEE MAIN 2021
28. (4)

Sol.

y  (A  A  B)  (B  A  B)
y  (A  A  B)  (B  A  B)
A B y
0 0 1
0 1 0
1 0 0
1 1 1

29. (120)

Sol.

10V
i  2mA
5k
14V
I  14mA
1k
 Iz  12mA
 P  Iz Vz  120mW

30. (3)

Sol.

Option (3)

31. (1)
Sol. Pentavalent activities have excess free e
So e density increases but overall semiconductor is neutral.
Option (1)
NUCLEUS-92, Rajeev Gandhi Nagar, Kota (Raj.) India 324005, Mob. 9358006181, 97831-97831 348
JEE MAIN 2021
32. (2)
IC I
Sol.  ,   C ; I E  IC  I B
IE IB
IC I /I 
  C B  
IC  I B IC  1   1
IB
1 1
1 
 
1 1 

 


1 

33. (20)
Sol. When unregulated voltage is 14 V voltage across zener diode must be 10 V so potential difference
across resistor VRs  4V and Pzener = 2W
VI = 2
2
I  0.2A
10
VRs  I R s
40
4  0.2R s  R s   20
2
34. (4)
Sol. Active region of the CE transistor is linear region and is best suited for its use as an amplifier
35. (5)
Sol. A diodes D1 and D2 are in forward bias, so they acted as negligible resistance
 Input voltage become zero

 Input current is zero  Output current is zero  V0 = 5 volt

36. (2)
Sol. VA = 5V  A=1
VA = 0V  A=0
VB = 5 V  B=1
VB = 0V  B=0
If A = B = 0, there is no potential anywhere here V0 = 0
If A = 1, B = 0, Diode D1 id forward biased, here V0 = 5V
If A = 0 , B = 1, Diode D2 is forward biased hence V0 = 5V
If A = 1, B = 1, Both diodes are forward biased Truth table for Ist
NUCLEUS-92, Rajeev Gandhi Nagar, Kota (Raj.) India 324005, Mob. 9358006181, 97831-97831 349
JEE MAIN 2021
37. (4)
Sol. To convert pulsating dc into steady dc both of mentioned method are correct.

38. (3)

Sol.

Y  (A.B).(A  B)

Y) (0,0)  1

Y)(0,1)  1

Y)(1,0)  1

Y) (1,1)  0

option (3) is correct

39. (3)
Ans. by Nucleus (Bonus)
Sol. When Vi> 3 volt, diode will be in forward biased state
When Vi  3 volt; diode will be in reverse biased state.
So, current will flow only for less than half cycle ⇒ VR will be only for less than half cycle.

NUCLEUS-92, Rajeev Gandhi Nagar, Kota (Raj.) India 324005, Mob. 9358006181, 97831-97831 350
JEE MAIN 2021
SUPPLEMENT_FOR JEE MAIN
1. An audio signal m = 20 sin 2 (1500 t) amplitude modulates a carrier
C = 80 sin 2 (100,000 t). [JEE MAIN 2021 (FEB)]
The value of percent modulation is _______.

2. Given below are two statement : [JEE MAIN 2021 (FEB)]


Statement-I: A speech signal of 2 kHz is used to modulate a carrier signal of 1 MHz. The band
width requirement for the signal is 4 kHz.
Statement-II : The side band frequencies are 1002 kHz. and 998 kHz.
In the light of the above statements, choose the correct answer from the options given below:
(1) Statement I is true but Statement II is false
(2) Statement I is false but Statement II is true
(3) Both Statement I and Statement II are true
(4) Both Statement I and Statement II are false

3. If a message signal of frequency 'fm' is amplitude modulated with a carrier signal of frequency 'fc'
and radiated through an antenna, the wavelength of the corresponding signal in air is :
[JEE MAIN 2021 (FEB)]
c c c c
(1) (2) (3) (4)
fc  f m fm fc  f m fc

4. The maximum and minimum amplitude of an amplitude modulated wave is 16V and 8V
respectively. The modulation index for this amplitude modulated wave is x × 10 –2. The value of x
is______. [JEE MAIN 2021 (FEB)]

5. If the highest frequency modulating a carrier is 5 kHz, then the number of AM broadcast stations
accommodated in a 90 kHz bandwidth are ......... [JEE MAIN 2021 (FEB)]

6. An electromagnetic wave of frequency 5 GHz, is travelling in a medium whose relative electric


permittivity and relative magnetic permeability both are 2. Its velocity in this medium is
________× 107 m/s. [JEE MAIN 2021 (FEB)]

7. Match List - I with List - II.


List - I List - II
(a) Source of microwave frequency (i) Radioactive decay on nucleus
(b) Source of infrared frequency (ii) Magnetron
(c) Source of Gamma Rays (iii) Inner shell electrons
(d) Source of X-rays (iv) Vibration of atoms and molecules
(v) LASER
(vi) RC circuit
Choose the correct answer from the options given below : [JEE MAIN 2021 (FEB)]
(1) (a)-(vi), (b)-(iv), (c)-(i), (d)-(v) (2) (a)-(vi), (b)-(v), (c)-(i), (d)-(iv)
(3) (a)-(ii), (b)-(iv), (c)-(vi), (d)-(iii) (4) (a)-(ii), (b)-(iv), (c)-(i), (d)-(iii)

NUCLEUS-92, Rajeev Gandhi Nagar, Kota (Raj.) India 324005, Mob. 9358006181, 97831-97831 351
JEE MAIN 2021
8. An electromagnetic wave of frequency 3 GHz enters a dielectric medium of relative electric
permittivity 2.25 from vacuum. The wavelength of this wave in that medium will be ____ × 10–2
cm. [JEE MAIN 2021 (FEB)]

9. A radiation is emitted by 1000 W bulb and it generates an electric field and magnetic field at P,
placed at a distance of 2 m. The efficiency of the bulb is 1.25%. The value of peak electric field at
P is x × 10–1 V/m. Value of x is_. [JEE MAIN 2021 (FEB)]
(Rounded-off to the nearest integer)
[Take 0 = 8.85 × 10–12 C2N–1 m–2, c = 3 × 108 ms–1]

10. The electric field intensity produced by the radiation coming from a 100 W bulb at a distance of
3m is E. The electric field intensity produced by the radiation coming from 60 W at the same
x
distance is E. Where the value of x = ___________ . [JEE MAIN 2021 (MARCH)]
5

11. A plane electromagnetic wave of frequency 500 MHz is travelling in vacuum along y-direction. At
a particular point in space and time, ⃗ ̂ . The value of electric field at this point is :
8 –1
(speed of light = 3 × 10 ms ) [JEE MAIN 2021 (MARCH)]
ˆx, y,
ˆ zˆ are unit vectors along x, y and z direction.
(1) 24xˆ V / m (2) 2.6 xˆ V / m (3) 24 xˆ V / m (4) 2.6 yˆ V / m

12. For an electromagnetic wave travelling in free space. the relation between average energy
densities due to electric (Ue) and magnetic (Um) fields is: [JEE MAIN 2021 (MARCH)]
(1) Ue = Um (2) Ue> Um (3) Ue< Um (4) Ue  Um

13. Red light differs from blue light as they have: [JEE MAIN 2021 (MARCH)]
(1) Different frequencies and different wavelengths
(2) Different frequencies and same wavelengths
(3) Same frequencies and same wavelengths
(4) Same frequencies and different wavelengths

14. Seawater at a frequency f = 9 × 102 Hz, has permittivity   800 and resistivity  = 0.25 m
Imagine a parallel plate capacitor is immersed in seawater and is driven by an alternating voltage
source V(t) = V0 sin (2Ft). Then the conduction current density becomes 10x times the
1
displacement current density after time t  s. The value of x is _________
800
 1 
 Given :  9 109 Nm2C–2  [JEE MAIN 2021 (MARCH)]
 40 

15. A plane electromagnetic wave of frequency 100 MHz is travelling in vacuum the x-direction. At a
ˆ . (where, k̂ is unit vector along z-direction)
particular point in space and time, B  2.0 108 kT
What is E at this point? [JEE MAIN 2021 (MARCH)]

NUCLEUS-92, Rajeev Gandhi Nagar, Kota (Raj.) India 324005, Mob. 9358006181, 97831-97831 352
JEE MAIN 2021
16. A plane electromagnetic wave propagating along y-direction can have the following pair of
   
electric field E and magnetic field B components. [JEE MAIN 2021 (MARCH)]
(1) Ey, By or Ez, Bz (2) Ey, Bx or Ex, By
(3) Ex, Bz or Ez, Bx (4) Ex, By or Ey, Bx

17. Consider a uniform wire of mass M and length L. It is bent into a semicircle. Its moment of inertia
about a line perpendicular to the plane of the wire passing through the centre is:
[JEE MAIN 2021 (MARCH)]
1 ML2 2 ML2 ML2 1 L2
(1) (2) (3) (4)
4 2 5 2 2 2 2

18. A 25m long antenna is mounted on an antenna tower. The height of the antenna tower is 75m.
The wavelength (in meter) of the signal transmitted by this antenna would be:
[JEE MAIN 2021 (MARCH)]
(1) 300 (2) 400 (3) 200 (4) 100

19. Two identical antennas mounted on identical towers are separated from each other by a distance of
45km. What should nearly be the minimum height of receiving antenna to receive the signals in
line of sight? [JEE MAIN 2021 (MARCH)]
(Assume radius of earth is 6400 km)
(1) 19.77m (2) 39.55 m (3) 79.1 m (4) 158.2 m

20. For VHF signal broadcasting, ____ km2 of maximum service area will be covered by an antenna
tower of height 30m, if the receiving antenna is placed at ground. Let radius of the earth be
6400km. (Round off to Nearest Integer) (Take  as 3.14) [JEE MAIN 2021 (MARCH)]

21. A carrier signal C(t) = 25 sin (2.512 × 1010t) is amplitude modulated by a message signal m(t) =
5sin (1.57 × 108t) and transmitted through an antenna. What will be the bandwidth of the
modulated signal? [JEE MAIN 2021 (MARCH)]
(1) 8 GHZ (2) 2.01 GHz
(3) 1987.5 MHz (4) 50 MHz

22. Match List – I with List – II. [JEE MAIN 2021 (MARCH)]
List – I
(a) 10km height over earth's surface (b) 70km height over earth's surface
(c) 180km height over earth's surface (d) 270 km height over earth's surface
List – II
(i) Thermosphere
(ii) Mesosphere
(iii) Stratosphere
(iv) Troposphere
(1) (a) – (iv), (b) – (iii), (c) – (ii), (d) – (i) (2) (a) – (i), (b) – (iv), (c) – (iii), (d) – (ii)
(3) (a) – (iii), (b) – (ii), (c) – (i), (d) – (iv) (4) (a) – (ii), (b) – (i), (c) – (iv), (d) – (iii)

NUCLEUS-92, Rajeev Gandhi Nagar, Kota (Raj.) India 324005, Mob. 9358006181, 97831-97831 353
JEE MAIN 2021
23. A TV transmission tower antenna is at a height of 20 m. Suppose that the receiving antenna is at.
(i) ground level [JEE MAIN 2021 (MARCH)]
(ii) a height of 5m.
The increase in antenna range in case (ii) relative to case (i) is n%.
The value of n, to the nearest integer, is.

24. AC voltage V(t) = 20 sin t of frequency 50 Hz is applied to a parallel plate capacitor. The
separation between the plates is 2 mm and the area is 1 m2. The amplitude of the oscillating
displacement current for the applied AC voltage is ___________
[Take 0 = 8.85 x 10-12 F/m] [JEE MAIN 2021 (JULY)]
(1) 21.14 A (2)83.37 A (3)27.79 A (4)55.58 A

25. In the electromagnetic wave the electric field vector and magnetic field vector are given as
E  E0 ˆi and B  B0K ˆ respectively. The direction of propagation of electromagnetic wave is
along: [JEE MAIN 2021 (JULY)]
ˆ
(1) (K) (2) Ĵ ˆ
(3) (  K) ˆ
(4) (  j)

26. Intensity of sunlight is observed as 0.092 Wm–2ata point in free space. What will be the peak value
of magnetic field at that point ? [JEE MAIN 2021 (JULY)]
–12 2 –1 –2
(0 = 8.85 × 10 C N m )
(1) 2.77 × 10–8 T (2) 1.96 × 10–8 T (3) 8.31 T (4) 5.88 T

27. A linearly polarized electromagnetic wave in vacuum is


E  3.1cos[1.8 z   5.4 106  t]iˆ N/ C [JEE MAIN 2021 (JULY)]
is incident normally on a perfectly reflecting wall at Z= a. Choose the correct option
(1) The wavelength is 5.4 m
(2) The frequency of electromagnetic wave is 54 104 Hz.
(3) The transmitted wave will be 3.1cos 1.8  z   5.4 106  t  ˆi N/C

(4) The reflected wave will be 3.1cos 1.8  z   5.4 106  t  ˆi N/C

28. In amplitude modulation, the message signal Vm  t   10sin  2105 t  volts and Carrier signal
Vc  t   20sin  2107 t  volts [JEE MAIN 2021 (JULY)]
The modulated signal now contains the message signal with lower side band and upper side band
frequency, therefore the bandwidth of modulated signal is  kHz. The value of  is :
(1) 200 kHz (2) 50 kHz (3) 100 kHz (4) 0

29. The amplitude of upper and lower side bands of A.M. wave where a carrier signal with frequency
11.21 MHz, peak voltage 15 V is amplitude modulated by a 7.7 kHz sine wave of 5V amplitude
a b a
are V and V respectively. Then the value of is ________.
10 10 b
[JEE MAIN 2021 (JULY)]

NUCLEUS-92, Rajeev Gandhi Nagar, Kota (Raj.) India 324005, Mob. 9358006181, 97831-97831 354
JEE MAIN 2021
30. The maximum amplitude for an amplitude modulated wave is found to be 12V while the minimum
amplitude is found to be 3V. The modulation index is 0.6x where x is _________.
[JEE MAIN 2021 (JULY)]

31. A carrier wave Vc(t) = 160 sin (26 t) volts is made to vary between Vmax = 200 V and Vmin=
120 V by a message signal Vm(t) = Am sin(2 t) volts. The peak voltage Am of the modulating
signal is ______. [JEE MAIN 2021 (JULY)]

32. What should be the height of transmitting antenna and the population covered if the television
telecast is to cover a radius of 150 km? The average population density around the tower is
2000/km2 and the value of Re = 6.5 × 106 m. [JEE MAIN 2021 (JULY)]
(1) Height = 1731 m
Population Covered = 1413 × 105
(2) Height = 1241 m
Population Covered = 7 × 105
(3) Height = 1600 m
Population Covered = 2 × 105
(4) Height = 1800 m
Population Covered = 1413 × 108

33. A message signal of frequency 20 kHz and peak voltage of 20 volt is used to modulate a carrier
wave of frequency 1 MHz ,and peak voltage of 20 volt. The modulation index will be.
[JEE MAIN 2021 (JULY)]

34. The maximum amplitude for an amplitude modulated wave is found to be 12V while the minimum
amplitude is found to be 3V. The modulation index is 0.6x where x is _________.
[JEE MAIN 2021 (JULY)]

35. The magnetic susceptibility of a material of a rod is 499. Permeability in vacuum is 4  107
H/m. Absolute permeability of the material of the rod is : [JEE MAIN 2021 (JULY)]
(1) 4  10 H / m (2) 2  10 H / m (3) 3  10 H / m (4)   104 H / m
4 4 4

36. Statement I : The ferromagnetic property depends on temperature. At high temperature,


ferromagnet becomes paramagnet. [JEE MAIN 2021 (JULY)]
Statement II : At high temperature, the domain wall area of a ferromagnetic substance increases.
In the light of the above statements, choose the most appropriate answer from the options given
below :
(1) Statement I is true but Statement II is false
(2) Both Statement I and Statement II are true
(3) Both Statement I and Statement II are false
(4) Statement I is false but Statement II is true

37. The value of aluminium susceptibility is 2.2 105 The percentage increase in the magnetic field if
x
space within a current carrying toroid is filled with aluminium is . Then the value of x
10 4
is________. [JEE MAIN 2021 (JULY)]

NUCLEUS-92, Rajeev Gandhi Nagar, Kota (Raj.) India 324005, Mob. 9358006181, 97831-97831 355
JEE MAIN 2021
38. Following plots show Magnetization (M) vsMagnetising field (H) and Magnetic susceptibility ()
vs temperature (T) graph : [JEE MAIN 2021 (AUGUST)]

(1) (2)

(3) (4)

Which of the following combination will be represented by a diamagnetic material?


(1) (a), (c) (2) (a), (d) (3) (b), (d) (4) (b), (c)

39. A carrier wave with amplitude of 250 V isamplitude modulated by a sinusoidal basebandsignal of
amplitude 150 V. The ratio of minimum amplitude to maximum amplitude for theamplitude
modulated wave is 50 : x, then value of x is ........... [JEE MAIN 2021 (AUGUST)]

40. Electric field of plane electromagnetic wave propagating through a non-magnetic medium is given
by E = 20cos (2 × 1010 t–200x) V/m. The dielectric constant of the medium is equal to :
(Take r = 1) [JEE MAIN 2021 (AUGUST)]
1
(1) 9 (2) 2 (3) (4) 3
3

41. A transmitting antenna at top of a tower has aheight of 50 m and the height of receiving antennais
80 m. What is range of communication for Line of Sight (LoS) mode ?
[JEE MAIN 2021 (AUGUST)]
[use radius of earth = 6400 km]
(1) 45.5 km (2) 80.2 km (3) 144.1 km (4) 57.28 km

A light beam is described by E = 800 sin  t   . An electron is allowed to move normal to the
x
42.
 c
propagation of light beam with a speed of 3 × 107ms–1. What is the maximum magnetic force
exertedon the electron ? [JEE MAIN 2021 (AUGUST)]
–18 –21
(1) 1.28 × 10 N (2) 1.28 × 10 N
–17
(3) 12.8 × 10 N (4) 12.8 × 10–18 N

43. A source of light is placed in front of a screen. intensity of light on the screen is I. Two Polaroids
P1 and P2 are so placed in between the source of light and screen that the intensity of light on
screen is I/2. P2 should be rotated by an angle of ………. (degrees) so that the intensity of light on
3I
the screen becomes . [JEE MAIN 2021 (AUGUST)]
8
NUCLEUS-92, Rajeev Gandhi Nagar, Kota (Raj.) India 324005, Mob. 9358006181, 97831-97831 356
JEE MAIN 2021
44. The electric field in a plane electromagnetic wave is given by
[JEE MAIN 2021 (AUGUST)]
 0.5 10  
3
11 rad  V
E  200 cos     1.5 10  t   ˆj
 m   s  m
If this wave falls normally on a perfectly reflecting surface having an area of 100 cm2. If the
radiation pressure exerted by the E. M. wave on the surface during a 10 minute exposure is
x N
. Find the value of x .
109 m 2

45. An amplitude modulated wave is represented by Cm(t) = 10(1 + 0.2 cos12560t) sin(111 x 104 t)
volts. The modulating frequency in kHz will be ........
[JEE MAIN 2021 (AUGUST)]

46. An antenna is mounted on a 400 m tall building. What will be the wavelength of signal of signal
that can be radiated effectively by the transmission tower upto a range of 44 km?
[JEE MAIN 2021 (AUGUST)]
(1) 37.8 m (2) 605 m (3) 75.6 m (4) 302 m

47. A plane electromagnetic wave with frequency of 30 MHz travels in free space. A particular point
in space and time, electric field is 6 V/m. The magnetic field at this point will be x  108 T. The
value of x is __________. [JEE MAIN 2021 (AUGUST)]

48. A transmitting antenna has a height of 320 m and that of receiving antenna is 2000 m. The
maximum distance between them for satisfactory communication in line of slight mode is 'd'. The
value of 'd' is ........km. [JEE MAIN 2021 (AUGUST)]

49. Electric field in a plane electromagnetic wave is given by E = 50 sin(500x – 10  1010t) V/m. The
velocity of electromagnetic wave in this medium is : [JEE MAIN 2021 (AUGUST)]
(Given C = speed of light in vacuum)
3 2 C
(1) C (2) C (3) C (4)
2 3 2

50. A bandwidth of 6 MHz is available for A.M. transmission. If the maximum audio signal frequency
used for modulating the carrier wave is not to exceed 6 kHz. The number of stations that can be
broadcasted within this band simultaneously without interfering with each other will be
_________ . [JEE MAIN 2021 (AUGUST)]

51. If the sum of the height of transmitting and receiving antennas in the line of slight of
communication is fixed at 160 m, then the maximum range of LOS communication is ______km.
(Take radius of Earth = 6400 km) [JEE MAIN 2021 (AUGUST)]

52. The electric field in an electromagnetic wave is given by E = (50 NC-1) sin (t  x / c)
The energy contained in a cylinder of volume V is 5.5  10-12 J. The value of V is _________cm3.
(given 0  8.8 1012 C2 N 1m 2 ) [JEE MAIN 2021 (AUGUST)]

NUCLEUS-92, Rajeev Gandhi Nagar, Kota (Raj.) India 324005, Mob. 9358006181, 97831-97831 357
JEE MAIN 2021
ANSWER KEY
1. 25 2. 3 3. 4 4. 33 5. 9 6. 15 7. 4
8. 667 9. 137 10. 3 11. 1 12. 1 13. 1 14. 6
15. 3 16. 3 17. 3 18. 4 19. 2 20. 1206 21. 4
22. 1 23. 50 24. 3 25. 4 26. 1 27. 4 28. 1
29. 1 30. 1 31. 40 32. 1 33. 1 34. 1 35. 2
36. 1 37. 22 38. 1 39. 200 40. 1 41. 4 42. 4
43. 30 44. 354 45. 2 46. 2 47. 2 48. 224 49. 3
50. 500 51. 64 52. 500

SOLUTION
1. (25)
Am
Sol. % modulation =  100
Ac
20
% modulation =  100
80
% modulation = 25%

2. (3)
Sol. fm = 2kHz
fc = 1MHz = 1000 kHz
Band width = 2fm = 4kHz
  Side frequencies will be
= fc ± fm = (1000 ± 2) kHz = 998 kHz & 1002 kHz
So statement-I & statement-II both are correct.

3. (4)
v c
Sol.  
f fc

4. (33)
A max  A min
Sol. Modulation index =
A max  A min
16  8 8 1
    0.33
16  8 24 3
x 102  0.33
x = 33.
5. (9)
Sol. B. W. (Bandwidth) = 2 × maximum frequency at modulating signal
= 2 × 5kHz
= 10 kHz
 No of stations accommodated
90
 9
10
NUCLEUS-92, Rajeev Gandhi Nagar, Kota (Raj.) India 324005, Mob. 9358006181, 97831-97831 358
JEE MAIN 2021
6. (15)
Sol. Given : Frequency of wave f = 5 GHz
= 5 × 109 Hz
Relative permittivity, r = 2
and Relative permeability, μr = 2
Since speed of light in a medium is given by,
1 1
v= 
  r  0 . r 0
1 1 C
v= 
 r r  0 r  r r
Where C is speed of light is vacuum.
3 108 30 107
v=  m/s
4 2
= 15 × 107 m/s
 Ans. is 15

7. (4)
Sol. (a) Source of microwave frequency is magnetron.
(b) Source of infrared frequency is vibration of atoms and molecules.
(c) Source of Gamma rays is radioactive decay of nucleus
(d) Source of X-rays inner shell electron transition.
Option (4) is correct.

8. (667)
c 3 108
Sol.  in vacuum =   0.1m
f 3 109
0.1
in medium 

Where refractive index
  r r
Assuming non – magnetic material r = 1
   2.25  1.5
0.1 1
m   m  6.67 cm
1.5 15
= 667 × 10–2 cm
Ans. 667

NUCLEUS-92, Rajeev Gandhi Nagar, Kota (Raj.) India 324005, Mob. 9358006181, 97831-97831 359
JEE MAIN 2021
9. (137)
1
Sol. Iavg   0 E 02 C
2
1.25 1000 1
   8.85 1012  3 108  e02
100 4  2 2
2
E 02  187.4
 E0 = 13.689 V/m = 136.89× 10–1 V/m
 x = 136.89
Rounding off to nearest integer
x = 137

10. (3)
100
Sol. c 0 E 2 
4 32
2
 x  60 x 3
c 0  E     x 3
 5  4 32
5 5

11. (1)
Sol. f  5 108 Hz
EM wave is travelling towards  ĵ
B  8.0 108 zT
ˆ
E  B  C  8 108 ẑ    3 108 yˆ 
 24xV
ˆ /m

12. (1)
Sol. In EMW. Average energy density due to electric (Ue) and magnetic (Um) fields is same.

13. (1)
Sol. Red light and blue light have different wavelength and different frequency.

14. (6)
E V
Sol. Jc  
ρ ρd
1 dq
Jd 
A dt
C dVc  dVc
 
A dt d dt
V sin 2ft 800
 0  10x  V0 (2f ) cos 2ft
d d
 900  40
tan  2π    10 
x
 900  x  6
 800  9 109

NUCLEUS-92, Rajeev Gandhi Nagar, Kota (Raj.) India 324005, Mob. 9358006181, 97831-97831 360
JEE MAIN 2021
15. (3)
Sol. E = BC = 6

(Dir. of wave) || E  B 
ˆi  ˆj  kˆ

E  6ˆjV / m

16. (3)

Sol.

17. (3)
L
Sol. r  L  r 

ML2
I  Mr 2  2

18. (4)

Sol. Length of Antena = 25m =
4
   100m

19. (2)
Sol. D  2 2Rh
D2 452
h  km  ~ 39.55m
8R 8  6400

20. (1206)
Sol. d = 2RH
A = d2
A = 2Rh
30
= 3.14 × 2 × 6400 ×
1000
A = 1205.76 km2
A 1206 km2

NUCLEUS-92, Rajeev Gandhi Nagar, Kota (Raj.) India 324005, Mob. 9358006181, 97831-97831 361
JEE MAIN 2021
21. (4)
Sol. Band width = 2fm
m = 1.57 × 108 = 2fm
108
BW  2f m  Hz  50MHz
2

22. (1)
Sol. Order of atmosphere stratification from bottom Troposphere, stratosphere, Mesosphere,
Thermosphere
(a)  (iv)
(b)  (iii)
(c)  (ii)
(d)  (i)

23. (50)
Sol. Range = 2Rh
Range  i   2RH
Range  ii   2RH  2RH'
where h = 20 m & h'= 5m
2Rh ' 5
Ans =  100%   100%  50%
2Rh ' 20

24. (3)

Sol.

From the given information,


 A  1
C  0  0 3 F
d 2 10
1 2 103 2 103
 XC    
C 2  500 25  4 0
2 103 18
 XC   9 109  106 
25 25
V0 20  25
 i0   106 A  27.47A
XC 18
The value of amplitude of displacement current will be same as value of amplitude of
conventional current.
Hence option 3.

NUCLEUS-92, Rajeev Gandhi Nagar, Kota (Raj.) India 324005, Mob. 9358006181, 97831-97831 362
JEE MAIN 2021
25. (4)
Sol. Direction of propagation = E  B  ˆi  kˆ  ˆj

26. (1)
B02 C 1
Sol. Iavg  &   0 C2
2 0  0
B02
I 0 C3
2
2I
B0 
 0 C3
B0 = 2.77 × 10–8 T

27. (4)
Sol. Reflected wave will have direction opposite to incident wave.

28. (1)
Sol. Bandwidth = 2 × fm = 2 × 105 HZ = 200 KHZ

29. (1)

Sol.

a b A C a
   1
10 10 2 b

30. (1)
Sol. A max  A c  A m  12
15 9
A min  A c  A m  3  A c  & Am 
2 2
Am 9 / 2
modulation index    0.6  x = 1
A c 15 / 2

31. (40)
Sol. Maximum amplitude
Amax = Am + AC Vmax = Vm + VC
200 = Vm + 160
Vm = 40
 Peak voltage Am = 40

NUCLEUS-92, Rajeev Gandhi Nagar, Kota (Raj.) India 324005, Mob. 9358006181, 97831-97831 363
JEE MAIN 2021
32. (1)
Sol. Radius covered r  2RHT
150 km = 2   6.5 106 m  HT
(150 km × 103)2 = 2 × 6.5 × 106 HT
HT = 1731m
Population covered = (r2) (2000/km2)
= 3.14 × (150)2 × 2000 = 1413 × 105

33. (1)
Sol. Modulation index
A 20
= m  1
A c 20

34. (1)
Sol. A max  A c  A m  12
15 9
A min  A c  A m  3  A c  & Am 
2 2
Am 9 / 2
modulation index    0.6  x = 1
A c 15 / 2

35. (2)
Sol.  = 0 (1 + xm)
= 4 × 10–7 × 500 = 2 × 10–4 H/m

36. (1)
Sol. As temperature increases, domains disintegrate so ferromagnetism decreases and above curie
temperature it become paramagnet.

37. (22)
Sol. B   (H  I)
 I
B    H 1  
 H
B = B0(1 + x)
B – B0 = B0X
B  B0
x
B0
B  B0
100  100x
B0
22
 2.2 10 3 
104

NUCLEUS-92, Rajeev Gandhi Nagar, Kota (Raj.) India 324005, Mob. 9358006181, 97831-97831 364
JEE MAIN 2021
38. (1)
Sol. Conceptual question
Option (1)

39. (200)
Sol. Amax = AC + Am = 250 + 150 = 400
Amin = AC– Am = 250 – 150 = 100
A min 100 1 50
  
A max 400 4 200
x = 200

40. (1)
2 1010
Sol. Speed of wave = = 108 m/s
200
3 108
Refractive index = 3
108
Now refractive index = r r
3= r 1 r = 9
Option (1)

41. (4)

Sol.

d t  2Rh1  2Rh 2
 2R  h1  h 2 
  2  6400 103  ( 50  80) = 3578 (7.07 + 8.94) = 57.28 km
1/2

42. (4)
E0
Sol.  B0
C
800
Fmax  eB0 V  1.6 1019   3 107
3 108

= 12.8 × 10–18N

NUCLEUS-92, Rajeev Gandhi Nagar, Kota (Raj.) India 324005, Mob. 9358006181, 97831-97831 365
JEE MAIN 2021
43. (30)
I
Sol. I = 0 cos 2 
2

I 3I
cos 2  
2 8
3
cos 2  
4
3
cos 2    = 30
2

44. (354)
Sol. E0 = 200
1
I   0 E 02 .C
2
radiation pressure
2I
P
C
 2  1 
   0 E 02 C 
 C  2 
  0 E 02
 8.85 1012  2002
354
 8.85 108  4  9
10

45. (2)
Sol. Wm  12560  2 f m
12560
fm 
2
= 2000 Hz
Ans. 2.00
NUCLEUS-92, Rajeev Gandhi Nagar, Kota (Raj.) India 324005, Mob. 9358006181, 97831-97831 366
JEE MAIN 2021
46. (2)
Sol. h : height of antenna
 : wavelength of signal
h
h
  400m

47. (2)
E 6
Sol. B   2 108 T
C 3 108

x  2

48. (224)
Sol. dm  2RHT  2RHR
d m  ( 2  6400 103  320  2  6400 103  2000)m
d m  224km

49. (3)
 10 1010
Sol. V   2 108
K 500
2C
V .
3

50. (500)
Sol. Signal bandwidth = 2 fm
=12 kHz
6MHZ 6 106
N    500
kHZ 12 103

51. (64)
Sol. hT = hR = 160 ...(i)
d  2Rh T  2Rh R

d  2R  h T  h R 

d  2R  x  160  x 
d(d)
0
dx

NUCLEUS-92, Rajeev Gandhi Nagar, Kota (Raj.) India 324005, Mob. 9358006181, 97831-97831 367
JEE MAIN 2021
1 1(1)
 0
2 x 2 160  x
1 1

x 160  x
x = 80 m
 80 20 
d max  2  6400   
 1000 1000 

80 2  2 80

10 10
 8  2  2  2 2  64km

52. (500)
  
Sol. E  50sin  t  .x 
 c 
1
Energy density  0 E 02
2
1
Energy for volume V  0 E 02 .V  5.5  10 12
2
1
8.8  10 12  2500V  5.5  10 12
2
5.5  2
V  .0005m 3
2500  8.8
 .0005 106 (c.m)3
= 500 (c.m)3

NUCLEUS-92, Rajeev Gandhi Nagar, Kota (Raj.) India 324005, Mob. 9358006181, 97831-97831 368
JEE MAIN 2021
ERROR
L
1. The period of oscillation of a simple pendulum is T  2 . Measured value of 'L' is 1.0 m from
g
meter scale having a minimum division of 1 mm and time of one complete oscillation is 1.95 s
measured from stopwatch of 0.01 s resolution. The percentage error in the determination of 'g' will
be : [JEE MAIN 2021 (FEB)]
(1) 1.13% (2) 1.03% (3) 1.33% (4) 1.30%

2. The pitch of the screw gauge is 1mm and there are 100 divisions on the circular scale. When
nothing is put in between the jaws, the zero of the circular scale lies 8 divisions below the
reference line. When a wire is placed between the jaws, the first linear scale division is clearly
visible while 72nd division on circular scale coincides with the reference line. The radius of the
wire is [JEE MAIN 2021 (FEB)]
(1) 1.64 mm (2) 0.82 mm (3) 1.80 mm (4) 0.90 mm

3. One main scale division of a vernier calipers is 'a' cm and nth division of the vernier scale
coincide with (n – 1)th division of the main scale. The least count of the callipers in mm is :
[JEE MAIN 2021 (MARCH)]
10 na 10 a  n 1  10 a
(1) (2) (3)  a (4)
 n  1  n  1  10 n  n

V
4. The resistance R = , where V = 50  2 V and I =  20  0.2 A The percentage error in R is
I
'x'%. The value of 'x' to the nearest integer is _____. [JEE MAIN 2021 (MARCH)]

5. In order to determine the Young's Modulus of a wire of radius 0.2cm (measured using a scale of
least count = 0.001cm) and length 1m (measured using a scale of least count = 0.001cm) and
length 1cm (measured using a scale of least count = 1mm), a weight of mass 1kg (measured using
a scale of least count = 1g) was hanged to get the elongation of 0.5cm (measured using a scale of
least count 0.001cm). What will be the fractional error in the value of Young's Modulus
determined by this experiment? [JEE MAIN 2021 (MARCH)]
(1) 0.14% (2) 0.9% (3) 9% (4) 1.4%

6. In the experiment of Ohm's law, a potential difference of 5.0V is applied across the end of a
conductor of length 10.0 cm and diameter of 5.00 mm. The measured current in the conductor is
2.00 A. The maximum permissible percentage error in the resistivity of the conductor is:-
[JEE MAIN 2021 (MARCH)]
(1) 3.9 (2) 8.4 (3) 7.5 (4) 3.0

7. The radius of a sphere is measured to be (7.50  0.85) cm. Suppose the percentage error in its
volume is x. The value of x, to the nearest x, is _______ . [JEE MAIN 2021 (MARCH)]

NUCLEUS-92, Rajeev Gandhi Nagar, Kota (Raj.) India 324005, Mob. 9358006181, 97831-97831 369
JEE MAIN 2021
8. The time period of a simple pendulum is given by. The measured value of the length of pendulum
is 10cm known to a 1mm accuracy. The time for 200 oscillations of the pendulum is found to be
100 second using a clock of 1s resolution. The percentage accuracy in the determination of 'g'
using this pendulum is 'x'. The value of 'x' to the nearest integer is:- [JEE MAIN 2021 (MARCH)]
(1) 2% (2) 3% (3) 5% (4) 4%

9. The vernier scale used for measurement has a positive zero error of 0.2 mm. If while taking a
measurement it was noted '0' on the vernier scale lies between 8.5 cm and 8.6 cm, vernier
coincidence is 6, then the correct value of measurement is ______ cm.
[JEE MAIN 2021 (MARCH)]
(least count = 0.01 cm)
(1) 8.36 cm (2) 8.54 cm (3) 8.58 cm (4) 8.56 cm

10. At an angle of 30° to the magnetic meridian , the apparent dip is 45°. Find the true dip:
[JEE MAIN 2021 (JULY)]
1 2 3
(1) tan 1 3 (2) tan 1 (3) tan 1 (4) tan 1
3 3 2
11. Three students S1, S2 and S3 perform an experiment for determining the acceleration due to gravity
(g) using a simple pendulum. They use different lengths of pendulum and record time for different
number of oscillations. The observations are as shown in the table.
[JEE MAIN 2021 (JULY)]

(Least count of length = 0.1 m least count for time = 0.1 s)


If E1, E2 and E3 are the percentage errors in 'g' for students 1, 2 and 3 respectively, then the
minimum percentage error is obtained by student no._____.
12. Student A and Student B used two screw gauges of equal pitch and 100 equal circular division to
measure the radius of a given wire. The actual value of the radius of the wire is 0.322 cm. The
absolute value of the difference between the final circular scale readings observed by the students
A and B is________ [JEE MAIN 2021 (JULY)]
[Figure shows position of reference ‘O’ when jaws of screw gauge are closed]
Given pitch = 0.1 cm.

NUCLEUS-92, Rajeev Gandhi Nagar, Kota (Raj.) India 324005, Mob. 9358006181, 97831-97831 370
JEE MAIN 2021
13. Assertion A : If in five complete rotations of the circular scale, the distance travelled on main
scale of the screw gauge is 5 mm and there are 50 total divisions on circular scale, then least count
is 0.001 cm. [JEE MAIN 2021 (JULY)]
Pitch
Reason R :LeastCount=
Total divisions on circular scale
In the light of the above statements, choose the most appropriate answer from the options given
below :
(1) A is not correct but R is correct.
(2) Both A and R are correct and R is the correct explanation of A.
(3) A is correct but R is not correct.
(4) Both A and R are correct and R is NOT the correct explanation of A.

14. The acceleration due to gravity is found upto an accuracy of 4% on a planet. The energy supplied
to a simple pendulum to known mass 'm' to undertake oscillations of time period T is being
estimated. If time period is measured to an accuracy of 3%, the accuracy to which E is known as
..........% [JEE MAIN 2021 (AUGUST)]

15. In a Screw Gauge, fifth division of the circular scale coincides with the reference line when the
ratchet is closed. There are 50 divisions on the circular scale, and the main scale moves by 0.5 mm
on a complete rotation. For a particular observation the reading on the main scale is 5 mm and the
20th division of the circular scale coincides with reference line. Calculate the true reading .
[JEE MAIN 2021 (AUGUST)]
(1) 5.00 mm (2) 5.25 mm (3) 5.15 mm (4) 5.20 mm

16. The diameter of a spherical bob is measured using a vernier calipers. 9 divisions of the main
scale, in the vernier callipers, are equal to 10 divisions of vernier scale. One main scale division is
1 mm. The main scale reading is 10 mm and 8th division of vernier scale was found to coincide
exactly with one of the main scale division. If the given verniercalipers has positive zero error of
0.04 cm, then the radius of the bob is __________  10-2 cm. [JEE MAIN 2021 (AUGUST)]

ANSWER KEY
1. 1 2. 2 3. 4 4. 5 5. 4 6. 1 7. 34
8. 2 9. 2 10. 4 11. 1 12. 13 13. 1 14. 14
15. 3 16. 52

NUCLEUS-92, Rajeev Gandhi Nagar, Kota (Raj.) India 324005, Mob. 9358006181, 97831-97831 371
JEE MAIN 2021
SOLUTION
1. (1)

Sol. T  2
g
4 2
g=
T2
g  2T
 
g T
g 1103 0.01
  2
g 1 1.95
g
= 0.0113 or 1.13%
g
option (1) is correct.

2. (2)
1mm
Sol. Least count =  0.01mm
100
zero error = + 8 × LC = + 0.08 mm
True reading (Diameter)
= (1 mm + 72 × LC) – (Zero error)
= (1 mm + 72× 0.01mm) – 0.08 mm
= 1.72 mm – 0.08 mm
= 1.64 mm
1.64
therefore, radius =  0.82 mm.
2

3. (4)
Sol.  n 1 a  n  a '
a'
 n  1 a
n
 L. C. = 1 MSD – 1 VSD
=  a  a ' cm

a
 n  1 a
n
na  na  a a
  cm
n n
 10a 
  mm
 n 

NUCLEUS-92, Rajeev Gandhi Nagar, Kota (Raj.) India 324005, Mob. 9358006181, 97831-97831 372
JEE MAIN 2021
4. (5)
R V I
Sol.  100   100   100
R V I
2 0.2
% error in R = × 100 +  100
50 20
% error in R = 4 + 1
% error in R = 5%

5. (4)
Stress FL mg  L
Sol. Y  
Strain Al πR 2 
ΔY Δm ΔL ΔR Δ
   2 
Y m L R
ΔY  1 1  0.001  0.001  1 1 1 14
100  100    2        1.4%
Y 1000 1000  0.2  0.5  10 10 5 10

6. (1)
 V
Sol. R 
A I
AV d 2 V  d 2 
  A  
I 4I  4 
 2d V I 
    
 d V I
  0.01  0.1 0.01 0.1
 2   
  5.00  5.0 2.00 10.0

 0.004  0.02  0.005  0.01


 0.039


% error  100  0.039 100  3.90%

7. (34)
4 3
Sol. v r
3
taking log & then differentiate
dV dr
3
V r
3  0.85
  100%  34%
7.5

NUCLEUS-92, Rajeev Gandhi Nagar, Kota (Raj.) India 324005, Mob. 9358006181, 97831-97831 373
JEE MAIN 2021
8. (2)
4 2
Sol. g
T2
 1 
g  T 0.1  200 
 2   2 
g T 10  0.5 
 
g 1 1
 
g 100 50
g
100  3%
g

9. (2)
Sol. Positive zero error = 0.2 mm = 0.02 cm
Main scale reading = 8.5 cm
Vernier scale reading = 6 × 0.01 = 0.06 cm
Final reading = 8.5 + 0.06 – 0.02 = 8.54 cm

10. (4)
Sol. A tan  = tan ' cos 
= tan45° cos30°
3
tan  = 1 ×
2
 3
 = tan–1  
 2 

11. (1)
42
Sol. T  2 g
g T2
g  2T
 
g T
least count of time  T0 
T 
number of oscillations (n)
g  2T0
 
g nT
g
As  and T0 are same for all observations so is minimum for highest value of , n and T
g
 Minimum percentage error in g is for student number-1

NUCLEUS-92, Rajeev Gandhi Nagar, Kota (Raj.) India 324005, Mob. 9358006181, 97831-97831 374
JEE MAIN 2021
12. (13)
Sol. For (A)
Reading = MSR + CSR + Error
0.322 = 0.300 + CSR + 5 × LC
0.322 = 0.300 + CSR + 0.005
CSR = 0.017
For B
Reading = MSR + CSR + Error
0.322 = 0.200 + CSR + 0.092
CSR = 0.030
Difference = 0.030 – 0.017 = 0.013 cm
0.013
Division on circular scale =  13
0.001
13. (1)
Pitch
Sol. Least count =
total division on circular scale
In 5 revolution, distance travel, 5 mm
In 1 revolution, it will travel 1 mm.
1
So least count =  0.02
50

14. (14)
T 2g
Sol. T  2  
g 4 2
2 2 T 
2 2
E  mg  mg
2 82
dE  dg dT 
 2   = (4 + 3) = 14%
E  g T 
15. (3)
0.5
Sol. Least count (L.C) 
50
0.5 0.5 0.5
True reading  5   20  5  5 (15)  5.15mm
50 50 50
16. (52)
Sol. 9 MSD = 10 VSD
9  1 mm = 10 VSD
1 VSD  0.9 mm
LC = 1 MSD – 1 VSD = 0.1 mm
Reading = MSR + VSR  LC
10 + 8  0.1 = 10.8 mm
Actual reading = 1.8 – 0.4 = 10.4 mm
d 10.4
radius    5.2mm = 52 × 10–2 cm
2 2

NUCLEUS-92, Rajeev Gandhi Nagar, Kota (Raj.) India 324005, Mob. 9358006181, 97831-97831 375

You might also like